Vous êtes sur la page 1sur 230

SCHAUM'S

OUTLINE

THEORY aiMl>ROBLEMS

SERIES

of

MATRICES
by

FRANK AYRES,

JR.

including

Completely Solved

SCHAUM PUBLISHING
NEW YORK

CO.

in

Detail

SCHAVM'S OUTLINE OF

THEORY

AI\[D

PROBLEMi;

OF

MATRICES

BY

FRANK AYRES,

JR., Ph.D.

Formerly Professor and Head,

Department of Mathematics
Dickinson College

^C^O
-^

SCHAIJM'S OUTLINE SERIES


McGRAW-HILL BOOK COMPANY
New

York, St. Louis, San Francisco, Toronto, Sydney

('opyright

United

1962 by McGraw-Hill, Inc.

States

of America.

No part

AH

Rights Reserved.

Printed in the

may

be reproduced,

of this publication

stored in a retrieval system, or transmitted, in any form or by any means,


electronic, mechanical, photocopying, recording, or otherwise,

written permission of the publisher.

02656

78910 SHSH 7543210

without the prior

Preface
Elementary matrix algebra has now become an integral part of the mathematical background
necessary for such diverse fields as electrical engineering and education, chemistry and
sociology,
as well as for statistics and pure mathematics. This book, in presenting the
more

essential matedesigned primarily to serve as a useful supplement to current texts and as a handy reference book for those working in the several fields which require some knowledge
of matrix theory.
Moreover, the statements of theory and principle are sufficiently complete that the book
could
be used as a text by itself.
rial,

is

The material has been divided into twenty-six chapters, since the logical arrangement is
thereby not disturbed while the usefulness as a reference book is increased. This
also permits
a separation of the treatment of real matrices, with which the majority of readers
will be concerned, from that of matrices with complex elements. Each chapter contains
a statement of pertinent definitions, principles, and theorems, fully illustrated by examples. These, in
turn, are
followed by a carefully selected set of solved problems and a considerable number
of supplementary exercises.

The beginning student in matrix algebra soon finds that the solutions of numerical exercises
are disarmingly simple. Difficulties are likely to arise from the
constant round of definition, theorem, proof. The trouble here is essentially a matter of lack of mathematical maturity,'
and

normally to be expected, since usually the student's previous work in mathematics has
been
concerned with the solution of numerical problems while precise statements of principles and
proofs of theorems have in large part been deferred for later courses. The aim of the
present

book

is

lems

in

to enable the reader,

if he persists through the introductory paragraphs and


solved probany chapter, to develop a reasonable degree of self-assurance about the material.

The solved problems, in addition to giving more variety to the examples illustrating the
theorems, contain most of the proofs of any considerable length together with
representative
shorter proofs. The supplementary problems call both for the solution of numerical
exercises
and

for proofs.

Some

of the latter require only proper modifications of proofs given earlier;

more important, however, are the many theorems whose proofs require but a few lines. Some are
of the type frequently misnamed "obvious" while others will be found to call for
considerable

None should be treated lightly, however, for it is due precisely to the abundance of
such theorems that elementary matrix algebra becomes a natural first course for those seeking
to attain a degree of mathematical maturity. While the large number of these problems
in any
chapter makes it impractical to solve all of them before moving to the next, special attention
ingenuity.

is

directed to the supplementary problems of the first two chapters.


mastery of these will do
to give the reader confidence to stand on his own feet thereafter.

much

The author wishes


Publishing

Company

to take this opportunity to express his gratitude to the staff of the

for their splendid cooperation.

Frank Ayres,
CarHsle, Pa.

October, 1962

Jr.

Schaum

CONTENTS
Page
Chapter

MATRICES

Equal matrices.
Products by partitioning.
Matrices.

Chapter

Sums

of matrices.

Products of matrices.

SOME TYPES OF MATRICES

10

Scalar matrices. Diagonal matrices. The identity


matrix. Inverse of a matrix.
Transpose of a matrix. Symmetric
matrices. Skew-symmetric matrices. Conjugate of a matrix. Hermitian
matrices.
Skew-Hermitian matrices. Direct sums.

Triangular matrices.

Chapter

DETERMINANT OF A SQUARE MATRIX.

20

Determinants of orders 2 and 3. Properties of determinants,


and cofactors. Algebraic complements.

Chapter

Minors

EVALUATION OF DETERMINANTS

32

Expansion along a row or column. The Laplace expansion. Expansion


along the first row and column. Determinant of a product. Derivative
of a determinant.

Chapter

EQUIVALENCE

39

Non-singular and singular matrices. Elementary


transformations. Inverse of an elementary transformation. Equivalent
matrices. Row canonical form. Normal form. Elementary matrices.
Canonical sets under equivalence. Rank of a product.

Rank

Chapter

THE ADJOINT OF A SQUARE MATRIX


The

Chapter

of a matrix.

adjoint.

The adjoint

of a product.

Minor of an

49
adjoint.

THE INVERSE OF A MATRIX.

55

Inverse of a diagonal matrix. Inverse from the adjoint. Inverse from


elementary matrices. Inverse by partitioning. Inverse of symmetric
X w matrices.
matrices. Right and left inverses of

Chapter

FIELDS
Number

64
fields.

General

fields.

Sub-fields.

Matrices over a

field.

CONTENTS
Page
Chapter

LINEAR DEPENDENCE OF VECTORS AND FORMS

67

Vectors. Linear dependence of vectors, linear forms, polynomials, and


matrices.

Chapter

10

LINEAR EQUATIONS

75

System of non-homogeneous equations. Solution using matrices. Cramer's


Systems of homogeneous equations.

rule.

Chapter

11

VECTOR SPACES
Vector spaces.
section

space.

85

Sub-spaces. Basis and dimension. Sum space. InterNull space of a matrix. Sylvester's laws of nullity.

Bases and coordinates.

Chapter

12

LINEAR TRANSFORMATIONS
Singular and non-singular transformations.
space. Permutation matrix.

Chapter

13

94

Change of

basis.

Invariant

VECTORS OVER THE REAL FIELD

100

Inner product.
Length.
Triangle inequality.
Schwarz inequality.
Orthogonal vectors and spaces. Orthonormal basis.
Gram-Schmidt
orthogonalization process. The Gramian. Orthogonal matrices. Orthogonal transformations. Vector product.

Chapter

14

VECTORS OVER THE COMPLEX FIELD

110

Complex numbers.

Inner product. Length. Schwarz inequality. Triangle inequality. Orthogonal vectors and spaces. Orthonormal basis.
Gram-Schmidt orthogonalization process. The Gramian. Unitary matrices.
Unitary transformations.

Chapter

15

CONGRUENCE

115

Congruent symmetric matrices. Canonical forms


of real symmetric, skew-symmetric, Hermitian, skew-Hermitian matrices
under congruence.
Congruent matrices.

Chapter

16

BILINEAR FORMS

125

Matrix form.
formations.

Chapter

17

Transformations. Canonical forms. Cogredient transContragredient transformations. Factorable forms.

QUADRATIC FORMS
Matrix form. Transformations. Canonical forms. Lagrange reduction.
Sylvester's law of inertia. Definite and semi-definite forms. Principal
minors. Regular form. Kronecker's reduction. Factorable forms.

131

CONTENTS
Page
Chapter

lo

HERMITIAN FORMS
Matrix form.

146

Transformations.

Canonical forms.

Definite

and semi-

definite forms.

Chapter lif

THE CHARACTERISTIC EQUATION OF A MATRIX


Characteristic equation and roots.

Chapter

20

21

Invariant vectors and spaces.

SIMILARITY

156

Similar matrices.

Chapter

149

Reduction to triangular form.

Diagonable matrices.

SIMILARITY TO A DIAGONAL MATRIX

163

Real symmetric matrices. Orthogonal similarity. Pairs of real quadratic


forms.
Hermitian matrices. Unitary similarity. Normal matrices.
Spectral decomposition.

Chapter

22

Field of values.

POLYNOMIALS OVER A FIELD


Sum, product, quotient

common

divisor.

Least

of polynomials.
multiple.

common

172

Remainder theorem. Greatest


Relatively prime polynomials.

Unique factorization.

Chapter

2o

LAMBDA MATRICES

179

The X-matrix or matrix polynomial. Sums, products, and quotients.


Remainder theorem. Cayley-Hamilton theorem. Derivative of a matrix.

Chapter

24

SMITH NORMAL FORM


Smith normal form.

Chapter

25

Invariant factors.

invariants.

derogatory matrices.

26

Elementary

divisors.

THE MINIMUM POLYNOMIAL OF A MATRIX


Similarity

Chapter

188

Minimum

polynomial.
Companion matrix.

Derogatory

196
and

non-

CANONICAL FORMS UNDER SIMILARITY

203

Rational canonical form.


second canonical form. Hypercompanion
matrix. Jacobson canonical form. Classical canonical form. A reduction
to rational canonical form.

INDEX

215

INDEX OF SYMBOLS

219

'

chapter

Matrices

A RECTANGULAR ARRAY OF NUMBERS


"2

enclosed by a pair of brackets, such as

and

-1

l'

(a)

(b)

and subject to certain rules of operations given below

is

called a matrix.

The matrix

(a) could

be

(2x + 3y + 7z =
linear equations
\ X- y + 5z =

considered as the coefficient matrix of the system of homogeneous

i2x + 3y = 7
-

or

as the augmented matrix of the system of non-homogeneous linear equations

we

Later,

shall see

how

the matrix

may be used

dinates of the points (1,3,

1),

(2, 1,4),

to obtain solutions of these

we might consider

The ma-

rows as simply the coor-

and (4,7, 6) in ordinary space.

The matrix will be used


same plane with the

or

such questions as whether or not the three points


on the same line through the origin.

later to settle

origin or

systems.

y = 5

its

could be given a similar interpretation

trix (b)

\x-

lie in the

In the matrix

^11 ^12

'^IS

'

(1.1)

*mi "m2 ""m3

the numbers or functions a^- are called its elements. In the double subscript notation, the first

subscript indicates the row and the second subscript indicates the column in which the element

stands.

Thus,

all

elements in the second row have

the fifth column have 5 as second subscript.

"m by

order

(In

used.

ra"

or

as

first

subscript and all the elements in

rows and n columns

said to be of

),

and double bars,

||

||,

are

sometimes

shall use the double bracket notation throughout.)

At times the matrix (1.1) will be called "the mxra matrix


[a^-]".

is

mxra.

indicating a matrix pairs of parentheses,

We

matrix of

When

we

the order has been established,

SQUARE MATRICES. When m

n, (1.1) is

[a^

"

or

"the

shall write simply "the matrix

mxn

matrix A =

4".

square and will be called a square matrix of order n or an

re-square matrix.
In a square matrix, the

The sum

elements

of the diagonal

a^,

022.

" are called

elements of a square matrix A

its

diagonal elements.

is called the trace of A.

MATRICES

EQUAL MATRICES. Two

matrices A = [a^] and B =

[CHAP.

[bij]

are said to be equal (A =

B)

if

and only

if

they have the same order and each element of one is equal to the corresponding element of the
other, that is,

if

and only

if

Thus, two matrices are equal

ZERO MATRIX. A

'V

^^J

and only

if

= 1,2,
if

one

which

matrix, every element of

ro;

1,

n)

duplicate of the other.

is a

is zero, is

When ^

called a zero matrix.

we

matrix and there can be no confusion as to its order,

is a zero

shall write A = Q instead of the

mxn

array of zero elements.

SUMS OF MATRICES.
is

If

defined as the

4 = [a^A and S =

mxn

corresponding elements of
1.

31

It

[fe.^-]

[c^A

mxn

are two

matrices, their

is the

sum

(difference),

sum

A B,

(difference) of the

'\

Example

C =

where each element of C


A and B. Thus, AB
[Oy bij]

matrix

14

and =

2
I

then

-12

+B

0'

Lo+(-

1)

5
2

+3

+2

+ 5j

and

Two

1-2

2-3

3-0

O-(-l)

1-2

4-5

matrices of the same order are said to be conformable for addition

matrices of different orders cannot be added or subtracted.


(b) above are non-conformable for addition and subtraction.

The sum

of k matrices

is

a matrix of the same order as

times the corresponding element of A.

We

define:

If

2.

If

[:

'11

A+A + A

subtraction.

A and each

(a)

Two
and

of its elements is k

k is any scalar (we call k a scalar to dis-

tinguish it from [k] which is a 1x1 matrix) then by kA = Ak


A by multiplying each of its elements by k.

Example

or

For example, the matrices

is

meant the matrix obtained from

then

-2

3A

A-3

and

-5A

r-5(l)

-5(-2)"|

r-5

10-1

L-5(2)

-5(3) J

L-10

-15j

by -A, called the negative of /4, is meant the matrix obtained from A by mulby -1 or by simply changing the sign of all of its elements. For
every A, we have A +(-A) = 0, where
indicates the zero matrix of the same order as A.
In particular,

tiplying each of its elements

Assuming
(a)
(b)
(c)

(d)

that the matrices

A,B,C are conformable

for addition,

we

state:

A + B = B + A
(commutative law)
A + (B+C) = (A + B)+C
(associative law)
k(A + B) = kA + kB = (A + B)k, A- a scalar
There exists a matrix D such that A + D = B.

These laws are a result of the laws of elementary algebra governing the addition of numbers
and polynomials. They show, moreover,
1.

Conformable matrices obey the same laws

of addition

as the elements of these matrices.

CHAP.

MATRICES

1]

MULTIPLICATION. By

the product

AB

in that order of the

Ixm

matrix A = [a^i a^g ais

and

a^m]

fell

the

^31

mxl

matrix

meant the 1x1 matrix C = [on fen + 012

is

fi

fesi

+ aimfemi]

fell
fe^i

That

[an

is,

Oi

'^imj

L^ii

fell

"*"

^12

feji

imi]

-|^^2

a^^fe^^J

femi

Note that the operation is row by column; each element of the row is multiplied into the corresponding element of the column and then the products are summed.
1'

Example

(a)

3.

[2

[2(l) + 3(-l) + 4(2)]

4]

[7]

2.

-2
(b)

[3

-1

[-6 - 6+ 12]

4]

3J

By
is

the product

meant the

mxn

AB

in that order of the

matrix

C =

\c;;'\
<-

ii^ij + "t2 ^2; +

Hj

-'

mxp

matrix A = [a^-] and the p

xn matrix B =

[bij]

where
p

^^^"ikbkj

"-ip i>,pj

(f

1, 2,

,m-;

1,

re).

Think of A as consisting of m rows and B as consisting of n columns. In forming C = AB


each row of A is multiplied once and only once into each column of B.
The element c^ of C is then
the product of the ith row of A and the /th column of B.

Example

4.

^11

*^ 1

."31

'011611 + 012621

O32

[611

612I

621

622J

021611+022^21
."31611+032621

11 ^12 + "12 622


0^21

^12 +

'^22

^22

"31612+002600

The product ^S is defined or A is conformable to B for multiplication only when the number
columns of A is equal to the number of rows of S. If ^ is conformable to B for multiplication
{AB is defined), B is not necessarily conformable to A for multiplication (BA may or may not
be
of

^^^ined)-

See Problems

Assuming

that

A,B,C

are conformable for the indicated

(e)

A(B + C) = AB

(/")

(A +

AC

(g)

A(BC) = (AB)C

B)C

AC
BC

3-4.

sums and products, we have

(first distributive law)


(second distributive law)

(associative law)

However,
(A)
(i)
(/)

AB
AB
AB

i=

=
=

BA, generally,
does not necessarily imply i =
or S =
does not necessarily imply B = C.

0,

AC

See Problems

3-8.

MATRICES

PRODUCTS BY PARTITIONING.

A=

Let

[CHAP.

be of order

[a^J]

mxp

8=

and

forming the product AB, the matrix A is in effect partitioned into

Other partitions may be used.

into n matrices of order pxl.

[b^j]

be of order pxn.

In

matrices of order Ixp and B

For example,

A and 6 be

let

parti-

tioned into matrices of indicated orders by drawing in the dotted lines as


(piXTii)

(mixpi)

(m]^xp2)

(P2X%)
Jmgxpi)

(m,2Xp2)

(p2Xra2)

(m2Xp3)_

(psxni)

Am

(pixnj)

(m-Lxps)

A^2

(p3Xre2)

Aig
"21

O22

A,

A^i
I

"31
In

any such partitioning,

in exactly the

necessary that the columns of A and the rows of 6 be partitioned


m^, mg, re^, rig may be any non-negative (including 0) integers
and rei+ ^2 = n. Then
it

is

same way; however

such that mi+ m2 =

^llOll + '4i2021

^8

"32

""

'4l3"31

A-^-^B-12

+ A-^qBqq + '^13832

Cii

A-2_iiB^^

A^^B^^ + ^22^22 + ^423632

_C21

C12

c
A21B11 + 422^21 +

Examples.

Compute

/4S, given

/I

1^22.

1110
and B
1

110

12

Partitioning so that

^12

11

A21 A22

AB

we have

'0

10

11

[A^-iB-i^i

111

^12

fill

and

S22

B21

+ A-12B21

A-ij^B'12 + A-i,

=
1

^21^12 *

AqiB^^ + ^22^21
1

i4'

'

+ [l][2

1]

-^2^

B,

[2]

[3

3]

[1

0]

[?

1"

[1

0]

_[l

3-1

[4

1]

To

+ [l][2]

[2

o]

1]

To]

Tol

[0]+[2]

5JL0J

[3

2] [2]

See also Problem

Let A, B.C... be re-square matrices.

9.

Let A be partitioned into matrices of the indicated

orders

(pixpi)

(piX P2)

(P2X pi)

'

""

^"
(P2X Ps)^

(piX Ps)"

All

A-^2

(P2XPS)

A21

A22

'
j

.(PsXpi)

and

let

8, C,

...

(psXp2)

...
I

(PsXPs)

/loo

be partitioned in exactly the same manner. Then sums, differences, and products
the matrices A^i, A^2< : Sii. 5i2. - ^n, C12

may be formed using

CHAP.

MATRICES

1]

SOLVED PROBLEMS
2-10

(a)

1.

2I
3

2_

2-2 3-1

'l

-1

O'

3-412

2-5

12

2.

"l

-1

o"

-2
-5

-3

Then D

= 4,

[4

[J
4
(c)

[1

3]

-6
-7
8

5
[

{2
.

Let

<4

-1

'2

-2
2

-3 -2

such that A + B -

2-p

-9

-1-s

"0

T)

0.

0"

-2-p

9-u_

4-r

and p = -2,

.0

A +

9.

[17]

2(5)

2(6)-

3(5)

3(6)

-1(4)

-1(5)

10

12

12

15

18

.-4 -5 -6

-1(6).

6"

-11 -8
1

(-6) + 2 (-7) + 3(8)

r2(l) + 3(2)+4(3)1

Ll(l) + 5(2) + 6(3)J

'

2J

1(9) + 2(10) + 3(-ll)

1 (6)

+ 2(7) + 3(-8)]

poT
L29J

ri(3) + 2(l) + l(-2)

l(-4) + 2(5) + 1(2)]

[4(3) + 0(1) + 2 (-2)

4(-4) + 0(5) + 2(2)J

8]
[s -I2J

1"

.1

A""

3-5

-2

1-3
2+1

4-r
9-

3(4)

10

"gl

_2

2J

0-2

-4 -4]

C
l]

(e)

2 + (-l)

+3

0"

-1

1(4) + 2(0) + 3 (5)

[19 4

(rf)

2(4)
=

e]

[4

(b)

D =

[4(2) +5(3) + 6(-l)]

6]

6,

3. (a)

find

-2
r

2-2- ?1
4-5-s
6+3- u.

3+1-r

_5+4-
and

4-202
5 2 4
5
4-741

0+2
1+3

2+0

-1 -2
1
-4
-2 -1
-2
5 -1 -2

and 6

A-\-B-D

-1 +

2+4 -1-1
2-0
0-5
-5+2
1-3

1-3
4-1
2-2

-15

+2

12

1-3-p
If

0+5
-5 + (-2)

6-3

-5

-5

2+(-4)

4+1

2-2 3-1

ri + 3
=

-1

If

15

2.

-4

-5

(d)

(b)

(c)

Then

1.

-1

The reader

\2

-1
1

will

r
2

1-

show

that

"5

-3

_3

-1

r
4

and

2.

A.a'^ and ^^./l^ = /4^.^^.

-3 f

-1

-1
1

.1

1'

2
1.

11
=

-8

8-18

.8-43

MATRICES

5.

Show

2
2:

k=i

a,-,-,

j=i 1=1

22

a--

1=17=1
(c)

that:
2

(b)

[CHAP.

aife(

2 (2

b^hCj^j)
^

h=t

h=i

k=i

aif,bkh)chj
^

2
(")

J,

"il^^i +

ife(*fei +'^fej)

+ '^t2(*2i+'^2i)

'^lj)

2
(6)

(il^i+''i2*2i)

(il<^17+'^i2'^2i)

2 2

a--

t=i j=i

(a^j^+ a-^ + a -p)

(an + 0^2 +

(Oil + O21)

+ (a^t + 092 +

a^g)

<223)

i=i

''

a-

"

i=l

+ (12 + "22)
2

+ (ai3 + "23)

=22
2

a +
a.
= l ^2
i = i 13

o--

j = ii = i

This is simply the statement that in summing all of the elements of a matrix, one may sum
elements of each row or the elements of each column.
2

(c)

^_j

222

^-^

(oil
=

a-,(bi, c
tfe^

fei

ij

6,

fee

&,

fe3

2JI

,^

Prove: It A = [a^-]
= AB + AC.

The elements
62J+

of the

j'th

fenj

+ 012(621^17 + b2QC2J + 623C3J)

(2

a{fefefe2)c2f
-^

fe=i

fe=i

ai<^b^^)c3j
**
-^

/.l/li^^^'fe'^^'^'^r

is of order

c'2j

oifefefei)cii

O
37'

+ ai2*2i)cij + (0^1612 + aiQh.22)cQj + (aiifeis + ai2b23)cgj

fell

k=l

feijj+cij,

the

a., (
b,,c,-)
tfe^^^j^ kh hy

0-il(^ll'^lj + ftl2C2J+ ^IsCsj)

6.

first

row

mxn
of

and

if

are a^^

+%j- Then

S =

[fe^-]
^

a^^,

...

and C =

[c--] are of order

nxp, then

%i)

C)

+C

are

a^^ and the elements of the /th column of S

the element standing in the ith row and /th column of

oil(feij + Cij) + afe(fe2j + C2j) +... +o.i(feni +

/}(B +

'

^^^"ikHj+^kj)
AB and ^C.

S^^iifefefej +^2^aifeC;^j,

A(B + C)

is

sum

Of

the

the elements standing in the ith row and /th column of

7.

Prove: If i = [aij] is of order


then ^(6C) = (iS)C.

The elements
p

mxn,

S =

[6^-] is of order rexp,

and

if

C =

[c.-A is of order

throw

and the elements ofthe/th column

of /4 areai^.a^g, ...,a-

BC

of

h=
b^j^c^j-,

hence the element standing


P

P
n
^^^^j_"ik''kh)<^hj

This

is the

...

and then

(/),

Using

(/")

and then

(e),

*n/i<;/ij

j'th

(A+B)(C+D)
(A+B)(C+D)

+ (X^aif^bf^2)<:QJ

+ (^^"ikbkp)<^pj

row and /th column of (AB)C; hence, A(BC)

in

+ (A

+B)D

(A

A(C +D) + B(C +D) = AC + AD ^ BC + BD


= AC +BC +A.D +BD.

(AB)C.

AC + AD
AC +BC +AD +BD.

two ways that (A +B)(C +D) =

+B)C

P
n
^^"ik<-j^,^hh<^hj^

(2^^0ifefefei)cij

Assuming A, B ,C,D conformable, show


(e)

J^

n
=

element standing in the

Using

+ tn

6ih c^
i

row and /th column of A (BC) is

in the ith

ait^^b^chj +ai2^^b2h%j
=

are

P
b^f^Cf^-

pxo

-^

P
of the

8.

if

BC

BD.

'iJ'

CHAP.

MATRICES

1]

o'

"l

l'

9. (a)

10

0'

10

'1

10
1

o"

"4

2"

[0]
2

:]

:o]
[0

j2

10

10

2]

10 3 10

[3

10

oil

10

o'

'l

2_

'

(b)

1
1

_3

'1

10

"l

[0]

[0:

[0]
3]

[0

"][o

2
3

12

10
18
0'

"1

0^0

-4- i

3|4

6_

4 15

8.

_0

4'

(c)
1

+i

-^-

Hiyi+

%i

Let

%2

[l]-[6 5 4]

'7

11'

"13"

10

13

16

19

"35

24

37
26

39"

20

33
22

13

13

.13

13

[6

10.

[l]-[8 7]

31

7"

[1]-[1]

11

13

13

16

19

41"

31

28

30

20

33
22

10
35
24

37
26

39 41
28 30

13

.13.

13

13

13

13

13

13

4]

[l]

"1272
11^1

= a^iy-i + 0^272 be three linear forms in y^ and yj ^-nd let

%3

os

yi +

% 2 72
new coordinates

linear transformation of the coordinates (yi, j^) into

0]^2^2

be a
.J1

(z^, zg).

O21 Zi +

The

h-2

result of applying

the transformation to the given forms is the set of forms

(on ii2 +

1) ^1

"*"

(^2

("31^11 + ^32^21) Zl

("3

1^12 + "32 ^22)

%i

(o-Li

%<2

((^2 1 C^l 1

X3

6^j +
"*"

aj^2

621) Zi

^2 2 ^2

^12

*i

Using matrix notation, we have the three forms

"*"

'^12^22) ^2
^2 2 ^22) ^2

%1

012

"21

"22

r-

Z2
-.

Vl

and the transformation

Vr,

^3

611

612

O21

022

The

'^3

result of applying the transformation is the set of three forms

"11

'xi

%2

"2

"^2

Os

^^3

pri ii2
^2

Thus, when a set of m linear forms


formation of the variables with matrix B

in n variables
,

622

-1

/,
^',

with matrix A is subjected to a linear transm linear forms with matrix C = AB

there results a set of

'

MATRICES

[CHAP.

SUPPLEMENTARY PROBLEMS
-3'

"l

Given A =

-1

.1

1_

-1

-2-4

Compute:

12.

Verify:

(d)

Find the matrix

-1

-1

-2

0-S

(A+B)-C.

D such

-3

Given A =

-51

t-2j

-4
-2

-10

A + {B~C)

(c)

3_

5-3

4j

-2

J -2

A-C

2I

-3

and C =

3_

-2 -4
(6)

-f

-1

_3

2I

_2

Compute: A + B =

(a)

A+D^B.

that

and B

AB

compute

=B-A

Verify that

= Q

-(A-B).

and

BA

-1
-2
-1

-11

-22

12

-11

that

AB

Hence, ABi^BA

generally.

\ -3
13.

Given A =

14

-3
-3 -1

-1

15.

Using the matrices of Problem

Given

Show
[

20.

BA

use the results of

Ans.

19.

AB

show

(b)

A, -I,

I,

AC

= 0,

(a) to

2
i

show

+ B^

AC

AC

or

more

A(BC).

and

(A+B)C

AC +BC.

-2 -3

= C.

ACB

as n

-2 -4

-13

(AB)C

that

- B^ ^ (A-B)(A+B).

and A^

and

show

CS^,

A'^

B"^ =

(A

-B)(A +

B),

= -1, derive a formula for the positive integral

any two

AC. Thus, AB

ij

AB

CA

that

A(B + C)

2AB

-3 -5

= A,

show

-A according

that the product of

that

-13

where

show

3 -4"!

0-2

[2

-13

-3 -4

^["12

and C

(ABf ^ A^

(a)

that

11,

-3 -5

-14

Given A =

18.

-1

16. Explain why, in general,

17.

1-1-2
-2 -1 -1

2-5-1

2
3

-1

Given A =

and C

does not necessarily imply B

14.

10

111
1-212

=4p,

4p-l-l,

ip +

2,

4p-^3, where

(A

Bf

powers of A

/=

[0

ij-

-'"'[;;][-:;] [rM'i

= A'^ + b'

-"]

[0 -]- ["0 3-

is a matrix of the set.

[? 0]

Given the matrices A of order mx, B of order nxp, and C of order rx^, under what conditions
on p, 9,
and r would the matrices be conformable for finding the products and what is the order of each" (a)
ABC
(b)ACB, (c)A(B + C)?
Ans.

(a)

p =

r;

mx-q

(b) r = n = g;

x p

(c)

= n, p = q;

x g

CHAP.

21.

MATRICES

1]

Compute AB, given:

10
A

(a)

10

11

0^

and

10

Ans.

and

Ans.

1
--1

12
1

10

and
I

'2

'

(a) trace (A

10

Ans
1

22. Prove:

10

(c)

o"

(b)

"

+ B) = trace A + trace B,

(6) trace (kA) = k trace

A.

h
=

V,

2n+r.-3y.

r -zi +

Y^l

y,\

^;^^^l^

[2

-3

[2

-2
1

-1

-3

722"]

[-221 - 622J'
24. If

-4

and B = [fe^J are of order

= [aiji

25. Let

= [a^-] and

/4

[fej,-;,]

where

(i

x n

1,

and

if

m;

= [c^j] is of order nx. p,

1,

p;

A;

show

1, 2, ...

,n).

that

(/4+B)C

Denote by

/S-

=^C
the

+ BC.

sum

of

Pi
182

the elements of the /th row of B, that is, let fij =

j)^.

Show

that the element in the ith row of

Pi
sum

is the

of the

Problems 12 and
26.

(II)

check the products formed

to

Determinative

Either a is in the relation to 6 or a is not in the relation to

Reflexive

a is in the relation to a, for all

(iii)

Symmetric

If

Transitive

If a is in

is called an

Show
relations.

in

congruency) between mathematical entities possessing the following properties:

(iv)

Show

Use this procedure

13.

A relation (such as parallelism,


(i

27.

elements lying in the sth row of AB.

b.

a.

a is In the relation to b then b is in the relation to a.

the relation to b and b is in the relation to c then a is in the relation to

c.

equivalence relation.

that the parallelism of lines,

Show

similarity of triangles,

and equality of matrices are equivalence

that perpendicularity of lines is not an equivalence relation.

that conform ability for addition of matrices is an equivalence relation while con form ability for multi-

plication is not.
28. Prove:

If .4, B,

are matrices such that

AC

CA and BC

CB,

then

(AB

BA)C

C(AB

BA).

chapter 2
Some Types

THE IDENTITY MATRIX. A


lar;

of Matrices

square matrix A whose elements a^: =

a square matrix A whose elements


"ii

"12

"13

tip o

tZo o

a^=

for

upper triangu-

Thus

upper triangular and

is

lower triangular.

is

both upper and lower triangular,

is call-

a,

...

It

is called

lower triangular.

is

which

Ogj

ed a diagonal matrix.

i>j

for

is called

"2n

03S

The matrix D

i<j

will frequently be written as

diag(0]^i, 022, 033,

'

^nn )

See Problem
If

in the diagonal matrix

D above,

Oi^ = Ogs =

k,

and

is

ci

in addition, k = l, the matrix is called the identity matrix

is

called a scalar matrix;

denoted by

/.

1.

if.

For example

'1

t"]
When
/+/ +

...

and

1
1

the order is evident or immaterial, an identity matrix will be denoted by /. Clearly,


to p terms = p / = diag (p,p,p
= I-I ... to p factors = /. Identity map) and

[1.

A.L

hAh

A, as the reader

may

readily show.

10

3I

(5

then l2-A =

CHAP.

SOME TYPES OF MATRICES

2]

SPECIAL SQUARE MATRICES.

A and B

If

are square matrices such that

called commutative or are said to commute.


matrix,

it

commutes with

11

is a

It

AB

= BA, then A and B are

simple matter to show that

if

is

any ra-square

itself and also with

See Problem

A and B are such

If

matrix

AB

that

-BA,

i =

matrix

1,

A and 6

are said to anti-commute.

,fe+i

for

which A

where

A,

A:

is a positive integer, is called periodic.

If

k is

See Problems

3-4.

k-t-i

the least positive integer


e for which
If

th^ matrices

2.

then A is said to be of period

A,

k.

so that A^ = A, then A is callled idempotent

for

aP

4=0,

which

where p

least positive integer for which A'^ =

0,

a positive integer, is called nilpotent.

is

ther

is said to

If

p is the

be nilpotent of index

See Problems

5-6.

THE INVERSE OF A MATRIX.

If A and B are square matrices such that AB = BA = I, then B is called the inverse of A and we write B = A'^ (B equals A inverse). The matrix B also has A
as its
inverse and we may write A = B~^

Example

1.

Since

-1

12

-1

-2 -3

o'

/,

each matrix in the product is the inverse of

( 1

the other.

We

shall find later (Chapter?) that not every square matrix has an inverse.
A has an inverse then that inverse is unique.

We can show

here, however, that if

See Problem

7.

If A and B are square matrices of the same order with inverses A^^ and B~^ respectively,
then (AB)''^ = B'^-A~^, that is,
1. The inverse of the product of
verse order of these inverses.

matrices, having inverses, is the product in re-

tijvo

See Problem

A
utory.

matrix

An

A such

that A^ = I is called involutory.

An

identity matrix, for example, isinvol-

own inve r$e.

involutory matrix is its

See Problem

THE TRANSPOSE OF A MATRIX. The


columns

of

an mxn matrix A

is

matrix of order

and /th column of A stands


If

In

^[-e]

in the /th

IS

i' =

nxm obtained by interchanging the rows and


A and is denoted by A' (A transpose). For

Problems 10 and
II.

11,

The transpose

(A'Y =

we

4'

6.

Note that the element

aIr

in the ith

A and

and

B,

a scalar, we have immediately

and

if

(b)

(kAy = kA'

A:

is

prove:

of the

row

row and ith column of A\

A' and ZTare transposes respectively qf


(a)

9.

called the trahspose of


1

example, the transpose of A

8.

sum

of two matrices is the

(A +

BY

A'+ S'

sum

of their transposes, i.e.

SOME TYPES OF MATRICES

12

[CHAP. 2

and
in.

The transpose

transposes,

of the product of

two matrices

product in reverse order of their

is the

i.e.,

(AB)'

B'-A'

See Problems 10-12.

SYMMETRIC MATRICES. A
A =

matrix

is

a^j]

/.

Thus, a square
For example,

4-5
3-5 6
Problem
IV.

symmetric and so also is kA

is

In

square matrix A such that A'= A is called symmetric.


symmetric provided o^-- = a,-,^ for all values of i and

k.

we prove

13,

If

any scalar

for

is

an ra-square matrix, then A + A'

symmetric.

is

A square matrix A such that A'= -A is called skew-symmetric. Thus, a square matrix A is
skew-symmetric provided a^.- = -Uj^ for all values of i and /. Clearly, the diagonal elements are
"

-2

For example,

zeroes.

3"

skew-symmetric and so also

is

is

kA

for

any scalar

k.

-3 -4
With only minor changes in Problem 13, we can prove
V.

If

is

any re-square matrix, then A -

A' is

skew-symmetric.

From Theorems IV and V follows


VI. Every square matrix A can be written as the sum of a symmetric matrix B = ^(A+A')
and a skew-symmetric matrix C = ^(A-A').
See Problems 14-15.

THE CONJUGATE OF A MATRIX.

Let a and b be real numbers and let i = V-1 then, z = a+ hi is


The complex numbers a+bi and a-bi are called conjugates, each
the other. If z = a+ bi, its conjugate is denoted by z = a+ hi.
;

called a complex number.

being the conjugate of


z^

If

= a + bi

and

Z2

z^

a~

Z2 =

then

bi,

z-^

a-

bi

= a+

bi,

that is, the conjugate of

the conjugate of a complex number z is z itself.

z.i=a+bi and

If
(i)

z-L

Z2

= c+

+ Zg = (a+c) + (b+d)i

sum

that is, the conjugate of the

(ii)

z^- Zg

then

di,

and
of

= (ac-bd) + (ad+bc)i

z^

= (a+c) - (b+d)i =

z^

two complex numbers

z^^

and

is the

sum

(a-bi) + (c-di)
of their conjugates.

= (ac-bd)- (ad+bc)i = (a-bi)(c-di) = F^-i^,

that is, the conjugate of the product of two complex

numbers

is the product of their conjugates.

When i is a matrix having complex numbers as elements, the matrix obtained from A by replacing each element by its conjugate is called the conjugate of /4 and is denoted by A (A conjugate).
l

2i

then
3

If

A and B

(i)

-21

and

(ii)

(.4)

above,

2-3i

are the conjugates of the matrices


(c)

Using

Example 2. When A

= A

we may prove

and

3i

A and B and itk


(c?)

is

any scalar, we have readily

(X4) = ~k-A

CHAP.

SOME TYPES OF MATRICES

2]

13

_VII. The conjugate of the sum of two matrices is the sum of their conjugates,
(A + B) = A + B.

Vin. The conjugat e of the product of two matrices


(AB) = A-B.

is the product, in the

same

i.e.,

order, of

their conjugates, i.e.,

The transpose
We have
IX.

A, i.e.,

Example

3.

of

is

denoted by A^(A conjugate transpose).

T^e transpose
(Ay = (A').

Prom Example
1

(AY

of the conjugate of

- 2i

3
2+

square matrix
Hermitian provided a^j = uj^ for
Hermitian matrix are real numbers.
1

square matrix

^ =

2i

and

[0^^]

[l-2i

3
2

such that
i and /.

A*.

= A

1
=

-i

A'

(A')

Zi

values of

all

1-i

+j

(AY

+ 3iJ

is called Herraitian.

Thus,

/I

Clearly, the diagonal elements of an

is Hermitian.

-I

if

+
i

is

kA Hermitian

sometimes written as

equal to the conjugate of the transpose of

A'

3i

HERMITIAN MATRICES. A

Hermitian provided

while
-i

Is

is

is

Example 4. The matrix A

It

k is any real number? any complex number?

^ =

such that

[0^^]

a^^ = -o^--

for all

1= -A

values of

is

called skew-Hermitian.

and

Thus, ^

is

skew-

Clearly, the diagonal elements of a


skew-Hermitian matrix are either zeroes or pure imaginaries.

Examples. The matrix A

l-i

-1-t

3i

-2

is

/.

skew-Hermitian.

Is

kA skew-Hermitian

it

k is any real

number? any complex number? any pure imaginary?

By making
X.

If

minor changes in Problem 13,

is

we may

an ra-square matrix then 4 +

Z is

prove

Hermitian and

A-T is

skew-Hermitian.

From Theorem X follows


XI. Every square matrix A with complex elements can be
written as the sum of an
Hermitian matrix B={(A + A') and a skew-Hermitian matrix
C=i(A-A').

DIRECT SUM.

Let A^, A^

As be square matrices of respective orders m^,

ization

^1

...

A^

...

diag(^i,^2
...

of the diagonal matrix is called the direct

A,

sum

of the Ai

As)

,ms.

The general-

SOME TYPES OF MATRICES

14

Examples. Let

/4i=[2], ^5

B:}

and An

-1

-2

[CHAP. 2

12
The

sum

direct

Problem 9(6), Chapter

A^,A^,Aq

of

diag(^3^,

is

/42, /4g)

12-1
2

1-2

illustrates

1,

XII. If A = Aia,g {A^,A^,


same order for (s = 1, 2

the

...,Ag)
s),

S = diag(Si, S2

and

AB

then

^s).

= diag(^iSj^, A^B^

where Ai and Behave

^s^s)-

SOLVED PROBLEMS

Or,

1.

Since

ii

"12

2i

"22

''ml

"7)12

'm

2n

"tn

11^1

OooOoo

22 21

''WB "Ml

"^22 ''2n

the product

AB

of

^mm"nn

''wm"m2

an m-square diagonal matrix A = diag(oii, a^^


an) and any mxn matrix B is obtained by multiplying the first row of B by %i, the second row of B by a^^- and so on.

2.

Show

This follows from

Show

-13

that

4.

Show

that

"^l

-2 -4'

-2 -3_

'

P^+*'^

-2

-4

'"^

-2 -4"

-13
.

-2 -3_

AB

A and BA =

B,

(AB)A

A-A

ABA

that if

.45.4 =

show

commute

for all

values of

'^l

["

o, 6, c, J.

*1

is idempotent.

-13

^1

-2 -3_

\"

and

-2 -4

2
3.

that the matrices

is idempotent.

A^ and

then i and
= .4(34) =

-2

-3

AB

are idempotent.
=

then 4^ =

.4

and 4

is idempotent.

Use BAB

to

CHAP.

5.

SOME TYPES OF MATRICES

2]

Show

that

is

Let

8.

A,B,C

Prove:

By

-1

-3
3"

-1 -3

A^ =

0,

a''

2,

...

^"

that

AOAf=

= 0.

Then A{IAf

(AB)(ABf

)AB

Supposed

is involutory if

11.

is involutory; then

(A+B)' =

A' +

A{lnA)

(AB)' =

B^-I-B

(AB)^

A-A

A(B-B''^)A~^

and only

A^ =

= [6^].

-2 -1 -3

AnA^

= A.

iCA)B =

CUB)

so that B

B-'.B
=

B-'.A-'

(IA)(I+ A) =

if

and

/I

0.

is involutory.

and (I-A)(I+A) =

We need

S' and (A+B)' are respectively a^,

Prove:

any positive integer.

for n
=

I-A^

I-I

= 0.

B'.

A = [ay] and B

Let

Now

/.

(AB)'^ is unique; hence,

7,

matrix

Suppose (I-A)(l+A) = I-A^ = 0; then A^ =

A'.

b\a''-A)B

AB(B~'^-A~'-)

Prove:

B~^-A'^

{ABf{AB)

definition

By Problem

10.

-1 -1 -3

be square^ matrices such that AB = 1 and CA = 1. Then


^ is
the unique inverse of ^. (What is S~^?)

(AB)^ =

5 = C= ^

and

Prove:

A^.A

-1 -3

-1

show

and

-2 -1 -3

(B' -A

9.

is nilpotent of order 3.

nilpotent of index

Thus,

C.

Since /? =

7.

5
-2

6. If

15

bjj_.

only check that the element in the ith row and /th column of

and aj^+

bj^.

S'i'.

Let A = [ay] be of order mxn,

[6y] be

element standing in the ith row and /th column of

of order

AB

is

cy

nxp
=

then

AB

[cy]

is of order

mxp.

The

J^aife. b^j and this is also the element stand-

ing in the /th row and ith column of (AB)'.

The elements of the

/th row of S'are

Hn- Then the element

"i2

in the /th

iy, b^j

Thus,

12.

Prove:

(AB)' =

bnj

and the elements of the ith column of ^'are a^^,

row and ith column of

B'/l'is

B'A'.

(ABC)' = CB'A'.

mite

ABC

(AB)C.

Then by Problem
,

1 1

(ABC)' = \(AB)C\' = C'(AB)' =

C'B'A'.

SOME TYPES OP MATRICES

2g

13.

Show

A =

that if

[o^j]

B =

is n-square, then

[bij] =

[CHAP.

+ A' is symmetric.

First Proof.

The element
oj^j]

hence,

in the ith

row and /th column of

The element

bij = aij + a^i.


bji

in the /th

a^^ + aij.

Thus,

and the corresponding element of

is aij

.4

row and ith column of A is

and B

bij = bji

is

a^i

/I'

hence,

is aji;

and the corresponding element of

.4'

is

symmetric.

Second Proof.

By Problem

14. Prove:

If

10,

A and B

(A+Ay

A' +

(A')'

A'+A

A + A' and

are re-square symmetric matrices then

(^ +.4') is symmetric.

AB

symmetric

is

and only

if

if

A and B

commute.
Suppose A and B commute so that
Suppose

is

Then (AB)' =

BA.

Now

symmetric so that (AB)' = AB.

B'A' =

BA

(AB)' = B'A' =

AB and AB

BA

AB

hence,

the m-square matrix A is symmetric (skew-symmetric) and


symmetric (skew-symmetric).

15. Prove:

If

is

If

.4

is

symmetric then (see Problem 12) B'

If

,4

is

skew-symmetric then B' = (P'AP)'

-P'AP

If

A and B are

16. Prove:

commute

for

Akl

if

P'AP

and B

BA and

the ma-

AB
=

= B.4

A and 5 commute

if

and only

and

AB - k(A+B)

+ k^I

BA - k(A+B)

+ k^I

(B-kr)(A-kl)

(B

and B kl commute.

(A-kI)(B-kI)

BA. and A and B commute.

is

mxn then B =

symmetric.

and B is skew-symmetric.

Suppose 4 -A;/ and B -kl commute; then

is of order

k.

{A-kI)(B-kI)

Thus,

(P'AP)' = P'A'(P')' = P'A'P

re-square matrices then

every scalar

Suppose A and B commute; then

AB

symmetric.

is

A and B commute.

trices

P'AP

AB

AB

AB - k(A+B)

+ k^I

BA - k(A+B)

+ k^I

-kr\(A-kI)

if

A~

kl and

B-

kl

CHAP.

SOME TYPES OF MATRICES

21

17

SUPPLEMENTARY PROBLEMS
17.

Show

that the product of

two upper (lower) triangular matrices

BA

18. Derive a rule for forming the product

19.

See Problem

Show

that the scalar matrix with diagonal element

where the order


20.

If .4 is

show

that

23. (a)

If

1.

-1

Show

that

that

a).

Wand that kA

klA =Aia.g(k,k

are idempotent.

that

(a)

o"

Show

that

29.

Show

that each of

2/1

0.

-4^

but

0.

9/ /

0.

-1

/.

-1 -1

is periodic, of period 2.

is nilpotent.

and

-12

-1 -6
2

commute.

_-l -1 -4_
-1/3'

2/3

and

-3/5

2/5

1/5

7/15 -1/5

1/15

commute.

anti-commute and (A+

Bf

The only matrices which commute with every n-square matrix

all matrices

(a) diag(a,fe, c)

A^ +

B'^

anti-commutes with the others.

31. (a) Find all matrices which

Ans.

= 0.

0-3

28.

Find

BA

"112'

30. Prove:

5! =

- 94

-1 -1

-1 ~1 -1

(b)

- 2A

-4

'12
that

4.A

-3 -4

Show

.4

AB

idempotent and that

-3 -4

-13

that

is

-2 -6

-3

Show

I-A

(b)

diag(aii,a22

and q are positive integers.

_3 _5

[-1

Show

that

show

that

27.

-4J

show

-1 -1 -1"

26.

-13
B

and

-3

show

is idempotent,

(b) If

25.

Using A and B, show that the converse of Problem 4 does not hold.

(b)

24.

can be written as

A:

A^ where

a'^ a'^ = a'^

-14

If

S and ^

matrix

row order of A

2-3-5
A

that

upper (lower) triangular.

1.

of / is the

re-square,

Show

21. (a)

22.

mxn

of an

Hint.

is

commute with diag(l, 2, 3).


which commute with diag(aii,a22
where a,6,

are arbitrary.

a^^).

are the n-square scalar matrices.

k) A,

32.

[CHAP. 2

SOME TYPES OF MATRICES

18

Show

(a)

that

"3-2-1
-4

is the inverse of

-2 -4 -5

10
2 10
4 2 10

(b)

"

34.

1.

-2100

is the inverse of

_ 8

-1 -1

to find the inverse of

[3:][:^E3

1.

^"'-

4]-

[3

that the inverse of a diagonal matrix A, all of

Show

10

0-2

-2311
33. Set

-1

-1/2]

[3/2

whose diagonal elements

diagonal matrix whose diagonal elements are the inverses of those of

A and

are different from zero, is a

in the

same

order.

Thus, the

inverse of / is /

-1

35.

Show

that

4-3
3-3

3"|

-1 0-1
-4 -44-3J

and

4
4

are involutory.

10
36. Let

10

I2

38. Prove:

(ABCy

39. Prove:

(a) (A-^)'^ =

/s

(c) (^^)' = (/l')^ for p

(b) (kA)" = kA',

C'^B-^A-'^.

A,

(a)(A)=A,

41. Prove:

a positive integer.

ABC=iAB)C.

Hint. Write

(c) (A'^Y^ =

(b) (kA)-^ = -j-A'^,

(a)

2- 3

-1 +

(A'^y forp a positive

integer.

iB is Hermitian,

that

{AB) = A B.

is

Hermitian,

is

skew-Hermitian,

-1

(<f )

+
2i

(c)

show

(d)

-i

1
i

-2-3J

is n-square,

+ 3

(c)(kA)=kA,

is Hermitian

A + B

(b)

B)

(b) (A +

42. Show:

that every real symmetric matrix is Hermitian.

Show

If

that

0-1

(a)(A')'=A,

43.

Show

'2

/I21

37. Prove:

40.

/j.

by partitioning.

-1

and B is skew-Hermitian.

(a)

AA' and A' A

are symmetric,

(6) A-irA',

Hermitian and A is any conformable matrix then (A)'

44. Prove:

If

45. Prove:

Every Hermitian matrix A can be written as B + iC where B

is

HA

AA', and A'A are Hermitian.

is Hermitian.

is real

and symmetric and C is real and

skew-symmetric.
46. Prove:

and C

Show

Every skew-Hermitian matrix A can be written as A = B + iC where B is real and skew-symmetric


(6) A' A is real if and only if B and C anti-commute.

is real and symmetric.

47. Prove:
48.

(a)

If

A and B commute so also do ^"^ and B'

that for

m and

n positive integers, ^4

A' and

B\

and S"^ commute

if

and A' and B"

A and 5 commute.

CHAP.

SOME TYPES OF MATRICES

2]

19

A
A
49.

Show

raA

(6)

If

51. Prove: If

is

symmetric

is

nX

A"

skew-symmetric then AA'= A'A

or

2n('-l)A
n-i

,ra

(a)

50. Prove:

nA

symmetric so also is

+6/1^

a-4

+.-.+/

and

where

are symmetric.

a, 6

are scalars and p is a positive

integer.

52. Prove:

Every square matrix A can

53. Prove:

If

54.

is real

if

is

Hermitian and C is skew-He rmitian.

complex and skew-Hermitian then iA are Hermitian.

is

that the theorem of Problem 52 can be stated:


Every square matrix A can be written as A =B+iC where B and C are Hermitian.

If^

are such that AB = A and


^ = B = / if ^ has an inverse.

A and B

If

idempotent,

(c)

show

is involutory.

Y=

(A

(a)

that k(.I+A) and

(A')

(A)^ = A-\

(b)

BA

kO-A)

n-square matrix A has an inverse A

57. If the

58.

and skew-symmetric or

B +C where B

/I

Show

55. Prove:

56.

written as

bfe

matrices which commute with (a) diag(i.

all

I.

(a) B'A'= A'

and

j(I+A)

A'B"= B\

^(I-A) =

(b) A"

and B' sue

0.

show:

Find

Ans. (a) dia.g(A. b.c).

then

(r)-^=(^-V

(c)

Prom the transpose

AA~^

are idempotent and

Hint, (a)

of

obtain

as the inverse of

(A'''-)

(6)

i, 2, 3),

diag(l.

/!'

1, 2, 2).

where A and B are 2-square matrices with arbitrary elements and

(b) dia.g(A.B)

b. c

are scalars.

A^.A^

59. If

with

A^

are scalar matrices of respective orders

diag(^i, ^2

Ans. dia.g(B^.B^

AB

60. If

= 0,

mj^.m^

m^.

find all matrices

which commute

'^s)

B^) where Si, S2

are of respective orders m-^.m^,

-85

m^ with arbitrary elements.

where A and B are non-zero n-square matrices, then A and B are called divisors of zero.
A and B of Problem 21 are divisors of zero.

Show

that the matrices

61. If
...,

s),

diae(Ai.A2

show

A^)

(a)

A+B

(b)

AB

diag(^iBi. /I2S2

(c)

trace

AB

and

di&giB^.B^

B^)

where ^^ and B^ are of the same order,

diag(^i+Si,^2 + S2
=

62. Prove: If

^ and B

63. Prove: If

-^s

trace /liB^ + trace /I2S2 +

+ trace A^B^.

...

are n-square skew-symmetric matrices then

is n-square

and B = rA+sI, where

and

AB

is

symmetric

are scalars, then

if

and only

if

A and S commute.

A and B commute.

65.

Show

66. If

1, 2,

+ Bs)

Let A and B he n-square matrices and let ri, rg, si, S2 be scalars such that
ri4+siB, C2 = r^A+SQB commute if and only if A and B commute.

/I

A^B^)

64.

(6)

(J

that

risj

7^

rssi.

Prove that

Ci =

that the n-square matrix A will not have an inverse when (a) A has a row (column) of zero elements or
has two identical rows (columns) or (c) ^ has arow(column)whichis the sum of two other rows(columns).

A and B

are n-square matrices and

A has an inverse, show that

(A+B)A~'^(A-B)

(A-B)A''^(A+B)

chapter 3
Determinant of a Square Matrix

PERMUTATIONS. Consider

together
the 3! = 6 permutations of the integers 1,2,3 taken

123

(3.1)

and eight of the

4!

132

312

231

213

321

= 24 permutations of the integers 1,2,3,4 taken together

4123
4213

3124
3214

2134
2314

1234
1324

(3.2)

we say that there is an


a given permutation a larger integer precedes a smaller one,
the permutation is
(odd),
even
is
inversion. If in a given permutation the number of inversions
there is no inversince
even
called even (odd). For example, in (3.1) the permutation 123 is
permutation
the
312 is even since in
sion, the permutation 132 is odd since in it 3 precedes 2,
If in

it

2,

and 3 precedes 2. In (3.2) the permutation 4213


4 precedes 1, 4 precedes 3, and 2 precedes 1.

precedes

is

even since in

it

4 precedes

Consider the re-square matrix

THE DETERMINANT OF A SQUARE MATRIX.

'11

"12 "IS

Uo 1 Oo p ^03

(3.3)

-'ni

"ne "n3

and a product
(3.4)

^j, "'^L
~^3
1 ~-'2 3Jo

'^"
"^n

comes from any row and one


of n of its elements, selected so that one and only one element
of convenience, the factors
matter
as
a
column.
In
from
any
(3.4),
comes
one
element
and only
re;
the
have been arranged so that the sequence of first subscripts is the natural order 1,2
of
the
intepermutations
re!
of
the
one
some
then
is
subscripts
second
sequence j^, j^
/ of
(Facility will be gained if the reader will parallel the work of this section bere.
gers 1,2

ginning with a product arranged so that the sequence of second subscripts is in natural order.)
For a given permutation /i,/2, ...,^ of the second subscripts, define ^j^j^....j^ = +1 or -1

according as the permutation is even or odd and form the signed product
(3.5)

-W-2-

By

k'^k ^^h

'^Jn

signed prodthe determinant of A, denoted by U|, is meant the sum of all the different
Ul, which can be formed from the elements of A; thus.

ucts of the form (3.5), called terms of


(3.6)

le Jlj2-

where the summation extends over

p=n\

The determinant

'^h

permutations hk---Jn of the integers 1,2,,

of a square matrix of order

20

re

is called

a determinant of order

re.

CHAP.

DETERMINANT OP A SQUARE MATRIX

3]

DETERMINANTS OF ORDER TWO AND THREE. Prom


-"ll

"12

^21

"22

(3.7)

12

^11^22

(3.6)

we have

21 '^12^^21

"'"

21

for n

11^22

and n = 3,

CE-j^2^21

and
"13

"12

'll

(3.8)

21
"31

"?

P*^

"32

"S3

^123

ail0223S
+

231

flu 022 %3

^132 Oll02s'^32

%2 02331

312 Ol3'%lC'32

^11023032

^213 Oi221<%3

^321

%3 ^^2 %!

012^1033

%l(22 033 - 02S32) " "12(021 "ss " ^S^Sl) + ^IS (a2lOS2 - 022^31)
022

^3

"^32

^3

O21

053

Example

"31

21

<^2

'^Sl

^^32

Oi

<^33

1.

21

-11

(a)

1-4

2-0

(fc)

2-3

(-1)3

-2

+ 3

3
2

5
10

(O

+
2

11

2(0-0
2
(d)

11

01

11

-1-1) - 3(1-0 -1-2) + 5(1-1-0-2)

2(-l)

- 3(-2) +

5(1)

-3 -4

0-2

2{0(-6)-(-2)(-5)! - (-3){l(-6)- (-2)0! + (-4) {l(-5) -

O-o!

-5 -6
-20

18

20

-18

See Problem

PROPERTIES OF DETERMINANTS.
nant

Ul

is

Throughout this section, A

is the

1.

square matrix whose determi-

given by (3.6).

Suppose that every element of the sth row (every element of the/th column) is zero.
Since
every term of (3.6) contains one element from this row (column), every
term in the sum is zero

and we have
I.

If

every element of a row (column) of a square matrix A

is zero, then U| =0.


Consider the transpose A' of A. It can be seen readily that every term of
(3.6) can be obtained from A' by choosing properly the factors in order from the
first, second, ... columns. Thus,
II. If 4 is a square matrix then
U'l = \A\; that is, for every theorem concerning the rows
of a determinant there is a corresponding theorem concerning
the columns and vice versa.

Denote by B the matrix obtained by multiplying each of the elements of


the ith row of A by
Since each term in the expansion of |5| contains one and only one
element from its
row, that is, one and only one element having A; as a factor,

a scalar k.
fth

\B\

1
P

Thus,

\: ,
,
a. , n^n
\
*%'2---i'iii2J2----anj^!
.

Jc\A\

DETERMINANT OF A SQUARE MATRIX

22

[CHAP.

every element of a row (column) of a determinant U| is multiplied by a scalar k, the


determinant is multiplied by k; if every element of a row (column) of a determinant \A\ has k as
If

III.

may be factored from

a factor then k
a7']^]^

ltd AQ

O\
OrQ

/l'

t*

CI

\A\

For example,

QO

f (X'

Let S denote the matrix obtained from A by interchanging its ith and (i+l)st rows. Each
\A\ is a product of |s|, and vice versa; hence, except possibly for signs,

product in (3.6) of

expansion of

(3.6) is the

term of

\b\,

\b\.

sign changed is a term of

its

IV.

If

In

counting the inversions in subscripts of any term of (3.6) as a

before i+1 in the row subscripts is an inversion; thus, each product of (3.6) with
|s|

and

= -

\e\

\A\.

Hence,

obtained from A by interchanging any two adjacent rows (columns), then \b\ =

is

- \a\.

As a consequence
V.
VI.
|s|

If

is

If

of

Theorem

we have

is

obtained from A by carrying

its ith

its

rows (columns), then

= -\A\.

\b\

row (column) over p rows (columns), then

= (-i)^UI.
VII.

If

two rows (columns) of A are identical, then

Suppose that each element


(j

IV,

obtained from A by interchanging any two of

= 1,2,

...,n).

row of A

of the first

is

\A\

expressed as a binomial

%,
^li

b-^:

c-^j,

Then

in

^Jd2

^^^ii ^

"^i^ "2 J23 J5

p^kk^ Jn ^^k'^k'^k-h,.

ii2

^13

^m

21

O22

"23

02n

""in
Os,

Os-,-

"njn^

"Jn

'In

Oni

n2

"na

'^nn

-^n2

'-"ns

In general,
VIII.

If

every element of the ith row (column) of A is the sum of p terms, then

\A\

can

be expressed as the sum of p determinants. The elements in the ith rows (columns) of these
p determinants are respectively the first, second, ..., pth terms of the sums and all other rows

(columns) are those of A.

The most useful theorem


IX.

If

is obtained from

is

A by adding

to the

elements of

its ith

tiple of the corresponding elements of another row (column), then

Cfrii

T"/CCEog

^22

Clnr

Clf,',

{-

Karin.

Clf=n

Clr^r,

Cfg ^

row (column), a scalar mul= \a\. For example.

\b\

H'Qri^

^Q'2

"^

i^Clr

Zgg +

ka^^

See Problems 2-7.

FIRST MINORS AND COFACTORS. Let A


by

(3.6).

When from A

be the re-square matrix (3.3) whose determinant \A\ is given

the elements of its ith row and /th column are removed, the determinant

of the remaining (re- l)-square matrix is called a first minor of

or of \a\

and denoted by

\M^j\.

CHAP.

DETERMINANT OF A SQUARE MATRIX

3]

More frequently,
cofactor of

Example

2.

a^-

If

The signed

called the minor of Oy.


denoted by a;

it is

and

is

23

minor,

is

(-if'*'^ \Mij\

called the

"V

11

12

''13

"21

'^SS

"23

"'31

^^32

''SS

"23

22

M 11

"21 "23

ki2

21

22

"31

"32

kic

"32 "33

"31

"33

and
1+1

1 + 21

kll

(-1)

1^11

(-1)

ai2

1^12!

14-31
I

ki3

(-1)

Then

\m.

(3.8) is

Ml

In

Problem

X.

The value

9,

aiilA/iil

"11*11

O12IM12I

ai20ii2

aislMigl

"130^13

we prove

of the determinant U|, where A is the matrix of (3.3), is the sum of the products obtained by multiplying each element of a row (column) of
U| by its cofactor, i.e..

n
(3.9)

Ul

aiiflii

(3.10)

Ml

"^if^if +

a^g a^g

^in * in

a^jd^j

^ aik'kk
n

Using Theorem
XI.

ra-square

Example

VII,

021*2/ +

(ij, = 1,2

fe?i"fei*j

n)

we can prove

The sum of the products formed by multiplying the elements of a row (column)
matrix A by the corresponding cofactors of another row (column) of A is
zero.

3.

If

is the matrix of

Example

and

2,

of

an

we have

"310^31

"320^32

"330^33

"12*12

"22*22

"32*32

"31*21

"32*22

"SS*23

"12*13

"22*23

"32*33

Ul
'4
I

while

and

See Problems 10-11.

MINORS AND ALGEBRAIC COMPLEMENTS. Consider the matrix (3.3). Let


i^
arranged in
h, i^
order of magnitude, be m, (1 < m < ra), of the row indices
1,2
n and let j^, j^
/^ arranged in order of magnitude, be m of the column indices. Let the
remaining row and cofumn indi,

ces, arranged in order of magnitude, be respectively

i^,^,,

i^

and /+;+2

a separation of the row and column indices determines uniquely


two matrices
a.-

.Jl.j2.-

(3.11)

im

'l'J'2

'^i.

i2.ii

''2.

i2 :h

"^m Ji

%J2

72

Jm

^n

^n

Such

DETERMINANT OF A SQUARE MATRIX

24

[CHAP.

and
''m+i'J'm+i

Jn

+ i'Jm+2'

.7ra

''m

+ S'Jm +

+ i'Jffl+s

n.7n

^m+2>7m+2

^ra+2>Jn

^n'j7ii+2

^ri'Jn

'-m

(3.12)

^^Jm-i-i

called sub-matrices of ^.

A and the

of each of these sub-matrices is called a minor of

The determinant

Jm

J1.J2.

J'm-'-i'Jn^-Q

Jn

^m+l'

''r

pair of minors

and

are called

complementary minors

Examples. For the

I
i

1,3
'^2,5

each being the complement of the other.

of A,

5 -square matrix

[iij.

'^21

'^23

f^Sl

63

and

'^l, 3.4-

'

I
I

"I'Z

%4

'^15

"32

^^34

'^SS

"42

"44

^^45

2,4,5

'-m-^2

are a pair of complementary minors.

Let

U +

(3.13)

In

+ in +

and
(3.14)

i-n

Ji, is.

The signed minor

fm+2 +

+1 +

(-1)

A,-

+ *n +

7(4-1.

and

/I.

(-l)'^

7n
is called

''m,+

,2 4 5
A.{3\i

of

Jn
^n

'to+2

the algebraic complement of


J1.72

Jm

H>h

^n

13

2+-5 + 14-31

complement

l>^m+2> ''-

Exainple4. For the minors of Examples.


Of

+ /m+2 + " + In

is called the algebraic

Jm-'-i'Jn-i-2

J+i'Jm,+2

J,

^ra

and

(-1)

l(-3 + 4-^2l-446

(-1)

.2,4,5
A-^ 34

Ap' g

=
I

-I

,1.3
/i^
I

2 4 5
^ijg',4

is

the algebraic complement

is

the algebraic complement of

1.3

^^is

Note that the sign given to the two complementary minors

is

the same.

Is this

always true?
Ji

.Ji

When m =

l,

(3.11)

becomes
J2.J3.

[%ii]

and

"HJi

an element of A.

The

Jn

complementary minor

is

Ml

in the notation of the section

above, and the

algebraic complement is the cofactor

OL-Lj.

minor of A, whose diagonal elements are also diagonal elements of A,

is

called a principal

The complement of a principal minor of A is also a principal minor


braic complement of a principal minor is its complement.

minor of A.

of A; the alge-

CHAP.

DETERMINANT OP A SQUARE MATRIX

3]

Examples. For the 5-square matrix A

25

H'
"?2
"11

"15

31

S3

and

^1.3

are a pair of complementary principal minors of

A,

,t

"24

(^25

0^2

044

045

52

"54

055

A What is the algebraic complement of each ?


The terms minor, complementary minor, algebraic complement, and principal minor as defined above for a square matrix A will also be used without change in connection
with U
.

See Problems 12-13.

SOLVED PROBLEMS
1- (")

!l
4|

1-1

2-4

- 3(-l)

4
(b)

15

21

5l

(1)

(l)(4-7

71

5-6)

-2-4

+ 2(3-6

4-5)

-6

(c)

11

1(4-21

-18

11

id)

2.

5-1)

first

column the corresponding elements

-4

-4

-4

-4

-4

-4

-4

-4

-4

of the other

columns,

I.

Adding the second column


using Theorem

4.

1(3-3

Adding to the elements of the

by Theorem

3.

to the third,

Vn

removing the common factor from this third column and

a+b+ c

+a

a+h

a+b+c
a+b+c

(a + b + c)

Adding to the third row the first and second rows, then removing the common factor
2; subtracting
the second row from the third; subtracting the third row from the first;
subtracting 'the first row
from the second; finally carrying the third row over the other rows

DETERMINANT OF A SQUARE MATRIX

26

Oj^+fej

a^+b^

ag+63

62+ ^2

^3+^3

i^+Ci

62 + ^2

&3+^3

Cj+a^

C2 + a2

C3+a3

a5^+5]^+Cj^

a2+fe2+"^2

"s+^s+'^s

6^ +

Without expanding, show that

02

Ul

Subtract the second row from the

l-2

'^3

Oi

02

^2

Og

Og

-(Oj^

Oi + a2

02

ar,

(a^

02

flg

Cl

C2

Cg

bi

^2

63

a-,

Oo 03

C]^

C2

Cg

'^s) ('^s

%)

a2

02)

O2

ag

by Theorem

III

Og

Og

oj^-oj is a factor of U|.

and

a^^

then

first;

^2

fl-j^

^3

- 02) (02 ~

b^+Cr

62

^3+^3

62+^2

C-L

b^+c^

b-i

63

62

a^

5.

bi + c^

6^+C-L

bi

[CHAP.

Og

Similarly,

a^-a^ and ag-a^

are factors.

Now M|

is of order three in the

letters; hence.
\a\

(i)

A:

k(a^^-a^)(a^-as)(as-a^)

The product of the diagonal elements. 0^02. is a term of \a\ and, from (i), the term is -ka^a^. Thus,
-l and \a\ = -{a^-a^){a^-a2){as-a^). Note that U vanishes if and only if two of the a^, og. os are
|

equal.

6.

Ul

7.

is

Since A

is

Prove:

If

skew-symmetric and of odd order 2pskew-symmetric, -4'= -A; then


Ut = -U! and Ul = 0.

\-A

- Ul- But, by Theorem

(-if^-^-UI

Ul; hence,

is

Hermitian, then \A\ is a real number.

Since A

is

Hermitian. A = A'

Prove:

If

then

Now Ul

8.

then Ml =

1,

Ul

and

(-ir'\l
12
,2+1

(-ly

3+

(-ly

II.

%;

Ul

|eii;2---Jn^iii"2i?--^"in

12

2,

U|

^iij2...j"yi%

is a real

ai2 =

a22 = (-1)

age

But

if

'^

+ *'

"

"

^'

number.

(-1) 1+2

U| by Theorem

U'| =

?1

\a\ =

Ul

requires 6 = 0; hence.

For the matrix A

a,,

|i

2+2I

(-1) 3+2I

2I

-2,

-1,

Mi3 =

1+3
(-ly

a,23

(-1)

a 33

(-1)

2+3

11

3+3I

2I

31

II,

CHAP.

DETERMINANT OF A SQUARE MATRIX

3]

Note that the signs given

to the minors of the

elements

in

forming the cofactors follow the pattern

where each sign occupies the same position


cupies in ^,

9.

27

in the display as the element,


Write the display of signs for a 5-square matrix.

whose cofactor

is required, oc-

Prove: The value of the determinant U| of an re-square matrix A is the sum of the
products obtained
by multiplying each element of a row (column) of A by its cofactor.

We

Now

e.

shall prove this for a row.

kh---Jn~

^jbjb- -in

The terms

^i"''^ i"

of (3.6) having a^^ as a factor are

^ P"'^tin

lJi..fe.

-in.

the

Then

is in natural order.

(a)

may

be written as
(6)

where the summation extends over the cr= (n-i)! permutations of the integers
2,3
022

2S

n,

and hence, as

<2n

"an

(c)

"ii

"11

'twill

"ns

^n2

Consider the matrix B obtained from A by moving its sth column over the first s-l columns.
By Theorem
\B\ = (-1)
U|. Moreover, the element standing in the first row and first column of B is
a^s and the
minor of a^^ in B is precisely the minor \M^\ of
a^s in A. By the argument leading to (c), the terms of
ais mis\ are all the terms of \b\ having a^s as a factor and, thus, all the
terms of (-1)^"^
having a.^
VI.

a factor.

Then

U|

the terms of

ais!(-if

are all the terms of \a\ having a^s as a factor

M^/isli

as

Thus

(3.15)

+
"11*11
since

(-1)

,s+i
= (-1)

m.r,JWlTr\T
B be the

ments. Let

umn over

the first

We have

+ ai5!(-lf||/,3|! +

+ ai2i2

(3.9) with

shall call (3.15) the expansion of

U| along

its first

"' ''^ '^ ^^^^^ '^' ^^-^^ '' ' = '^ '^ '''^i"^'J by f^P^^ting the above arguJ'*' ^iT^
matrix
obtained from A by moving its rth row over the first r-1
rows and then its .th col^

s-l columns. Then

T~l
(-1)

The element standing in the


^ Thus,

the minor of a.rs in

are all the terms of

first

row and the

s-l

(-!)

first

for

having a^^ as a factor.

r.

\a

column of 5

(-1)

is

a and

the terms of

u
the minor of a^^ in
^^

Then

,l,-rkU-l)
and we have (3.9)

+ ai !(-!)'*" M,

+ ain*in

We

...

r+fel

M.rk\

2
k=i

a^j^a.
rk^rk

is yreciseiy
i
precisely

10.

[CHAP.

DETERMINANT OF A SQUARE MATRIX

28

When

A =

is the cofactor of aij in the ra-square matrix

oLij

'll

(i)

fC^GC^-j

"T

fCi^OL^j

"T

[a^j]

show

that

%,j4-i

^'i.n

(h,j-i

02

*2n

"W,j-i

<%,

%,f-i

"12

"'1

i +

itj^dj^j

'^i

^2

'^n

In making these
with k^
0^7 with Atj,
This relation follows from (3.10) by replacing a^j with k^, 027
^2J
appearing is affected since none contains an element
(t^j
replacements none of the cofactors OLij.QL^j.
,

from the

th

column of A

By Theorem

when A^= a^^,

VII, the determinant in {i) is

and VII, the determinant in

(i) is

when

+ fea^g,

Ay
\7

Write the eauality similar to (0 obtained from (3.9)

by

= 1,2

(r

and

= 1,2

and

when the elements

By Theorems Vin,

j).

/).

of the ith row of

are replaced

/ci,/c2.

11. Evaluate:

(a)

04
-5

3 4

-2

-4

(c)

28
(e)

25

38

42 38

65

56 47

83

3-4
5-6 3
2-3
4

4 8

-2 1 5

(b)

((f)

-3 2 4

(a)

(r

Expanding along the second column (see Theorem X)


1

-5

Cl-^^fX^Q

022^22 ^ ^32 32

"f"

ai2 +

a22 +

(-5)o;g2

1
1 2

34.2I
-5(-l)-'

-10

5(4-6)

3 4l

(b) Subtracting

twice the second column from the third (see Theorem IX)
4 8

-2 15

-3 2 4

8-2-4
5-2-1
4-2-2

3 4

-2 5

-4

-3

second row from the

3(-l)''

1 3

-2 +

2(1)

-4

5-11

-2 -4

5-3(3)
=

+ 2(2)

-2 -3

-4+

27
8

-11 -41
-2 - 11

to the third

-2 -4
9

2(3)

-32
in (c)

-4 + 4(1)
2-2(1)
1
5-2(-ll) -11
3 + 4(-ll)
4-2(-2)
-2 -3 + 4(-2)

column from the second and then proceeding as

3-4
5-6 3
2-3
4

-3

and adding twice the second row

first

3-3(1) 4-3(2)

(-4 + 36)

(d) Subtracting the first

-42

-3(14)

(c) Subtracting three times the

-2

27

-41
-31
-11

CHAP.

(e)

DETERMINANT OF A SQUARE MATRIX

3]

Factoring 14 from the

28

first

column, then using TheoremIX to reduce the elements in the remaining columns

38

25

42 38 65

14

47 83

56

-2

25

38

38

65

47 83

Show

-12
6

P + q

Now

13.

25-12(2) 38-20(2)
38-12(3) 65-20(3)

47-12(4) 83-20(4)

-1

-14

-1

-14(-l-54)

770

that p and q, given by (3.13) and (3.14), are either both even or both
odd.

Since each row (column) index

(-1)

14

14

4-13
12.

29

is

found in either p or 9 but never in both,

+ 2+-+7J) +

(1

(l

2+-"+n)

2-kn{n + l)

n(n +1)

even (either n or n + 1 is even); hence, p and q are either both even


and only one need be computed.

p+q

= (-1)^

is

For the matrix

[..]

12

11

12

13

14

10
15

16

17
22

18

19

23

24

.21

'4^'4^'5|

complement

of

Ao's

Thus,

is

20
25

13

,^2+3+2+4| .1,3,51
,
(-1)
Ml,4,5l

and the algebraic complement of

the algebraic

or both odd.

16

18

20

21

23

25

(see Problem 12)

is

I12 141

SUPPLEMENTARY PROBLEMS
14.

Show

that the permutation 12534 of the integers

1, 2, 3. 4,

is even,

24135

is odd,

41532

is even,

odd, and 52314 is even.

15.

List the complete set of permutations of

16.

Let the elements of the diagonal of a 5-square matrix A be a.b.cd.e.


diagonal, upper triangular, or lower triangular then \a\ = abcde.

17.

Given

-4

[j J]
each product is 4.

18.

and B = [^

(a)

-1

2 4

-1

^^^^

AB^BA^ A'b

27

4 AB' ^

a'b'

Show, using

^ B'a'

(b)

2-2

12

2 3

2 3
=

(c)

-2
-3 -4

(3.6), that

when ^

is

but that the determinant of

1,

is

2,3,4, taken together; show that half are even and half are odd.

6J

Evaluate, as in Problem
2

^^^^

1,

53142

DETERMINANT OF A SQUARE MATRIX

30

19.

(a)

Evaluate

\A

1 2

10

2 3

[CHAP.

4 5 11
(b)

Denote by B
Evaluate \B

to verify

(c)

Denote by

Show

that

2 7

by multiplying the elements of

Obtain from \A

2 3 4

4 5 8

4 5 3

.4

by interchanging

|o| =

the determinant

2 3

and third columns.

its first

by subtracting three times the elements of the

Evaluate

to verify

Theorem

/I
I

\A
I

multiply the first column by three and from

has been tripled.

an n-square matrix and 4

20.

If -4 is

21.

Prove: (a)

If

(b) If

22.

Compare with

then

= k.

\a\ = k =

(a)

Count the number of interchanges


and thus prove the theorem.

(b)

Same,

for

Theorem

it

subtract the third column.

not confuse (e) and

use (3.6) to show that

is a scalar,

is skew-Hermitian, then

Do

(e).

Evaluate

Evaluate to show that

(g).

U^

/t^M

|.

\a'\.
|

-4

is either real or is a pure imaginary

of adjacent

number.

rows (columns) necessary to obtain 6 from A

in

Theorem V

VI.

23.

Prove Theorem VII. Hint: Interchange the identical rows and use Theorem V.

24.

Prove:

If

first

IX.

subtract twice the first row from the second and four times the first row from the third.
the resulting determinant.

(g) In

Evaluate

5-1

column from the corresponding elements of the third column.


(/) In

5.

thus verifying Theorem VIII.

second column by

its

2 3

2 3 5

(e)

-4

III.

the determinant obtained from

(d)

Theorem

Theorem V.

to verify

the determinant obtained from

any two rows (columns) of a square matrix A are proportional, then

25.

Use Theorems

26.

Evaluate the determinants of Problem 18 as

VIII, III,

,4

o.

and VII to prove Theorem IX.

in

Problem

11.

a b
27.

Use

(3.6) to evaluate

\A\

d
e

".P/.

then check that \a\ =

Thus,

if

= diag(A-^. A^).

where

g h
A^,

A^ are 2-square matrices,

U4i|

2/3

-2/3 -2/3
1/3 -2/3

2/3

-2/3

-1/3
28.

Show

that the cofactor of each element of

1/3

-4
29.

Show

that the cofactor of an element of any row of

is that element.

-3 -3

is the

corresponding element of the same

numbered column.
30.

If

is

symmetric then

(b) If

is

n-square and skew-symmetric then

Prove: (a)

<Xij=

0^j^

when

aij = (-1)

"P when

Ot--^

CHAP.

31.

DETERMINANT OF A SQUARE MATRIX

3]

For the matrix A of Problem


(a)

show

(b)

form the matrix

that

32.

12

(^^22

32

^23

Otgg

the result in (6) is

show

that

Without evaluating show that

known as soon as
a

a^

6^

ca

52

c^

c2

ab

be

ab

ca

ab

ca

be

ca

be

ab

a^

bed

6^

aed

e^

d^ d

a-^

a^ a

abd

c^

c'^

abc

d d^ d

(a

Show

that the ra-square determinant

a\

n-2

ra-i

n-2

a^

1 ... 1

0...1

...0

remove the common factor from each of

1...1

("-!)

re-i

known.

(a) is

respectively by a,b.c

34.

AC = [.

and show that

be

Multiply the columns of

the rows to

33.

why

_'^13

(c) explain

31

b)(a

c)(a

...

...

...

d)(b

...

...

d)(e

- d).

n-1
(-1)

c)(i

(n~l).

1
1

ar, 1

35.

Prove:

n-i

2)(i

as).

(ai-a^H(a2- 03X02-04)...

na^+b-^

na^+b^

na^+bg

nb-j^+e^

nb^ + c^

nb^+Cg

nc-i^+a.^

nCQ+a^

ncQ + a^

X
37. Without expanding,

show

that the equation

38.

a+ b

a
b

+6

(n

x+c

{na + b).

+ l)(rf--n +

1)

O]^

02

O3

61

62

is

Ci

Cg

Cg

xb
x-c

,n-i

Prove

x-i-a

x+b

-a^j

Without expanding, show that

+6

(02

ra-2

36.

S(i

has

as a root.

7i-i

- "ni

chapter 4
Evaluation of Determinants

determinants of orders two and three are found in Chapters. In


Problem 11 of that chapter, two uses of Theorem IX were illustrated: (a) to obtain an element 1
or 1 if the given determinant contains no such element, (b) to replace an element of a given

PROCEDURES FOR EVALUATING

determinant with

0.

For determinants of higher orders, the general procedure is to replace, by repeated use of
Theorem IX, Chapters, the given determinant U! by another |b| -= \bij\ having the property
that all elements, except one, in some row (column) are zero. If &>, is this non-zero element
'Pq

and ^p^

is its cofactor.

Then

bpq

is treated in similar

the minor of bp

i-ir

dp^

minor of

^Pq

b.

'pq

fashion and the process is continued until a determi-

nant of order two or three is obtained.

Example

1.

3-2 4
3-2 12

2 + 2(3)

-2

3 4

3-3(3)

2-3(-2)

-2

-2405

+ 2(-2)

8-1
=

(-1)"

+ 2(1)

-2

4 + 2(2)
2

+ 8(-l) -1

-6

-2

-6 +

8(8)

-2

-2 +

8(4)

'^

-(-l)- = (-l)P'

30

0-2

-2405

3-2
-6

4-3(2)

3-3(1)

8-10

0-10

+ 8(-l)

-2 + 8(8)

58

62

+ 8(4)

30

37

-286

37

See Problems 1-3

For determinants having elements of the type in Example 2 below, the following variation
divide the first row by one of its non-zero elements and proceed to obtain zero

may be used:

elements in a row
Example
0.921

or

column.

2.

0.185

0.476

0.614

0.517

0.667

0.201

0.201

0.517

0.667

0.123 -0.384

0.782

0.157

0.527

0.138

0.782

0.157

0.527

0.138

0.872

0.484

0.637

0.799

0.872

0.484

0.637

0.799

0.309

0.196

0.217

0.555

0.841

0.448

0.312

0.555

0.841

0.448

0.492

0.680

0.240

0.921

0.921

0.312

-0.320

0.123 -0.384
0.921

0.309

0.196

0.217

0.921(-0.384) 0.309

0.196

0.217

0.492

0.680

0.240

0.492

0.680

0.240

0.921(-0.384)

0.309

0.265

0.217

0.492

0.757

0.240

0.921(-0.384)(0.104)

32

-0.037

0.921(-0.384)

0.309

0.265

0.492

0.757

CHAP.

EVALUATION OF DETERMINANTS

4]

THE LAPLACE EXPANSION.

The expansion

of a determinant

33

of order n along a row (column) Is

a special case of the Laplace expansion.

bered

I'l,

i^

J2

Instead of selecting one row of \a\, let


when arranged in order of magnitude, be selected. Prom these

n(n-

m + l)
m

l)...(ra

1-2

....

minors

can be formed by making all possible selections of m columns from the n columns.
minors and their algebraic complements, we have the Laplace expansion
>

|(-1)^

(4.1)

i.

Jrr.

Jn+i' jm+2

i.

Example

Using these

Jn

^m4-i'% + 2

where s = i^ + i^+... + i^ +A + /2 +
column indices taken m at a time.

m rows numm rows

^n

and the summation extends over the/? selections of the

+h

3.

Evaluate

3-2 4
2
3-2 12

using minors of the

two rows.

first

3 4

-2405
Prom

(4.1),

1+2+1+21
(-1)

1 + 2+1 + SI

U1.2

_l + 2 + l + 4|

-2

1 + 2 + 2 + 31

+
,,1+2 + 2+4. .2,4
.13
,
(-1)
Mi'jl-Us'.

-2

(-1)

-2

-2

_2

(-13)(15)

.2,31

^1,:

1 + 2+3+4

(-ly

2,41

,1,31

"Mi',|-U
1,2!
3,4!

1,4

(-1)

(-1)"

U-5aI
^3,4

_2

-2

- (8)(-6) + (-8)(-12) + (-1)(23) -

.34
*l,2r 1^3, 4I

-2

(14)(6)

-2 4

+ (-8)(16)

-286
See Problems 4-6

DETERMINANT OF A PRODUCT.

If

A and B

Us

(4.2)

are n-square matrices, then

Ul-lsl

See Problem 7

EXPANSION ALONG THE FIRST ROW AND COLUMN.

If

^ = [aid
n

(4.3)

Where

^11 ^1
i=2

a^

is the

cofactor of

o^ and

a^^-is the

n-square, then

is

ii
IJ

IJ

j,-=2

algebraic complement of the minor

"ii^iil

of^.

ii Otj
I

DERIVATIVE OF A DETERMINANT. Let


tiable functions of a variable x.

the ra-square matrix

Then

A =

[a^,-]
^tj.

have as elements differen-

EVALUATION OF DETERMINANTS

34

The

[CHAP. 4

\A\, of \A\ with respect to x is the sum of n determinants obtained


dx
by replacing in all possible ways the elements of one row (column) of
by their derivatives
with respect to x.
I.

derivative,

Example

4.

x^\

x^
1

dx

2x-\

2x-\

-2

x2 x +

2x
=

x""

x^

4x

-2

x+

3^^

-2

2x-i

x-"

6x

12x^

See Problem 8

SOLVED PROBLEMS
2

-2

-2 4
-3 10

-2

4-2(3) -3-2(-2) 10-2(4)

2(2)

3-2

-2

-2

1.

(See Example 1)

-286
3

There are, of course, many other ways of obtaining an element +1 or -1; for example, subtract the
column from the second, the fourth column from the second, the first row from the second, etc.

-2

2+2

2-2(1)
-2-2(2)

+2

1-2(2)

-3

+ 3 -3-2(3)

18-9

-1 +

4-3

1-

1+

2 +

Multiply the second row by

: i

and the third row by

(-1+3j)|-^|

2J;

-4 + 7i
1+i

25 -

+j

5j

-6

1+2;

5-J

-4 + 7j -10 +

2j
I

14i

then

5-J

,4

1+2j

+i

2
5

and

4-3

-11 -4

2j

(l+J)(l+2j)Ml

-9-3(-3)

8-3(4)

-4

2-3i

2i

-5

-72

-11 -4

\A\

-6-2(-3)

4-2(4)

1-3(4)

-4

4-6
4-3

4-3

3-2(4)
=

-9

-5

Evaluate

-6

3.

10

-1

first

18

+J

82J

14j

2J

25 -

5j

-4 + 7J -10 + 2i

CHAP.

4.

EVALUATION OF DETERMINANTS

4]

Derive the Laplace expansion of

\A\

Now

into the first

row

(I'l- 1)

numbered

which the row and column indices are arranged

of 1^1 in

%\

in

interchanges of adjacent rows of |^


the row numbered ii can be brought
row. by i^- 2 interchanges of adjacent rows the tow numbered is can be brought into the second
-m interchanges of adjacent rows the row numbered % can be brought into the mth row. Thus.
i;

by

after

by

m<n

Jrn

A;
H-''^

order of magnitude.

of order n, using minors of order

\aij\

ji'h

Consider the m-square minor

35

+ (ig-

2)

i^

I'l,

+ (ijj -m)
11 + ^2+
occupy the position of the

+ 'm-2'"('"+l) interchanges of adjacent rows the rows


m rows. Similarly, after /i + j^ + + /^ - \m(m+l)

first

interchanges of adjacent columns, the columns numbered /i,/2


occupy the position of the first m col/jj
umns. As a result of the interchanges of adjacent rows and adjacent columns, the minor selected above occupies the upper left corner and its complement occupies the lower right comer of the determinant; moreover.
I

A has changed sign


[1

+ ^2+ +

in

= j- + i +
+
+ /i +
+ ii + + in changes. Thus

cr

Ji.J2'

/i

Jm

Ji~h-

'-m

'm-H'''m +2-

Jn

Jn-H' Jm+2'

-m)! terms of (-1) \a\

m!(ra

is

equivalent to

yields

w!(n-

or

' Jn

n+2'

"TOJ-l'

times which

+ /-"('" +1)

yields

(-ir

(a)

Jn

Jm+i'Jn-i-2

AH'''^

/2

m)! terms of \a\.

n(n~l)...(n-m + l)
different m- square
l'2....m
m\{n m)\
minors may be selected. Each of these minors when multiplied by its algebraic complement yields m!(/j-m)'
terms of U|. Since, by their formation, there are no duplicate terms of
U| among these products.

Let

I'l,

in be held fixed.

12

Prom these rows

S(-i/

I
,

5.

where

/i, 72

in of the column indices.

i^

+ i^+

12 3 4
12 1
11
3 4 12
(-1)^

Jn

different selections

using minors of the first two columns.

21

|1

ll

(-2)(1)

(-ir

21

ll

41

ll

(-If

11

13

41

ll

(5)(-l)

A,B, and C are re-square matrices, prove

C B
Prom the

7.

and the summation extends over the

+ in

(-3)(1)

6. li

Jm-1' Jvn

I'm

Evaluate

+ i^ +j^ + j^ +

Jn

rows of |P| only one non-zero n-square minor, U|, can be formed.
Hence, by the Laplace expansion, |p| = |.4|.|b|.

first n

plement

is

Prove

\AB\ =

|s|.

Suppose
Problem 6

= [a^j]

Its

algebraic com-

S
and

= [bij]

are n -square.

Let

= [c^j] =

AB

so that

^Hkhj-

i^rom

EVALUATION OF DETERMINANTS

36

"m

2n

"nn

ail

012

021

022

"ni

n2

-1
-1

[CHAP. 4

6ii

612

..

hi

621

^22

..

?,2

-1

To

the (n+l)st column of \P\ add fen times the first column, 621 times the second column

the nth column;

On

012

"m

Cii

"21

"22

2ra

C21

"ni

"n2

"nn

Cm

-1

Next, to the

(n

+ 2)nd column of |P| add

times the nth column.

fei2

times the

Let A =

h,

622

..

62

column, 622 times the second column,

Hence,

\p\

in

Cii

C12

2n

C21

C22

"nn

Cm

Cn2

Its

(-i)Vlf" ^"^"lc|

^13

^m

*23

b^n

\P\

zero n-square minor, 1-/| = (-l)'^ can be formed.

8.

first

612

We have

Continuing this process, we obtain finally

= (-lf'2"^^'|c|.

-1

Oil

6i times

we have

Prom

the last n rows of

only one non-

algebraic complement is (_i)i+24--"+ra+(n+ii+-"+2n|(-|


=

and

\c\

\c\

\ab\ = U1.|b|.

%2 %S

021 '%2 '^a

where

Oif

aij(x),

(i, j

= 1,2,3),

are differentiable functions of x.

031 032 033

%l"22*33 +
and, denoting -ra;:

dx

^^

by

a,',-,

^J

122Sa31

%s"S2"21

"

(^ix"^"^ "

0:l202lOs3

"

(hs^^O'Sl

Then

CHAP.

EVALUATION OF DETERMINANTS

4]

'^ll"22''S3

dx

+ 22ll"33 + ''33"ll"22

+ %3032021

+ O32OISO2I

a^OQiOg^

Oil

0^11

o-L20^i2

OisOi^g + a^-^dj^x

"*"

oil 012 Ol3


022

031

032 033

by Problem 10, Chapter

023

021

Oj^jCggOgg

11

O12 Ol3

21

O22

O23

31

O32

O33

Og^a^a^^

022(^22

>i23'^n''S2

0210-12033

+ O31O12O23

+ "l2"23"31 + 023012131

021013032

37

air^a^^a^.^

+ 023(^23

"^

Og-LCXgi

OgjOj^jOgg

02205^303^

Ogj^o^gOjg

+ 032(^32 + 033(^33

"13

'11

"12

i2i

"22 "23

'31

"32 "33

3.

SUPPLEMENTARY PROBLEMS
9.

Evaluate:

(o)

11

-2
-1

3
1

-4
-2

-1

156

-2000
113 4
1116
16
12 9

ic)

41

(d)

7
3

11.

5
3

-2 -2

show

Is n-square,

that \A^A

is real

aJ"'

Let

"^

and B =

[^-02 ojj

4B|
Use |^B|

(6)

Let

''

1^-62

|.4|"|B|
i.4|-|B| to

show

+ t03

a2+iaA

-Og + ja^

Oi-iogJ

tOj^

and non-negative.

inii.
\ts\

Use \AB

to

first

two rows; also using minors from the

'^

0000

bjj

that

and

118

-4 -3 -2 -5
-2
2
2 -2

Evaluate the determinant of Problem 9(o) using minors from the


first two columns.

12. (o)

-304

{b)

10. If

-4
4 -3
-4 -5

(<
+ 02)(6i+62)
(oi

61 + 163

(0161-0262)

+ (0261+0163)

62 + 164

[-62 + 864

6i-j6gJ

222222,2,2

express (01 + 02+03 + 04X61+62+63+64) as a sum of four squares.

13.

Evaluate

using minors from the

first three

rows.

Ans. -720

EVALUATION OF DETERMINANTS

38

112 12
14.

12
^1,^2

15. If

111
110
112

Evaluate

using minors from the

first

two columns.

.4ns. 2

11

square matrices, use the Laplace expansion to prove

^"^^

^^s

[CHAP. 4

|diag(4i,.42

As)\

Uil-U,!

....

a^ a^ Og a^

16.

17.

*1

*2 ^3 *4

a-^

a^

flg

a^

*1

^2

^3

*4

a^

a2

62

Expand

using minors of the

two rows and show that

60

Use the Laplace expansion


A

first

to

show

62

6..

A
B C

that the n-square determinant

\A\

umn

to

is fe-square, is

zero when

aiiOiii + ai2ai2 + OisO^is

expand each of the cofactors a^g.

+ ai4i4.

.-^

.^

ffi,-

is the

algebraic complement of the minor

"11

"12

"21

22

"ni n2

Use

92

ire

Pi

?i

92

"2ra

P2

Pi "11

'^12

"

"nra Pre

"712

Pn "ni

|.4|

If

"m

Pilj
.^
'J
t=i j=i

Cti

^V

"nn

Zx +

I
2*:

2a:

-1

a:

(c)

;<:

x-l

a;

a:

2a:

+5

3-2

Prove

?n

find the derivative.

x^

(b)

(a)

that the bordered determinant

(4 .3).

()

-4ns.

show

[a^,-],

In

For each of the determinants

2x

"ij

denotes the cofactor of a^j in the n-square matrix A =

li

Hint.

first col-

11 "IJ l^lj
^il^lj^lj

Of
"ii

19. If a^j

along its

ti

1=2 J=2

where

OL^a. tti*

show
^11^11

21.

where

> 2"-

18. In

20.

63

+ 9a:^-

8a;=^

A and B are

(6)

6a:

x^+l

+ 21a:^ +

x+l

12a;^

real n-square matrices with

15a:*,

(c) 6a:^

A non-singular and

5*"^

if

ff

x^

- 28x^ +
=

4 + iS

9a:^

20a;

is Hermitian, then

chapter 5
Equivalence

THE RANK OF A MATRIX. A

non-zero matrix A is said to have rank

minors is different from zero while every (r+l)-square minor,


said to have rank

Example

1.

if

r if

at least one of its r-square

any, is zero.

zero matrix is

0.

The rank

of

'l

3"

is

r=

-1^0

since

while

= 0.

See Problem

An
A

A is called non-singular if its rank r=n,


The matrix of Example 1 is singular.

re-square matrix

called singular.

is

Prom
I.

AB\

that is,

if

A ^

0.

1.

Otherwise,

A\-\B\ follows

The product

of

two

more non-singular re-square matrices

or

is non-singular;

the prod-

uct of two or more re-square matrices is singular if at least one of the matrices is singular.

ELEMENTARY TRANSFORMATIONS.
on a matrix do not change either
(1)

(2)

order or its rank:

The interchange

of the ith and /th rows, denoted

The interchange

of the ith

The multiplication

The
(3)

The following operations, called elementary transformations,


its

by Hij;

and /th columns, denoted by K^j

of every element of the ith row

by a non-zero scalar

k,

denoted by H^(k);

multiplication of every element of the ith column by a non-zero scalar k, denoted by Ki(k).

The addition

elements of the sth row of


denoted by Hij(k)

to the

of the /th row,

k,

a scalar, times the corresponding elements

The addition

to the elements of the ith column of


ments of the /th column, denoted by K^j(k)

k,

a scalar, times the corresponding ele-

The transformations H are called elementary row transfonnations;

the transformations

are

called elementary column transformations.

The elementary transformations, being precisely those performed on


determinant, need no elaboration.
order of a matrix.

In

Problem

It

2, it is

the rows (columns) of a


an elementary transformation cannot alter the
shown that an elementary transformation does not alter its

is clear that

rank.

THE INVERSE OF AN ELEMENTARY TRANSFORMATION. The

inverse of an elementary transformawhich undoes the effect of the elementary transformation; that is, after A
has been subjected to one of the elementary transformations and then the resulting matrix has
been subjected to the inverse of that elementary transformation, the final result is the matrix A.

tion is an operation

39

EQUIVALENCE

40

2.

Let A

Example

[CHAP.

4 5 6

7 8 9
2

"l

The

effect of the elementary row transformation

H2i(-2)

is to

produce B

3"

10

.7

8 9

The

effect of the elementary row transformation ff2i(+ 2) on B is to produce A again


Thus, ff2i(-2) and H2x(+2) are inverse elementary row transformations.

The inverse elementary transformations


-1

"ij

are

^ij

(2')

Hi\k) = H^(i/k)

(3')

H--(k)

= H^A-k)

K^jik)

= Kij(-k)

We have
II.

same

The inverse

of an elementary transformation is an elementary transformation of the

type.

EQUIVALENT MATRICES. Two

matrices A and B are called equivalent, A'^B,


from the other by a sequence of elementary transformations.

one can be obtained

if

Equivalent matrices have the same order and the same rank.
Examples. Applying

in turn the elementary transformations

W2i(-2), ^siCD. Ws2(-1).

-1

12-14

-1

5-3

-1

-2

-7

-7

-3
-3

-1 -2

Since all 3-square minors of B are zero while

I
I

t^

0,

12-1

5-3

'^

the rank of

is 2

hence,

^ is 2. This procedure of obtaining from A an eauivalent matrix B from which the


rank is evident by inspection is to be compared with that of computing the various minors of -4.
the rank of

See Problem

ROW EQUIVALENCE.
lone,

If

is said to

3.

a matrix A is reduced to B by the use of elementary row transformations abe row equivalent to A and conversely. The matrices A and B of Example 3

are row equivalent.

Any non-zero
(a)

matrix

of rank

r is

row equivalent to a canonical matrix C in which

one or more elements of each of the

first r

rows are non-zero while

all other

rows have

only zero elements.


(b) in the ith row, (i

this
(C) ]\

<

=1,2, ...,r), the first non-zero element


element stands be numbered
't

is 1;

let the

column

in

which

;'-

/2

<

(d) the only

<

j^.

non-zero element in the column numbered

the ith row.

j^, (i

=1,2

r),

is the

element

of

CHAP.

EQUIVALENCE

5]

To reduce A
(ii)

If

aij
171

a; J

to C,
7^

suppose

use

0,

//i(l/oi,iji

o^
^^7

but

/i is

(is)

If

(ii)

Use row transformations

Vi

0,

the

number of the

reduce

to

use

41

it

to

non-zero column of A.

first

when necessary.

1,

and proceed as in

ffij,

(i^).

of type (3) with appropriate multiples of the first row to obtain

zeroes elsewhere in the /^st column.


If

non-zero elements of the resulting matrix B occur only in the

wise, suppose

use ^2(1/^2^2) as in
in

(ii);

if

column

first

but

&2J2=

bqj^

B =

first row,

Other-

C.

which this does not occur. If &2j ^ 0,


use H^^ and proceed as in (ii). Then, as

in

0,

clear the /gnd column of all other non-zero elements.

(il),

If

number of the

72 is the

non-zero elements of the resulting matrix occur only in the

Otherwise, the procedure is repeated until

Example

4.

The sequence of row transformations


to A of Example 3 yields
-1

-2

-7

having the properties

first

two rows, we have C.

is reached.
ff2i(-2), ffgiCD

-1

^\j

-3

-3

2(l/5)

-1

'\^

17/5

1
'%^

-3/5
-3

applied

//i2(l). //ssC-S)

-3/5

(a)-(rf).

See Problem

THE NORMAL FORM OF A MATRIX. By means


r

>

can be reduced

to

of elementary transformations any matrix A of rank

one of the forms

(5.1)

/.

called its normal form.

\l%

zero matrix is its

"'"'
own normal

form.

Since both row and column transformations may be used here, the element
obtained in the section above can be moved into the

first

column can be cleared of other non-zero elements.


row can be brought into the second column, and so on.
first

For example, the sequence


/^32(-l), ^42(3)

ff2i(-2),

^31(1).

column.

of the first row

Then both the

Similarly, the element

Ksi(l),

^2i(-2),

applied to 4 of Example 3 yields

first

of the

row and

second

K23, K^{\/%),

X4i(-4).

the normal form.

See Problem

ELEMENTARY MATRICES.

4.

5.

The matrix which results when an elementary row (column) transforma-

tion is applied to the identity matrix /^ is called an elementary

row (column) matrix. Here, an


elementary matrix will be denoted by the symbol introduced to denote the elementary transformation which produces the matrix.
0'
1

Example

5.

Examples

of elementary matrices obtained from

/g

1
1_

"0
1

= K,

_0

'1

0'

1_

Ha(k)

0"

k_

0'

'l

K-sik).

H^g(k)

k
1_

K^^yk)

EQUIVALENCE

42

Every elementary matrix

The

To

A by an elementary

To

mxn

matrix

A can be produced by

matrix.

effect a given elementary row transformation on

to form the corresponding elementary matrix

Ijn

(Why?)

is non-singular.

effect of applying an elementary transformation to an

multiplying

to

[CHAP.

effect a given elementary

of order

mxn, apply the transformation


left by H.

and multiply A on the

column transformation on A, apply the transformation to


K and multiply A on the right by K.

/ to

form the corresponding elementary matrix


~1

Example

6.

When A

3~

4 5 6
_7

H^^-A

4/^13(2) =

3'

"l

_2

"7

9"

interchanges the

4 5 6

7 8 9_

_1

first

and third

3_

"723'

o"

10

8 9

3~

4 5 6
0_

4 5 6

'l

1
1

9_

1
rows of A

"O

_25

1_

adds to
t(
the

16 5 6

first

column of A two times

39J

the third column.

LET

AND B BE EQUIVALENT MATRICES. Let the elementary row and column matrices corresponding to the elementary row and column transformations which reduce /I to 5 be designated
as //i./Zg
where //^ is the first row transformation, //g is the second, ...;
^s< J^\,T^Q.
K^ is the first column transformation, K^ is the second
Then

'^t

(5.2)

IU-H,.A K-i-K^

//.

PAQ

K,

where

H^-H^

Ih

(5.3)

and

We have
III.

P and Q

Two

matrices A and B are equivalent

defined in (5.3) such that


"1

Example

7.

When

2
_1

-1

["100"! r
1

and only

if

there exist non-singular matrices

^3i(-l) //2i(-2) -^ ^2i(-2) -Ksid) .K4i(-2) -K^sd) .Ks(i)

2J

1-200
o"j

0-2
ij

[j^i

if

= B.

2~|

5-23,
2

PAQ

~1

10
10

0~

2"

10
10

"1000"

10

10 1
10

"1000"
10
5

iJ
1

_0

1_

_0

1_

_0

1_

1-25-4
10

[:;=}

o"l
1

PAQ

[1

10
1

oj

Since any matrix

is

equivalent to

its

normal form, we have

IV. If ^ is an re-square non-singular matrix, there exist non -singular matrices


as defined in (5.3) such that PAQ = 1^

P and Q

See Problem

6.

CHAP.

EQUIVALENCE

5]

43

INVERSE OF A PRODUCT OF ELEMENTARY MATRICES.


P

as in (5.3). Since each

H^...H^-H^

and

Let

and

K-^-K^-.-Kt

has an inverse and since the inverse of a product is the product

in reverse order of the inverses of the factors

P~^

(5.4)

H;\hI\..H^^

Q''

and

Let A be an re-square non-singular matrix and


=

let

Kt...Kt-Kt-

P and Q defined above be such

that

PAQ

Then
A

(5.5)

P'\PAQ)Q^

P'^-k-Q^

P'^-Q'^

We have proved
V.

Every non-singular matrix can be expressed as a product

of

elementary matrices.

See Problem

From

7.

this follow

VI.

If

VII.

If

is non-singular, the

P and Q

rank of

Two mxn

(also of

In

Problem

8,

BA)

PAQ

are non-singular, the rank of

CANONICAL SETS UNDER EQUIVALENCE.


VIII.

AB

is that of B.

is that of A.

we prove

matrices A and B are equivalent

if

and only

if

they have the same rank.

A set of my.n matrices is called a canonical set under equivalence if every mx-n matrix is
equivalent to one and only one matrix of the set. Such a canonical set
is given by (5.1) as r
ranges over the values 1,2
m or 1,2. ...,re whichever is the smaller.
See Problem

RANK OF A PRODUCT.
matrices

Let A be an mxp matrix of rank

P and Q such

NQ

Let S be a

AB

pxre matrix

(5.6)

By Theorem

By Theorem

III there exist

non-singular

that

PAQ
Then 4 = P

r.

9.

P''

and consider the rank of


=

P~'NQ'^B

AB is that of NQ'^B. Now the rows of NQ~'b consist of the firstr


m-r rows of zeroes. Hence, the rank of AB cannot exceed r
the rank of A
Similarly, the rank of AB cannot exceed that of S. We
have proved
rows of

VI, the rank of

Q B and
IX.

The rank

of the product of two matrices cannot exceed


the rank of either factor.

suppose iS =
then from (5.6). NQ-'b = 0. This requires that the
first r rows of Q'^B
be zeroes while the remaining rows may
be arbitrary. Thus, the rank of Q-'b and, hence
the
rank of B cannot exceed p-r. We have proved
:

X. If the mxp matrix A


rank of B cannot exceed p-r.

is

of rank r

and

if

the pxn matrix

is

such that

AB

the

EQUIVALENCE

44

[CHAP.

SOLVED PROBLEMS
1.

(a)

The rank

-4

The rank

12

of

is 2

since

and there are no minors of order three.

-4

2 3

(b)

2 3]
sj

of

since

is 2

and

2 4 8
"0

(c)

The rank

Show

since

is 1

4 6

each of the nine 2-square minors

0,

is 0, but nov

6 9_

_0

every element

3"

of

^0.

is

that the elementary transformations do not alter the rank of a matrix.

We

shall consider only row transformations here and leave consideration of the column transformations
Let the rank of the mxn matrix ,4 be r so that every (r+l)-square minor of A, it any, is zero.
Let B be the matrix obtained from .4 by a row transformation. Denote by \R\ any (r+l)-square minor of A and
by Is] the (r+l)-squaie minor of B having the same position as \R\

as an exercise.

Let the row transformation be H^j


two of
in the

Its effect on |/?| is either (i) to leave


interchange one of its rows with a row not of \R\

its rows, or (lii) to

case

(ii),

= -\r\ =

\S\

case

in the

In the

case

(ii) to
(i),

\S\

interchange
= \r\

=0;

except possibly for sign, another (r+l)-square minor

\s\ is,

(iii),

unchanged,

it

of l^l and, hence, is 0.

Let the row transformation be Hi(k). Its effect on \R\ is either (1) to leave
A:.
Then, respectively, |S| = |/?| = o or |S| = ;i:|/?| = o.

unchanged

it

or (ii) to multi-

ply one of its rows by

Let the row transformation be Hij(k). Its effect on |/?| is either (i) to leave it unchanged, (ii) to increase
one of its rows by k times another of its rows, or (iii) to increase one of its rows by k times a row not of S|.
In the cases (i) and (ii), |S|=|ft| = 0; in the case (iii),
\s\ =
=
/?| + A: (another (r+i)-square minor of /I)
|

k-0

0.

Thus, an elementary row transformation cannot raise the rank of a matrix.


lower the rank

tor,

did, the inverse transformation

if it

would have to raise

On
it.

the other hand, it cannot


Hence, an elementary row

transformation does not alter the rank of a matrix.

For each of the matrices A obtain an equivalent matrix B and from

it,

by inspection, determine the

rank of A.
"1

(a)

3"

1
'-^-/

"1

-3 -3

_3

1_

-4 -8

_0

2_

The transformations used were


"1

(b)

2 3

0"

2 4 3 2
3
_6

i
+

2j

0"

-3

-8

-44 -11

8 7 5_

Note.

1+i
(c)

1
''V.

1_

_0

//2i(-2). ffsi(-3);

''\j

-4

-8
-3

H^(-l/3), HQ(-l/i); Hg^i-l).

0'

0"

-8

-3

5_

-3

p -4 -11

5_

_1

3"

-""J

"1

'~^

r^

-4

The rank

is 3.

-8 3

S.
-3

The rank

is 3.

'~^

2j

l+i_

'1

3~

"^

2j

2J_

'-Vy

2J

B.

The rank

is 2.

The equivalent matrices 5 obtained here are not unique. In particular, since in (o) and (i)
only
row transformations were used, the reader may obtain others by using only column
transformations.
When the elements are rational numbers, there generally is no gain in mixing row and column
transformations.

CHAP.

4.

EQUIVALENCE

5]

(a)

(b)

A =

13
12 6

113

2 3 9

2 3 9

113

13

2 4
1

.1

5.

C row equivalent

Obtain the canonical matrix

-2
-2
-3
-1

Reduce each of

(a)

12

6_

GOOD

-2_
3"

-1

'^

10
4
13-2

'->-'

-2 3

l"

of the given matrices A.

13
-1

p -1

each

13-2
13-2

-2

6 4

113

--v^

to

5_

45

'10

10
10

^\y

2
1

-1

--O

pool

4_

10
1
10 0-1
10 2

7~

2_

the following to normal form.

-1

1
^\y

-1

-2

7 2

"l

fl

o"|

0-2

7 2 sj

o"

1-2

p-

Lo 2

o'

'l

10

1-2

11

7 3_

o'

'l

10

'V.

-7_

0_

Us o]

The elementary transformations

are:

^2i(-3). 3i{2); K2i(-2), K4i(l); Kgg; Hg^-^y. ^32(2). /f42(~5); /fsd/ll), ^43(7)

(6)

[0234"! P
23 5 4^-0

4 8 13 I2J

|_4

354
2

"1000"

'-^

8 13 12

2 8 13

The elementary transformations

13
13

'Xy

2 3 4

"1000'
^\j

10
p

4.

"1000"
<^

10

0_

0_

are:

^12: Ki(2); H3i(-2); KsiC-S), X3i(-5), K4i(-4); KsCi); /fs2(-3), K42(-4); ftjgC- 1)

6.

1
2 3-2
2-2 1 3
3
04 1

Reduce A

Since A is 3x4,

we

to

normal form

A'

and compute the matrices

shall work with the array

seven elements and each column transformation

is

and

(^^

Each row transformation

such that

is

P-i_AQ^

-3

-2 -3

1
1

12

2-2

10

-6

-5

-6

-5

3-2

1/3 -3

-1/6

1-57-210
0-1-11

-5

7/6

10

10
1

or

0-210
0-1-1

-2

0-1-1

1/3 -4/3 -1/3

10
1

-6 -5 7 -2
-6 -5 7 -3

0-1/6 -5/6

1
1

Pi

A^.

performed on a row of

performed on a column of seven elements.

10

P^

EQUIVALENCE

46

[CHAP.

1/3 -4/3 -1/3

1
1

Thus,

Pi

-1/6 -5/6

-2

-1

-1

7/6

10

and

PiAQ-i =

10
10

N.

7.

Express A =

.1

4_

as a product of elementary matrices.

The elementary transformations

H^-H^-A-K^-K^

//2i(-l).
=

^siC-D; ^2i(-3). 'f3i(-3) reduce A

Hl-H'^-K^-Kt

(5.5),

Prove:
If

other.

Two mxn

9.

that is,

[see (5.2)]

ij

[l

if

3"|

010

ij

[p

and only

if

ij

fl 3

O'

010
1_

_0

they have the same rank.

same rank, both are equivalent to the same matrix (5.1) and are equivalent
^ and B are equivalent, there exist non-singular matrices P and Q such that B
A and B have the same rank.

Conversely,
VII,

fl

0"j

010

matrices A and B are equivalent

A and B have

By Theorem

fl

0"1

110
_0

8.

ff3i(-l)-^2i(-l)-'4-X2i(-3)-?f3:i.(-3)

fl

Prom

to

the

if

to

each

PAQ.

canonical set for non-zero matrices of order 3 is

[!:][::]

[i:l-[:i:]
A

canonical set

tor

non-zero

Vh o]

10. If from a

3x4

[:=:]

square matrix

nm

matrices is

of order n and rank

is selected, the rank r^ of

is

r^,

a submatrix

equal to or greater than

The normal form of A has n-rj^ rows whose elements


whose elements are zeroes. Clearly
'A

from which follows

>

- n as required.

r^

&:]
B consisting

+ s -

of s rows (columns) of

n.

are zeroes and the normal form of

6 has s-rg rows

CHAP.

EQUIVALENCE

5]

47

SUPPLEMENTARY PROBLEMS
2

4 5

11.

Find the rank of

(a)

2
(c)

(c) 4.

(b) 3,

Show by considering minors

13.

Show

14.

Find the canonical matrix row equivalent


Tl

A,

that

2-3"]^ri 0-7]
[o

-4j

[2 5

A. and

A'.

-2

10 11 12 13 14

4-16

15 16 17 18 19

have the same rank.

(b)

3 4

3 4

12

11/9

-1

-1

(b).(c)

[/g

/a

12

12

^V/

(e)

-2

o]

P^

(d)

(e)

j]

From
Prom

10
1
10-1

[I, 0],

(a)

Let A

1/9

(c)

-3

-3

10
10
12

10 0-12
10

normal form of each of the matrices of Problem

15. Write the

Ans.

10

2j

-1

1/9"

1
'X^

14.

P'

jl

form Z/^^. /^O). //i3(-4) and check that each HA effects the corresponding row transformation.
form K^^. Ks(-l). K^^O) and show that each AK effects the corresponding column transformation.

(c) Write the inverses H^l,

H^

{?,),

H^lc-i) of the elementary matrices of

(d) Write the inverses K^l. ifg^C-l).

K^lo)

Compute B

^12

ft,(3) -//isC-

4)

(a).

of the elementary matricesof (6)

"0

(e)

(d)

each of the following:

to

(6)

5-46

.4'

12
_4

(a)

that the canonical matrix C, row equivalent to a given matrix A, is uniquely determined by A.

(a)

16.

(d) 2

12.

(d)

-1 -3

7 4 6

(a) 2,

(b)

4 3 4

Ans.

12-23

Check
Check

0'

-4

that for each

"0
and C

H^^(-4:)-H^(3)-Hi

1/3

(/)

17. (a)
(b)

Show

that

BC

Show
Show

that

/?',-=

that if

/?

CB
H-.

H,H-H~^^l
KK~^ = I

that for each K.


1

4"|

0.
ij

K-(k) = H^(k).

is a product of

and K^,(/t)

= H^-(k)

elementary column matrices. R'is the product in reverse order of the same

elementary row matrices.

18.

Prove:

(a)

(b)

19.

UP

AB
AB

and
and

BA
BA

are non-singular if
are singular

if at

.4

and B are non-singularra -square matrices.

least one of the n-square matrices

A and B

is singular.

and Q are non-singular, show that A,PA,AQ, and PAQ have the same rank.
P and Q as products of elementary matrices.

Hint. Express

20.

Reduce B

13 6-1
14 5 1
15 4 3

to

normal form

/V

and compute the matrices

P^

and

Qr,

such that P^BQ^ = N

EQUIVALENCE

48

21. (a)
(6)

Show
Show

m+l

22.

that the

number of matrices

number
and n+1.
that the

12
13

Given A

2 5 6

10

in a canonical set of n-square matrices under equivalence is

of matrices in a canonical set of

of rank 2.

23.

The matrix A

24. If the

25.

mxn

of Problem 6 and the matrix

AB

matrices A and B are of rank

show

Let A and B be n-square matrices,

Show

rj

and

rg

respectively,

show

Find P and Q such that B - PAQ.

that the rank of

that

\AB\ = |^|

(a) If at least

A+B cannot exceed

By considering each

of the

|fi|

one is singular show that


\AB\ = \a\-\B\

\AB\ =

|/4|-|s|

(6) If

both are

that equivalence of matrices is an equivalence relation.

29. Prove:

Hint.

= 0.

A_

Let ^ be an arbitrary n-square matrix and S be an n-square elementary matrix.

28. Prove:

30.

of Problem 20 are equivalent.

non-singular, use (5.5) and Problem 25 to show that

27.

such that

7^

are arbitrary.

six different types of matrix S,

26.

n+l.

matrices under equivalence is the smaller of

abed
_e

and take

Q-'b

where a.b

mxn

Find a 4-square matrix S

Follow the proof of Theorem

Hint.

[CHAP.

The row equivalent canonical form

of a non-singular matrix

Not every matrix A can be reduced

to

is I

and conversely.

normal form by row transformations alone.

Exhibit a matrix which cannot be so reduced.

Show how

to effect

types (2) and

31. Prove: If

.4

on any matrix A the transformation H^: by using a succession of row transformations of

(3).

is an

mxn

matrix, (m ^n), of rank

theorem when the rank of A is < m.

m,

then AA' is a non-singular symmetric matrix.

State the

chapter 6
The Adjoint of a Square Matrix

THE ADJOINT.

Let A =

[a^ ]

be an n-square matrix and

adjoint

(6.1)

adj

oij-

be the cofactor of a-; then by definition

12

^22

'^ire

'^sn

Note carefully that the cofactors of the elements of the ith row (column) of A are the elements
of the jth column (row) of adj A.

Example

i.

For the matrix A

11=

2
3

ai2 = -2.

6,

Cl-lS

-3.

fflsi

= 1,

0122 =

and

adj

a^g

-5,
6

ffai

= -5,

Otgg

4,

agg = -1

1-5
1

-2 -5
-3

= 3,

-1

See Problems

Using Theorems

and XI of Chapter

"In

^^2

0271

'^

diag(M),

Examples. For the matrix ^

/I

By

of

Example

U|.

ni

1,

adj

CE 7

A-L

1^1)

6
1 -5I
-2 -55 4
-3 33 -ij

/I

|,

There follow
I.

(6.4)

If

is re-square

and non-singular, then


adj
I

-7 and

U|

(adj A)

f-T

4
I

49

.
I

-7/

-7

[_

adj

0'

0-7

we have

taking determinants in (6.2),

(6.3)

1^1

(adj /I)

find

%1 %2
i(adj A)

(6.2)

we

3,

1-2.

THE ADJOINT OF A SQUARE MATRIX

50

If

II.

is of

If

A and B are

AB

adj

IV.

In

Problem

^ii.i2

^l'''-2

Let

7m + i' 7m+2'
let

we

is of rank ra-1, then

Jn

%.

^2

adj

adj

-square minor of the

be

Jn
its

complement

re-square matrix

in A,

A =

the m-square minor of adj A


dei
denote

in adj

A as those

Jm

occupy

of

j\

If in (6.6)

S2

+ +

in A.

,?i.

M
When m =

2, (6.7)

Jn

Jm+i' Jm + 2

Jn

(-D^'i^l'

%+

/i

is non-singular,

(6.7)

+ +

72

Jt,

then

' Jm

J2<

(-1)

H, %,

uaH-i
I

i-

^M+1' 'm+2. . ^n

becomes
Jg>J4-

Jn

(_l)V*2+Jl-'j2m

(6.8)

H'J2

^2'J2
.J1.J2

\A\

When m = n-l,

(6.7)

Jl'j2

When m =

n, (6.7)

algebraic complement of

becomes
Jre-1

M,

(6.9)

tjj

whose elements occupy the

Jm + 2

Jvi + i'

Jm

Ji' J2

M:
s

[o,-,-].

and

Then

where

3-

prove

Ji' J2

(6.6)

See Problem

is of rank 1.

Ji'J2

same position

adj

we prove

6,

be

^m+i' ^ra+2'

let

4,

If

re-square matrices,

(6.5)

MINOR OF AN ADJOINT.

0.

Problem

In

(ad}A)A

rank < ra-l, then adj A =

THE ADJOINT OF A PRODUCT.


III.

and singular, then

is re-square

A(a,diA)

If

[CHAP.

becomes

(-1)

(6.4).

Ul

a.

CHAP.

THE ADJOINT OF A SQUARE MATRIX

6]

51

SOLVED PROBLEMS
1.

The

adjoint of

i4

a h

'^ll

IS
c

0^21

d -b

3 4

4 3

4 3l

3 4
-7

2.

The

A =

adjoint of

[;]

IS

-2

3.

Prove:

If

is of order n

we

First

-1
-1

and rank n-\, then adj A

rank

is of

1.

note that, since A is of rank n-\. there is at least one


non-zero cofactor and the rank of adj A
By Theorem X, Chapter 5, the rank of adj^ is at most n-{n-l) =
i.
Hence, the rank is

IS at least one.

exactly one.

4.

Prove:

adj

^S

= adjS

adj

^S-adjS-adj^

and

(adjB-adj^)4fi

we conclude

5.

Show

that

By

(6.2)

^Badj^S

^y(6.2)
Since

y4

.4

(S adj

S ) adj

/I

adjS {(adj^)^lfi =

that

adj

adj(adji) = U|
and

\ab\-I

-A,

if

^S

\a\

{a.aiAB)AB

^(l^l-Oadj/i

jslc^adj/^) =

adjS-U|./.B

U|{(adjB)Si

adj

adj

|b|.U|./
=

Ab\

\AB\-I

/I

0.

?^

(6.4),

adj

adj (adj

^)

diag(| adj ^|,

diagd^l

adj/l|

|adj^|)

Ul

1^-1

Ul

Then

adj (adj ^)

and

6.

Prove:

Ji'

Let

A;

adj (adj

^)

\aT' -A

adj (adj

^)

UT'^-^

^^ ^" m-square minor of the re-square matrix


I

Jm+i. Jw4-2

in

^m+i. ^m+2

^n

let

nH.i2

UT'^-^

in

;
'"n

^^^

in\

A =

[a:-]
tj-

be its complement in A, and

^*^e the m-square minor of adj


J1.J2

position in adjv4 as those of

/I

whose elements occupy the same

is

^H,i^, ...,i

occupy in A.

Then

THE ADJOINT OF A SQUARE MATRIX

52

u
where

s =

Ji

+ f2+ +

Ji.

J/2'

H'

^2-

+/1 + 72+

Jra + i' Jii

Jm

>

'

(-ifur

^m

[CHAP.

^i

'

Jn

7m

Prom
a,-

a,-

a,-

a,,

"im.im

"^mJ's

Wl'A

''m+iJs

''m+iJm

'

"W2

"ht.ji

in.i

"^m^n

'^m+iJm+i

'

"

"^raJm+i

"traJn

tti,

*i

a,-

a,-

a,-

a,-

a,-

>

'00

,/2

ai

a,-

a,-

''m+i-Jn

"V.im4.i

Ul

^^2

h-j9

"^ra.ii

1-

h.Jm^i.

\a\

Ul

''i2Jn

" "^-Jn

"^M'Jn^i

"V+i'in

"%-i-i'Jn-^i

^ri'^m+i

"in.in

by taking determinants of both sides, we have


Ji. Js'

(-ifUI- M %
where

7.

s is

Prove:

If

as defined in the theorem.

is a

H\

Jn

Jm+1' Jm-t2
4.

^2

Prom

this, the required form follows

skew-symmetric of order

^m+l' ''m+2

''w

2ra,

immediately.

then \A\ is the square of a polynomial in the elements

of i.

By

its definition.

above this polynomial

The theorem

2.

By

we

a\ is a polynomial in its elements;

is true for n = l

Assume now
order2A: +

are to

show

that under the conditions given

is a perfect square.

since,

when A = r
\

that the theorem is true

partitioning, write

oi

,1

\A\ = a

\,

2
.

\-a Oj

when

P^l
\

n = k

and consider the skew-symmetric matrix A = [a^j] of

E==
where

'

r
\

-zri^i.-zii^-zy
'^2k^..2n^\

Then S

is

skew-sym-

"

CHAP.

THE ADJOINT OP A SQUARE MATRIX

6]

metric of order 2k and, by assumption,


If

\b\ =

a^j denotes the cofactor of a^j in

Moreover,

/I,

*2fe+l, 2fe+l

0^2^+2. 2fe+l

f''2fe+l,

0'2fe4-2,

Otsfe+s.sfe+i =

2fe+2

-fflgfe+i^sfe+s

1^1/

where /is a polynomial

we have by Problem

elements of B.

in the

Chapter

6,

53

and (6.8)

3,

0^2^+2, 2fe+l

^2^+1, 2/!e+2

2fe+2

hence,

Ll2/fe+l.

and

2fe+2

a perfect square.

SUPPLEMENTARY PROBLEMS
8.

Compute the adjoint


3~

"l

(o)

~1

(b)

0_

_0

of:

(i)

_3

10
4

-2

110
(rf)

1_

1_

_0

9.

-2

0-2

(a)

(c)

1_

Ans.

2*

'l

_0

2"

S'

(c)

-2

-5
-2

(rf)

-16

10 3-5

-2

10

Verify:
(a)
(b)

(c)

The
The
The

adjoint of a scalar matrix is a scalar matrix.


adjoint of a diagonal matrix is a diagonal
matrix.
adjoint of a triangular matrix is a triangular
matrix.

10. Write a matrix

11. If

is a

/I

7^

of order 3 such that

adj^ =0.

2-square matrix, show that adj(adj/4) = A.

-1 -2 -2
12.

-4

-2

Show

that the adjoint of

13. Prove:

If

-2

2-2

an n-square matrix A is of rank

14. Prove: If

is

15. Prove: If

is Hermitian,

16. Prove: If

is

-4 -3 -3
is 3^' and the adjoint of

<n-\.

then

adj^

is

itself.

= 0.

symmetric, so also is adj-4.

so also

is adj/1.

skew-symmetric of order

, then

adj^

is

symmetric or skew-symmetric according as . is


odd

THE ADJOINT OF A SQUARE MATRIX

54

theorem similar to that of Problem 16

17. Is there a

18.

[CHAP. 6

skew-Hermitian matrices?

for

For the elementary matrices, show that


-1
(a) adj Hij =

-Hij

-1

H^ (k)

diag(l/A:. l/k

(c) adj Hij


Hjk(k)

Hij(k),
HJ

(6) adj

19. Prove: If

1,

l/k

stands in the ith row

with similar results for the K's.

-1

Use

n-l

and

H^...H^

if

-H-^

-A K-^-K,2---^t

where \

is

then
adj

20.

where the element

l/k),

an n-squaie matrix of rank n or

is

or

1/k,

adj

Xi

-1

adj

-1

K^

K^

adj

-1

adj

adj H^

-1

adj

H^

-1

adj H^

the method of Problem 19 to compute the adjoint of

1110
(o)

of

12

Problem 7 Chapter 5

(b)

2-2

-14
7

Ans. (a)

-3 -3

-1
-1

14

= [a^--]

23.

is n-square then

Let A^ =\.%j\
example,

1-1

and B = [^k-a^A be 3-square matrices.


S(adj-4)

22. Prove: If

-2

(b)

-7
21. Let

-2

and

S(adjB)

adj (adj

^)

If

S{C) = sum of elements of matrix C, show that

\b\

UI

k S(adj,4)

Ui

be the lower triangular matrix whose triangle is the Pascal triangle; for

('/ = 1. 2, ...,n)

10

110

12 10
13
3

Define

bij

"V

= (-1)^ ^ai,
Hj

and verify

for n

adJ-4

(i)

24.

Let B be obtained from A by deleting

where

=2,3,4

o!^-

its ith

^ij

^pj

iq

pq

is the cofactor of a^j in \A\

that
[6.^,-]

tjj

and pth rows and /th and gth columns.

Show

that

chapter 7
The Inverse of a Matrix

IF A

AND B
A

is

are n-square matrices such that

called the inverse of B, (A =


In

Problem
I.

An

II.

THE INVERSE

If

ra-square matrix

BA

A has an Inverse

if it is

of a non-singular n-square matrix is unique.


is non-singular,

then

AB

AC

of a non-singular diagonal matrix

:^i' -^2

is called the inverse of A,

and only

if

implies

B =

diag (i k,

diag(i/k^,l/kr,.

^^

=1,

(B=A~^) and

we prove

l,

The inverse

AB

B''^).

non-singular.

(See Problem?, Chapter2.)


C.

A)

diagonal matrix

is the

1A)

...,

are non-singular matrices, then the inverse


of the direct

diag(Al^.

A^

sum diag(/}

A~s)

Procedures for computing the inverse of a general


non-singular matrix are given below.

INVERSE FROM THE ADJOINT. From

adj

i = U|

/.

if

is non-singular

n/M|

a^^/\A\

...

ai/U|

i2/U|

a22/!.4|

...

a2/UI

am/Ml

a^n/\A\

...

ann/\A\

adj^

(7.1)

Example

(6.2)

1.

From Problem

2,

Chapters, the adjoint of A

"7/2

Since

Ml

= -2,

A~^ =

4 3

-1
-1

-2

-3

^'^^ -^

-k
2

-7
is

^J
See Problem

55

2.

THE INVERSE OF A MATRIX

56

INVERSE FROM ELEMENTARY MATRICES.

[CHAP.

Let the non-singular n-square matrix A be reduced to

by elementary transformations so that

H^.-.H^.H^. A-K^-K2...Kt

Then A ^ P

Q^

Example

2.

From Problem

Q-P

(P'^-Q'Y

K^-K^... KfH^... H^-H^

K^K2H2'i

o"

-1

In Chapter 5

-3 -3
1

-10

that a non-singular matrix can be reduced


Then, from (7.2) with Q =1, we have

normal form by row

to

H,...H^.H,

(7.3)

That

-1

was shown

it

transformations alone.

-1

1
1

1__

"106" '100"

-3

_0

-3"

'l

-3

1_

-3

"l

-1

-1

Chapters,

7,

HqH^ AK^Kq

Then

(S~V^ = S,

by (5.5) and, since

A'^

(7.2)

PAQ

is,
III.

If

is

reduced to

by a sequence of row transformations alone, then A~

equal

is

to the product in reverse order of the corresponding elementary matrices.

Examples. Find the inverse

of

.4

4 3

13
Write the matrix [A
Ir,

3 3

4 3

3 4

of

Example

using only row transformations to reduce

/I

to/.

and perform the sequence of row transformations which carry A into

on the rows of six elements.

[AI^]

We have
1

1
"X^

1
1

-1
-1

-3

-"o

'\j

-1

-li

-3 -3
1

[/3^"']
7

by

(7,3).

Thus, as A

is

reduced to

/g, /g is

carried into

-1
-1

-3 -3
1
1

See Problem

INVERSE BY PARTITIONING.

Let the matrix A = [a^j] of order n and

its

inverse

B =

titioned into submatrices of indicated orders:

41

A12

^11

"12

(pxp)

(pxq)

(pxp)

(px?)

^21

A22
(qxq)

"21

S22

(qxp)

(qxq)

and

(qxp)

where p + q = n

[6.^,-]

3.

be par-

CHAP.

THE INVERSE OF A MATRIX

7]

Since

AB

BA
(i)

57

we have

I^,

/4iiSii + /lisBgi

4iiSi2 +

Ip

(iii)

Bgi'^ii + B^z'^si

(iv)

Bjiiia + Bss^^as

(7.4)
(ii)

Then, provided

/4ij^

!Bxi
where

.422

A-^^B^-z

Iq

is non-singular,

'4ij^

(A.^^^

"-i-zK

^21

(^^si '^n)

~C

(^21 '^u)

- A^i^Ali A^^).

See Problem

To

taken of order ra-l.

In practice, A.^^ is usually

used.

4.

obtain A^^, the following procedure is

Let

[Oil

%2l

2i

^^aj

"11
Osi

'

31

%2

13

022

O23

32

G^ =

11

%2 %3 %4

021

022

023

Ogj

Ogj

033

6(34

41

%2

'''43

''44

33_

'^24

After computing C^^, partition G3 so that /422 = [o33] and use (7.5) to obtain C^^. Repeat the proc

ess on

G4. after

partitioning

so that ^22 = [044], and so on.

it

Example 4. Find the inverse of A

Take ^11
11

^=

[!'].

^22

4 3

3 4

^12

'42i(^;' ^12)

using partitioning.

^421 =

[13],

and ^22= [4]-

[4]-[l3]M

and

[1],

Now

f^

[l]

Then
Sii

4\

S12

-('4i\^i2)r^ "

('4i\/ii2)rw^ii)

S21

-^

(^2i'4la)

522

r'

[j

i]^[o]fii-'^^^

oj

[-1,0]

[1]
"
fill

fil2

and

-1

-3 -3
1

S21 B22
_

-1

See Problems

5-6.

THE INVERSE OF A MATRIX

58

THE INVERSE OF A SYMMETRIC MATRIX. When A


tors

need be computed instead of the usual

[CHAP. 7

symmetric, aij =

is

ciji

and only ^(n+1) cofac-

n^ in obtaining A~^ from adj A.

If there is to be any gain in computing A~^ as the product of elementary


matrices, the elementary transformations must be performed so that the property of being symmetric is preserved.
This requires that the transformations occur in pairs, a row transformation followed immediately

by the same column transformation. For example,


'0

...

b
.

Ht

...

...

^l

b
c

^i(- ^)

^^i(- |)

However, when the element a in the dia^gonal


and K^{l/\Ja).

H^(l/\Ja)
is not

In general,

replaced by

is

1,

the pair of transformations are

^Ja is either irrational or imaginary; hence, this

procedure

recommended.

The maximum gain occurs when

the method of partitioning is used since then (7.5) reduces to

(7.6)

where

A^^ - A.^^(Af-^A^^).

See Problem

When A
is

is

not symmetric, the above procedure

symmetric, and then the inverse of A

to find the inverse of A'A,

which

found by

is

A'^

(7.7)

may be used

7.

(A'Ai^A-

SOLVED PROBLEMS
1.

Prove:

An

re-square matrix

Suppose A
that

is

non-singular.

By Theorem

PAQ=I. Then A=P~^-Q'^ and


-1

Supposed
A

A has an inverse

A''^

if

IV,

Q-P

and only

Chapter

5,

if it is

non- singular.

there exist non-singular matrices

-1

exists.

is non-singular.

The A- A

=1

P and Q such

exists.

is of rank n.

If

/4

were singular,

_i
/l/l

would be of rank <

n; hence,

CHAP.

2.

(a)

THE INVERSE OF A MATRIX

7]

When A

2 3

(b)

3.

When A =

!]

[?

r4-3l,andi"=r4/5-3/5-]

=5, adji

[-1

2 3

-5

-5

then U! = 18, adj A

2 4

3 6

2 5

-3

_4 5

14

4 =

2 4

4 5

14

1 1

2/5J

-5

and A

-5

14_
0'

10

[_-l/5

2j

Find the inverse of

[AIJ

/I
I

59

3/2

3 6

2 5

-3

14

14

1
"K^

1/2

1
1

^\j

3/2

1/2

'

1/2 -1

-3/2

-1 -5

-1

10

-2

1
'

- 3 10

10

-3

3/2

-1 -5

0'

1/2

1
]

'Xy

_4

-1

-3

10

10

18

1
'X'

-3/2

7/2

11

5/2

1-1-5

-1

1-2
5-5

-3

-2

"1

'V^

1/2 -1

h
-7 -2

13

-7| -4
-2 -3

0'

10

18

-7

-2

-7-2

-4

-3

'

--10

10

11

1_

-23

29

10

26/5

7/5

-12
-2

6/5

2/5

-2

3/5

1/5

100]

'^-/

'-\J

13

0'

-2

-5

-2 3/5 1/5

-64/5 -18/5

Ih

The inverse

(i)

4.

Solve

is

iiiSn

(..,
/fiiS,2
(11)

Set

A-']

S22 = f"^

^ti - A'l^A^^B^^ =

29

10

26/5

7/5

-12
-2

6/5

2/5

-2

3/5

1/5

+ iisSsi
^

+ A^^B^^

Prom

4\

-64/5 -18/5

-23

(ii),

(iii)

S^i^ii + B^^A^^ =

(iV)

^21^12 + 52,42

for

B^^ = -(-4^1-412)^"^;

from

(iii),

Bii,Si2,S2i, and S

= /

B21 =

- ^~\A^^At^y,

and, from

+ (^lVi2)f~\'42iA\).

Finally, substituting in (iv),

-f

(-42i'4ii)^i2 + ^'^422

*.

and

'^'2

('^21'*lll)'^12

(i),

B-^^

THE INVERSE OP A MATRIX

60

5.

12
13

Find the inverse of A

[CHAP.

by partitioning.

1111
(a)

Take Gg

and partition so that

[24],
-1

Now

r 3 -2l
jj.
^_^

11

,f

Then

^22 -

Sii = 4l\ +

,-1

,
^11-41. -

'42i('4ii^i2)

[3

-2

|^_^

(.4lVi2)rV2i^i\)

3L-3

4]r

[j

Hr-i] [2

"^l +

-'^

f (/I2A1)

and

D01

A^^

3 3

4 3

-3

Oo

-421

=[

-6

0],

A~'^

0-1

and

1 ]

/I22 =

[ I ]

A^iA-^-^

l](i-)

and

&

3'

^1

3 [3]-|[2

B12 =

-3 2]

-1

-1
"

"X"L2

3 2J,

-6

C'^=[3]

3'

-1

[-2

S21 =

B22

-2].

4
=

1_

1-210
[Sii
S21

S12

1-2

2-3

1-11

B22J

-2

-2

1-2

-2

-1

6-9
-2

0'

-3
_

and

4-fl

0-1

-1

[o oJ

[l]-[l

.f

B,

ij

Then

=[-l/3]

A^Q

3-6

4i

[2 0],

Now

-2

-2] _ [2 ol

-1

3-6

that

r 3

f
3

0]

i[2

-3

A so

[24]

and

[-3],

[3]

3J

B12 = ('4ii'4i2)C

(6) Partition

'421-4 11

[3] - [2

and

3-2

[3]

CHAP.

6.

THE INVERSE OP A MATRIX

7]

Find the inverse of A =

We cannot

take

4 3

-4,

by partitioning.

since this is singular.

By Example 3,

61

3 3

4 3

3 4

7 -

-3

the inverse of

//

-3 -3'

B^Ho

"1

0'

1
1

-1
-1
7

is

-3

-3'
1

Then

Thus, if the (ra-i)-square minor A^.^_ of the n-square non-singular matrix A is singular, we first bring a
non-singular (n-l)-square matrix into the upper left corner to obtain S, find the inverse of B, and by the proper transformation on B~^ obtain A''^

1-12

2
7.

Compute

the inverse of the symmetric matrix

13

-1

2-3
1-1

2-3-1
1

-1

-1

Consider

first the

submatrix G^

partitioned so that

-421

^22=

[-1 2],

[1]

Now

^22 - ^2i(Ai^i2) =

Then

[l]-[-l

2]rM

[-2]

and

f^=

[-i]

B,

B12

["jj.

B21

[-k

10

-5

-1

-5

-5

and

B22 = [-i]

k].

Consider now the matrix A partitioned so that

'^11

A<

_1_

10

-1

-1

-5

3-1

Now

-5

-5

[2 -3 -1],

-1/5
2/5

-2

<f= [18/5

],

[4]

[5/I8].

THE INVERSE OF A MATRIX

62

5-7

Then

5-15

JS^

[CHAP.

-7

^[1

521

-2 10],

B22 =

[5/I8]

11

5-71

5-1

and

-7

5-2
11

10

-2 10

SUPPLEMENTARY PROBLEMS
8.

Find the adjoint and inverse of each of the following:


"

2-1"

"2

4"

_1

3"

"1

3
(a)

-1

-1

(b)

2
1_

(O

(d)

n
w

n
U

n
^

1
J.

3-15
Inverses

(a)

-10

^ -153-1
5

-5

9.

10.

Find the inverse of the matrix of Problem

8((?)

-3

-3

-1

-2/3
1/3

(d)

1/2 -1/6

2-1

-J

(c)

-4 -14

15

^'^i

1/3

as a direct sum.

Obtain the inverses of the matrices of Problems using the method of Problems.

13

3-1

13

11

3
1

11.

Same,

for the

matrices

(a)

-4 -5

(a)

16

-6

-144

- 30

-1

48

-10 -44
4 -13

30

-2
-1

(c)

11

-1 -7 -3

48

-26
16

-1

-1 -1

Example 4

(d)

36

60

12

21

-5
-20 -12
-4 -12 -13
48
12 -12
3
30 -20 -15

25

-18

21

30 -11

(^)i

-5
-8

-30
-15

-9

12

6-9

15

-7

-6 -1

-1

12

12.

Use the result

13.

Obtain by partitioning the inverses of the matrices of Problems 8(a), 8(6), 11(a) - 11(c).

of

11-1
1-2-12 2

41

2
(c)

(b)

-1
(b)

22

18

-4
-5

to obtain the inverse of the matrix of

Problem 11(d) by partitioning.

CHAP.

14.

THE INVERSE OF A MATRIX

7]

63

12-12

12

2-11

112

Obtain by partitioning the inverses of the symmetric matrices (a)

-1

-1 -1 -1
-1 -1
1

-1

-1 -5 -1

Ans. (a)
-1

15. Prove: If

16.

Show
(a)

17.

andS,

Show

that

if

A^A

Hint:

18.

Show

A und B

(b)

20.

(AA"^)' =

AC

is said to

and only

Find a

if

The rank

is the

A and B commute, so also do


B
Hint, (a) A (AB)A

(A~^)'A

is

symmetric, so also is

/I

(BA)A

^.

A and B commute, then

(a) A~'^B.

(b) AB"'^.

and

(c) A~^B~^

have a right inverse B if AB = I and a left inverse C it CA = I. Show that A


has
A is of rank m and has a left inverse if and only if the rank of A is n

13 2 3
14 13
13 5 4

if

one exists.

of A is 3 and the submatrix S

4x3 matrix

= C.

a.nd

17

if

right inverse of

{A'^ B)' = {BA^'^)' = (A'^)'B' = A~^B.

Hint: (a)

a right inverse

implies

that if the non-singular symmetric matrices

An mxn matrix A

Hint.

(c)

the non-singular matrix


:r

are symmetric.

19.

AB

then

that if the non-singular matrices

1-12

3-3 2
3-4 4-2
-3
4-5 3
2-2 3-2

(b)

is non-singular,

.4

-1

-3

-1 -1

(b)

-1 -1

2
is

non-singular with inverse S

right inverse of

-9 -5

-4
-1

L J

Show

that the submatrix

3 3

21,

4 3

3 4

of A of Problem 20 is non-singular and obtain

-1
-1

-1 -1

-3

23.

Show

|/+

l|

that

24. Prove: If

25. If

-310b
13
14
2

|.4ii|

0,

0,

then

;^

26. Prove:(i) of

as a

inverse of

left

has neither a right nor a

/111

A21
+

where a,b. and

then

.4)-^

and

(I

Problem 23, Chapter

6.

(/

^12
Aq2

|.4ii|

- A) commute.

left inverse.

-1^22 - ^^si^ii/ligl

as another

right inverse of A.

22. Obtain

-3 -3

c are arbitrary.

chapter 8
Fields

NUMBER

collection or set S of real or complex numbers, consisting of more than the ele-

FIELDS. A

ment 0, is called a number field provided the operations of addition, subtraction, multiplication,
and division (except by 0) on any two of the numbers yield a number of S.

Examples

number fields

of

(a) the set of all rational

(b) the set of all real

are:

numbers,

numbers,

(c) the set of all

numbers of the form a+b\f3

(d) the set of all

complex numbers a+

bi,

where a and b are rational numbers,


where a and b are real numbers.
,

set of all integers and the set of all numbers of the form

The

bvs

where

i is

a rational

number, are not number fields.

GENERAL FIELDS. A

collection or set S of two or more elements, together with two operations called

addition (+) and multiplication

is called a field

().

provided that

a, b, c,

being elements of

...

F, i.e. scalars,

a + b is a unique element of

a+b

= b+a

a + (b + c)

= (a +

b) + c

For every element a in

For each element a in

As

ab =

a-b

is a

F
F

there exists an element

in

F such

there exists a unique element

unique element of

-a

in

that

a+

F such

+a =

that

a.

a + (-a) =

0.

ab = ba
(ab)c = a(bc)

For every element a in

For each element a ^


a~

Di

in

?^

such that

there exists a unique element a'^ in

1-a =

F such

that

o.

a-

a'''^

-0=1.

a(b +

there exists an element

c)

= ab+ ac

(a+b)c = ac + bc

D^'-

In addition to the number fields listed above, other examples of fields are:
(e) the set of all quotients

(/) the set of all

2x2

^^^
Q{x)

of polynomials in

matrices of the form

ti

x with real coefficients.

where a and

b are real

numbers.

0+0=0.

This field, called of characteristic 2, will be excluded hereafter.


customary proof that a determinant having two rows identical
not valid. By interchanging the two identical rows, we are led to D = -D or 2D =

(g) the set in

which

In this field, for example, the


is

but

is

is not

necessarily

0.

64

CHAP.

FIELDS

8]

SUBFIELDS.

S and T are two sets and

If

65

every member of S is also a member of T, then S is called

if

a subset of T.
S and T are fields and

If

if

S is a subset of T, then S is called a subfield of T.

For exam-

the field of all real numbers is a subfield of the field of all complex numbers; the field of

ple,

all rational

numbers

is a subfield of the field of all real

numbers and the

field of all

complex

numbers.

MATRICES OVER A FIELD, When


'Mis over F". For example.
1

A =

1/4

1/2
2/3

elements of a matrix A are in a field F, we say that

all of the

and

over the rational field

is

11

- 3i

is over the

complex

field

Here, A is also over the real field while B is not; also A is over the complex field.

Let

A.B, C,

is,

let

F be

the smallest field

the elements are rational numbers, the field

if all

rational field and not the real or complex field.


fined

F and

be matrices over the same field

...

contains all the elements; that

An examination

which
is the

of the various operations de-

on these matrices, individually or collectively, in the previous chapters shows that no


F are ever required. For example:

elements other than those in

The sum,

difference, and product are matrices over F.

If

If

A ^l then

If

is non-singular, its inverse is

P and Q over F such

there exist matrices

is over the rational field

over F.

and

rank

is of

r,

its

that

PAQ

and

is over F.

rank is unchanged when considered over

the real or the complex field.

Hereafter when A is said to be over


taining all of its elements.

it

will be

assumed

that

is the

smallest field con-

In later chapters it will at times be necessary to restrict the field, say, to the
real field.
At other times, the field of the elements will be extended, say, from the rational field to the real
field.
Otherwise, the statement "A over F" implies no restriction on the field, except for the
excluded field of characteristic two.

SOLVED PROBLEM
1.

Verify that the set of all complex numbers constitutes a field.

To do

this

we simply check

the unit element (M^) is

product

(A/ 1)

is

1.

(a+bi){c +

If

the properties

a + bi

di) = (ac

and

di

A^-A^, Mi-Mg, and D^-D^. The zero element


are

-bd) + (ad + bc)i

a + bi

a^

bi

+62

two elements, the negative (A^) of a +


;

the inverse (M5) of a +

a^+

b^

hi

a^ + b^

Verification of the remaining properties is left as an exercise for the reader.

bi^o

is

bi

(/I4) is

is

-a-bi.

and
the

66

FIELDS

[CHAP.

SUPPLEMENTARY PROBLEMS
2.

Verify (a) the set of all real numbers of the form a + b\r5
(6) the set of all quotients

numbers and

Verify (a) the set of all rational numbers,

numbers a + bvS, where a and b are rational numbers, and


numbers a+bi, where a and b are rational numbers are subfields of the complex

(b) the set of all

(c) the set of all

4.

b are ra:ional

of polynomials in x with real coefficients constitute fields.

Q(x)

3.

where a and

Verify that the set of all

Show

where a and

that this is a subfield of the field of all 2x2 matrices of the form

5.

Why does not

6.

set

a ring.
ring.

2x2 matrices of the form

2x2

the set of all

b are rational

numbers, forms a

w]\eK a and

h are real

field.

field.

numbers.

matrices with real elements form a field?

of elements a,b.c.... satisfying the conditions {Ai, A^. A^. A^, A^; Mi.

D^, D^) of

M.y,

Page 64

is called

To emphasize the fact that multiplication is not commutative, R may be called a non- commutative
When a ring R satisfies Mg, it is called commutative. When a ring R satisfies M^. it is spoken of as

a ring with unit element.


Verify:

even integers 0,2,4,

(a) the set of

(b) the set of all integers


(c) the set of all
(d) the

...

0,+l,2,+3,

is an

n-square matrices over

set of all

2x2

example of a commutative ring without unit element.

is an example of a commutative ring with unit element.

...

is an

example of a non-commutative

matrices of the form

where a and

ring with unit element.

are real numbers, is an example of a

commutative ring with unit element.


7.

Can the set


ments 1

8.

Problem 6 be turned into a commutative ring with unit element by simply adjoining the ele-

(a) of

to the set?

By Problem

4,

the set (d) of Problem 6 is a field.

Is every field a ring?

element a field?

9.

Describe the ring of

10.

=
.

show

all

that

Let C be the field of


u, V are real

Is every

commutative ring with unit

numbers.

2x2

LA

all

To

matrices

= A.

Call

L a

ol
^

where a and

left unit element.

complex numbers p + qi and

Take the complex number a +

bi

b are in

F.

If

is

A.

any matrix of the ring and

Is there a right unit element?

be the field of

all

2x2

-b^

and the matrix

matrices

where

p, q,

^
as corresponding elements of

the two sets and call each the image of the other.
[2

-3l

To

-4l

2j

L4

Oj

(b)

Show

that the image of the

(c)

Show

that the

(d)

What

is the

(e)

What is the image of the inverse of

3+ ^^2i,

sum (product) of two elements of

image of the identity element

image

5.

of

of the conjugate of a + bi?

"!]
[:

This

is

is the identity

an example of an isomorphism between two sets.

is the

sum (product)

element of C.

of their

images

in C.

chapter 9
Dependence

Linear

and Forms

of Vectors

pair
of real numbers (%, x^) is used to denote a point Z in a plane. The same
2-vector
or
vector
two-dimensional
the
to
denote
used
here
will
be
as
x^],
written
[x^,
of numbers,

THE ORDERED PAIR

OX

(see Fig. 9-1).


X2

''^3(*11

^21..

^12 +

^2!

X.2(X^1. X22)

X(Xi. Xq)

Fig. 9-1

If

.Yi

= [%i,xi2]

sum (see Fig.

their

X^=

and

[x^^.x^q]

are distinct 2-vectors, the parallelogram law for

9-2) yields

A3

A^ + A2

^11

Treating X^ and Xg as 1x2 matrices, we see that this


en in Chapter 1. Moreover, if k is any scalar,
kX-i

L rC%'\

is the familiar multiplication of a vector

VECTORS. By

-1

"^

-^12

"^

-'^221

merely the rule

is

-^21

fv

X-y

for

adding matrices giv-

by a real number of physics.

an n -dimensional vector or re-vector

over

is

meant an ordered set

of n

elements x^

of F, as

(9.1)

The elements

x^,X2

Later we shall find

%
it

and

[a;i,x2, ...,%]

are called respectively the first, second,

...,

reth

components of X.

more convenient to write the components of a vector in a column, as

(91)

Now

I X-i

X2,

Xi

same vector; however, we shall speak of (9.1) as a row vector


We may, then, consider the pxq matrix A as defining p row vectors

(9.1) and (9.1') denote the


(9.1')

as a column vector.

(the elements of a row being the

components of a

67

17-vector) or

as defining g column vectors.

LINEAR DEPENDENCE OF VECTORS AND FORMS

68

The

whose components are

vector, all of

The sum and difference


tor are

of

zero, is called the zero vector and is denoted by 0.

two row (column) vectors and the product of a scalar and a vec-

1.

Consider the 3-vectors

(a)

2X-^-iX^

(6)

IX^+X^.

(c)

2X^

{d)

2X^ -

2[3,l,-4] - 5[2,2,-3]

2[2,2,-3]+ [-4,-4.6]

^3= [0,-4,1],

-^2= [2.2,-3],

A:i= [3,1,-4],

-3X2-^3

X^- Xq+

X^

are said to be linearly dependent over

[-4,-8,7]

that if

primed to denote col-

is

^mJ

|.%]^,%2'

each bracket

provided there exist

elements

h-^^.k^,

,k^

of F, not

such that
+

k.^X-1

(9.3)

Otherwise, the

k^X^^

k^X-^

vectors are said to be linearly independent.

Example 2. Consider the

Example

four vectors of

so also are X^, X^, and X^ by

The vectors X^ and


fel^l

Then

3fti + 2k^ =
and then ^2 = 0.

n-vector

+
0,

Xq^.

(c)

[Zk^ + 2k^,

+ 2^2 =

ft^

By

(6) the vectors

^2 ^"d

Xj^_

are linearly dependent;

{d).

however, are linearly independent. For. assume the contrary so that


=

;c2A'2

1.

and the entire set by

and

0,

k^+

-ik^ -

2k^, -'iJc.i_~ Zk^l


lik^

= 0.

Prom

[o,

0,0]

the first two relations A^ =

are linearly dependent.

and the n-zero vector

vector ^,B+i is said to be expressible as a linear combination of the vectors Ai, X^

there exist elements

k^, k^, ...,k^

Xfn+i

BASIC THEOREMS.

If

in (9.3),

^i
(9.4)

-I5]

[o,0.o] =

re-vectors over

Ai

Any

10,

LINEAR DEPENDENCE OF VECTORS. The m

all zero,

-S] - [lO,

[6. 2,

^4= [-4,-4,6]

and

The vectors used here are row vectors. Note


umn vectors, the results remain correct.

formed by the rules governing matrices.

Example

if

[CHAP.

=
Xi

k^

?^

0,

F such

of

%A]^ +

we may solve

- T"!^!'^! +
=

SiJVi

X^

that
:2A2

k^Xf^

for

+ ^i-i^t-i + -^i+i^i+i +

+ s^_iZi_i + s^j-i^i+i +

+ ^m^m!

or

+ s^J^

Thus,
I. If m vectors are linearly dependent, some one of them
a linear combination of the others.

may always be expressed as

CHAP.

LINEAR DEPENDENCE OF VECTORS AND FORMS

9]

II.

If

vectors X^, X^

ing another vector


bination of Xi, X^,

is

X^j,-^

, X^

Examples. Prom Example

X^ are linearly independent while the set obtained by addlinearly dependent, then Jt^+i can be expressed as a linear com-

If

X^.X^.^niXg
Zg=2A^i-3^2-

the vectors X^, and X^ are linearly independent while

2,

Xg=

linearly dependent, satisfying the relations 2X.i^-2X^-

III.

69

among the m vectors

X^ there

X^, X^

is

0.

Clearly,

a subset of

r<m

are

vectors which are

linearly dependent, the vectors of the entire set are linearly dependent.

Example

4.

By

(b) of

Example

the vectors

1,

X^ and X^ are linearly dependent; by

(d),

the set of four

See Problem

vectors is linearly dependent.

1.

IV. If the rank of the matrix

%2

Xi 1

m<n

(9.5)

associated with the

^n2

*TOl

vectors (9.2) is r<m, there are exactly

vectors of the set which

are linearly independent while each of the remaining m-r vectors can be expressed as a
linear combination of these r vectors.
See Problems 2-3.

V.
is

necessary and sufficient condition that the vectors (9.2) be linearly dependent

that the matrix (9.5) of the vectors be of rank

r<m.

If

the rank is m, the vectors are

linearly independent.

The
If

set of vectors (9.2) is necessarily linearly dependent

if

m>n.

the set of vectors (9.2) is linearly independent so also is every subset of them.

A LINEAR FORM

over

in

ra

variables
n

(9-6)

x^, x^

OiXi

is a

a^x^

polynomial of the type


+

a^x^

a^A:^

i=i

where the coefficients are


Consider a system of

in

F
linear forms in n variables
/l

CIu% + %2^2 +

(9.7)

1^1

/m

0~foXo

'^2n*re

%n^n

and the associated matrix

-'ml

If

there exist elements k^.k^

Kk

k^
+

^^^2/2

"m2

not all zero, in


+

...

A;4

F such
=

that

LINEAR DEPENDENCE OF VECTORS AND FORMS

70

[CHAP. 9

the forms (9.7) are said to be linearly dependent;

independent.
to the linear

Example

5.

otherwise the forms are said to be linearly


independence of the forms of (9.7) is equivalent
independence of the row vectors of A.

Thus, the linear dependence

dependence

The forms

/i =

or

2xi

- 2 +

3*g,

or

2%

x.^+

/2 =

ix^

/g =

4^=3.

Tx^ + Xg

are linearly depend-

2-13
A =

ent since

The system

-7

is of rank 2.

(9.7) is necessarily

dependent

Here,

if

3/^

"if^

fs =

m>n. Why?

SOLVED PROBLEMS
1.

Prove:

If

among

the

vectors X^,X^, ...,X^ there is a subset, say, X^,X^

linearly dependent, so also are the


Since, by hypothesis,

k^X-^ +

k^X^

k^X^ +

+ k^X^ +

X^,

r<m,

which is

vectors.

+ k^X^ =

with not all of the k's equal to zero, then

+ k^X^ +

0-.Y^+i +

0-.Yot

with not all of the k'a equal to zero and the entire set of vectors is linearly dependent.

2.

Prove:

If

the rank of the matrix associated with a set of

ra-vectors is

r<m, there are exactly

vectors which are linearly independent while each of the remaining m-r vectors can be expressed

as a linear combination of these

vectors.

Let (9.5) be the matrix and suppose first that m<n


If the r-rowed minor in the upper left hand comer
equal to zero, we interchange rows and columns as are necessary to bring a non-vanishing r-rowed minor
.

is

into this position and then renumber all rows and columns in natural order.

11

11?

21

25-'

Thus, we have

Consider now an (r+l)-rowed minor

*11

12

%-r Xiq

%1 %2

*rl X^2

xp^

where the elements xp; and


...,A;^+i =

by (3.10)

xj^q

Xpr,

Xqt x^q

^rr Xrq
.

xpf xpq

are respectively from any row and any column not included in A.

be the respective cofactors of the elements x^g. x^q

x^q. xpq,

Let h^,k^,

of the last column of V.

Then,

CHAP.

LINEAR DEPENDENCE OF VECTORS AND FORMS

9]

Now

+ krXri + ^r+i*^i

k^x^q + k^xQq +

and by hypothesis

u.

k2X2i +

fci^ii

= 1,2

(i

+ krx^q + krA-ixpq

71

r)

V be replaced by another of the remaining columns, say the column numbered


cofactors of the elements of this column are precisely the k's obtained above

column of

let the last

The

not appearing in A.

so that
k^x^n + ^2*2W +

+ ^rXru + ^r-n^pu

Thus,

and,

summing over

all

k^2t +

k^x^j;

values of

"

/i:,^+i

was any one


binatlon of

0,

ji^

Xp

is

a.

X^

k^X^ +

linear combination of the

of the m-r vectors

^j^,

(t

f'r-n'^pt

= 1,2

n)

t,

k^X^ + k^X^ +

Since

f'r'^rt

^I

^r+2

-V^+i,

k^^-^Xp

linearly independent vectors

hence, each of these

X-^^.

X^

X^.

But

^^j

may be expressed as a linearcom-

X^.

m>n.

consider the matrix when to each of the given m vectors m-n additional zero compoThis matrix is [^ o]. Clearly the linear dependence or independence of the vectors and
also the rank of A have not been changed.

For the case

nents are added.

Thus, in either case, the vectors


tors

3.

X-^.X^.

...,

X^

as

was

X^

Xr+^

are linear combinations of the linearly Independent vec-

to be proved.

Show, using a matrix, that each triple of vectors


X^ = [1,2,-3.4]

X-L

^2 = [3,-1,2,1]

(a)

and

^2=

(b)

In

1,-2]

[2, 3,

^3= [4,6,2,-3]

^^3= [1,-5,8,-7]
is linearly dependent.

= [2,3,1,-1]

each determine a maximum subset of linearly independent vectors and

express the others as linear combinations of these.

(a)

Here,

12-34
3-121
1-5

is of

rank

2;

there are two linearly independent vectors, say

-1

j^

Consider then the minor

Here

1-1
1-2
2-3
;

The minor

-3

3-12
1-5

respectively -14,

(b)

and X^

8-7
1

X.^

7,

and -7.

Then -1^X^ +

is of rank 2;

we interchange

The cofactors

of the elements of the third column are

1X2-1X2=

and Xg

-2X^ + X^.

there are two linearly independent vectors, say

the 2nd and 4th columns to obtain

The cofactors

of the elements of the last

column of

-1

-2

are

2-113
2-213
4-326

and

Xg

for

Xi +

X,

Now

-1

-2

which

2,2,-2 respectively. Then

4-32.
2X^ + 2X2 - 2Xs =

X^ and Xg.

the

5^0.

LINEAR DEPENDENCE OF VECTORS AND FORMS

72

4.

Let Pi(l, 1, 1), PsCl, 2, 3), PsiZ, 1, 2), and P^(2, 3, 4) be points in ordinary space.
and the origin of coordinates determine a plane tt of equation

X y z

12

The

points

Pi,

P^

1111
(i)

[CHAP. 9

2y +

1
1

Substituting the coordinates of P^ into the left


2

member

1111

1110

12

12

We have

The

tt.

Any

verified:

we have
4

significant fact here is that

[P^.,

Px.Pq

]'

is of rank 2.

three points of ordinary space lie in a plane through the origin provided the matrix

of their coordinates is of rank

Show

(i).

Thus, P4 lies in

of

2.

that Pg does not lie in v.

SUPPLEMENTARY PROBLEMS
5.

Prove:

If

vectors

-^m+i is linearly

6.

Show

X,^.

X^

are linearly independent while the set obtained by adding another vector

X-^

dependent, then ^m+i can be expressed as a linear combination of X^.X^

that the representation of /^^+i in

m
Hint: Suppose

^7^+1

Prove:

kiXi

1,

and consider

siXi

(A:^.

- s^ ) X^

i=i

i=i

necessary and sufficient condition that the vectors (9.2) be linearly dependent

(9.5) of the vectors be of rank

Hint: Suppose the

vectors are linearly dependent so that (9.4) holds.

converse, see Problem

is that the matrix

r<m.

product of the first row by

8.

is unique.

i=i

7.

Problems

X^^.

s^,

the product of the

second row by

In (9.3) subtract
S2.

from the ith row the

^s Indicated in (9.4).

For the

2.

Examine each of the following sets of vectors over the real field for linear dependence or independence. In
each dependent set select a maximum linearly independent subset and express each of the remaining vectors
as a linear combination of these.

^1
Xj_

[2,-1,3,2]

X^

[1,3,4.2]

X2 = [2,1.4]
(a)

(a)

Xq

2X^ - X^

[2,1, 3,2, -1]

^2 =

[4,2 1.-2 .3]

[0,0 5.6, -5]

(c)

(6)

Xa

[4,5,6]

Xs

^4

[1.8.-3]

X^

[6,3 -1,--6,7]

Xs

x^

2^1--X^
2X2--^1

X3 = [3,-5,2,2]

Ans.

^1

[1,2,1]

A3 = 2a^ +
(b)
A. A

A.Q

5a 1 2a o

(c)

CHAP.

9.

10.

LINEAR DEPENDENCE OF VECTORS AND FORMS

9]

Why can
Show

73

there be no more than n linearly independent -vectors over F'>

that if in (9.2) either Xi = Xj

X^ = aXj, a

or

in F. the set of vectors is linearly

dependent.

Is the

converse true?

11.

Showthatanyn-vectorZandthen-zero vector are linearly dependent; hence,


k^-O =
where fc^ = o and ftj ^ 0.

A"

ando are considered

proportional.

Hint: Consider k^^X +

12. (a)

Show

that

[l.l+i,i], X^ = [j,_j,i_i] and X^ = [l-i-2i,l-i, 2-j

X._ =

are linearly dependent over

the rational field and, hence, over the complex field.


(b)

Show

Zi

that

[l.l+i.i], X^ = [i.-i.l-i], and Xq = [o.l-2i.2-i]


complex field.

are linearly independent over

the real field but are linearly dependent over the

13. Investigate the linear

()

dependence or independence of the linear forms:

Xg + 2Xg + x^

/i

fz

2::i

+ 3x2 - Xg+ 2x^

/a

5x^

Arts, (a)

14.

3% -

3/i

95C2

2/2

+ 8xq -

2Xx

f^ =

3%-L

+ 2^2 -

fg =

5Xj^

- X2+ 2Xq +

fx

(b)

x^.

3Xg +

4A!g

- 2*4

2x3 + 5*4
X4.

/g

Consider the linear dependence

or

Hox

independence of a system of polynomials

+ aij^x

+ ai_^x

+ a;

(i =

1,2

m)

and show that the system is linearly dependent or independent according as the row vectors of the coefficient matrix
"10

"11

20

21

"^0

"ni

nn

are linearly dependent or independent, that is, according as the rank

of

is less than or

equal to

polynomials of either system are linearly dependent, find a linear combination which is identically

15. If the

zero.

Pi =
(a)

~ 3x2 + 4^ _

- 6 +

P2 =
Ps =

Ans.

16.

2x2

2*2 +

2Pi + Pg

(a)

Pj_

(b)

2x* +

P2 =

P^ +

(6)

P^-

2Pg

-4x^+5x

x +

X* +

Pg =

2P3 =

3:c^

2x-

+ 3

2x2- Zx +
x^ +

X + 2

Consider the linear dependence or independence of a set of 2x2 matrices M.

[::]-Ci]-[::]

over F.

Show

that

fe^il/i

A:2^^2

+ ^3^3 =

when

not all the k's (in F) are zero, requires that the rank of the

abed
matrix

g h

be <

3.

(Note that the matrices

components,)

Extend the result

to a set of

mxn matrices

M-^.Mz.Mq

are considered as defining vectors of four

LINEAR DEPENDENCE OF VECTORS AND FORMS

74

17.

Show

that

18.

Show

and

are linearly dependent.

3
-

any 2x2 matrix can be written as a linear combination

that

[CHAP.

of the matrices

[o

oj'

[o oj

and
[i

oj'

Generalize to nxn matrices.

X^

X^^.X^

19. If the ra-vectors

are linearly independent,

show

that the vectors

Y-^.Yr,

1^

where

7^

20. If

aijXj.

is of rank

where

21.

C^,

C^

(6)
(c)

Show
Show
Does

Show

if

and only

if

\_aij'\

is non-singular.

show how to construct a non-singular matrix B such that AB


C^ are a given set of linearly independent columns of A.

r,

Given the points Pi(l,


(a)

22.

are linearly independent

that the rank of [Pi, P3]' is


that [P^, P^. P3,

PaY

is

C2

C7-,

o]

Ps(2, 2, 2, 2), and /VO, 4. 5, 6) of four-dimensional space,


so that the points lie on a line through the origin.
of rank 2 so that these points lie in a plane through the origin,

Pjd,

1, 1, 1),

= [C^,

2, 3. 4),
1

P5(2, 3. 2. 5) lie in the plane of (6)?

that every n-square matrix

A^ +

A over F satisfies an equation of the form


k-^A^

+ kr,A^

+ kp^^A + kpl

...

where the k^ are scalars of F


Hint: Consider

23.

/,

4,^^,/!^

in the light of

Problem

16.

Find the equation of minimum degree (see Problem 22) which


(a)

Ans.

24. In

(a)

4^-24=0,

Problem 23(b) and

(b)

(c),

thus verify: If A over


cients are scalars of F.

L J,
[:;]

4^-24

is satisfied

A
'-[:-:]
(b)

+ 27 =

0,

(c)

\_

A^ - 2A +1

multiply each equation by 4"^ to obtain


is non-singular, then

A'

(c)

\.

by

[;:]
=

(b)

A'''^

I-^A.

(c)

A~'^=2l-A,

and

can be expressed as a polynomial in A whose coeffi-

chapter 10
Linear Equations

DEFINITIONS. Consider a system

of

linear equations in the n

022X2 +

OqiX-i +

(10.1)
"Wl*!"'" Ob2*^2 +

+ c!2n*-n

^2

Aa

OIj,

unknowns

xi.a;.

>

*r?

x^

in

in

which the coefficients

By

a solution in

(o's)

and the constant terms (A's) are in

of the system is meant any set of values of %,%2.

isfy simultaneously the

equations.

When

the system has a solution,

otherwise, the system is said to be inconsistent.


tion or infinitely

many

it

is

F which

sat-

said to be consistent;

consistent system has either just one solu-

solutions.

Two systems

of linear equations over F in the same number of unknowns are called equivevery solution of either system is a solution of the other. A system of equations equivalent to (10.1) may be obtained from it by applying one or more of the transformations: (o) interchanging any two of the equations, (b) multiplying any equation by any non-zero constant in

alent

if

F, or (c) adding to any equation a constant multiple of another equation. Solving a system of
consistent equations consists in replacing the given system by an equivalent system of prescribed form.

SOLUTION USING A MATRIX.

In matrix notation the

system of linear equations (10.1) may be written

as
^11

02

(10.2)

\'

1
02 2

*2

^2n

hm

r".
or,

more compactly, as

AX

(10.3)
?here

A =

[o^^-] is

Consider now

(10.4)

the coefficient matrix,

for the

xj\ and H

[x^.Xr,

[h^h

A^]'

system (10.1) the augmented matrix

Oil

ai2

"i?i

021

02 2

^271^2

ml

0^2

<^nn

[A

H]

(Each row of (10.4) is simply an abbreviation of a corresponding equation of (10.1); to read the
equation from the row, we simply supply the unknowns and the + and = signs properly.)

75

LINEAR EQUATIONS

76

[CHAP. 10

the system (10.1) by means of (10.4), we proceed by elementary row transformations


A by the row equivalent canonical matrix of Chapter 5. In doing this, we operate on
rows of (10.4).

To solve
to replace

the entire

Example

3xi +

Solve the system

1.

x^

2X2

^.X-^

Xg

2x-j^

+ ^Xq + 2xq

^Xq^

The augmented matrix VA H\

-2

-3

-1

-5 -5
-5 -5

-]

2'

"1

-1
1

1
1

Thus, the solution

is the

equivalent system of equations:

pressed in vector form, we have

= [l,

0, l]

1'

1
1

x-i

.0

OJ

.0

-11 -5 -5

0.
'1

0.

=1, 2

= 0,

xq =

1.

Ex-

FUNDAMENTAL THEOREMS.

When the coefficient matrix A of the system (10.1) is reduced to the


A:]'.
row equivalent canonical form C, suppose {A H] is reduced to [C K], where K= ^1,^5
If A is of rank r, the first r rows of C contain one or more non-zero elements. The first non-zero
element in each of these rows is 1 and the column in which that 1 sta,nds has zeroes elsewhere.

The remaining rows consist


the variables x:

bleS X:

Jr+1

If

X:

Jr+2

X,-

On

a solution.
0,

...

...

Prom

of zeroes.

r rows of [C K], we may obtain each of


Chapter 5) in terms of the remaining varia-

the first

(the notation is that of

,xj

Jr
X; and one of the i^, Uq
Jn

k^

= k

If

values for

kj; 7^

x:

then (10.1) is consistent and an arbitrarily selected set of

0,

together with the resulting values of

X.

ac,-

k^.

the other hand,

if at

least one of

V+i'

"r+s

'

%Ji
,,

x
,

...

J2
is different

X- constitute
Jr

from zero, say

the corresponding equation reads


Qx.^

and (10.1)

0%

+ 0*71

7^

is inconsistent.

A and

In the consistent case,

have different ranks.


I.

[A

have the same rank; in the inconsistent case, they

H]

Thus

A system AX

of

unknowns is consistent if and only


system have the same rank.

linear equations in n

the coefficient matrix and the augmented matrix

if

of the

r<n, re-r of the unknowns may be chosen


When these n-r
r unknowns is of rank r.
are
uniquely
determined.
unknowns
other
r
values,
the
whatever
assigned
any
unknowns are
II.

In a consistent

system (10.1)

of rank

so that the coefficient matrix of the remaining

3*^3

+ 3^2 +

Xi +

Example

2.

For the system

aci

2^1 +

2*2

5a:2

~ 4^4

xg

2x4.

2%3

5x^1

10

CHAP.

LINEAR EQUATIONS

10]

12-3-4
[A

12-3-4

13 1-2
2 5-2-5

H]

77

">-/

10_

-11

"l

-V

-11 -8 10

2-2
3-2

'Xy

2-2

10

_0

lO"

0-2

[C K]

10

^0

Since A and [A H] are each of rank r = 3, the given system is consistent; moreover,
the general solution contains n-r = 4-3 = 1 arbitrary constant. Prom the last row
of [C K], x^= 0. Let xs = a, where a is arbitrary; then aj-l = 10 + 11a and xq = -2-4o.

The solution

= [lO

of the

system

-2-4a,

+ llo,

a consistent system of equations over

If

is over

F.

If

by x^ = 10 + lla, xj =

is given

-2-4o,

xg = a,

x^,

or

a, o]'

the system has infinitely

has a unique solution (Example

many solutions (Example

2)

it

1) that

solution

has infinitely many solu-

F when the arbitrary values to be assigned are over F. However, the system has
many solutions over any field 2F of which F is a subfield. For example, the system
of Example 2 has infinitely many solutions over F (the rational field) if o is restricted
to rational
numbers, it has infinitely many real solutions if a is restricted to real numbers, it has infinitely
many complex solutions if a is any whatever complex number.
tions

over

infinitely

See Problems

NON-HOMOGENEOUS EQUATIONS. A
a-^

is called

non-homogeneous

equations provided

1-2.

linear equation

Xi + 0^X2 +

+ n*n

A 7^ 0. A system AX = H is called a system of non-homogeneous


zero vector. The systems of Examples 1 and 2 are non-homogeneous

if

is not a

systems.
In

Problem 3 we prove
ni.

A system

of

re

non-homogeneous equations

provided the rank of its coefficient matrix

in n

is n, that is.

unknowns has a unique solution


provided \a\ ^

0.

In addition to the

method above, two additional procedures for solving a consistent


system
non-homogeneous equations in as many unknowns AX = H are given
below. The first of
these is the familiar solution by determinants.
of n

(a) Solution

by Cramer's Rule. Denote by 4, (i = 1,2


n) the matrix obtained from A by replacing Its Jth column with the column of constants (the
h's). Then, if \A\ y^ 0, the system
AX = H has the unique solution

<1"'5)

777

X2

'

"'

See Problem
X2 + Sxg +

2xi

Example

3.

X2

Solve the system

We

*4

- 3x3 - 4x4
6x2 - 2x3 +
X4.

Xl +
3x:i

2%-^

+ 2*2 + 2x3

using Cramer's Rule.

- 3x4

find
1

-3 -4
-2 1

2-3

-120.

-3

-4

-2

-3

-240

4.

LINEAR EQUATIONS

78

-1 -3 -4

8-21

-24,

2-3
2

and

Then

-240

x-i

[CHAP. 10

15

1-1-4

2-3

-3 -1
-2 8
2

-96

2_

A^

-24

Ml

-120

= 0,

-120

and

-120

-96
^4

(b) Solution using

^.

-120

If

^ #

0,

AX

A~^ exists and the solution of the system

is

given

by

A-^-AX

(10.6)

A-^H

or

A-^H

2xi + 3X2 +

Example

The

4.

coefficient matrix of the system

From Problem

2(6), Chapter 7,

x^ +

2%

3^1 +

a;2

A
18

+ 3xg
+

2a:3

-5

-5

-5

A-^H

AX

18

-5

is

J_

Xg

-5

-5

Then
"35'

["9"

29

18
L8_

5_

solution of the system is x^ = 35/18, x^ = 29/18, x^ - 5/18.

The

See Problem

HOMOGENEOUS EQUATIONS. A
is called

homogeneous.

A system

"2*2 +

unknowns

+ ''n'^n

of linear equations

AX

(10.8)
in n

linear equation
'^1*1 +

(10.7)

5.

system of homogeneous equations. For the system (10.8) the rank


and the augmented matrix [A 0] are the same; thus, the system is
=
is always a solution; it is
Note that X = 0, that is, %i = xs = =

is called a

of the coefficient matrix A

always consistent.

called the trivial solution.


If

the rank ot

unique solution

is

xj^

r<n. Theorem

=
II

is n,

X2=

then n of the equations of (10.8) can be solved by Cramer's rule for the
= x^=
and the system has only the trivial solution. If the rank of

...

V.

Thus,

assures the existence of non-trivial solutions.

A necessary and sufficient condition


solution is that the rank of A be r < n.

IV.
trivial

for (10.8) to

have a solution other than the

necessary and sufficient condition that a system of n homogeneous equations in

n unknowns has a solution other than the trivial solution is

|/4

=
|

0.

VI. If the rank of (10.8) is r < n, the system has exactly n-r linearly independent solutions such that every solution is a linear combination of these n-r and every such linear
See Problem 6.
combination is a solution.

CHAP.

LINEAR EQUATIONS

10]

79

two distinct solutions of AX = H. Then AX^ = H, AX^ = H, and A (Xx- X^) =


Y = X^~ X^ is a non-trivial solution of AX = 0.

LET Iiand X^he


Thus,

then
Zy, +
'P

AY

0.

Conversely, if Z is any non-trivial solution of AX =


and if X^ is any solution of AX = H,
X = Xl+ Z is also a solution of AX = H. As Z ranges over the complete solution of AX = 0,
Z ranges over the complete solution of AX = H. Thus,
VII.

the system of non-homogeneous equations

If

AX

solution of the system is given by the complete solution of

AX

lution of

Example

5.

E
AX

is

consistent, a complete

plus any particular so-

= H.
i

system

In the

solution is

A:^,

2x2

Xi

Xj +

= [o,

+ 3x3

^2 + 2 Xg

set x^ = 0; then xg = 2 and x^ =

The complete

1, 2]'.

solution of < *^
\^Xi

where a

~
+

*^

Xq + 2*3 =

[-7a,a,3o]' +

[-7a,

[O, 1,2]'

IS

-^

Then the complete solution of the given system

is arbitrary.

1.

particular

[-7a,a,3oJ

is

+a, 2+3a]'

Note. The above procedure may be extended to larger systems. However, it is first
necessary to show that the system is consistent. This is a long step in solving the
system by the augmented matrix method given earlier.

SOLVED PROBLEMS
X2 ~ 2xs +

xi +

X4 + 3 K5

+ 2x4 + 6*5

+ 2 X2 - 4 Xg - 3 X4 - 9 xg

2i -

ail

^2 +

2%

tion

The augmented matrix

1-2
2-1 2

'l

[A H]

-4 -3 -9

"1

l'
2

3_

-3
-1

-v-

-6

8
3

-2
-2

0-1

1
'\J

0_

1-2

-18

-2

-18

-6
1

1
1

-2000
13

Then

1, x^- 2xs = 0, and


may be given as xx=

x^ -

solution

x^ +

Solve

^4

+ 33=5 =

x^^a,

X4

s - 2x4
+ 3xg

X2 + 2Xg +

5%

Take

2a,

2%i + 3x2 -

4xi +

0.

x^^

1.

xg = a and x^ = b. where a and b are arbitrary; the complete


x^ = -3b. x^ = b or as AT = [l, 2a, a, -3 6, 6]'.

Snliitinn*

'11
[A H]

The

last

3-1-22

row reads 0-xi +

'112

5"'

7
O-^tj +

"V-

1-5-4
1-5-4

-8
-13

1
1

13

-5 -4 -8
-5

O-^g + 0-4 = -5; thus the given system

is inconsistent

and has no solution.

LINEAR EQUATIONS

80

3.

A system AX

Prove:

provided \A\
If

7^

non-homogeneous equations

of n

is non-singular,
[/

K].

it

is

equivalent to

Then X

When A

/.

reduced by row transformations only to

is

is a solution of the

X = L is a second solution
non-singular, K = L, and the solution is

Suppose next that

4.

unknowns has a unique solution

in n

0.

[A H] is reduced to

Since A is

[CHAP. 10

/,

suppose

system.

AK

of the system; then

and

AL

H, and

AK

AL.

unique.

Derive Cramer's Rule.


Let the system of non-homogeneous equations be

(1)

il*l + a^jAiQ +

+ ain*n

h^

uqiXi + 022*2 +

+ 2n*n

^z

"ni*i + an2*2 +

+ "nn^n

^n

Denote by A the coefficient matrix

[a.

let a^,-be the cofactor of

and

the last equation

tion of (J) by ttn, the second equation by Ksi

S
X

which by Theorems

ai^di-^x-^ +

ai20iii% +

and XI, and Problem


hx ai2

22

^^2

^n

""no.

10,

Chapter

.S

1=1

i=i

i=l

3,

''is

Oqi

'^23

/12

\A\.x^

and add.
n

Multiply the first equa-

We have

ain^ilXn
1

=1

reduces to

"m
'^2n

so that

x-i

^11j-

^1

"n
n2 and sum to obtain

Next, multiply the equations of (i) respectively by a^i, ^ii


"11 ^1

in

ttni.

by

"m
"2n

so that

Continuing in this manner, we finally multiply the equations of

(i)

*<,

respectively by a^n. 2n

(^,71

and sum to obtain


Oil

"l,n-i "1

"21

"2,n-i ^2

so that

x^.

'ni

2Xi +
5.

Solve the system

X2 + 5 *3 +

*^4

X2 ~ 3x3 - 4*4
Xi +
:4
3 Xi + 6 Xj - 2 aig +
+
+
3x4
2a;3
2x1
2x:2

-1
using the inverse of the coefficient matrix.

Solution:

The inverse

of

15

1-3-4

3
2

6-2 1
2-3
2

120

-120'

120

-69 -73 17
120 -15 -35 -5

.24

80

40

-40.

Then

CHAP.

LINEAR EQUATIONS

10]

"120

120

.24
(See Example

Solve

40

1/5

-40.

"

-1

80

4/5

2.

3.)

s +

*4

i + 3*2 + 2xq +

4a;4

Xj^

Xl +
6.

5"

-120"

120

-69 -73 17
-15 -35 -5

81

^2 +

Xg -

2Xi

Solution:

\A

U\

-1

o"

1
'X/

1,

6 =

x^ =

What can be said

7.

1
''\J

i 1

The complete solution of the system is


we may obtain exactly n-r = 4-2
taking a =

0.

and then a =

X2 = -2, *3 =

3,

1,

11110
13

= 2

6 =

X4 =

-i

3
2

-^a + 16, ^^

x^ =

is 2,

first

-2 -1 -3

11110
1

o"

-! - |6, xs=a, x^ = h. Since the rank of


One such pair, obtained by

linearly independent solutions.


is

and

of the pair of solutions obtained

x^ = -1, x^ = -3, ^3 =

by taking a =

b =

and a =

3,

:x:4

b =

37

Prove: In a square matrix A of order n and rank n-1, the cofactors of the
elements of any two rows

(columns) are proportional.


Since

AX

1^1

(A'X =

=0,

the cofactors of the elements of any row (column) of

are a solution X-^ of the

system

0).

Now the system has but one linearly independent solution since A (A^) is of rank n-l.
Hence, for the
cofactors of another row (column) of A (another solution X^ of the
system), we have X^ = kX^.

8.

Prove:

If

/"i,

f^

f^ are

m<n

linearly independent linear forms over

in n variables, then the

linear forms

^j
are linearly dependent if and only if the

The g's

are linearly dependent

if

mxp

and only

0-=1.2

^^'ijfi-

p)

matrix [5^] is of rank r<p.

if

there exist scalars a^,a^

a.

in

not all zero, such

that

lgl + "2g2 +

+apgp

ai

^^Hxfi

.2

1=1

si^fi

.|

2
J=l

a;:S--)f.
J

^J

...

ap

.2

sj^fi

LINEAR EQUATIONS

82

[CHAP. 10

Since the /'s are linearly independent, this requires

j?/i^v

Now, by Theorem IV, the system of m homogeneous equations

trivial solution

9.

Suppose A =

[a-

apV

= [a^.a^,

] of order n

AB

order n such that

and

if

only if [^^j] is of rank

Show

is singular.

sider any one of these,

that there

in p unknovras

say AB^

1,

m)

always exists a matrix R =

Find

of

AB^

Then, by hypothesis,

B.

^65

..

AB^

ii&it + "iQ^'st +

"m ^nt

ni*it + n2^2t+

"nn^nt

Similarly,

AB^

= 0,

AB^

= 0,

+ *3

2^:5

...

(c)

3*'4

i>nt

Xj^

X^ +

/4ns.

(a)

! =

(b)

xi =

(d) Xi

11.

Find

\2x^

+ 5

1 + 2a
-7a/3 +
% = 1,

Xq +

Xg

2 Xg

3C2

:;3

x-i

1 x^

+ %2 +

^1 + %2 +

ajj

x:2

X\

-^

Xq

X-i

Xq +

~ 2%3
7^3

->=3

+ %4

-^S

x^ +

:>:4

-4

+ x^

Xr,

= o, X3 = b, x^ =

4a/3 =

5/3, Xg

2x^

*1 + 2*2

+ 3x3
(c)

<

12*1

2*1 +

+ 5*2 + 6*3
3*i

2x-i
(.h)

(6) *i =
(d)

2*1

*2
5

*i

*3
J.

*3

- 4*2 + 5*3

*i = -3o, X2 =

3,

"*'

+ 2*2 +

0,

2*1

*g = a

= a

*o =

*4

3*3

X2 + 3*3

4*1
X2 + ^x^

3*1 + 22 +
*i

(o)

of

Con-

has solutions other

all non-trivial solutions of:

(a)

/4ns.

-^

(d)

+ 3c,

17/3,

x^ +

2%i + 5%2
x-^

(b)

= 0.

have solutions, each being a column of S.

x-^

Xj_

?^

all solutions of:

{a)

[h^j]

= 0, or

SUPPLEMENTARY PROBLEMS
10.

has a non-

s^j xj -

r<p.

Since the coefficient matrix ^ is singular, the system in the unknowns h^t,^'2t

than the trivial solution.

0.

B be the column vectors

Let Bi.Bg

(i

+ "p'iP

^i^-ii

*3

+ 2*3 +

*3
^ *47*2 4*3 5*4
3

*2

11*5

1*3

5*4

CHAP.

12.

LINEAR EQUATIONS

10]

Reconcile the solution of 10(d) with another

A =

13. Given

6_

the solutions of

14.

15.

AX

AX

= c, x^ = d, x^ =

x-i

of rank 2

AB

such that

^c -

= 0.

Hint.

x^ =

c +d.
o

Select the columns of

B from

= 0.

Show that a square matrix


Let

find a matrix

83

and only

is singular if

if its

rows (columns) are linearly dependent.

be a system of n homogeneous equations in n unknowns and suppose A of rank r = n-1.


[a^i, a^j
OLinV of a row of ^ is a solution of AX = 0.

Show

that any non-zero vector of cofactors

16.

Use Problem

ixg
3%

\2x-^ + 5%2 + 6*3


Xg

^1

Hint.

third

To

row of

Ans.

17.

15 to solve:

- 2%2 +

(b)

the equations of (o) adjoin


"l

-2

+ 2x^

y'ix^

4%2

+ 2%3

Xg

Oxi + Ox^ + 0%3

(c)

%2 + 6 Xg

+ 3*2 + 4

2.^1

1 2xi

and find the cofactors of the elements of the

a""

-27a. X2

(a) xi =

/ 2xj_

9a

= 0, X3 =

[3a,

or

-a]',

0,

(6) [2a,

-7a, -17a]',

[lla, -2a, -4a]'

(c)

Let the coefficient and the augmented matrix of the system of 3 non-homogeneous
equations in 5 unknowns
be of rank 2 and assume the canonical form of the augmented
matrix to be

AX =H

1
1

613

614

fcj^g

ci

623

624

625

C2

_0

with not both of

AX

tion of

ci, C2

equal to

Then choose

= H.

First choose

0.

tain other solutions X^,Xg, and X^.

18. Consider

that

for (O,

19.

20.

IS a

AX

if

Hint. Follow

If ^ is an m xp
Use Theorem VI.

Prove:

x^ = xg =

1,

AX

0,

s,X^ + s^X^ + s^X^ + s^X^

and only

complete solution of

Prove: Theorem VI.

Hint.

21.

i^

the linear combination

is a solution of

X3 = x^ = x^ =

and obtain X^ = [c^, c^, 0, 0, o]' as a solualso X3 = x^ = 0, x^ = 1 and X3 = x^ = 0, Xg == 1 to obshow that these 5-2 + 1=4 solutions are linearly independent.

X3 =

if

s, +.2 +-3 +^4 =

(i)

r^

of the solutions

Thus, with s

of

Problem 17

s,. .3,

Show

.4 arbitrary except

= H.

Problem 17 with c^ = c^

matrix of rank

1-

and S

is a p

= 0.

matrix of rank

r^

such that

AB

= 0, then

r,

^-<

r^

n
f

Using the 4x5 matrix

.1 = [a^^-]
of rank 2, verify: In an x matrix A of
rank .. the r-square determinants formed from the columns of a submatrix consisting
of any r rows of A are proportional to the
r-square
determinants formed from any other submatrix consisting
r

Hint.
(7 =

of
rows of A
two rows are linearly independent so that a^j = p^.a^j
+Ps^a,.. a^.-p^a^,f^
ij +p42a2,ha2 2j.
j
Evaluate the 2-square determinants
j

Suppose the
1.2

5).

first

"17

ai5

\'^3q

agg

a^q

025

and
"4(7

"AS

22. Write a proof of the theorem of Problem 21.

'

^'"

'''''"''

''

f?ctors''hcJ?'"

(a)

where (h,i,j,h =

1,

n).

*'' ""''"'''' '""'"'

a^^-a^^

a^J^(x^,

'" ' '^"' ""'


(t)

*^'" "'' '''^'"^ ^^1^""^ ^"-""S

a^^aJJ

a^jUji

"^

LINEAR EQUATIONS

84

11114
123-4
21126

10

24.

Show

that

system

m>n

-3

-,,.

row equivalent to

ij

01000
00100
00010
00001

From B =[A H]

infer that the

[_0

unknowns has 5 linearly independent equations. Show that a system of


unknowns can have at most re +1 linearly independent equations. Show that when

of 6 linear equations in 4

linear equations in n

there are

25. If

IS

32-1-13

122-2

[CHAP. 10

ra

AX =H

the system is inconsistent.

is consistent

and of rank

r,

for

what set of

variables can one solve?

equations in n unknowns with coeffi26. Generalize the results of Problems 17 and 18 to m non-homogeneous
and the augmented matrix of the
coefficient
If
the
r
to
prove:
cient and augmented matrix of the same rank
Xn-r-n are
rank
r and if X-^^.X^
have
unknowns
=
in
n
equations
non-homogeneous
AX H of m

system

linearly independent solutions of the system, then

si^i + 51^2 +

^n-r+i^n-r+i

n-r+i

where

1,

a complete solution.

is

i=i

i and

27. In a four-pole electrical network, the imput quantities

2 and

Iq

by

E^

Show

6/2

"11

cE^ + dlQ

_/lJ

oEq +

'eI

h.

'b

I^,

and

I^,

and E^

AX

29. Solve the set of linear forms

Now

write

down the solution

of A'

'l

-1

X2

_s_

1"

0,

71

matrix with

72

AX

= K.

Show that the

for the x^ as linear forms in the y's.

p.
= E^,

(i

1,

^n^%

n).

where ^is the

Identify the matrix [S^, Sg

and let S^ be a solution of AX - E^, {i -1,2,.


and whose other components are 0. If K = \k^. k^.
k^Si + A:2S2

is a solution of

have a unique solution.

m<n

m-vector whose ith component is

>-

"n"

'xi

A be n-square and non-singular, and let S^ be the solution of AX


n-vector whose ith component is 1 and whose other components are 0.
Let 4 be an m X

'e2

30. Let

31.

Let the system of n linear equations in n unknowns 4X = H, H ^


system AX = K has a unique solution for any n-vector K ^ 0.

and

that

Solve also for 2 and

28.

are given in terms of the output quantities

m).
,

S^].

where E^ is the

k^' show
,

that

Chapter

11

Vector Spaces

UNLESS STATED OTHERWISE,


played,

we

all

now be column vectors. When components are disThe transpose mark (') indicates that the elements are

vectors will

shall write [xj^.x^

a^]'.

to be written in a column.

A
them
if

F is said to be closed under addition if the sum of any two of


Similarly, the set is said to be closed under scalar multiplication

set of such 7z-vectors over

is a vector of the set.

every scalar multiple of a vector of the set is a vector of the set.

Example

1. (a)

The set of

all vectors [x^. x^. x^Y of ordinary space havinr equal


components (x-^ = x^^ x^)
closed under both addition and scalar multiplication. For, the sum
of any two of the
vectors and k times any vector (k real) are again vectors having equal
components.
is

(6)

The set

of all vectors

[x^.x^.x^Y of ordinary space

is

closed under addition and scalar

multiplication.

VECTOR SPACES. Any

set of re-vectors over

plication is called a vector space.


linear combinations

<"!)

K^i

is a vector

F which

Thus,

if

is

X^, X^

KX^

closed under both addition and scalar multiX^ are n-vectors over F, the set of all

KX^

(kiinF)

space over F.

For example, both of the sets of vectors (a) and (b) of Example
1 are
Clearly, every vector space (ll.l) contains the zero
re-vector while the zero
re-vector alone is a vector space. (The space
(11.1) is also called a linear vector space.)
vector spaces.

The

totality V^iF) of all re-vectors over

is called the re-dimensional vector

space over

F.

SUBSPACES. A

set V of the vectors of V^(F) is called a subspace


of V^(F) provided V is closed under addition and scalar multiplication. Thus,
the zero re-vector is a subspace of F(F)- so
also
IS V^(F) Itself. The set (a) of Example
1 is a subspace (a line) of ordinary
space. In general
If X^, X^, ..., Z^
belong to V^(F), the space of all linear combinations
(ll.l) is a subspace of

vector space V

vided (a) the Xi


vectors X^, X^

lie in

X^_

is

said to be spanned or generated by the re-vectors


(b) every vector of F is a linear
combination

V and

X^,

X^

(11.1).

X^ proNote that the

are not restricted to be linearly independent.

Examples. Let F be the field R of real numbers


so that the 3-vectors
X^
^3= [1.3,2]' and X^= [3,2,1]' lie in ordinary space S =

[i.i.lY X^
Any vector

V^R).

s can be expressed as

85

2 i]'

\i

cY

\a
b
,

ot

VECTOR SPACES

86

[CHAP. 11

3y4.

+ 2X2 + Sys +

274.

y2 +

yi +

+ 72^2 + ys'^s +

Ti ^1

yi

y4-^4

3y2 +

yi +

ys

2y3

Ja

since the resulting system of equations

yi +

y2

yi +

(i)

yi

Thus, the vectors

is consistent.

ys

2y2 +

3y2

Xj^,

3x4

3ys + 2y4

2y3

Xj. Xg,

y4-

X4 spanS.

and ^2 are linearly independent. They span a subspace (the plane


real numbers.
S which contains every vector hX^ + kX.^. where /i and k are

The vectors

of

tt)

X-y

The vector X^ spans

hX^. where
a subspace (the line L) of S which contains every vector

A is a real number.

See Problem

BASIS

1.

maximum number of linthe dimension of a vector space V is meant the


number of linearly inminimum
same thing, the
early independent vectors in F or, what is the
is considered as
space
ordinary
geometry,
dependent vectors required to span V. In elementary
it as a
considering
been
have
we
Here
(a, 6, c).
a 3-space (space of dimension three) of points
is of
line
L
the
and
is of dimension 2
3-space of vectors ia,h,c \. The plane n of Example 2

AND DIMENSION. By

dimension

A
we

1.

vector space of dimension

consisting of 7i-vectors will be denoted by F(F).

When

n,

shall agree to write p;(F) for %\F).

basis of the space. Each veclinearly independent vectors of V^(F) is called a


of this basis. All bases of
vectors
the
combination of
tor of the space is then a unique linear
independent vectors of the
linearly
r
V^(F) have exactly the same number of vectors but any

set of

space will serve as a basis.


Example

The vectors

3.

X^.

Xr,.

X^

of

X-y
71-^1

Example

span S since any vector [a,

+ j^Xq + yg^a

yi

ys

ys

yi

2y2

3ys

3X2

2ys

yi +

ys =

,^ + 3X3 =
yi + 2y2

3X2 + 2xs =

!yi
xi
The

resulting system of equations

Xi+

y2 +

b. c ]'

of S can be

unlike the system

expressed

i),

has a u-

are not a
The vectors X^.X^.X^ are a basis of S. The vectors X^.X^.X^
X^. X^
set
the
basis
is
whose
tt
Example
2,
of
(Show this.) They span the subspace

nique solution.
basis of S

Theorems I-V

of

Chapter 9 apply here, of course.

In

particular,

Theorem IV may be

re-

stated as:
I

If

X X^

..

X^

the rank of the raa:ro matrix


Unearly independent vectors may be selected. These

are a set of re-vectors over

F and

if r is

of their components,' then from the set r


the remaining m-T vectors lie.
T vectors span a V^iF) in which

See Problems

2-3.

CHAP.

VECTOR SPACES

11]

Of considerable importance are


II.

If

^m4.2. .

JYi,

^n

^2,

...,

Zm

m<n

are

linearly independent n-vectors of

are any n-m vectors of V^iF)

independent set, then the set

87

X^

X^,

which together with

X.^,

X^

V^iF) and

J^+j^,

if

X^ form a linearly

is a basis of V^iF).

See Problem
III.

If

Z^.Zg,...,! are m<7i linearly independent -vectors over F, then the


p vectors
m
^i

are linearly dependent

IV. If

4.

Zi,

.2, ...,

p>m

if

or,

i^'ij^i

when p<m,

if

(/=l-2

P)

[s^,] is of rank r<p.

are linearly independent re-vectors over F, then the vectors

n
Yi

=
7

are linearly independent if and only

IDENTICAL SUBSPACES.
only
is

If

X(F)

,V^(F) and

each vector of X(F)


a subspace of the other.
if

if

a^jXj
"

(i

= 1,2

re)

[a^-] is nonsingular.

are two

subspaces of F(F), they are identical if and


and conversely, that is, if and only if each

is a vector of ^V;[(F)

See Problem

SUM AND INTERSECTION OF TWO SPACES.


sum

is

meant the totality of vectors

5.

Let V\F) and f3^) be two vector spaces. By their


where X is in V^(F) and Y is in V^iF). Clearly, this

X+Y

is a vector space; we call it the sum space


V^^iF).
The dimension s of the sum space of two
vector spaces does not exceed the sum of their dimensions.

By

the intersection of the two vector spaces is meant the


totality of vectors common to the
Now if Z is a vector common to the two spaces, so also is aX;
likewise if X and
y are common to the two spaces so also is aX^bY. Thus, the intersection of two spaces
is a
vector space; we call it the intersection space V\F).
The dimension of the intersection space
of two vector spaces cannot exceed the smaller of
the dimensions of the two spaces.

two spaces.

V. If two vector spaces FV) and V^(F) have


V^\F) as
section space, then h + k = s + t.

sum space and V^(F) as

inter-

Example 4. Consider the subspace 7f, spanned by X^ and X^ of


Example 2 and the subspace tt^ spanned
by Xs and X^. Since rr^ and tt^ are not identical (prove this)
and since the four vectors span
S, the sum space of tt^ and tt^ is S.

Now 4X1 - X2 = X^: thus, X^ lies in both tt^ and 7t^. The subspace (line
L) spanned
by X^ IS then the intersection space of 77^ and 77^
Note that 77^ and 77^ are each of dimension
2, S IS of dimension 3, and L is of dimension 1.
This agrees with Theorem V.
.

See Problems 6-8.

NUIXITY OF A MATRIX.

For a system of homogeneous equations AX = 0, the


solution vectors X
constitute a vector space called the null space of A.
The dimension of this space, denoted by
^
A'^
IS called the nullity of A.
,

Restating Theorem VI, Chapter


VI.

If

A has

nullity N^

10,

then

we have

AX

has N^ linearly independent solutions X^. X^

VECTOR SPACES

88

Xti
A

AX

such that every solution of

[CHAP. 11

is a linear

combination of them and every such

linear combination is a solution.

basis for the null space of A is any set of N^ linearly independent solutions of

AX

0.

See Problem

Vn. For an

mxre matrix

of rank rj

(11.2)

rA

SYLVESTER'S LAWS OF NULLITY.

AB

>

of order

'^

Nab > Na

(11.3)

Nab

El = [1,0,0

N/^,

and respective ranks q and

ra

rg

the

satisfy the inequalities


''AB

BASES AND COORDINATES. The

Nji

A and B are

If

rank and nullity of their product

and nullity

9.

<

- "

'B

Nab > Nb
Na + Nb

See Problem

10.

ra-vectors

1]'

En = [0,0,0

0]',

E^ = [0,1,0

0]',

are called elementaiy or unit vectors over F. The elementary vector Ej, whose /th component
The elementary vectors E^, E^
constitute an
is 1, is called the /th elementary vector.

important basis for f^^Cf).

Every vector

uniquely as the
^nl' of 1^(F) can be expressed

[%,%

sum

1=1

Of the elementary vectors.

X
X

XxE-^

xiEi

The components %,

2-^2

x^ oi

x^

are

now called the coordinates of


we shall assume that a vector

relative to the E-basis. Hereafter, unless otherwise specified,


is

'^nEr,

given relative to this basis.

Zn be another basis of 1^(F).

Let Zi, Zg
in F such that

exist unique scalars

Then there

%,

03

a^

X
i

These scalars
JY^

= [oi, og

is the matrix

If

02

Zg +

o are called the coordinates of


a^Y, we have

Examples.

01,05

(11.4)

where Z

a.^Z.^

aiZi

Zi =

Z^ = [l,

2,

-1

]',

Z3

= [l, -1,

dj^Zj^

relative to the Z-basis.

-1

Zi,

]'

Writing

Z-Xz

Zn]Xz

[Zi.Zg

whose columns are the basis vectors

[2, -1, 3]',

=i

is

Z.

Zg

a basis of

Fg

(F) and

Xz

[1.2.3]'

is a vector of Vg(F) relative to that basis, then

[Zi, Zg, Zg]A:^

-1

-1

relative to the -basis.

-1 -1

"1

[7,0,-2]'

-2
See Problem 11.

CHAP.

VECTOR SPACES

11]

Let

W^,

be yet another basis of f^(F).

(11.5)

From

(11.4) and (11.5),

=
=

X^

(11.6)

where P =

JV^

IT

Suppose

WX^

\_\,\

89

Zj^

-r

K\' so

= [61,^2

that

Xy

and

^'^.Z-X^

Xig

PX,

IF"^Z.

Thus,
VIII.

If

a vector of f^(F) has coordinates Z^ and X^ respectively relative to two bases

of P^(F), then there exists a non-singular matrix

given by (11.6) such that

Xf/

P determined
,

solely by the two bases and

= PX^

See Problem

12.

SOLVED PROBLEMS
1.

The

set of all vectors

A'

= [%,

x^, Xg,

x^Y,

where x^ + x^ + x^ + x^ = Q

is

a subspace V of V^(F)

since the sum of any two vectors of the set and any scalar multiple of a vector of the set have

components whose sum

2.

is zero, that is, are

vectors of the set.

Since

is of rank 2, the vectors

X.^

= [1,2,2,

1 ]',

X^ = [3,4,4,3]', and X^ = [1,0,0,

are linearly dependent and span a vector space

(F).

Now any two of these vectors are linearly independent; hence, we may take
and Xg as a basis of the V2{F).

14
13 12
12
2

3.

Since

1 ]'

X-^

and X^, X^ and

A:g.

or

X^

is of rank 2, the vectors

^"1

= [1,1,1,0]',

A's

= [4,3,2,-1

]',

A'g

= [2,1,0,-1]',

-1 -1 -2
and

^"4

= [4,2,0,-2]' are linearly dependent and span a V^(F).

For a basis, we may take any two of the vectors except the pair Xg, X^.

4.

The vectors

X^, X^, Xg of

Problem

2 lie in V^(F).

Find a basis.

For a basis of this space we may take X^,X^,X^ = [l.O.O.O]', and Xg = [o, 1,0,0]' or X^.X2.Xg =
since the matrices [X^, X2, X^.Xg] and [X^. X2.Xg. Xy] are of rank
[1.2,3.4]'. and X; = [1,3,6,8]'
4.

5.

[CHAP. 11

VECTOR SPACES

90

Let Zi = [l,2,l]', Z2 = [l,2,3]', A:3 = [3,6,5 ]', Y^ = [0,Q,lY, i'2=[l.2,5]' be vectors of


Show that the space spanned by X-^, X^, Xq and the space spanned by Y.^, Y^ are identical.

we

First,

note that X^ and X^ are linearly independent while

of dimension two. say

2Zj^

X^..

Thus, the X^ span a space

Also, the Yi being linearly independent span a space of dimension two. say

iF|(''').

Thus, any vector


iX.^, Y^ = 2^2- ^li ^i = ^i^ - 4^1. X^ = Y^- 271.
(50 + 26)^2 - (2a + b)X.^ of iPg^CF) and any vector cXj^ + dX^ of iff(f)
+ d)Y2 - (4c + 2<i)Ki of QVi(F). Hence, the two spaces are identical.
Fi =

Next,

2lg^(F)
(c

Xq

6. (a) If

kXz -

is

a vector

= [%, 2.%]' lies in the Vg(F) spanned by

-2% +

Vs(F).

X^ =

aY-^

fcK,

1,-1,1]' and X^ = [3,4,-2]',

of

is a vector

then

5X2 + IXg

(b) If

= [xi,x^,X3,x^Y lies in the Pi(F) spanned by X^ = [1,1,2,3]' and X^ =

3Ci

-2

%4.

xi

4.

%
1

Since

2.

= - 2*1 + 4% -

this requires

0,

then

1,0,-2,1]',

and

%+

2*2 -

s;*

These problems
linearly independent

7.

rank

is of

verify: Every ^(^ ) may be defined as the totality of solutions over F of a system
homogeneous linear equations over F in n unknowns.

of n-k

two vector spaces Vn(F) and I^(F) have Vn(F) as sum space and V^iF) as intersection
space, then h + k = s + t.
Prove:

If

Suppose
s

= h; then Vr!XF) is a

h+k. The reader

will

show

subspace of (^^F) and their sum space


that the

same

Suppose next that t<h. t<k and let X^^.X^


Yh so that X1.X2
^t.i't-n
A:i,A:2
^t.Zt+1
2fe span I^'^F).
^t^-i, yt-+2

Now suppose

is true if

is 1^' itself.

X^ span

=h

and

Then by Theorem H there exist vectors


Z^ so that
Zj^^. Z^+j

V^(F).

span l^(F) and vectors

Yh

X
1=1

"iXi

.S

a^Yi

t=t+i

ft

i=i

i=t+i

biZi

i=t+i

i=t+i

The vector on the


V^(F). But X^.X^

left

belongs to P^(F),and from the right member, belongs also to ^(F); thus

X-t

span Vn(Fy, hence,

from (11.4),

2
i=i

a^+i = at+2 =

"f,

it

belongs

= 0.

Now

there exist scalars a's and 6's such that

(11-4)

to

Thus, s=k.

= k.

"iXi

2;

b^Z^

i=t+i

But the X's and Z's are linearly independent so that a^ = 05 = = t =


the ^f's.y-s, and Z's are a linearly independent set and span ^(F). Then

''t^i =
s

t*^"^,
*t+2 = = ^fe =
as was to be proved.

=h + k-t

CHAP.

8.

VECTOR SPACES

11]

Consider ^FsCF) having X^ = [l,2,2Y and Z2 =

1,1,1

]'

91

as basis and jFgCF) having 71 = [3,1,2]'

113
2 110
1

and

Fj

1.0,1

]'

as basis. Since the matrix of the components

12

space

is V,(F).

Prom

h + k

As a basis, we may take


= s

t.

X-^,

and

X^,

Y^

To

the intersection space is a VsiF).


2

sum

is of rank 3, the

find a basis,

we equate

linear combinations

of the vectors of the bases of iFg(F) and s^'sCF) as

- 3e

take d =

aX;]^

9.

for

convenience, and solve

+ 6^2 = [-1.-2/3.-1/3

Determine a basis

]' is

for the null

^2a

6-

( 3a

- 2e

a basis of the intersection space.

space of A

Consider the system of equations

AX

a = 1/3,

obtaining

113

10

113

The vector [3,2,1]'

basis for the null space of

.4

c =

-2/3.

Then

is also a basis.

x^ =

which reduces

to

\x2+
A

6 = -4/3,

Xs + 2x^ =

independent solutions [1.2,0,-1]' and [2,l,-l,o]'

is the pair of linearly

of these equations.

r >

10. Prove:

AB

Suppose

r
A

- n.

first that

A has the form

the remaining rows are zeros.

PAQ

By Problem

r,

rows of

Chapters, the rank of

10,

the special

Solution

(a).

Find

AB

AB

is

are the first

^45

>

+ %

'k

r.

rows of B while
"

aZ^ + 6Z2 +

Thus

coordinates relative to a new basis Zi =

its

cZr^.

that is.

+ h

c = 2.

[l 1 0]'

Write
1

case when

Let X=[l,2.lY relative to the S-basis.


Z2 = [1,0,1]', and Zg = [1,1. l]'.

(i)

first

Suppose next that A is not of the above form. Then there exist nonsingular matrices P and
Q such that
has that form while the rank of PAQB is exactly that of AB (why?).

The reader may consider

11.

Then the

relative to the Z-basis,

we have X^

!a

[0,-1,2]'

Then

+ i + c =

a
b

and a =

0,

6 =

1,

VECTOR SPACES

92

Rewriting

Solution (6).

(i)

as

-1

X^

Z X

-1

Let X^ and

be the coordinates of a vector


Z3= [1,1,1]' and f4 = [l,l,2]',

X-^

Z2=[l,0,l]',
trix ? such that X^ = PX^

Then

Zg]

\_Z^, Z^,

W'^Z

1
1

-1

_ _

with respect to the two bases

ff^

= [2,2,1

]',

Ifg

= [1,2,2

]'.

7,^

= [1,1,0]'

Determine the ma-

Here

[0,-1,2^

2
1
.

12.

11

we have

{Z-^.Z^.Z^^X^ = ZXg,

[CHAP.

'l

l'

2-3 2
0-1
2

and

-3

by (11.6).

0-3

SUPPLEMENTARY PROBLEMS
13.

Let

[x-i^.

x^. x^.

X4,y

be an arbitrary vector of Vi(R). where

R denotes

the field of real numbers.

Which of the

following sets are subspaces of K^(R)'?


(a)

All vectors with Xj_= X2 = X3 = x^.

(6) All vectors


(c) All

with x^ = x^.

vectors with %4 =

Ans. All except {d) and

x^=2x^.

(d) All vectors with

x-^

(e) All vectors with

x^.Xr,.Xs.xj^

and [2,3.3,2]' are a basis of the

fi^(F) of

Problem

Determine the dimension of the vector space spanned by each set of vectors.

]'

integral.

2.

15.

1.1.1.1

0.

Show

(e).

14.

that

Select a basis for each.

[1.1,1.1]'

[l.l.O.-l]'

[1,2,3.4.5]'
(b)

[5.4,3,2,1]',

(a)

16. (a)

(6)

[3,4,5,6]'
^'^

[2.3.3,3]'

[1.1,1.1,1]'

Ans.

[1,2,3,4]'

(a), (b), {c).

[1.2,3,4]'

[1,0,-1,-2]'

r= 2

Show that the vectors X-^ = [l,-l,l]' and X^ = [3,4,-2]' span the same space as Y^ = [9,5,-1 ]' and
72= [-17,-11.3]'.
Show that the vectors X^ = [ 1,-1,1 ]' and A'2 = [3,4,-2 ]' do not span the same space as Ti = [-2,2,-2]'
and K, = [4,3,1]'.

n. Show

that if the set

X^.X^

Xfe is a basis lor Vn(F). then

sented uniquely as a linear combination of

X-^,

Hint.

Assume

51

aiXi

biXi-

X^

X^

any other vector Y of the space can be repre-

CHAP.

18.

VECTOR SPACES

11]

4x4

Consider the

93

matrix whose columns are the vectors of a basis of the Vi(R) of Problem 2 and a basis of
3. Show that the rank of this matrix is 4; hence. V^R) is the sum space and l^(R), the

the \i(R) of Problem

zero space, is the intersection space of the two given spaces.

19.

Follow the proof given

20.

Show

Problem

in

that the space spanned by [l,0,0,0,o]', [0,0,0,0,1 ]', [l.O,l,0,0]', [0,0,1,0,0]' [l,0,0,l,l]' and the
space spanned by [l,0.0.0,l]', [0,1,0,1,0 ]', [o,l,-2,l,o]', [l,0,-l,0,l ]', [o,l,l,l,o]' are of dimensions
4 and 3, respectively. Show that [l,0,l,0,l]' and [l,0,2,0,l]' are a basis for the intersection space.

Z^=

21. Find, relative to the basis


(a)

[l.l.o]',

Ans.

[1,0, l]',

(b)

(a) [2,-1,0]',

Ans.

Let X^ and
trix

Zi=

PX^

[l/3, 1/3,

]'

(c) [0,0,l]'.

[-6,7,-2]',

(6)

that Xj^ =

(c)

]',

Zi=[o,l,o]', Z2=[i,l,l]', Z3=[3,2,l]' the coordinates of the vectors


(c)

[-1/2, 3/2, -1/2

]'

with respect to the given pair of bases.

[1,0,0]',

Z2=[i,o,l]',

Z3=

[1,1, il-

= [0,1,0]',

[^2= [1,2,3]',

If'3=

[1,-1,1]'

Zi = [0,1,0]',
^^^

1^1

(a)

2j

is a solution of

AX

= [l,1.0]'.

2^

23

= [1.1,0]',

1^2= [1,1.1]',

= [1.2.3]'

1^3= [1,2.1]'

-2"1

-1

!fi

ij

n
=

Ej

(j = 1,2

n).

then

hjPj

is a

solution of

AX

= H.

where H =

KV-

[''1.^2

Hint.

(6)

24. Prove: If Pj

41

Ans.

Determine the

ea)

25.

-1

be the coordinates of a vector

X^^

P such

[l.l.l]'.

(6) [4/3, 1/3,

(b) [1,-3,5]',

(a) [-2,-1,1]',

[l,1.2]', Zg = [2.2,l]', Zg = [l,2,2]' the coordinates of the vectors

(c)

[-1/3,2/3,0]',

(a)

22. Find, relative to the basis

23.

Chapter 10, to prove Theorem HI.

8,

h^Ej^ +

h^E^+

+hnE^.

The vector space defined by all linear combinations of the columns of a matrix A
is called the column space
of A. The vector space defined by all linear combinations of the
rows of A is called the row space of ^.
Show that the columns of AB are in the column space of A and the rows of AB are in
the row space of

26.

Show

that

AX = H a system of m non-homogeneous
H belongs to the column space of A

equations in n unknowns, is consistent

the vector

27. Determine a basis for the null

space of

Ans.

28. Prove:

Hint:

[1,-1,-1]',

(a)

(a)

r^g

5.

r^g

Consider n~r^g

29. Derive a procedure for

Problem

1,1,-1, -i

N^,>N^. N^^>N^

(a) /V^g
(b)

(6)

<

and only

if

the

1111

1-1,

(a)

if

fi.

(6)

12 12
3

[l, 2,-1, -2]'

]',

(b)N^,<N^^N^
r^

and

rg

using the theorem of Problem 10.

Problem 16 using only column transformations on A = [X^. X^, y^ Y^].

Then resolve

chapter 12
Linear Transformations

DEFINITION. Let

%]' and Y =

[x,., x^, ....

JnY ^^ ^^ vectors

lyx. y^.

ordinates being relative to the same basis of the space.

of l^(F), their co-

Suppose that the coordinates of

are related by
yi

'^ll'^l

"^

^12 "^2

"^

^^Yi J^

df^Yj.^ri

(12.1)

AX

or, briefly,

where A =

[a^.-]

Then (12.1) is a transformation T which carries any vector X


Y of the same space, called its image.

is over F.

of

V^(F) into (usually) another vector

(a)
(fe)

If

(12.1) carries X^ into F^ and X^ into Y^, then

it

carries kX^ into ^y^, for every scalar k, and

it

carries aX^ + bX^ into aY^ +

every pair of scalars a and

feFg. for

b.

For this reason, the

transformation is called linear.

Example

Consider the linear transformation Y =

1.

(a)

(6)

The image

of

A"

= [2,0,5]' is

The vector X whose image

112
Since

BASIC THEOREMS.
if

then
I.

If in (12.1),

Y =

12
13

[a^j.a^j

5
3

is

= [\,Q
"nf]'-

'l

13

in ordinary

A"

space Vq(R).

'12"

27

[12,27.17]'.

17

= [2,0.5]' is obtained

10
10

AX

by solving

'l

'x{

*2

.1

3_

3.

'2'

=
5

13/5
11/5

[13/5,11/5,-7/5]'.

-7/5

0]'='i then
Hence,

Y =

an,

linear transformation (12.1) is uniquely determined

Ogi,

...,

a^J'

and, in general,

when the images (Y's)

of the

basis vectors are known, the respective columns of A being the coordinates of the images

See Problem

of these vectors.

94

l.

CHAP.

LINEAR TRANSFORMATIONS

12]

95

if the images of distinct vectors Xi


Otherwise the transformation is called singular.

linear transformation (12-1) is called non-singular

are distinct vectors Y^.

II.

linear transformation (12.1) is non-singular if and only if A, the matrix of the

transformation, is non-singular.

in.

See Problem

2.

non-singular linear transformation carries linearly independent (dependent) vec-

tors into linearly independent (dependent) vectors.

See Problem

3.

Prom Theorem HI follows


k

IV. Under a non-singular transformation (12.1) the image of a vector space Vn(F) is a
vector space VjJ,F), that is, the dimension of the vector space is preserved. In particular,
the transformation is a mapping of ^(F) onto itself.

When A

is non-singular, the inverse of (12.1)

A'^y

carries the set of vectors Y^, Y^, ...,\ whose components are the columns of A into the basis
vectors of the space. It is also a linear transformation.

V. The elementary vectors ^ of \{F) may be transformed into any set of n linearly
independent n-vectors by a non-singular linear transformation and conversely.
VI. If

CZ

IT

into

Y = AX carries a vector X into a vector F, if Z = BY carries Y into Z, and if


Z into W, then Z = BY = {BA)X carries X into Z and IF = (CBA\X carries

carries

IF.

VII. When any two sets of re linearly independent re-vectors are given,
there exists a
non-singular linear transformation which carries the vectors of one set into the
vectors of

the other.

CHANGE OF

BASIS. Relative to a Z-basis, let 7^ = AX^, be a linear transformation of ^(F). Suppose


changed and let X^ and Y^ be the coordinates of X^, and Y^ respectively relative to the new basis. By Theorem VIH, Chapter 11, there exists
a non-singular matrix P such
that X-^ = ?X^ and Yy, = PY^ or, setting ?~^ = Q, such that
that the basis is

X^

Then

Y^

Q-%

QX^
=

and

Q-^X,

Y^

QY^

Q-^AQX^

BX^

where

(12.2)

Two

Q-^AQ

matrices A and B such that there exists a non-singular matrix


We have proved

for

which B = Q'^AQ

are called similar.

vm. If Y^ = AX^ is a linear transformation of V^(F) relative to a given basis (Z-basis)


and Yjf = BX^ is the same linear transformation relative to another basis (IF-basis),
then
A and B are similar.
Note. Since

will be
for

might have been written as B = PAP-^. A study of similar matrices


There we shall agree to write B = R'^AR instead of S = SAS'^ but

P"^, (12.2)

made

later.

no compelling reason.

LINEAR TRANSFORMATIONS

96

Example

Let

2.

AX

[l.2,l]', W^ = [1,-1.2]',

be a linear transformation relative to the -basis and

= BXjf

IFg

(a)

to the W-basis.

(b)

corresponding to

AX.

(c)

let

W^

Given the vector X = [3,0,2]',


Find the linear transformation
Use the result of (b) to find the image ijf of Xy =

[1,-1,-1]' be a new basis,

image relative

find the coordinates of its


Yjr

[CHAP. 12

[1.3,3]'.
1

Write

[W^.W^.W^] =

(a)

Relative to the

The image

of

If -basis,

is

-1 -1

-1

1
9

-1

the vector

AX

then

-2

-1 -3

= [3,0,2]' has coordinates

[9,5,7]'

Xff =

W X

which, relative to the IF-basis is

Yf^

= [l,l,l]'.
=

W Y

[14/3,20/9.19/9]'.

(b)

w\

(c) Yj.

W^AX

36

21

-15

21

10

-11

-3 23

-1

(W~^AW)Xjf = BXj^

36

21

-15

21

10

-11

-3 23

-1

6
=

[6,2,7]'

L.

See Problem

5,

SOLVED PROBLEMS
1. (a)

{h)
(c)

(d)

(a)

(6)

Set up the linear transformation


and 3 into Fg = [2,1,3]'.

carries E^ into

Y^

= [1,2,3]',

E^ into [3,1,2]',

Find the images of li= [1,1,1]', I2 = [3,-1,4 ]', and ^3 = [4,0,5]'.


Show that X^ and Zg ^-^^ linearly independent as also are their images.
Show that Xi, X^, and Z3 are linearly dependent as also are their images.

By TheoremI, A

The image

of

image of Xg

(c)

Y = AX which

The rank

is

= [y^, Fg, K3]

X^=

K3

[l,l.l]' is

the equation of the linear transformation is

Y-^

13

[6.4,8]'.

The image

of

=^

AX

-Yg is

Ys = [8,9,19]' and the

=[ 14.13,27]'.

of [A'^.Xg] =

-1

_1

is 2 as also is that of [^i, Kg]

19

Thus, X^ and X^ are linearly

independent as also are their images


(rf)

We may compare

the ranks of \_X^. X^. X^] and {Y^.Y^.Yq\; however,

both sets are linearly dependent.

X^

X^ + X^ and Fg = Fi+Zg so that

CHAP.

2.

LINEAR TRANSFORMATIONS

12]

Prove:

linear transformation (12.1) is non-singular

97

and only

if

if

is non-singular.

Suppose A is non-singular and the transforms of X^ ^ X^ are Y = AX^ = AX^. Then A{X-i^-Xi)
homogeneous linear equations AX = Q has the non-trivial solution X = X-^-X^. This
sible if and only if .4| = o, a contradiction of the hypothesis that A is non-singular.

the system of

and

is pos-

3.

Prove:

non-singular linear transformation carries linearly independent vectors into


linearly in-

dependent vectors.

Assume the contrary, that is, suppose that the images Yi = AXi. (i = 1,2
ent vectors X-^.Xr,
Xp are linearly dependent. Then there exist scalars s-^.s^

of the linearly independ-

p)
sp

not all zero, such that

^lYi +

H'^i

s^Y2+

1=1

s^Yfy
^^
f^

P
'

.|^ ^(-4^1)

Since A is non -singular,


linearly independent.

4.

s^X^ + s^X^ + -. + spXp =

Hence, the

Y^ are linearly

certain linear transformation

[3.0,-2]', and I3

iZ

F =

But this

o.

carries Z^ =

=[

a + b +
aX.i^

+ bXr,+

cXg= E^;

-6

then

a + b

and

its

imageis

Y2 = 1-1.3,1

Y,

-^2, 3,-1

and the image of Eg

Yy

= AXf, =

ter 11, find the

X^

]'

1,0,1

into

[2,3,-1]',

Find the images of

and a

-^,

6 = 1,

H-2.7.-1 ]' =
The equation

]'.

7^ =

BX^

fj^,

^^s

Chapter

11,

Xf^r

PX^

-1

P''X,.,

c =

-1

-2

i.

Thus,

]'

E^= -hX^ + X^

1"

^z-

Then

0_

-1"

-1

^!i

Q^w

-2
PY

=[ 1.-1.1

into

and write the equa-

Q-^AX,

Problem

relative to the f'-basis of that problem.

--3

and

,!

^Xg

[l,2,-2]'. Similarly, the image of 5 is


of the transformation is

[Y^.Y^.Yg]X

12,

hypothesis that the Xi are

is a linear transformation relative to the Z-basis


of

same transformation

From Problem

is contrary to the

c =

is K, = [l,l,l

+ 2c =

]'+ [3,0,-2]' +

5. If

+ spXp)

independent.

1.2,-1]' into [-2,7,-1]'.


tion of the transformation.

Let

A(Sj^X^+ S2X^+

Q^^Q^r.,

\^

14 -6
14

-9

X,

12,

Chap-

LINEAR TRANSFORMATIONS

98

[CHAP. 12

SUPPLEMENTARY PROBLEMS
Problem

6. In

show:

(a) the transformation is non-singular,

(b)

Af

= [1,1,2]',

A Y

carries the column vectors of

into

the elementary vectors.

7.

Using the transformation of Probleml, find (a) the image of


[-2.-5.-5]'.
-4ns.
(a) [8,5.11]', (b) [-3.-1. 2]'

8.

Study the effect of the transformation Y = IX. also Y = klX.

9.

Set up the linear transformation which carries E^ into [l,2,3]', 5 into [3.1. 2]', and 3 into [2,-1,-1]'.
Show that the transformation is singular and carries the linearly independent vectors [ 1,1,1 ]' and [2,0,2]'
into the same image vector.

10.

11.

Suppose (12.1) is non-singular and show that


ages Y^.Y^
Y^.

Use Theorem
changed.

to

III

show

....

X^

X whose

image

is

are linearly dependent so also are their im-

that under a non-singular transformation the dimension of a vector space is un-

Hint. Consider the images of a basis of P^ (F).


1

12.

X-^.X^.

if

(h) the vector

Given the linear transformation Y

-2
dependent vectors

^i=[l,l,l]',

JVg =

X.

show

]',

and A:3=[i.2,3]' are linearly dependent,

(a) it is singular,

(6) the

images of the linearly

in-

3 5

[2.I.2

(c) the

image

of V^{R) is a Vs(R).

13.

Given the linear transformation Y

2 4 X.

show

(a) it is singular,

(6) the

image of every vector of the

113
2
V,

14.

(R) spanned by

]'

Yi,

= 1,2

]'

lies in the K,(fl)

spanned by [5,7.5]'.

Prove Theorem Vn.


Let Xi and

Hint.

Y
15. Prove:

16.

and [3.2.0

1.1,1

Let

BZ

(j

n) be the given sets of vectors. Let

carry the E^ into

AX
^2

12

_1

1_

= [1.0.1]',

Z3

(6)

X. when referred to the

= [14,10,6]' is the

= [1.1.1]'

image of

new

A^

carry the set Xi into E{ and

be Chosen.

Yy =

PX

and Y^ = PY.

Q ^AQX.

AT

=P

basis, say

= [1,2,3]' relative to the E-basis.

basis, has coordinates X^, = [-2,-1.4]' and

where P

where Q

Let

new

Show

Z, =

that

under the transformation.

Y has coordinates

Y^ = [8,4,2]'

0-1"

(d)

AX

be a linear transformation relative to the -basis and let a

A^

(a)

X^

Similar matrices have equal determinants.

[1,1,0]',

(c)

Kj.

-1

-1

iZ^,

Zf2,

Z^i

^.

0"
1

17.

Given the linear transformation

7^

1
1

1
1

Xjf

relative to the IF-basis:

W^= [o,-l,2]',

IK,= [4,1,0]'

CHAP.

IK5

LINEAR TRANSFORMATIONS

121

[-2.0,-4]'- Find the representation relative to the Z-basis: Z^ = [i,-l,l]', Z2 = [l,0,-l]', Z3=[l,2,l]'.

-10

Am

18.

If.

99

2-5
-10 2
2

in the linear transformation

whose vectors transforms

AX. A

is singular,

then the null space of A is the vector space each of

Determine the null space of the transformation of

into the zero vector.

123"
(a)

Problem

-4ns.

19.

12.

(6)

Problem

13.

(c)

X.

(R) spanned by [l.-l.l]'

(a)

(6)

I^\R) spanned by [2,l,-l]'

(c)

I/^()

spanned by [2,-l,o]' and [3,0,-1

]'

If y = AX carries every vector of a vector space I^ into a vector of that same space, v^ is called an Invariant space of the transformation. Show that in the real space V^{R) under the linear transformation

-f

(a)

12

X. the

\l^

spanned by [l.-l.o]', the V^ spanned by [2,-1.-2]', and the V^ spanned by

[1.-1,-2]' are invariant vector spaces.


2
(6)

13

spaces.

X. the Vq spanned by [l.l.l]' and the

^ spanned

by [l,0,-l]' and [2,-l,0]' are invariant

(Note that every vector of the V^ is carried into itself.)


1

(c)

10

X, the li spanned by [l,l,l,l]' is an invariant vector space.

-14-6
20.

Consider the linear transformation


tion of

1,

n.

PX

yi

(i

^h

1.2

n)

(a)

Describe the permutation matrix P

(b)

Prove: There are n! permutation matrices of order n.

(c)

Prove:

If

Pj and fg are permutation matrices so also are P3 = P-lP2

(d)

Prove:

If

(e)

Show that each permutation matrix P can be expressed as a product


umn matrices K^2, ^28
^n-T-nWrite
P = [^^, E^^.
^^] where ii, ij
% is a permutation

(/)

is a permutation matrix so also are P' and

= [s, 1, 4,

/^ is

/1./2

a permuta-

and

P^^P^Pt.

of a

number of the elementary

/.

1,2

of

and ^

col-

are the ele-

Find a rule (other than P~ = P') for writing P~


For example, when n = 4 and
2], then P'^ = [2. '4. 1. 3]; when P = [E^ E^. 1, 3], then P~^ = [g, 2, 4, 1].

mentary n-vectors.

PP'

which

in

chapter 13
Vectors Over the Real Field

INNER PRODUCT. In this chapter


If Z = [%,%, ..., x^y and y

all

vectors are real and l^(R) is the space of

= [ji,

all real re-vectors.

are two vectors of l^(R), their inner product is

YnY

72, ,

defined to be the scalar

X-Y

(13.1)

Example

1.

x^y^

^2=

For the vectors X^=\\.\,\\',


(a)

X^-X^

1-2

(6)

X-^-X^

1-1

+ l(-2)

(c)

X.yX^

1-1

(rf)

^1-2^:2

1-1

1-4

1- 1

1-

[2.1,2]',

+1-1

1-4

XnJn

^3 = [l.-2.l]':

=
=

1-1

1-2

x^j^

10

2(^1-^2)

Note. The inner product is frequently defined as

X.Y

(13.1')

X'Y

Y'X

The use of X'Y and Y'X is helpful; however, X'Y and Y'X are 1x1 matrices while
X-Y is the element of the matrix. With this understanding, (13.1') will be used
In vector analysis, the inner product is callhere. Some authors write Z|y for X-Y
.

ed the dot product.

The following

(13.2)

rules for inner products are immediate

(a)

X^-X^ = X^-X^,

ib)

X^-(X^ + Xs)

(c)

(X^+ X^)

0.

(X^+Xs)-X^

(Xs+ X^)

ORTHOGONAL VECTORS. Two


product is

X^-hX^ = HX^-X^)

X and Y

vectors

The vectors Z^ and Xg

THE LENGTH OF A VECTOR X of


thus.
ner product of X and X

X^-X^

^i(R),

of

X^-X^ + X^-X^

=
+

X^-X^ + X^-Xs + X^-X^

of V^iR) are said to be orthogonal if their inner

Example

denoted by

\\

are orthogonal.

X\\

is

defined as the square root of the in-

(13.3)

Examples. Prom Example

II

1(c),

^11

\\

X^\\ =

\/

V3

X-X

\/

xl +

xl+

---

X.

See Problems

100

1-2.

CHAP.

VECTORS OVER THE REAL FIELD

13]

Using (13.1) and

(13.3),

x.Y

(13.4)

A
are

may be shown

it

vector

X whose

length is

that

iil|z+y||'

||z||

101

WxW

- ||y|h

is called a unit vector.

The elementary vectors E^

examples of unit vectors.

THE SCHWARZ INEQUALITY.

If

and Y are vectors of ^(/?), then

\X-Y\

(13.5)

<

\\x\\.\\y\\

that is, the numerical value of the inner product of two real vectors is at most the product of
their lengths.

See Problem

THE TRIANGLE INEQUALITY.

If

and Y are vectors of

(13-6)

ORTHOGONAL VECTORS AND SPACES.


n-vectors and
0.

If

X^,

CiZi + ^2^2+ ...+ c^^ =


Since this requires 0^ =
for i = 1,2
if

Any

I.

set of

m<

vector

0,

IUII + ||y||

X^

X^

then for

then

are

= 1,2

m<n

mutually orthogonal non-zero


+ c^X^) Xi =

m. (c.lX^+ 0^X2+

we have

n mutually orthogonal non-zero re-vectors is a linearly independent

set and spans a vector space

<

l!^+yll

)/(/?),

3.

is said to be

Ijf(/?).

orthogonal to a vector space Vn(R)

orthogonal to every

if it is

vector of the space.


II.

If

a vector

to the space

is

orthogonal to each of the re-vectors X^, X^

X^,

it is

orthogonal

spanned by them.
See Problem

HI. If Vn(R) is a subspace of I^(/?),

which

is orthogonal to

k>h. there exists

at

least one vector

4.

of V^CR)

V^\R).

See Problem

5.

Since mutually orthogonal vectors are linearly independent, a vector space V'^(R), m>0,
can contain no more than m mutually orthogonal vectors. Suppose we have found r<m mutually
orthogonal vectors of a V^(R). They span a V^iR), a subspace of V*(R), and by Theorem HI,
there exists at least one vector of V^(R) which is orthogonal to the I^(/?). We now have
r+l
mutually orthogonal vectors of l^(R) and by repeating the argument, we show
IV.

Every vector space V^(R), m>0, contains m but not more than

mutually orthog-

onal vectors.

Two vector spaces are said to be orthogonal if every vector of one is orthogonal to every
vector of the other space. For example, the space spanned by X^ = [1,0,0,1]' and X^ =
[0,1,1,0]' is orthogonal to the space spanned by X^ = [ 1,0,0,-1]' and X^ = [0,1,-1,0 ]'
since (aX^ + bXr,) (0X3+ dX^) =
for all a,b,c,d.

VECTORS OVER THE REAL FIELD

102

The set

V.
tor

[CHAP. 13

of all vectors orthogonal to every vector of a given Vn,(R) is a unique vec-

space
Vn'^(R).
^
n
J

ggg Problem

6.

We may associate with any vector ^ 7^ o a unique unit vector U obtained by dividing the
components of X by \\X\\ This operation is called normalization. Thus, to normalize the vector
X = [2,4,4]', divide each component by ||^|| = V4 + 16 + 16 = 6 and obtain the unit vector
.

[1/3,2/3.2/3]'.

basis of Vn(R) which consists of mutually orthogonal vectors is called an orthogonal baif the mutually orthogonal vectors are also unit vectors, the basis is called a

sis of the space;

The elementary vectors

normal orthogonal or orthononnal basis.

are an orthonormal basis of ^(R).

See Problem

THE GRAM-SCHMIDT ORTHOGONALIZATION PROCESS.


V^(R).

Suppose

X^,

X^

are

7.

a basis of

Define

y,

X,

^S

Y
^3

Y V

-'m

'^

^g'^3

-'w-l

Then the

unit vectors

Gj =

(i

^1-^3

%-l

Xi

= l,2,...,m)

V V

ll*^!

"

y'

"'l

are mutually orthogonal and are an orthonormal

1^11

basis of

Example

F(/?).

3.

Construct, using the Gram-Schmidt process, an orthogonal basis of V2(R). given a basis

A'i= [1,1,1]', a:2= [1,-2,1]',


(i)

Y^

X^ = [1.1.1]'

(ii)

Y^

X^- ~^Yi

(ill)

^3 =

^3 -

[1,-2,1]'

^Y, - ^Y,

The vectors

G2 =

Xs=[i.2.zY.

-^,

Note that Fg =

-^2

^1-1 =

[l/\/l. l/\/3, l/^/3l

[l/Ve, -2/V6, l/Ve]'

here because

Each vector
X.^

and

[1,-2.1]'

- -^y, - ^[1,1.1]'

[1,2,3]'

G^ = -jpj =

are an orthonormal basis of ^(fl).


0.

and

Gg

[-1,0,1]'

]',

-ii-

[-I/V2,

0,

1/\A2]'

G^ is a unit vector and each product G^

A^2 a^re

Gj =

orthogonal vectors.

See Problems 8-9.

CHAP.

VECTORS OVER THE REAL FIELD

13]

Let

^m be a basis

^2

Zi,

of a f^(/?) and suppose that

103

X^,

Xg,(l< s<

X^

m),

are

Then, by the Gram-Schmidt process, we may obtain an orthogonal basis


of the space of which, it is easy to show, Yj^ = X^, (i = 1,2
Thus,
s).

mutually orthogonal.
y^, Yg

}^

VI. If

X-i^,

X2,

exist unit vectors

, Xs,(l< s<m),
X^^^,
^m

are mutually orthogonal unit vectors of a Vn(R), there


i" the

X^^.^-

space such that the set

X^, X^, ...,X^

is an

orthonormal basis.

THE GRAMIAN.

Let X^, X^

be a set of real n-vectors and define the Gramian matrix

Z>,

A^ A^

A^ A2

A2 Aj X2 A2
'

'

Xp- X^

Xp- X2

...

'

(13.8)

A>)

A]^

A2

'

Ajj

A2 A^ A2 A2

X-^ A,

X^Xp

Xp- Xp

XpX-i

Clearly, the vectors are mutually orthogonal if and only if


In

Problem

if

the vectors are linearly dependent.

14,

Chapter

17,

we

ORTHOGONAL MATRICES. A

XpX^

...

XpXp

is diagonal.

shall prove

For a set of real re-vectors

VII.

A^ A^ A2

A-L

Z^, X^,

....

Xp,

G >0. The
|

square matrix A is called orthogonal

AA'

(13.9)

A'A

equality holds

if

and only

if

that is, if
(13.9')

A'

Prom

(13.9)

it

is clear that the

A'

column vectors (row vectors) of an orthogonal matrix A are

mutually orthogonal unit vectors.

Example

4.

By Examples, A

l/\/3

l/x/6

l/\[Z

-2/\/6

l/\/3

\/\[&

-1/^2
is orthogonal.

l/x/2

There follow readily


VIII.

If

the real re-square matrix

is orthogonal, its

column vectors (row vectors) are

an orthonormal basis of V^(R), and conversely.


IX.

X.
XI.

The inverse and


The product

the transpose of an orthogonal matrix are orthogonal.

of two or more orthogonal matrices is orthogonal.

The determinant

of an orthogonal matrix is 1.

ORTHOGONAL TRANSFORMATIONS.
(13.10)

Let

AX

VECTORS OVER THE REAL FIELD

104

[CHAP. 13

be a linear transformation in Xi(R) and let the images of the n-vectors I^ and
X^ be denoted by
Yi and Y^ respectively. Prom (13.4) we have

x^-x,

u\\x,^x,f -

\\X,f

Y,-Y,

k\\\Y,^Y,f -

II

\\X,f]

and

Comparing

and

right

left

members, we see that

products, and conversely.

XII.

A
lem

if

y,

Y,f]

(13.10) preserves lengths

it

preserves inner

Thus,

linear transformation preserves lengths

linear transformation

Y=AX

is called

and only

if

orthogonal

if its

if it

matrix

preserves inner product s.

is orthogonal.

we prove

10,

XIII.

linear transformation preserves lengths

Examples. The linear transformation Y

AX

if

l/\/2

l/v/6

l/\/3

-2/\/6

I/V3

l/\/6

and only

if its

In Prob-

matrix is orthogonal.

-I/V2

ii3

orthogonal.

The image of

l/v^

[a,i,c]'is
"

_^

26

_f

_1_

c "]/

[
and both vectors are of length yja^ + b^ + c^

XIV. If (13.10) is a transformation of coordinates from the -basis to another, the Zbasis, then the Z-basis is orthonormal if and only if A is orthogonal.

SOLVED PROBLEMS
1.

Given the vectors

= [1,2,3]' and X^ = [2,-3,4]',

Zj.

(o) their inner product,

(b) the

find:

length of each.
2

(a)

X^-X^

XiX^

[1.2.3] -3

1(2)

2(-3) +

3(4)

(6)

\\X^f

X^.X^

lA-jf

2(2)

X[X^

[1,2,3]

+ (-3)(-3) + 4(4)

29

14

and

and

\\X^\\

\\Xji

V29

vTi

CHAP.

VECTORS OVER THE REAL FIELD

13]

105

Show

(b)

that 1 = [1/3, -2/3, -2/3 ]' and Y ^ [2/3.-1/3, 2/3]' are orthogonal.
Find a vector Z orthogonal to both X and Y.

(a)

X-Y

2. (a)

2/3

Xy

[1/3,-2/3,-2/3] -1/3

and the vectors are orthogonal.

2/3
1/3
(6) Write

2/3

-2/3

column of zeros.

3.

Then by

and compute the cofactors -2/3. -2/3, 1/3

2/3

Clearly, the theorem is true

of the elements of the

0_

(3.11)

Prove the Schwarz Inequality:

If

O"

-2/3 -1/3

[A:,y,o]

= [-2/3, -2/3, 1/3]'

If

if A"

is

and Y are vectors of


F

or

is the zero vector.

orthogonal to both

Vn(R), then

Assume then

A:

\X-Y\ <
that

and K.

||A'||.||y||

and Y are non-zero vectors.

a is any real number,

llaA^

yf

(aX + Y)-(aX + Y)

[ax^ + y^.ax^+y^

(a x^

a^xf

+ 2ax^y^ +

axn+yn]-[ax^+y^. ax^ + y^
+ (a^x^ + Zax^y^ + y^)

yj^)

+ 2aX.Y

axn+jnY

+ (o^^ + 2a%j^ + y^

\\Yf >

Now a quadratic polynomial in a is greater than or equal to zero for all real values
of o
discriminant is less than or equal to zero. Thus,
i(X.Yf - i\\xf- \\Yf
and

4.

\x-y\

Prove:

If

a vector

is orthogonal to

each of the n-vectors

Since

Xi-Y

= 0, (i = 1,2

orthogonal to the space.


orthogonal to that space.

5.

Prove:

which

If

a^X^)-Y

X^

X^,

Thus, Y

m).

In particular, if

a^X^-Y + a^X^-Y +

is
is

X^.

it

a^X^-Y

is orthogonal to the

Then

orthogonal to every vector of the space and by definition is


orthogonal to every vector of a basis of a vector space, it is

a ^() is a subspace of a V^(R), k>h. then there exists at least


one vector

is orthogonal to the f^(fi).

^^*

-^I'-^s

^h be a basis

of the

FV).

let X^,^^

be a vector

in the

vM)

A"

of

v\R)
"

but not in the P^(R)

consider the vector

<*)

The condition

that

if its

m-\\Y\

<

of the space spanned by the X's can be written as a^X^+a^X^-^


^-c^Xtji

{aiXi + a^X^+... +

and only

<

space spanned by these vectors.

Any vector

if

a^X^ + a^X^ +

be orthogonal to each of X^.X^

...

Xf,

aj^X,^

+ a^^^Xf,^^

consists of h homogeneous linear equations

and

VECTORS OVER THE REAL FIELD

106

[CHAP. 13

a^X^-X^ + a^X^-X^ +

...

+ af,Xh-X-, + a,,^^Xf^^^. X^

a^X^.X^ + a^X^.X^ +

...

+ a^Xh-X^ + af^^^Xf,^^- X^

ft + l
unknowns a^.a^
ay,^^. By Theorem IV, Chapter 10, a
non-trivial solution exists. When these
values are substituted in (i), we have a non-zero (why?) vector
X orthogonal to the basis vectors of the kA)
and hence to that space.

In the

^ ^^^ ^^''^'''^ orthogonal to every vector of a given V^{R) is a unique


vector space

""^^

Vn-k'J'^^

^^* -^i-^s
be a basis of the V^{R).
A-fe
system of homogeneous equations

The

..-vectors

X^.X=o.X^.X=Q

^'>

Since the

Jt,-

n-k linearly independent solutions (vectors) which span


a K"-\/?).

system

(i) is

of rank k

hence, there are

(See Theorem VI, Chapter 10.)

Uniqueness follows from the fact that the intersection space of the
V^iR) and the V^'^(R)
space so that the sum space is Xi{R).

7.

Find an orthonormal basis of

V^(R),

given

is the zero-

= [1/^6, 2/x/6, l/x/6]'.

Note that A- is a unit vector. Choose Y = [l/Vl,


Then, as in Problem 2(a), obtain Z = [l/^Jl. -l/VI,

0,

-IA/2 ]'

1A/3]'

8.

satisfy the

Xk.X=0

are linearly independent, the coefficient matrix of the

X^^

orthogonal to each of the

to

another unit vector such that

complete the

-Y =

set.

Derive the Gram-Schmidt equations (13,7).


^^'

^^' -^2

be a given basis of V^(R) and denote by

>^n

Y^.

Yr,

the set of mutually orthogonal

Y^

vectors to be found.
(a)

Take

Y-^^

(b)

Take

Y^ = Xr, + aY-^

X^.

Since

Y-^

Y^.Y^
and

(c)

Take

o =

- -i-2

Y^ =

and Y^ are to be mutually orthogonal.

X3 +

aYr,

Thus.

+ bY^

K, =

Y^-X^ + Y^-aY^

Y X
^Sill
X^ ~ ^1^
Xo-

Since

Y^. K,,

Y^

Y^-X^ + aY^-Y^

Y^ are to be mutually orthogonal,

yi-Yj,

Yi-Xg + aY^.Y^ + bY^-Y^

Y^-

X^ + bY^-Y^

Y^.Y^

Y^-Xs + aY^-Y^ + bY^.Y^

Y^-X^ + aY^-Y^

and

Then

(d)

Continue the process until K,

- ^ll^

is obtained.

and

7, =

Z3 -

^ki^ K _

2W^ v

CHAP.

9.

VECTORS OVER THE REAL FIELD

13]

Construct an orthonormal basis of

Take

Y^ =

X^

= [2.I.3Y.

K,

X^ -

^^i-i

"2

[1.1.1]'

Fg,

107

given the basis Z^ = [2,1,3]', X^ = [1,2,3]', and Xg = [1,1,1]'.

Then
=

^3 V

- ii[2,1.3]'

[1,2.3]'

[-6/7.15/14,3/14]'

^1 '"^3

- ^

Normalizing the y-s. we have

14

I4J

[2/\/l4. l/i/Ii, 3/v'l4

]',

[_3

3j

[-4/\/42, 5/V42. l/\/42]', [l/Vs. 1/V3.

-1/^3

]'

as the required orthonormal basis.

10. Prove:

Let

linear transformation preserves lengths

y^. Yq

Suppose A

and only

if its

is

orthogonal so that A/l =

Y^-Y^

Theorem XII lengths

YiY^

AX.

Then

/.

(X'^A')(AX^)

X\X^

X^^-X^

are preserved.

Conversely, suppose lengths (also inner products) are preserved.


Y^-Y^

and A

matrix is orthogonal.

be the respective images of X^,X2 under the linear transformation

(1)

and, by

if

X{(A'A)X2

Then

A'A=1

X^X^.

is orthogonal.

SUPPLEIVIENTARY PROBLEMS
H. Given

the vectors A'l = [l,2,l

.Yg =

]'.

[2.I.2]', vYg = [2.1,-4

]'.

find:

product of each pair,


(6) the length of each vector.
(a) the inner

(c)

a vector orthogonal to the vectors X^, X^

Ans.

(a) 6, 0,

-3

Vb,

(6)

12.

Using arbitrary vectors of

13.

Prove (13.4).

14.

Let

a:

= [1.2.3,4]' and

V^

3,

^3(7?).

(c)

X.^,

Xq

[l,0,-l

]',

[3.-2.1]'

verify (13.2).

= [2,1,-1.1]' be a basis of a V^(R) and

= [4,2,3,l]' lie in a V^{R)

containing

and y.
(a)

Show

that

(b) Write

15. (a) Prove:

Z is not in the ^^{R).


aX + bY + cZ and find a

vector of

I^(ft) is

vector

of the V^{R) orthogonal to both

orthogonal to itself

if

and only

if it is

and Y.

the zero vector.

If X-^. X^. Xg are a set of linearly dependent non-zero ^-vectors and


X^ and Xq are linearly dependent.

(6) Prove:

if

Z^ Xg = Xj_-Xq=

0,

then

VECTORS OVER THE REAL FIELD

108

16.

Prove:

vector

is orthogonal to every vector of

P^"(/?) If

and only

[CHAP. 13

is orthogonal to every vector of

if it

a basis of the space.

17.

18.

Prove:

20.

two spaces V^iR) and

t^(fl) are orthogonal, their intersection

space is

I^(if).

Prove: The Triangle Inequality.


Hint.

19.

If

Show

Prove:

that

A"

||

!|Z +

1|

<

(||.Y||

+ \y\

|1a:|1

if

Normalize the vectors of Problem


Ans.

[l/Ve, 2/V6. l/\/6]',

21.

Show

that the vectors X.

22.

(o)

Show

using the Schwarz Inequality.

||y||)^,

and only

if

and Y are linearly dependent.

11.

[2/3, 1/3, 2/3]',

Y .Z

X^

that if X^.

Problem

of

X^

2 are

[2/V2i, l/V^I, -4/V2i

]'

an orthonormal basis of V^{R).

are linearly independent so also are the unit vectors obtained by normalizing

them.
(6)

Show

that

if

the vectors of (a) are mutually orthogonal non-zero vectors, so also are the unit vectors

obtained by normalizing them.

23.

Prove: (a)

is

orthogonal and

.4

is

orthogonal and \a\ = -i, each element

24.

Prove Theorems

25.

Prove;

26.

each element of A

If

(6) If

If

Prove that AA'

.4

1,

equal to

is

of^

its

cofactor in

^4

is equal to the negative of its cofactor in |^|

VIII, IX, X, XI.

and B commute and C

.4

where A

(or A'A),

is orthogonal,

then C'AC and

is n-square. is a

C'BC commute.

diagonal matrix

if

and only

if

the rows (or columns) of A

are orthogonal.

27.

Prove:

UX

28.

Prove:

If

29.

Prove:

If

and Y are n-vectors, then XY'-YX' is symmetric,


and Y are n-vectors and A

is n-square, then

X-(^AY) = {A'X) -Y

(b)X.X
30.

be an orthonormal basis and

A'

cl+4+...

(a)

X^.

(6)

[3/VI3,

{a)

[1,-1,0]',

(6)

[1.0,1]',

(e)

[2,-1,0]',

VsiR). given

2/\/l3]',

c^X^,

then

(a)

X-X^ =

c^,

(i = 1,2

(a)

X^ =

3/VT7, -2/\/T7, 2/1/17]';

[-4/\^, -3/^34,

[2/VI3.
V^i^R)

0,

(6) [3,0,2]'

3/\/34]'

-3/VT3]', [0,1,0]'

by the Gram-Schmidt process, using the given vectors

in order:

[3.2,1]'

[4,-1,0]',

(a)

[iV^. -iV^,

(b)

[^\/~2, 0,

(c)

[2V5/5, -V5/5,

[4,0,-1]'

[V^/6, V2/6, -2v'V3]', [-2/3,-2/3,-1/3]'

0]',

iV2]',

[0,1,0]',
0]',

Obtain an orthonormal basis of


Hint.

Take

Ans.

[\A3/3, V'3/3, -V^/3]',

Y^ = X^,

[1,-1,-2]'

[2,-1,-2]',
[1,3,1]',

A^

^=1

[0.l/^^2.l/\f2Y,
0,

if

cl

Construct orthonormal bases of

Ans.

32.

Z^

Find an orthonormal basis of


Ans.

31.

A"]^,

[iV^.

-iV2]'

0,

[^/5/5.2^/5/5.oy,
I^(ft),

given

^"1

[o,0,-l]'

=[ 1,1,-1

]'

and

ATg =

[2,1,0

]'.

obtain Y^ by the Gram-Schmidt process, and Y^ by the method of Problem 2(6).


[5\/2. 0, 2\A2]',

[\/6/6, -\/6/3, -\/6/6

]'

n);

CHAP.

VECTORS OVER THE REAL FIELD

13]

33-

Obtain an orthonormal basis of V^iR). given

34.

Show

35.

Prove:

36.

Use Problem 35

two ways that the vectors

in

If

skew-symmetric. and

is

[7,_i,_i

10

(b)

-5

oj'

38.

71 =

-5 -10

-10

(b)

5 -I2J'

is an orthogonal matrix

P^^APX^

and

it

Prove:

40.

Let

If

and

is orthogonal

= [xj^.x^.xsY

as

or

-11

AP

where P

is

non-singular, then

PB^

is orthogonal.

-basis to an orthonormal Z-basis with matrix P. Y =

7^= BX^ (see Chapter

12).

Show

that if

is

orthogonal so also is

B.

and

ZxF

and

+ .4 is non -singular then

= [yi.yQ.ygY

where

25, 23]'

= (I

- A) (I + A)'"^

is

Jl

21 =

3 ys
,

Z2 =

X^, Y^.

^1
.

the z^ as cofactors of the elements of the third column


mn of

],

Zg =

yi
estciblish:

of

(6)

The vector product

of two linearly independent vectors is orthogonal to each of the two vectors.

(c)

XxY

(d)

(kX)xY

41. If W. X. Y.
(a)

(d)

k(XxY)

XxikY),

k a scalar.

are four vectors of V^{R), establish:

X x{Y+Z)

(b)X-(YxZ)
(c)

is the zero vector.

-(YxX-)

XxY

XxZ

Y-(ZxX)
W-Y
(WxX)-(YxZ) =
X-Y
X-X
(XxY)-(XxY) =
Y-X
=

Z-(XxY)
W-Z
X-Z

X-Y
Y-Y

\XYZ\

of

After identifying
2 yi

The vector product

two linearly dependent vectors

X xY

yi

(a)

be-

skew-symmetric.

be two vectors of VsiR) and define the vector product,


X2

= [21,

^3 ys

AX

and con-

Theorem XIV.

versely, to prove

39.

-51

In a transformation of coordinates from the

comes

is orthogonal.

10

.4

-A)(I + A)~^

given

"5-14

If

= (I

2-3

-12

37. Prove:

12
s"!

(a)

and [5,4,5,6]' are linearly dependent.

+ A is non-singular, then

to obtain the orthogonal matrix

Ans.

]'.

[l. -1.-2, -3]',

[l.2,3,4]',

"

(a)

X-^ =

109

chapter 14
Vectors Over the Complex Field

COMPLEX NUMBERS.

x and j are real numbers and i is defined by the relation j^ = 1, z = x^iy


is called a complex number. The real number x is called the real part and the real number y is
If

called the imaginary part of x +

Two complex numbers

fy.

are equal if and only if the real and imaginary parts of one are equal

respectively to the real and imaginary parts of the other.

A complex number
The conjugate

x + iy =

of the

if

complex number

(product) of any complex number and

It

and only

its

if

x = y =

= x+iy

0.

is given

by

= x+iy = xiy.

The absolute value |z| of the complex number z = x+iy is given by


follows immediately that for any complex number z = x + iy,
|z|

(14.1)

VECTORS. Let X

and

> \A

>

\z\

be an ra-vector over the complex field C.


Since ^(R) is a subfield,

the vector space I^(C).

The sum

conjugate is a real number.

\J

z-z =

\fW+y^

\y\

The
is to

it

|z|

such vectors constitutes

totality of

be expected that each theorem con-

cerning vectors of I^(C) will reduce to a theorem of Chapter 13 when only real vectors are considered.
If

is

= [%, x^

XnY and y =

[ji,

y^Y are two vectors of

72

P^(C), their inner

product

defined as

X-Y

(14.2)

XY

%yi

The following laws governing inner products


(a)

(14.3)

I-y =

hold.

2?.{X.Y)

where R(X-Y)

is the real part of

X-Y.

(c)

X-(cY) = c(X-Y)

(d)

X-(Y+Z) = X-Y + X-Z

= 2CiX-Y)
where C(Z-y) is the imaginary part of Z-F.

(e)

(Y+Z)-X =

Y-X+Z-X

See Problem

of a vector

vectors

is

given by

and Y are orthogonal


V^{C), the

X-Y-Y-X

(g)

||Z||

if

\/

X-X =

X-Y = Y-X =

Q.

Triangle Inequality
\\X+Y\\

and the Schwarz Inequality (see Problem


(14.5)

XnJn

(cX)-Y = c(X-Y)

For vectors of
(14.4)

are readily verified;

X-Y+Y.X

(/)

(b)

The length

Two

y^

x^y^

<

||Z||

<

\\X\\-\\Y\\

||i'll

2)

\X-Y\

Moreover, we have (see Theorems I-IV of Chapter 13)

110

\/%%

%^2

+ XnXn-

1.

CHAP.

VECTORS OVER THE COMPLEX FIELD

14]

I.

Any

set of

111

mutually orthogonal non-zero re-vectors over C is linearly independent

and, hence, spans a vector space I^(C).


II. If a vector Y is orthogonal to each of the re-vectors
thogonal to the space spanned by these vectors.

III.

If

X^,

X^

X^,

then

it

is or-

V^iC) is a subspace of V^C), k>h, then there exists at least one vector

in

V^(C) which is orthogonal to V^(C).


IV.

Every vector space

J^(C),

m>0, contains m

m mutually

but not more than

orthog-

onal vectors.

basis of I^(C) which consists of mutually orthogonal vectors is


called an orthogonal
If the mutually orthogonal vectors are also unit
vectors, the basis is called a nonnal
or orthonormal basis.
basis.

THE GRAM-SCHMIDT PROCESS.

(14.6)

Let

X,. X^

Y,

X,

In

Xn

Yd

Xn

yn

_
~

X^

^2-^3

Y .Y
l2-l2

be a basis for F^^CC).

Yi-Xs

Y^
Y,.Y,

Y
y
'a-i'^m
y

T;

"Si

'y^

-'m-i

^m-i'-'m-i

The

unit vectors

Gi

7.

(i

= 1.2

m)

Define

are an orthonormal basis for ^^(C).

V. If X^, X^
X^, (l<s<m), are mutually orthogonal unit vectors
of ^(0) there
exist unit vectors (obtained by the Gram-Schmidt
Process) Z,,,, X, ^
;i'
in the snaoe
such that the set Z Z^
Z, is an orthonormal basis.

THE GRAMIAN.

Let
Gramian matrix.

Xp be a

X^

set of

;s-

vectors with complex elements and define the

Xi'Xi Xj^-Xq

X^-Xp

Aj

X-y

X-^Xq

X^ Xp

X^-X^

X^-Xp

Ag X^

XqXq

X2 xp

Xp-Xp

XI X-^ XI x^

x^x.
p^p

X^'X'i

(14.7)

Xp-X^ Xp-X^

Clearly, the vectors are mutually orthogonal if and


only if

Following Problem

14,

Chapter

17,

is diagonal.

we may prove

VI For a set of re-vectors X^. X^


Xp with complex elements,
holds if and only if the vectors are linearly
dependent.

\G\

>

0.

The equality

VECTORS OVER THE COMPLEX FIELD

112

UNITARY MATRICES. An

n-square matrix A is called unitary

The column vectors (row vectors)

fCHAP. 14

(AyA = A(A)'=

if

that is

if

(i)'=

^.

of a unitary matrix are mutually orthogonal unit vectors.

we have

Paralleling the theorems on orthogonal matrices of Chapter 13,

The column vectors (row vectors)

VII.

I,

of an re-square unitary matrix are an orthonormal

basis of l^(C), and conversely.


VIII.

The inverse and

The product

IX.

of two or more unitary matrices is unitary.

The determinant

X.

the transpose of a unitary matrix are unitary.

of a unitary matrix has absolute value

UNITARY TRANSFORMATIONS. The

linear transformation

(14.8)

where A

1.

AX

is unitary, is called a unitary transformation.

XI.

linear transformation preserves lengths (and hence, inner products) if and only

matrix is unitary.

if its

XII.

If

AX

is a transformation of

coordinates from the i'-basis to another the Z-

basis, then the Z-basis is orthonormal if and only

if

is unitary.

SOLVED PROBLEMS
1.

Given X = [l+j, -J, 1]' and Y = [2 + 3J, 1- 2J, i]',


(c) verify X-Y + Y-X = 2R(X-Y)
(a) find X-Y and Y-X
(d) verify X-Y -Y-X = 2C(X-Y)
(b) verify X-Y = Y-X

2+
(a)

X-Y

X'Y

[l-i,f,i;

3/

l-2i

(l-i)(2+3!) + i{l-2i) + 1(0

7 4 3j

Y-X

Y'X

+i

[2-3i.l + 2i.-i]

3i

3i

= X-Y.

2.

Y-X. the conjugate of Y-X. is

(b)

From

(c)

X-Y + Y-X

(7

3i)

+ (7-3i) =

14

2(7)

(d)

X-Y

-Y-X

(7

3j)

-(7-30

6j

2(30 =

(a):

Prove the Schwarz Inequality:

As

in the

\X-Y\

<

2R(X-y)

p||

case of real vectors, the Inequality

2C(X-Y)

11Y||.

is true if

or

= 0.

When X and Y

a'Wxf

+ 2aR{X-Y) +

are non-zero

vectors and a is real, then


\\aX +

Yf

{aX+Y)-{aX+Y)

a X-X + a{X-Y + Y-X) + Y-Y

\\Y\\

>

0.

CHAP.

VECTORS OVER THE COMPLEX FIELD

14]

Since the quadratic function in a is non-negative

\xf\Yf

R{X-Yf If

X-Y

then

0,

|;f

-F

if

and only

<

discriminant is non-positive,

if its

Ra-Y)

and

< ||x|

fl(;5:-y)

||y

X-Y

if

113

<

,i

IkMyll
define

O.

X-Y

c =

\x-y\

Then

R(cX-Y) <

R[ciX-Y)]

3.

Prove:

B =

(B)

4. If

\(A)'AV

i = B + iC

5.

B+iC

is real if

Prove:

If

is

(A'A)

(A)A

while, by (14.3(6)),

R(cX-Y)

and B is Hermitian.

(B + iCUB+iC)

and only

if

BC + CS

= o or

is skew-Hermitian. then

(B + iC)(B+iC)

BC

iA

-CB

if

B and C anti-commute.

The reader

B^ + i(BC + CB) - C^

and only

if

if

B and C anti-commute.

is Hermitian.

(riA)'

thus,

Since A is skew-Hermitian,
(S)'

is Hermitian.

and only

if

(B+iC)' = B + iC; thus.

Hermitian,

Consider B = ~iA.

and S

|UM|y||

forall^andy.

Hermitian, show that (I)/l is real

is

(A)A
This

|c||U||-iyl|

U-yl < U||-||y||

Thus,

(A)'A is Hermitian for any square matrix A.

Since

\\cX\\-\\y\\

\X-Y\.

(A)' =

t(Z)'

will consider the case

-A. Then
i(-A)

-iA

= iA.

SUPPLEMENTARY PROBLEMS
6.

Given the vectors X^=[i.2i,iY.


find X^-X^ and X^-X^,

A:2 = [l,

1+

/,

o]',

and Xg =

[i, 1

j,

2]'

(a)
(b)

find the length of

(c)

show

(d)

find a vector orthogonal to both

Ans.

(a)

that

each vector Xi

[l-i, -1, i-j]'

2-3i.-i

(b) ^/e

7.

Show

8.

Prove the relations (14.3).

9.

Prove the Triangle Inequality.

that [l + i.i.lV.

orthogonal to both X^ and X^.

is

X^ and

V^ V^
.

[iA-i.oy, and

10.

Prove Theorems

11.

Derive the relations (14.6).

I-IV.

Jf g

(d)

[-1 -5iJ .3 -i]

[l -i.

1. 3j ]'

are both linearly independent and mutually orthogonal.

VECTORS OVER THE COMPLEX FIELD

114

12.

in order, construct

Using the relations (14-6) and the given vectors

[CHAP. 14

an orthonormal basis

for iy.C)

when

the

vectors are
(a)

[0,1,-1]', [l + j,l,l]'. [l-j,l,l]'

(b)

[l + i.i.lY.

Arts,

[2.1-2i.2 + iY, [l-i.O.-iY.

[O.^V2.-iV2]', [1(1 + 0.

(a)

ri

[2(1+0. 2..2

(&)

13. Prove: If

is

/I

T
J.

[3;^.

I. 2

5t

]'.

[-^i,

3+

3jy

^d

^(1 + 0.

]'

-6 + 3i
7
-5
-5
3j
-b
1-i
-t
y
L2V30 'aVso- 2\Am^
r

4^. 4^ J-

a matrix over the complex field, then A + A has only real elements and

A- A

has only pure

imaginary elements.
14.

Prove Theorem V.

15. If

(b)

show

is n-square,

(a) A'A is

A'A =

diagonal
if

and only

if

and only

if

Prove:

17.

Prove Theorems VII-X.

18.

Prove:

If

the columns of

and Y are n-vectors and A

16.

If

if

the columns of

is

A are mutually orthogonal vectors.

are mutually orthogonal unit vectors.

is re-square,

skew-Hermitian such that

X-AY

then

A'X-Y

+ A is non-singular, then B = (l-A){I + A)~^ is unitary.


I

19.

Use Problem 18

to form a unitary matrix, given

(6)

"^
i

L-1+.-

_9 + 8i
Ans.

r-l +

|_

-4-2i"|

2j

If

2-4i

4-lOj

^ and B are unitary and

Follow the proof

22. Prove: If

29

-2-jJ'

in

Problem

is unitary

Show that

same

1(1 +

J//3

-k

l/\/3

+t

-1 + j

-10 -4t
-9 + 8j

AB

-16-18i

12i

J-

-2-24J

order, then

13. to prove

and Hermitian. then A


3+

23.

of the

Chapter

10.

-10 -4i

-2-24J

(b)

(a)

20. Prove:
21.

i+j1

and

BA

are unitary.

Theorem XI.

is involutory.

2^
4 + 3i

is unitary.

2y/T5

-^
2Vl5_

-i/y/3

-1

24. Prove: If

is unitary

and

is unitary

and I+A

25. Prove:

if

= .4P

where P is non-singular, then PB

is non-singular, then

(I

- A)

(,!

is unitary.

+ A)'^

is

skew-Hermitian.

chapter 15
Congruence
CONGRUENT MATRICES. Two

A and B over F
P over F such that

re-square matrices

there exists a non-singular matrix

(15.1)

are called congruent,

over

if

FAP

Clearly, congruence is a special case of equivalence so that congruent matrices have the

same

rank.

When P

is

expressed as a product of elementary column matrices, P' is the product in reis, A and B are congruent provided A can

verse order of the same elementary row matrices; that

be reduced to B by a sequence of pairs of elementary transformations, each pair consisting of

an elementary row transformation followed by the same elementary column transformation.

SYMMETRIC MATRICES.
I.

whose
Example

1.

In

Problem

we prove

1,

Every symmetric matrix A over F of rank


first r

r is congruent over F to a diagonal matrix


diagonal elements are non-zero while all other elements are zero.

Find a non-singular matrix P with rational elements such that

12

10

10

-8

- P'AP

is diagonal, given

reducing A to D, we use [A /] and calculate en route the matrix P' First we use
and XgiC-S), then H^-ii-2) and K^^{-2) to obtain zeroes
in the first row and in the first column.
Considerable time is saved, however, if the three
In

ff2i(-2) and K2i(-2). then //gjC-S)

row transformations are made first and then the three column transformations.
transformed into a symmetric matrix, an error has been made. We have

[AH^

12

10

10

-8

10

1
1

[DP']

115

-2

-1

-1

10

is not then

-2100
-3010
-2001

0-100

4-12

10

0-1-1
0-1-1

c
'V-

If

0-100

-2

10

~\^

-10

-1-110

CONGRUENCE

116

[CHAP. 15

-2 -10 -1

4-1

Then
1

10
The matrix D

to

which A has been reduced

The additional transformations

is not unique.

10
0-100
ffgCi) and Kg(^).

for

D by

example, will replace

the diagonal meitrix

while the

10

10
transformations

H^d) and K^Ci)

replace

0-900

D by

There

however, no pair of

is,

rational or real transformations which will replace

D by

a diagonal matrix having only non-neg-

ative elements in the diagonal.

Let the real symmetric matrix A be reduced by real elementary


transformations to a congruent diagonal matrix D, that is, let P'AP = D. While the non-zero
diagonal elements of D depend both on A and P. it will be shown in Chapter 17 that the number

REAL SYMMETRIC MATRICES.

of positive non-zero diagonal elements

depends solely on

A.

a sequence of row and the same column transformations of type 1 the diagonal elements
may be rearranged so that the positive elements precede the negative elements. Then a

By
of

sequence of real row and the same column transformations of type 2 may be used to reduce the
diagonal matrix to one in which the non-zero diagonal elements are either +1 or 1. We have
II.

real symmetric matrix of rank

r is

congruent over the real field to a canonical

matrix
P

(15.2)

The

'r-p

integer p of (15.2) is called the index of the matrix and

p-(r p)

is called the

signature.

Example

2.

Applying the transformations H23. K^a and H^ik), Kr,(k) to the result of Example

[A\n

10

0-100
4

-2

-10

-1 -1
and (/AQ = C.

if

Thus, A

is of

rank

C
1

we have

10

-5

0-10

-2

10

IC\(/]

-1-110

= 3,

1,

index p =

2,

and signature

= 1.

III. Two re-square real symmetric matrices are congruent over the real field if and only
they have the same rank and the same index or the same rank and the same signature.

In the real field the set of all re-square matrices of the type (15.2) is a canonical set over

congruence

for real ra-square

symmetric matrices.

CHAP.

IN

CONGRUENCE

15]

THE COMPLEX FIELD, we

117

have

IV. Every ra-square complex symmetric matrix of rank


complex numbers to a canonical matrix

congruent over the field of

is

/^

(15.3)

Examples. Applying the transformations

H^ii)

and

10

-5

0-10

-2

[^1/]

-1 -1

Two

V.

numbers

if

Example

to the result of

10
10
10

2,

-5

-2i

-1 -1

if

SKEW-SYMMETRIC MATRICES.

[D

']
1

See Problems 2-3.

complex symmetric matrices are congruent over the field of complex


they have the same rank.

ra-square

and only

we have

^&:]

R'AR

and

K^f^i)

If

is

skew-symmetric, then

(FAP)'

FAT

r(-A)P

-FAP

Thus,
VI.

Every matrix

B =

FAP

congruent to a skew-symmetric matrix A is also skew-

symmetric.

In

Problem

we prove

4,

Every n-square skew-symmetric matrix A over F

VII.

congruent over

is

to a canoni-

cal matrix

(15.4)

where

D,-

= r

diag(Di, Dj

0^,0,..., 0)

,(f=l,2

t).

The rank

of

/}

is

2t.

See Problems.

There follows

if

Vin. Two ra-square skew-symmetric matrices over


they have the same rank.

The set

are congruent over

if

and only

of all matrices of the type (15.4) is a canonical set over congruence for re-square

skew-symmetric matrices.

HERMITIAN MATRICES. Two

[^

],

(15.5)

Thus,

or

conjunctive

if

n-square Hermitian matrices A and B are called Hermitely congruent,


there exists a non-singular matrix

FAP

P such

that

CONGRUENCE

118

Two

IX.

[CHAP.

re-square Hermitian matrices are conjunctive

if

and only

if

15

one can be obtain-

ed from the other by a sequence of pairs of elementary transformations, each pair consisting of a column transformation and the corresponding conjugate row transformation.

An Hermitian

X.

matrix

of rank

conjunctive to a canonical matrix

r is

Ip

-Ir-p

(15.6)

The

p-(r-p)

integer p of (15.6) is called the index of A and s =

Two

XI.

is called the signature.

have the same

re-square Hermitian matrices are conjunctive if and only if they

rank and index or the same rank and the same signature.

The reduction

of an Hermitian matrix to the canonical form (15.6) follows the procedures

Problem 1 with attention


troublesome case is covered

to the proper pairs of

of

in

The extreme

elementary transformations.

Problem?.
See Problems 6-7.

SKEW-HERMITIAN MATRICES.

If

skew-Hermitian, then

is

(FAPy

FAP

(PAT)

Thus,

Every matrix B =

XII.

conjunctive to a skew-Hermitian matrix A is also skew-

FAP

Hermitian.

By Problems, Chapter

14,

= -iA is Hermitian

P such

there exists a non-singular matrix

if

is

By Theorem

skew-Hermitian.

that
Ip

FHP

iFHP

Then

= iF{-iA)P =

FAP

= iC

-Ir-p

and

Up
B

(15.7)

FAP

- ilr~p

Thus,
XIII.

which

Every re-square skew-Hermitian matrix A

is the rank of

A and

p is the

is conjunctive to a matrix (15.7) in

index of iA.

XIV. Two re-square skew-Hermitian matrices A and B are conjunctive


they have the same rank while -iA and -iB have the same index.

if

and only

See Problem

if

8.

CHAP.

CONGRUENCE

15]

119

SOLVED PROBLEMS
1.

Prove: Every symmetric matrix over


r non-zero elements in the diagonal.

of rank

can be reduced to a diagonal matrix having exactly

Suppose the symmetric matrix A = [a^,-] is not diagonal. If a^ / 0. a sequence of pairs of elementary
3, each consisting of a row transformation and the same column transformation,
will

transformations of type
reduce A to

"n2

Now

"ns

the continued reduction is routine so long as b^^.c^^.

that along in the reduction,

we have obtained

S+1, s+2

k^
S+2, s+ 1

'^ri

which the diagonal element k^^^^^^^

however, some

k^j

say

s+^ /

0,

Suppose then

hss

in

are different from zero.

the matrix

= o.

s+i

every

If

we move

4y

*s+l, n

"n,

s->-2

= 0,

we have proved

the theorem with s =

into the (s+l,s+i) position by the proper row and

it

r.

If,

column

transformation of type 1 when u = v; otherwise, we add the


(s+u)th row to the (s+t;)th row and after the
corresponding column transformation have a diagonal element
different from zero. (When a^, = o, we proceed
as in the case Ar^+i ^^.^ = o above.)

we

Since

whose

2.

first t

Reduce

In

are led to a sequence of equivalent matrices, A is


ultimately reduced to a diagonal matrix
diagonal elements are non-zero while all other elements
are zero.

the symmetric matrix

'12

.2

to

canonical form (15.2) and to canonical form


(15.3).

each obtain the matrix P which effects the reduction.


1

2
1

u\n

-1

-2

-2

-1

c
1

[0|^']

To

obtain (15.2),

we have

[0

Pi']

-1

-4

-2

1
1

0-1

-2^f2

-2

k\f2
1

i\Pi

\C\P'\

-1

-4

-2

CONGRUENCE

120

[CHAP. 15

-av^ -2

j\/2

and

5a/2

To

obtain (15.3),

we have

[D\Pl]

-1

Find a non-singular matrix ? such that ?'A? is

2J

k\f2

-i

2\/2

2i

-i

2-j

Vc\?'\

iV2

canonical form (15.3), given

in
1

-2\/2

-2V2

and

3.

2-i

10 + 2i

^
1

[^1/]

1+J

2-j

+i

2-i

10 +

2j

'-\J

3-2J

-i

2j

-1-j

10

10

5+12J

2i

2i

-i

7+
13

4i!'

-5+12i 3-2t
13

13

[c\n
-I

7+

4i

13

-5+12i
1

Here.

13

3-2j
13

4.

Prove: Every ?i-square skew-symmetric matrix A over

B
where

It

D^

= Q.

U :]
then S = ^.

(J

If

= 1,2,

-4

0,

diag(Di, Dg

of rank 2t is congruent over


7)^,0

to a matrix

0)

then some

mj

and
/th and second rows; then Interchange the Jth

oy
a..
j

^ by the skew-symmetric matrix

-a,;,
-Oy

g
2

'S*
i

first

-aji ?^ 0. Interchange the sth and first rows and the


columns and the /th and second columns to replace

Next multiply the

first

row and the

first

column by

l/a^^-

CHAP.

CONGRUENCE

15]

121

1
''2

to obtain

-1

and from

by elementary row and column transformations of type

it,

3,

obtain

4
1

Di

-1

F4

If

5.

/v=

0.

the reduction is complete; otherwise, the process is repeated on F4

Find a non-singular matrix P such that P'AP

a^s 4 0,

we need

-2 -1
-4
3

10
10

1-3

and second columns of

10

-4320

0-3

10
10

-4230

-1 -2

to obtain
1

Next, multiply the

first row and first column by 2; then proceed to clear the


two columns of non-zero elements. We have, in turn.

10

0-3

-2230

-2

to obtain

10

Di

1/10 -1/5

0-10

1/2

-1

Thus when

-2i

-1

-1/5

P'

Do

0-2

-1

1/10 -1

-1/5
=

10-2

P'A?

= diag (Di.Dj).

6.

Find a non-singular matrix P such that P'AP

is in

canonical form (15.6), given

-3 - 2j

two rows and the

-5

Finally, multiply the third row and third column by

1/2

and

first

10
10

-1 -2

10

-2

-2

-2 -1

-1

is obtained.

only interchange the third and second rows followed by the interchange of the
2

third

canonical form (15.4), given

is in

Using

until

-3 + 2i

first

CONGRUENCE

122

1-j -3 + 2i

1+i

[^1/]

[CHAP. 15

-3 -

-i

2j

10
25

2-3i

5_

13

-13

13

13

2j

00

-l-i

-13

2i

2-3f

13

5V13

5\/l3

\/l3

0-1

2i

Vl3

VT.5

vc\p'^

2+

3-2i

3t

5\/l3

\fl3

13

and
5\/T3

7.

^/I3

\/T3

Find a non-singular matrix P such that P'AP is in canonical form (15.6), given
l

- 2i

2+
1

[41/]

2i

2+

3i

2i

3j

-4 +

13

2i

10
HC

10

13

2f

10

5j

-5i

HC

-2-Zi

-5J

-4 +

5i

-4 - 2s

3f

- 3i
-4 - 2

+ 2i

10
0-1

-4 -

-2-2J

5J

-5/2

10

\/T6

x/Io

x/To

[cin
-4 +

4i

ao

\/T0

10

10

and

vTo

4j

\/To

-2-3!

10

'10

10

10
HC

HC

2j

-1 +

CHAP.

8.

CONGRUENCE

IB]

Find a non-singular matrix P such that P'AP

canonical form (15.7), given

is in

-l +

-J

1-i
1

The non-singular matrix

2i

-l-2j

-i

is

+ 2t

1
1

Consider the Hermitian matrix

1 +

123

2j

-i

+i

such that P'HP = diag[l,

1,

-l].

10
Then P'AP

= diag[i.

i.

-i

SUPPLEMENTARY PROBLEMS
9.

Find a non-singular matrix P such that P'AP

(a)

[-:

(6)

-:]

3-1

(a)

"O

O'

(c)

-2

(b)

.2

"

2"

4
4

(d-)

(c)

is in
2i

ky/2

-k

2i

11.

(a)

1
/I

-1
-2

H- j

+ 4iJ

- 4;

-1 -

(b)

(1

(l-J)/\/2

-1
1

(6)

is in

canonical form (15.4). given

-"

(b)

-3/2"'

(c)

12-2

(d)

A =

-2 -3
-

+ 2i)/l3

(c)

+ 0/2

(-3-2i)/l3
(3

5i

-1

4j

i/\/2

Ans.

-3 -

2i

2(1 -J)

2(-i-2,)]

(a)

(d)

-l-2i

+i

Find a non-singular matrix P such that P'AP


(a)

canonical form (15.3). given

+j

Ans.

11.

I
'

|_1

i + 2i
1+211

Find a non-singular matrix P such that P'AP

-1 o1

-1

0,

kV2

[::]

10.

"^V2 -^v^ -1

-1 -2_
1

Ans.

canonical form (15.2), given

is in

0-1
2-3
"0010"

10
10

-2

-2103
2-1-3

'1

(d)

3
2

0-1

-1

-1/3
-2/3 2

1/3

CONGRUENCE

124

12.

Find a non-singular matrix P such that P'AP is

in

+i

tl+

(a)

(6)

'

1-i

Ans.

13.

[1

(a)

canonical form (15.6). given


2

-i +

sfl

(6)

-i

(c)

in

(c)

A
I

(6)

6i

[:
1

(6)

(-2-J)

-1

1-i

-1-j

-i.

2+j

l-2i

-1

-1-2J

_-2 + i

(c)

-r]

-i
1

^1

(l-J)/\/2

-l'

l/\/2

-1

1,

,0

-2 + 3j
-2 -

(rf)

show

l/x/2

(l-2i)/VT0

i/y/2

(-2-i)/yJT0

-l/\/2'
i/\/2

l/\/l0

(a)

(i)

2i

-1-j (-2 + 51)

-1-i

i
Ans.

(c)

l+i

18

1-j

-l+i

3 + 4t

canonical form (15.7). given

-1

3 + 2f

i/Vs

L-i+j

2i

3-4i

(2-f)/V5

1-j

-1-i (-5-i)/\/5

Find a non-singular matrix P such that P'AP is

(a)

3i"|

10

3i

[CHAP. 15

that a 2-square matrix

satisfies

C'DC

14

If

15.

Let A be a non-singular n-square real symmetric matrix of index

if

and only

|Cl=l.

if

L-i oj
Show

p.

that

>

if

and only

if

-p

is even.

16- Prove:

Hint.

17.

non-singular symmetric matrix A is congruent to its inverse.

Take P

BB' where B'AB

= I

and show that P'AP = A

Rewrite the discussion of symmetric matrices including the proof of Theorem

to obtain (15.6) for Hermitian

matrices.

18.

19.

Prove:

It

then A is symmetric (skew-symmetric)

Hint.

if

is

symmetric (skew-symmetric).

7")

(S

- T)

Show

is non-

(S+TT^(S-T)

P'SP =[(S-Tf^(S+T)S~'^(S-T)<.S+T)'^r^.

Let S be a non-singular symmetric matrix and let T be such that (S+T)(S - T) is non-singular.
then T is skew-symmetric.
if P'SP = S when P = (S + TT^iS- T) and I+P is non-singular,
Hint.

21.

and only

Let S be a non-singular symmetric matrix and T be a skew-symmetric matrix such that (S +


singular. Show that P'SP = S when

20.

if

S(/-P)(/ + P)-^

S(I + PrHl-P).

that congruence of n-square matrices is an equivalence relation.

Show

that

chapter 16
Bilinear Forms

AN EXPRESSION
and

(y^, y^

which
Jri)

is linear

and homogeneous in each of the sets of variables


fonn in these variables. For example,

(x^, x^

m)

is called a bilinear

XxJi

2x172

is a bilinear form in the variables {x^, Xg)

The most general

ISxija

and

4x271

15x272

^s/s

(y.^, y^, y-g).

bilinear form in the variables (x^, Xg

and {y^.y^

x)

Jn)

maybe

written as

or,

more

%i%yi

oi2%y2 +

+ 021^271

022^72 +

+ ami^myi +

Om2%y2 +

X i7?i

%r!

fl2ra%7ra

""mv^n Jn

briefly, as

(16.1)

/(^.y)
t

S ai-x^y

=i

j=i

where

[x^, x^

The matrix A

]',

^ =

[0^^],

'

-^

*2> .

and

i'

%1

012

in

'7i'

Ojl

^22

2n

72

_0'n\

Om2

Omra_

= [yi,72

_Yn_

7]'.

of the coefficients is called the matrix of the bilinear form and the rank of

is called the rank of the form.

See Problem
Example

1.

The bilinear form


1

^iXi + ^irs + *2yi + ^2X2 + ^373

Ui.

%.%

X'AY

125

Ti

73

y2

1.

BILINEAR FORMS

126

CANONICAL FORMS.

Let the

[CHAP. 16

new variables

x's of (16.1) be replaced by

u's

by means of the

lin-

ear transformation
(16.2)

Xi

(f=l,2

bijUj,

1.

m)

or

BU

and the n y's be replaced by new variables v's by means of the linear transformation
(16.3)

Ti

(f=l,2

c^jVJ,

J=i

or

n)

Y = CV

We have X'AY = (BU)"A(CV) = U'(BAC)V. Now applying the linear transformations U =


V = lY we obtain a new bilinear form in the original variables X\B'AC)Y = X'DY

Two

bilinear forms are called equivalent

and only

if

if

IX,

there exist non-singular transfor-

mations-which carry one form into the other.

Two bilinear forms with mxn matrices A and B over


they have the same rank.

I.

only
If

if

the rank of (16.1) is

sire

equivalent over

there exist (see Chapters) non-singular matrices

r,

if

and

P and Q such

that
Ir

FAQ
and C =

Taking B = P'

in (16.2)

(16.4)

UXPAQ)V

reduced

in (16.3), the bilinear form is


Ir

U'

U^V^

^2^2

to

U^V^

Thus,
II.

Any

mations over

bilinear form over

of rank

to the canonical form

can be reduced by non-singular linear transfor-

u^^v^

+ u^v^ +

+ u^i'r-

Examples. For the matrix

X^AY

of the bilinear form

X'

ofE^xamplel,

10

10
10

10

10 110
110 10
10
1

10

1
1

10

10

1-1-110

-1

10

10

1-1-110

10

0-110
1

p'

-1

Thus,

-1

10

PU

-1

and

QV

-1

1
1

reduce X'AY to

-l'

1
1

1
1

U%V

U-^V-:^

+ U^V^ + Ugfg

CHAP.

BILINEAR FORMS

16]

The equations

127

of the transformations are

X-j^

X2

Xg

Ur,

and

U,

Ti

72

Ts

^1

"3

1^3

V^

^3

See Problem 2

TYPES OF BILINEAR FORMS. A

bilinear form

1 S

X'AY

ij x
'^iJj

a,-,-

symmetric

symmetric

alternate

skew-symmetric

according as A is

Hermltlan

Hermltlan

alternate Hermltlan

COGREDIENT TRANSFORMATIONS.
^) and

(i. 2

tion

CU

is

IV.

CV

skew-Hermltlan

Consider a bilinear form X'AY in the two sets of n variables


When the ;'s and y's are subjected to the same transforma-

the variables are said to be transformed cogredlently.

Under cogredient transformations

III.

where A
If

(yi.ys. .3^)-

and Y =

is called

CU

Y = CV,

and

is re-square, is carried into the bilinear form

We have

the bilinear form X'AY.

U\C'AC)V.

symmetric, so also is C'AC; hence,

symmetric bilinear form remains symmetric under cogredient transformations of

the variables.

V.

Two

variables

if

F are equivalent under cogredient transformations of the


matrices are congruent over F.

bilinear forms over

and only

From Theorem

I,

if their

ChapterlS, we have

VI. A symmetric bilinear form of rank


transformations of the variables to

(16.5)

%%yi

From Theorems

II

and IV, Chapter

15,

can be reduced by non-singular cogredient

02*272

OrXryr

follows

VIl. A real symmetric bilinear form of rank r can be reduced by non-singular cogredient
transformations of the variables in the real field to

(16.6)

and

iyi
in the

complex

(16.7)

%y2 +

xpyp

2y2

a;^4.i

7^4.1

x^Jr

field to

%yi +

x^y^

See Problem

CONTRAGREDIENT TRANSFORMATIONS.

Let the bilinear form be that of the section above. When


X = (C~^yU and the y's are subjected to the trans-

the x's are subjected to the transformation

formation

Y = CV,

3.

the variables are said to be transformed contragredlently.

We have

BILINEAR FORMS

128

X'AY, where A
IX.

only

if

The

= (C-K^T
)'U and

X'lY =

bilinear form

x^y^ + x^y^ +

%y

Y = CV,

the bilinear form

U\C AC)V.

is n-square, is carried into the bilinear form

transformed into itself

is

if

and

the two sets of variables are transformed contragrediently.

FACTORABLE BILINEAR FORMS.


X.

Under contragredient transformations

VIII.

[CHAP. 16

Problem 4, we prove

In

non-zero bilinear form is factorable

and only

if

if its

rank is one.

SOLVED PROBLEMS
1.

%yi

+ 2iy2

13%yg

4a^yi + 15x^y^

%. 2

-13]

x^y^

= X'

Ji

-1

4 15

Jx
1

-13l

-4

15

-ij

73

2.

Reduce %yi + 2x^y^ + 3%y3

Ix^y^ + 2%yi

2^y2 + x^y^ + 3x2y4 +

+ 4%y3 + x^y^ to canon-

"ix-^y^

ical form.
2

The

matrix of the form is

-2

-1/6 -5/6

and

-1 -1

By Problem

6.

Chapter

5.

the non-singular matrices

1/3 -4/3 -1/3

10

3-2

2-2

7/6

10

/g

PAQ

are such that

Thus, the linear transfor-

14

mations

P'U

or

X^

:x:2

Ui

2U2
U2

Uq

U3

and

y
6

QV

or

"3

xs =

7a
reduce X'AY to u^v^ +

3.

U2t)2.

Reduce the symmetric bilinear form

mations

X'AY = X'

to (16.5) in the rational field,

12

10

10

-8

(b) (16.6) in

by means of cogredient transfor-

the real field, and (c) (16.7) in the complex

field.
1

(o)

Prom Example

1,

Chapter

15. the linear transformations

-2 -10 -1
4 -1
1

u.

=
1
1

-2 -10 -1

4-1

=
1

10

CHAP.

BILINEAR FORMS

16]

reduce

XAY

to

uj^'i

"2% +

129

^usfg.

(6)

Prom Example

Chapter

2,

the linear transformations

15,

-5 -2 -1
1

-1

u.

-5 -2 -1
2

1-1
1

reduce X'AY to u^v^ + ug^s


(c)

Prom the

result of

Example

- "3%Chapter

2,

000'

-5
-2

-1

we may obtain

15.

-1

-5202
-2j

-1-110

Thus, the linear transformations

10
10
10

-5 -2J -1
i
-1
2

u.

-5 -2J -1
-1
2
t

reduce X'AY to

U^V^ + U2V2 + UgVg.

4.

Prove:

non-zero bilinear form f(x,y) is factorable

Suppose the form

if

and only

if its

1.

is factorable so that

and, hence, a^j = b^cj.

Clearly, any second order minor of

aij

aj5

a^bj

aib^

fk^'j

fe*s_

= [a^-], as

bib^
Lfei "ks_

vanishes.

rank is

Thus the rank

of

/4

"k

"k

is 1.

Conversely, suppose that the bilinear form is of rank 1. Then by


Theorem I there exist non-singular
linear transformations which reduce the form to U'(BAC)V = u^v^.
Now the inverses of the transformations
=

carry u^u^ into

(S rijXj)(l s^jyp

jnr

f(x.y)

and

'^jyj

and so f(x.y) is factorable.

BILINEAR FORMS

130

[CHAP. 16

SUPPLEMENTARY PROBLEMS
5.

Obtain linear transformations which reduce each of the following bilinear forms to canonical form (16.4)
(a)

x^j^

- 2^73 +

+ ^272 - 3*273 - ^sTi - ^72

"ix^j^

(6)

-11

-4

7 y.

(c)

-2

6.

-2
-2

X'

5-510

^sTs

10

2-510

(rf)

12

10

Obtain cogredient transformations which reduce

(a)

14

y and

Ans.

(a)

(6)

X'

10

5_

-2 -7

to canonical form (16.6).

'l

(6)

-3 -4V3/3

\/3/3

\/3/3

Ir
7. If Bj^,

B2. Ci

C2 are non-singular n-square matrices such that

B-^A^C^ = B^A^C^ =

find the transfor-

mation which carries X'A^Y into UA2V.


Arts.

= (B'^B^iU,

8. Interpret

Problem

23,

C^C^V

Chapter

5, in

terms of a pair of bilinear forms.


-

9. Write the

transformation contragredient to

.Y

1
1

Prove that an orthogonal transformation

11.

Prove: Theorem IX.

is

/.

.4ns.

-2
i
2

10.

221

~2

contragredient to itself, that is

.Y

PV Y
.

PV.

-1
12. If

X'AY

is a real

non-singular bilinear form then

XA Y

is called its reciprocal bilinear form.

Show

that

when

reciprocal bilinear forms are transformed cogrediently by the same orthogonal transformation, reciprocal bilinear forms result.

13.

4, Chapter 15, to show that there exist cogredient transformations


duce an alternate bilinear form of rank r = 2t to the canonical form

Use Problem

U1V2

14.

W2V-L

+ U3V4.- u^V3 +

+ ugt-i "^t - "sf^et-i

Determine canonical forms for Hermitian and alternate Hermitian bilinear forms.
Hint. See (15.6) and (15.7).

PU. Y

PV

which

re-

chapter 17
Quadratic Forms

A HOMOGENEOUS POLYNOMIAL

of the type

n
(17.1)

whose coefficients
^It ^2

Example

a^,-

"u

X'AX

are elements of

is called a quadratic form

over

in the variables

^7J'

1.

2
as^^

+ 2*2 -

trix of the form

2
"^x^

The maways according as the cross-product terms -4x1x2

+ 8x^x^ is a quadratic form in the variables xi.x^.xg.

'ix^x^

may be

written in various

and 8x1x3 are separated to form the terms

012X1X2.021^*1

and

013X1X3,031X3X1.

We

shall

agree that the matrix of a quadratic form be symmetric and shall always separate the crossproduct terms so that a
a^ Thus,

The symmetric matrix A =


A

-2

0-7

4Xj^X2

SXj^Xg

is called the matrix of the quadratic form

[o^^- ]

is called the rank of the form.

X'

7Xg

"-^s

'

If

the rank is r<n,

and the rank of

the quadratic form is called singular;

otherwise, non-singular.

TRANSFORMATIONS. The

linear transformation over F,

with symmetric matrix A over

(17.2)

(.BYyA(BY)

BY

carries the quadratic form (17.1)

into the quadratic form

YXB'AB)Y

with symmetric matrix B'AB.

Two

quadratic forms in the same variables %, xz

there exists a non-singular linear transformation

one of the forms into the other.


I.

The rank

X=BY

are called equivalent if and only if

which, together with Y =IX, carries

Since B'AB is congruent to A,

we have

of a quadratic form is invariant under a non-singular transformation of the

variables.
II.

Two

quadratic forms over

are equivalent over

if

and only

if their

matrices are

congruent over F.

Prom Problem

1,

Chapter

15, it

follows that a quadratic form of rank

the form
(17.3)

K^i

Kr^

131

+ h^y^,

hi/=o

can be reduced to

QUADRATIC FORMS

132

in

[CHAP. 17

which only terms in the squares of the variables occur, by a non-singular linear transformaX = BY. ViTe recall that the matrix B is the product of elementary column matrices while 6'

tion

is the product in reverse order of the

-2

Example

2.

Reduce

X'

same elementary row matrices.


4

Example

of

to the form (17.3).

-7

We have

[A I]

-2

-2

o'

BY

0-2

-23

-4

0-2

Thus.

0-7

reduces q to

1
1

[D Bl

=
1

q'

y=-2v^+9v=.

See Problems

1-2.

LAGRANGE'S REDUCTION. The

reduction of a quadratic form to the form (17.3) can be carried out


by a procedure, known as Lagrange's Reduction, which consists essentially of repeated com-

pleting of the square.

Example

3.
1

4x^*2

7%;
3

"*"

'

^*1*3

[x^- 'ix^{x^-2x^)\ + 2x1

7^:

{xl-'^x^{x^-2x^) + i{x^-2x/\ + 2x1 - Ixl -

(x^-2x^ + ix/ - 2(xl-8x^x^)

Ux^-2xJ

2Sx

(x^-2x^ + 'ix^f - 2(.xl-Zx^x^-\-\Qxl, + Qxl

(\-2^2+H)'

Thus,

yi

ys

xx

- 2x2 +
xq^

4acs

- 4x3

ot

Xg

73

reduces q to

- 2(A:^-4^3f + Qxl
+

2y2 +

xi

X2

y2 + 4y3

X3

ys

yi

4ys

y^ - 2y^ + 9y^.

See Problem

REAL QUADRATIC FORMS.

3.

q = X'AX be reduced by a real non-singular


one or more of the h^ are negative, there exists anonsingular transformation X= CZ, where C is obtained from 6 by a sequence of row and column
transformations of type 1, which carries q into

Let the

real quadratic form

transformation to the form (17.3).

(17.4)

in

sz^
s,z7

11

s z^
s^z2 2

If

...

Sfyzj,

s*+iZa+i

...

s^;.

which the terms with positive coefficients precede those with negative coefficients.

CHAP.

QUADRATIC FORMS

17]

Now

133

the non- singular transformation

Wi
"'i

VSi Zi

'j

= 1,2

(/

=r+l,r+2

r)

71)

or

'

;-

'''4k-

.1.1
J

carries (17.4) into the canonical form

wl + wl +

(17.5)

w^^

;2^^

wl

Thus, since the product of non-singular transformations is a non-singular transformation,

we have
Every real quadratic form can be reduced by a real non-singular transformation to
the canonical form (17.5) where p, the number of positive terms, is called the index and r
III.

is the rank of the given quadratic form.

Example

4.

In

Example
o

y^
q" =

into

J
reduces
q

"

^12

the quadratic form

2,

a^ + 2x^

8xx
1213

- Ix^ - ixx
3

was reduced

to

The non-singular transformation yi = z^, y^ = 23, ys = 22 carries q'


2y^ + 9yQ
~
+
^^3 ^^^ ^^^ non-singular transformation 21 = w^, 22 = w^/S, zg = wa/v^
922
zf
.

i.

to

"'

2,2
w^+
w^ -

w^.

Combining the transformations, we have that the non-singular linear transformation


Wi + ^W2 + V^M>3

Xl

X2

|mJ2 + 5V^;g

X3

3"^

4/3

v^

4/3

2^

1/3

qr

to

SYLVESTER'S LAW OF INERTIA.

W^ +

In

wl-

The quadratic
q

w%.

Problem

we prove

5,

form is of rank 3 and index

2.

the law of inertia:

IV. If a real quadratic form is reduced by two real non-singular transformations to


canonical forms (17.5), they have the same rank and the same index.

Thus, the index of a real symmetric matrix depends upon the matrix and not upon the
mentary transformations which produce (15.2).

ele-

The difference between the number of positive and negative terms, p - {r-p), in (17.5) is
called the signature of the quadratic form. As a consequence of Theorem IV, we have
V.

Two

and only

if

real quadratic forms each in n variables are equivalent over the real field if
they have the same rank and the same index or the same rank and the same sig-

nature.

COMPLEX QUADRATIC FORMS.

Let the complex quadratic form X'AX be reduced by a non-singular

transformation to the form (17.3).

It is

clear that the non-singular transformation

2i

^iVi.

(i

Zj

yj

(7

= r+1, r+2,

1.

r)

...,7i)

QUADRATIC FORMS

134

[CHAP. 17

Ul
1

1, 1,

-'-

''''{/h^'/h;'-

)^

carries (17.3) into

-4--

<

(17.6)

Thus,

Every quadratic form over the complex

VI.

field of rank

can be reduced by a non-

singular transformation over the complex field to the canonical form (17.6).

Two complex

VII.

quadratic forms each in n variables are equivalent over the complex

field if and only if they

have the same rank.

DEFINITE AND SEMI-DEFINITE FORMS. A

if its

rank and index are equal.

positive definite quadratic form can be reduced to

of values of the x's, ^ >

rank and index are equal,

the

q>

a;'s,

be reduced

-yl

i.e.,

y^ + yj

to

=X'AX,

= p<

n.

"^

\a\

=0,

is called positive semi-definite if its

Thus, in the real field a positive semi-definite quad+ y^ r< re,


and for any non-trivial set of values of
,

0.

real non-singular quadratic form

A
i.e.,

-^

q = X'AX is called negative definite if its index p = 0,


= n,p = 0. Thus, in the real field a negative definite form can be reduced to -y^ - y^ - y^ and for any non-trivial set of values of the x's, q < 0.

i.e.,

j\-^ J^-^

0.

real singular quadratic form

ratic form can

q = X'AX, \a\ f 0, in
Thus, in the real field a
y^ and for any non-trivial set

real non-singular quadratic form

n variables is called positive definite

real singular quadratic form q =


<

re,

p =

- y^ Clearly,

X'AX

is called negative semi-definite if its index p = 0,

Thus, in the real field a negative semi-definite form can be reduced


^f ^y non-trivial set of values of the x's, q 5 0.
^"^
y^

if

0.

to

q is negative definite (semi-definite), then -q is positive definite(semi-definite).

For positive definite quadratic forms, we have

=X'AX

VIII. If q

is positive definite, then

U| >0.

PRINCIPAL MINORS. A

minor of a matrix A is called principal if it is obtained by deleting certain


rows and the same numbered columns of A. Thus, the diagonal elements of a principal minor of
A are diagonal elements of A.
In Problem
IX.

6,

we prove

Every symmetric matrix of rank

has

at least

one principal minor of order

r differ-

ent from zero.

DEFINITE AND SEMI-DEFINITE MATRICES. The

matrix i of a real quadratic form q = X'AX is called definite or semi-definite according as the quadratic form is definite or semi-definite. We have
X.

real symmetric matrix

singular matrix
XI.

C such

that

A =

real symmetric matrix

exists a matrix

of rank

is positive definite if and only if there exists a non-

C'C.

A of rank r
A = C'C.

is positive semi-definite if

and only

if

there

such that

See Problem

7.

CHAP.

QUADRATIC FORMS

17]

XII.

If

135

is positive definite, every principal minor of

is positive.

See Problem
XIII. If

is positive semi- definite, every principal minor of

REGULAR QUADRATIC FORMS.

For a symmetric matrix A =

is non-negative.

over F,

[a^,-]

8.

we

define the leading

principal minors as

PO =

(17.7)

In

Problem

Pl=

1,

9,

O12

021

022

P2 =

Oil.

04.2

%3

Ogi

052

O23

Oqi

O32

'^3

%1

Oil

Pa

Pn= Ul

we prove

XIV. Any re-square non-singular symmetric matrix A can be rearranged by interchanges


of certain rows and the interchanges of corresponding columns so that not both p_^ and
Pn-2 are zero.

XV. If A is asymmetric matrix and


have opposite signs.

Example

5.

For the quadratic form

X'AX

Oigs

t^

0;

12

po =

Kg^X

X'

12 14
2

for

which po =

1,

Pi =

1,

P2 =

0.

Ps = -1.

and p

P4=

pi =

1,

1,

p2 =

0,

p3 =

0,

P4 =

m=

1.

yields

'1112
112 2
(i)

then p_2

X,

the transformation

but Pn-i = 0,

Pn-iPn^^

112
112
2

Here

if

1-

Thus,

for

i) not both p2 and ps are zero.

symmetric matrix A of rank r is said to be regularly arranged if no two consecutive p's


sequence po.pi, -.Pr ^^^ zero. When A is regularly arranged the quadratic form X'AX
is said to be regular. In Example 5, the given form is not regular; the quadratic form (i) in the
same example is regular.

in the

Let i be a symmetric matrix of rank

r.

By Theorem

A contains

IX,

least one non-

at

vanishing r-square principal minor M whose elements can be brought into the upper left corner
of A. Then p^ ^^
while p^^.^ = p^^.^ = ... = p = 0. By Theorem XIV, the first r rows and the
first r columns may be rearranged so that at least one of Pr-i and p^_2 is different from zero.
If p^_^ j^
and p^_2 =0, we apply the above procedure to the matrix of Pt--i; if Pt--2^0,

we apply

the procedure to the matrix of P7-_2;

^^^^

so on, until

XVI. Any symmetric matrix (quadratic form) of rank

is regularly arranged.

Thus,

can be regularly arranged.

See Problem

all

XVII. A real quadratic form X'AX is positive definite


leading principal minors are positive.
XVIII.

real quadratic form

each of the principal minors po.Pi

X'AX of rank
Pr

if

and only

if its

rank is

re

10.

and

is positive semi-definite if and only if

is positive.

QUADRATIC FORMS

136

[CHAP. 17

KRONECKER'S METHOD OF REDUCTION. The


into one in

method of Kronecker
which only the squared terms of the variables appear

XIX.

X'AX

q =

is a quadratic form over

for

is

reducing a quadratic form

based on

in n variables of rank

singular linear transformation over F it can be brought to q' =


singular r-rowed minor C oi A occupies the upper left corner of B.

F which reduces

a non-singular linear transformation over


quadratic form in r variables.

X'BX

r,

then by a non-

which a nonMoreover, there exists


in

q" = X'CX, a non-singular

q to

XX. li q = X'AX is a non-singular quadratic form over F in n variables and


a ^ 0, the non-singular transformation

if

Pn-i =

re-1)

1, 2.

^nnyn
or

10

a^n

BY
1

<^n-i,

carries q into

HiJiYj + Pn-xPnYn

which one squared term in the variables

in

has been isolated.


1

-2

-2

4
1

Example

6.

For the quadratic form

X'AX

= X'

-^i

P2 =

-2 ^ 0.

-2

0-7

-2

O^S

non-singular transformation

Xl

xs

ys +

a.13

ys

- 8y3

yi

a23 ys

8y3

2ys

0-2

ss ys

-8

ys -

-8

reduces X'AX to
1

-2

-2

0-7

0-2

Y'

8 -8 -2

in

XXI.
but

which the variable yg appears only

q =

a,
're,?!-! ^
'

0,

-8

in

-8

Y'

-2

-2

36

squared form

X'AX is a non-singular quadratic form over


the non-singular transformation over F

F and

if

o^-i, n-i

"

xi

yi

CLi,n-iyn-i

Xn

^n-i.n yn.

*ra-i

(i

'^inyn,

1, 2,

...,n-2)

a n n-i Yn-
,

or
1
1

sy

...

!, ^.

...

a2 n-1

=
1

a.-2,

_i
n-1

...

,0

...

a
'-'-n.-2, n
'-n-i,

a__i

= ^nn

The

CHAP.

QUADRATIC FORMS

17]

n-2

n-2

carries 9 into

(Hjyiyj

1'^n,n-\Pnjn-\yn-

i-i j-\

The

137

further transformation
'

Ji

Jn-x

2
V

1,

n-2)

2n

Zn

n-2

n-2

yields

Zn-i

Zn-i

Jn

(i

Zi,

a^jz^zj +

.2

2a_^_j^p(zn-i

- z^

in

which two squared terms with opp-

J -1

site signs are isolated.

Example

7.

For the quadratic form

X'AX

^22

- O'ss =

but

reduces X'AX

xi

X2

xq

*i3ys

*23r3

-1

or

0-10

a 32 72

-1

The non-singular transformation

0.

cti272

yi

i^

to

-1

tXg2 =

X'

-1

-]

rSY

yl

+ 2y,y3

The transformation
)'l

72

22

23

-1

73

22 +

23

21

carries

Y'BY

into
1

Z'

-1

o'
1
1

-1

z'

<

2zi

-2

Consider now a quadratic form in n variables of rank r. By Theorem XIX, q can be reduced to 9i = X'AX where A has a non-singular r-square minor in the upper left hand corner
and zeros elsewhere. By Theorem XVI, A may be regularly arranged.
If

pr-i

0,

Theorem

XX

can be used to isolate one squared term

(17.8)

Pr-iVrYr

If pr-i =
but a.r-1, r-i ^0, interchanges of the last two rows and the last two columns
yield a matrix in which the new pr-i = a^_i^^_i f 0. Since pr-2 ^ 0, Theorem XX can be used
twice to isolate two squared terms

(17.9)

Pr-2

r-i,

r-iJ

a r-i., r-iPrYr

which have opposite signs since pr-2 and pr have opposite signs by Theorem XV.

QUADRATIC FORMS

138

If

used

to

and a^_i_r-i = 0,
isolate two squared terms

[CHAP. 17

then (see Problem 9)

pr-L =

a,-,

and Theorem XXI can be

r-i ^

2ar,r~ipr{yr-i-yr)

(17.10)

having opposite signs.

This process may be repeated

until the given quadratic form is

reduced

to

another con-

taining only squared terms of the variables.

sequence pr-i,Pr
presents a permanence or a variation of sign. In (17.9) and (17.10) it is seen that the sequences p^_2, oLr-i. r-v Pr ^"^^ Pr-2' ^r, r-v Pr Present one permanence and one variation
Thus,
of sign regardless of the sign of dr-i, r-i and a.^, r-i
In (17.8) the isolated term will be positive or negative according as the

XXII. It q = X'AX, a regular quadratic form of rank r, is reduced to canonical form


by the method of Kronecker, the number of positive terms is exactly the number of permanences of sign and the number of negative terms is exactly the number of variations of
sign in the sequence

where a zero in the sequence can be counted either

P^,

po, pi, P2

as positive or negative but must be counted.

See Problems 11-13.

FACTORABLE QUADRATIC FORMS.

Let X'AX f

0,

with complex coefficients, be the given quad-

ratic form.

Suppose X'AX factors so that


X'AX

(1)

+ 02*2 +

(oj.%1

+ ;)( iiXi +

b.2_X2

+ + b^x^)
0--

to.;

hi
J

.^

tj

is non-singular.

becomes

Let the

The non-singular transformation

transforms
If

i )

into

y-^y-z

of rank

yx

OiaCi

+ 02*2 +

+ 0

y2

^1*1 + ^2^2 +

+ ^n^n

ys

Xs

Thus,

2.

x^

1) is of rank

2.

the factors are linearly dependent, at least one element a^^f 0.

their coefficients be renumbered so that o^ is ci

y?

transforms (i) into

Conversely,

if

!Y^

a^Xx + 02*2 +

72

X2

of rank

X'AX has rank

Thus, (1) is of rank

1.

or 2

it

Let the variables and

The non-singular transformation


+ (^n^n

Xn
1.

may be reduced respectively by Theorem VI

to y^ or

r^ + y^, each of which may be written in the complex field as the product of two linear factors.

We have proved
XXIII.

linear factors

quadratic form
if

and only

if its

X'AX ^
rank is

with complex coefficients is the product of two

r<2

CHAP.

QUADRATIC FORMS

17]

139

SOLVED PROBLEMS
1.

Reduce

q = X'AX = X'

Prom Example

1,

12

10

Chapter

[^1/]

10-8

15,

12

10

10

-8

PY

10
-2100

0-100

c
^\J

-10

-2

-10 -1

4 -1

iD'.P']
4

-1-110

Thus, the transformation

form (17.3).

to the

reduces q to the required form y^

yl^'^yl-

2.

Reduce

We

q = X'AX = X'

4 8

to the

form (17.3).

find
1

[^1/] C

2
1

Thus, the transformation

-4

PY

-i Y

1
1

3.

[D\P']

reduces q to y^ + 8y^ - 2y^

Lagrange reduction.
(a)

2
,...2
2
2x^ + 5^2 + ,
19*3

2{xf+ 2x^(2x^ + 3x^+2x^)1 + 5x^+ 19x1 " 24*4 + ^S^j^g - 8x^x^ - IGx^x^

2{xl+ 2x^(.2x^+Zx^+2x^) + {2x^ + 3x^ + 2x^f\

- 24x^+

8a:^a=2

+ I2x^x^ + Sx^x^ + l%x^x^ - 8x^x^ - IGx^x^

+ bxl + 19*2 - 24*2 ^ jg^^^^


~ 8*2^^
g^^^^ _ ^^
"2*3 _
=

2 (*i

+ 2*2 + 3*g + 2*^)2 - 3

{*^ + 2*2(*g+ 4*4)

2 (*i

+ 2*2 + 3*g + 2*^)2 _

(^ + ^ + 4^^)2

+ 4 (*g

\yi =
Irs

+ *3 - 32*2 -

^q_^

- 2*4)2

*i + 2*2 + 3*3

ryi

Thus, the transformation

_ 2(2*^ + 3v +2*,)2

*2 +

+ 2*4

*s + 4*4

*3-2*t

'Educes,

to

00^

2y2_3y22+4y2.

*4

(i)

For the quadratic form of Problem


9

*2 +

4*^*2 +

2,

we have

4*^*3 + 4*2 +

Since there is no term in *2 or *2 but a

tenii in

16*2*3 + **

*2*3

we use

(*^

+ 2*2 + 2*3)2 + 8*2*3

the non-singular transformation

QUADRATIC FORMS

140

Xl

i )

Zl

A;2

22

[CHAP. 17

Z2 + ^3

Xs -

to obtain

(2i

+ 4z2+2z3f'

+82^+82223

422+2z3)2+8(22+2Z3f - 2z2

(z^ +

Now Y

4 2
1

X
2

Z and

from

i),

-1

'1

Thus, the non-singular transformation

4 2
1

y^

+ ^y^ -

-1

2yi^

-1

ii
1

6"

-4

2
I

X\ hence,

y effects the reduction.

4.

Using the result of Problem

2,

[^1/] C

-4

-2

-2

i
2

and applying the tiansfoimations HgCiV^). K2(i+-\/2) and ^3(5 a/2). ^3(2

[^!/]

0-2

1
1

-2

Thus, the transformation

Prove:

-iVl

-V2
k^f2 -\\f2 y

= (^y

[cic']

0-1

q = X'

reduces

iv^

i\f2

2,2

5.

10

we have

),

10

-|

10

V2

ii\/2

to the canonical form

,2

If

a real quadratic form q is carried by two non-singular transformations into two distinct

reduced forms
(i)

y; +

yg

y^

and
<ii)

y' + yl +

yq

yq,i

yj.2

y'

then p = q

Suppose q>p. Let


mation which produces ( ii

X
).

FY

be the transformation which produces (1) and

Then

F~^X

+ binxn

+ b^nXn

+ bn2X2 +

cii% + ci22 +

C21X1 +

filial

bi2X2 +

621X1

622*^2

bjiixi

+ bnnpcn

and

C222 +

+ C2ren

Cin^n

G'^X
cr!.i;i

cn2*:2

cnnXn

GY

be the transfor-

CHAP.

QUADRATIC FORMS

17]

respectively carry (i) and (ii) back into

(iii)

*i2*2+- +

*in^n)^ +

...

+ {bp^'^^ + bp^x^-\-- + bp^Xyf

(CiiA:^

+ c^2*2+- + c^^f +

+ (cq^x^ +

+ l.l^l'''

('^g

r-q+p <

Consider the

+ 6i2 *2 +

bpiH
By Theorem

+ *;^2*2 +

(*;b+i,i'i

"*"

+1,2*2

equations

^g+i,i*i + '^(7+1,2*2 +
't?+2,l*l

+ 6jbn% =

+ ^^+1,2*2 +

(-^11

i +

%22 +

^ri

+ ^2*2

*i

+ bp^i,rf-J^

q<p

6.

will also lead to

'^g

M*re =

+ c^^x^

(Ot^.ttg

0!^^).

When

this

solutionis

(Vll + V22+--- + V?In)

+ c.^n'^nf +

+ '^9+2,2*2 +

+ (<^7ii + ^(?22 +

Clearly, this requires that each of the squared terms be zero.

q<p. A repetition
a contradiction. Hence q=p.

contrary to the hypothesis.

x^f

+ l,/!^^

'^q

+ 6^

x^+- + c

Chapter 10, they have a non-trivial solution, say


iii ), we have

IV,

substituted into

"^g

+ *in*n =

''21*1+^22*2 +

Thus,

q.

(''ii*i

*ii

141

Thus,

Prove: Every symmetric matrix A of rank

has

>an)^

But then neither F nor G is non-singular,

of the above argument under the

at least

assumption that

one principal minor of order

different

from zero.
Since A is of rank

r,

it

has

at least

stands in the rows numbered Ji,i2


matrix and let the columns numbered

Now

one r-square minor which is different from zero. Suppose that it


these rows be moved above to become the first r rows of the
i^ be moved in front to be the first r columns.

I^^t

'rfi,s2

rows are linearly independent while all other rows are linear combinations of them. By
taking proper linear combinations of the first r rows and adding to the other rows, these last n-r rows can
be reduced to zero. Since A is symmetric, the same operations on the columns will reduce the last n-r
columns to zero. Hence, we now have
the first

ili2

H2i2

%i2

"Vii

in

which a non-vanishing minor stands

pal minor of

7.

Prove:
matrix

A
C

in the

"vv

upper left hand corner of the matrix.

Clearly, this is a princi-

real symmetric matrix

of rank

of rank

is positive semi-definite if

and only

if

there exists a

such that A = C'C.

Since A is of rank

r,

its canonical form

'--[t:]

Then there exists a non-singular matrix B

QUADRATIC FORMS

142

such that A
of rank

B%B.

and A

Since

Ai = A^

A'i'=

we have A

[CHAP. 17

= B'NxB = B'N^Ny^B = B'N(-N^B

Set

= A^B; then

is

as required.

= C'C

Conversely, let C be a real n-square matrix of rank

r;

then A = C'C is of rank s<r.

Let

its

canonical

form be

N^

diag(di,

^5,0,0

(^2

0)

where each di is either +1 or -1. Then there exists a non-singular real matrix E such that E'(C'C)E =
CE = B = [bij\. Since B'B = N^, we have

A^g

Set

(i =

1,

ii =

s+l,s+2

s)

and
+

*ii

d;>0 and A

Clearly, each

8.

Prove:

If

is positive

*i2

^jn

0'

semi -definite.

is positive definite, then every principal minor of

n)

is positive.

Let q = XAX. The principal minor of A obtained by deleting its ith tow and column is the matrix A^ of
the quadratic form q^ obtained from q by setting x^ = 0. Now every value of qj_ for non-trivial sets of values of its variables is also a value of g and, hence, is positive. Thus, Aj_ is positive definite.

This argument may be repeated for the principal minors


... rows and the same columns of A.

A^j,

obtained from A by deleting two,

A^^j-^, ...

three,

By Theorem

Ai>

VI,

A^j>

0,

0,

...

thus, every principal minor is positive.

Prove: Any ra-square non-singular matrix A = [ay] can be rearranged by interchanges of certain
rows and the interchanges of corresponding columns so that not both p_^ and p_2 are zero.
Clearly, the theorem is true for A of order

when

a^

p^_^ =

Suppose

(a)

j^

Suppose

0.

some CL^

?^

0.

01^ =

and of order

then either (a) some a^^

2.

Moreover,

it is

true for

of order

n>2

or (b) all a^^ = 0.

?^

After the ith row and the ith column have been
new matrix has p_^ = CC^j^ 0.

moved

to

occupy the position

of the last row and the last column, the

Suppose (6) all a^i = 0, Since \A\ J^ 0.


and the ith column into the (n-l)st position.
a

a
n, n-i

^n-i,n-i

and

at

least one a^i i^


new matrix

0.

Move

_i,n =

In the

Vi,n

^n-i,n

the ith row into the (n-l)st position

\n-i

^-

-a.n-i,n

^^

(^-^^^ '^ ^^''^

PnsPn

Vi,n

Pn^^O.
Note that this also proves Theorem XV.

10.

Renumber the variables so

that

q =

X'AX = X'
2

13

is regular.

1111
Here

po

1,

pi =

0,

ps =

0,

Since

ps = -4, p* = -3.
2

of p3

The cofactor B22

?^

pi = P2 =

0,

ps ^

0,

we examine

the matrix

the interchange of the second and third rows and of

CHAP.

QUADRATIC FORMS

17]

the second and third columns of

143

A yields
2
2

11

1111
for

11.

which p^ -

1,

p^ =

p^ = -4, pg = -4, p^ = -3.

0,

Reduce by Kronecker's method

Here, X2 has been renumbered as xq and

12 3
15

as xg

X'

;<:g

X.

Here Pq - 1, p^ - 1, p^
and q is regular. The sequence of p's presents one
3, pg - 20, p^ = -5
permanence and three variations in sign; the reduced form will have one positive and three negative terms.
Since each pj /

repeated use of Theorem

0,

XIX

yields the reduced form

PoPiTi + PiPgrl + PsPsrl + PsP^rl

12.

Reduce by Kronecker's method

Here A is

of rank 3 and

fflgg

= X'AX = X'

An interchange

i 0.

y\

12

13

- eo^g -

"^yl

looy^

X.

of the last two rows and the last two columns carries

12 13
2

into

2 4 3 6

13

which

in
5

4 3

3 6 2 9

Now

has been reduced

to

XCX

= X'

12 10

Since S is of rank

0.

3, it

can be reduced to

2 1

4 3

3 5

for

form will contain two positive and one negative term.

which p^ =

Since p =

p^ =

1,

1,

p^ =0, pg = -1.

but

= 4

i^

0,

2 4 3

13

The reduced

the reduced

form is by (16.8)

PoPiri +

Pxy-i-zJi

1-2
13.

Reduce by Kronecker's method

-2

2-12
Here p^

tive terms.

- 1, p^ -

1,

p^ -

0,

Pg

Consider the matrix B


L

9,

p^ = 27

-2

ij

yl + 4y| -

4y|

1-1

1112

= X'

y-iiP^yl

X.

the reduced form will have two positive and two nega-

of pg.

Since /^gg =

but /3g2

=-3^0

the reduced for

is by (16.8) and (16.9)

PoPiTi^ + 2/332 Pg (y| - y|) + pgp^y^

yl + 54y|

54^3^

- 243y|

QUADRATIC FORMS

144

For a set of real ra-vectors Xi, X^,

14. Prove:

Ai

[CHAP. 17

Xp,

Ai

A]_

A2

;t2-x

A2 ^2

Ag Ai

'

>

Z^ Zi

Xp- X2

the equality holding

and only

if

Xp- Xp

the set is linearly dependent.

if

P
(a)

Suppose the vectors Xi are linearly independent and

0<Z-Z

and

)( 2 X:xA

X-x.

j=i

1=1

X\X':XAX
t-j'

Since this quadratic form is positive definite,

(6)

let

G >

= [i,2

r(A:^-;f,-)^
t

X^x^

i=i

X'GX

J'

0.

Then there

Suppose the vectors X^ are linearly dependent.

Then Z

xp]' f 0.

exist scalars

kp, not all zero,

k^^^.k^

such that

S
f = i=i

4,,jy.^

X..^

and, hence, such that

+ k^Xj

kiXjj-Xi

Thus the system

X2 +

+ kpXj

Xp

= 1.2

p)

homogeneous equations

of

Xj-X^xx + Xa-Xqxq +

'J

has a non-trivial solution xi=

We have proved

that

k;

G >
1

(i = 1,2

+ Xa-X,
'j
'

p),

P ^P

and \G\

p)

0.

To prove the converse of

0.

= 1,2

we need only

(b).

assume

to

reverse the steps of (6) to obtain

^=

^.

0,

if

= 0,

;?,

= 0, (/ = 1,2

and

=
|

S kjXy^ =

Thus,

f= l^k^Xi.

where

p)

p
i= 1

and the given vectors Xj are linearly dependent.

SUPPLEMENTARY PROBLEMS
15. Write the following quadratic forms in matrix notation
(a)

xl + 4*1^2 +

34

(6)

2x1 -

6j;i2

<<^)

^s

^1

" 2^| -

3a:

12

2-3
Ans.

(a)

rP

^JX

-3

X'

(b)

(c)

Ans.

17.

2x1

- 6x^x^ +

Reduce by the method

(a)

X'

Ans.

6-2 X

(a)

of

Problem

+ 2y^ - 48y|

is

(6)

Hint. In (c) and (d) use

y,=

a;i

-3

-5

and by Lagrange's Reduction:

1-13-3

13

012
X

(c)

2^2^

+ 4y^

= zg, X2 = zi, xg = Z2.

-1

(d)

2-10

1-3 1-3
(6)

-3

5<

1111

-2 18
yl

whose matrix

+ 2x^ + 8X2^3

2x.^Xg

xi.x^.xs

-2 -4
-4 -3

16. Write out in full the quadratic form in

+ 4j:i%2 + 6;c^*g - ^x^x^

(c)

y^

72 + 8yg

,,

id)

2
Ji

-2

-2

2,2
-y^z^ya

CHAP.

18.

19.

20.

QUADRATIC FORMS

17]

\x = ^'

145

(a)

Show

that

(b)

Show

that the symmetric matrix of a quadratic form is unique.

Show

X'\

that over the real field

Prove:

\x but the matrices have

different ranks.

xf+xl- ixl+ix^x^ and 9xf+2x^+2xl + 6x^x^-6x^x^-8x^x^

are equivalent.

real symmetric matrix is positive (negative) definite if and only


if it is congruent over the real

field to / (-/).

21.

Show that X'AX


Theorem XIX.

22.

(a)

Show

(6)

Show

23.

of

Problem 12

is

reduced

to

X'CX hy X

RX.

where

Ks^K^^{-5)K^^a).

Then prove

two real quadratic forms in the same variables are positive definite, so also is their
sum.
that if q^ is a positive definite fonn in x^.x^
xs and 92 is a positive definite form in xs^i. xs+2.
..x^, then g = ?i + 92 is a positive definite form in xx,x2
x^.
that if

Prove: If

is any real non-singular matrix, then C'C is positive definite.


XlX = Y'ClCY

Hint: Consider

24.

Prove: Every positive definite matrix


of

25.

26.

Theorem

X.)

Hint: Consider

A can be
D'AD=I.

written as 4 =

CC. (Problems

23 and 24 complete the proof

Prove: If a real symmetric matrix A is positive definite, so also is A^ for


p any positive integer.
Prove: If ^ is a real positive definite symmetric matrix and
CB is orthogonal.

B and C

if

B'AB =1 and A

are such that

= C'C

then

27.

Prove; Every principal minor of a positive semi-definite matrix A is


equal to or greater than zero.

28.

Show

29.

Verify the stated effect of the transformation in each of Theorems

30.

ax^ - 2bx^x^ + cx\ is positive definite

that

By Kronecker's

if

and only

reduction, after renumbering the variables

a>0 and \a\

if

XX

ac-l^ >0.

and XXI.

when necessary, transform each

of the following

into a canonical form.


1

(a)

-1
2

-1

0-1

-1

X'

12 12
12
r

(c)

1112

2
1

4-4
(b)

X'

-4

-3

10-2

-3 X

(d)

X'

2
2

X'

12

-2

(g)

9
3

(/)

X'

-12

(a)

(b)
(c)
(d)

31.

Show

that

p^=p^=p^=p^=l;

^^j

iyl - 16y| + 16y|

yl
jI
q

^yl

+ 4y= - 3y2

8y,

xf-

Gxl -

6^

- Zxl - x^x^ -

xx

Pq = Pi =

(h)

22 =

-!

P3

Pi=l. 2S=-4. P3

(g)

See

(e).

(A)

4y2 - 16y| + 16y| + 12y^

11

Uxx

3_

X'

17(rf).

Po =

2xx

-2

1111

(/)

+ y| + y2
yf

-2

2 -1"

Hint: In (g), renumber the variables to obtain (e) and also as in Problem

Ans.

X'

2-462
3

(e)

XUx
2

4-

-1

-16;

2
Ti

y^^

2,2
72

^3

+ 128y|- 128y=

^xx^
can be
3
4-

"^

factored

chapter 18
Hermitian Forms

THE FORM DEFINED

by

where

is Hermitian

Hermitian form.

X'HX

(18.1)

v=\

j=i

and the components of

The rank

of

__

h^jXiXj,

= hj^

hij

are in the field of

complex numbers, is called an


If

the rank is Kra, the form is

we

shall find that the theorems

is called the rank of the form.

called singular; otherwise, non-singular.


If

and

are real, (18.1) is a real quadratic form;

hence,

here are analogous to those of Chapter 17 and their proofs require only minor

changes from

those of that chapter.


Since H is Hermitian, every h^i is real and every h^^^x;^ is real.
cross-products h^j'x^Xj and hj^XjXj^,
h^jx^xj
is real.

hjiXjXi

Moreover,

for

the pair of

+ hijXiXj

hijXiXj

Thus,
1.

The values

of an Hermitian form are real.

The non-singular linear transformation X = BY carries the Hermitian form (18.1)

into an-

other Hermitian form

{mH{BY)

(18.2)

Y(WHB)Y

Hermitian forms in the same variables x^ are called equivalent if and only if there exists
a non-singular linear transformation X = BY which, together with Y = IX, carries one of the
forms into the other. Since B'HB and H are conjunctive, we have

Two

H. The rank of an Hermitian form is invariant under a non-singular transformation of


the variables,

and
HI.

Two

Hermitian forms are equivalent

REDUCTION TO CANONICAL FORM. An

if

and only

if their

Hermitian form (18.1) of rank

matrices are conjunctive.

can be reduced

to

diagonal

form
(18.3)

Kjxn

k^y-iLji

'crfryr'

^i^^

and real

by a non-singular linear transformation X = BY. From (18.2) the matrix B is a product of elementary column matrices while B' is the product in reverse order of the conjugate elementary
row matrices.

By a
(18.4)

further linear transformation, (18.3) can be reduced to the canonical form [see (15.6)]

%Zi + i^22 +

+ 'zpzp

146

z^+i2^ + i

z-rV

CHAP.

HERMITIAN FORMS

18]

147

of index p and signature p-(r-p). Here, also,


p depends upon the given form and not upon
the transformation which reduces that form to (18.4).
IV. Two Hermitian forms each in the same n variables are equivalent if and
only
they have the same rank and the same index or the same rank and the same
signature.

if

DEFINITE AND SEMI-DEFINITE FORMS. A

non-singular Hermitian form h = X'HX in a variables


and index are equal to n. Thus, a positive definite Hery^y^ + y^y^ + +Jnyn and for any non-trivial set of values of

is called positive definite if its^rank

mitian form can be reduced to


the x's, A>0.

A
a.:e

X'HX is called positive semi-definite if its rank and index


Thus, a positive semi-definite Hermitian form can be reduced
to
and for any non-trivial set of values of the x's, h>0.

singular Hermitian form h =

equal,

rj= p <

i.e.,

yiTi + y2y2 +

+ yrYr

n.

r<

n,

H of an Hermitian form XliX is called positive definite or positive semidefinite according as the form is positive definite or positive
semi-definite.
The matrix

An Hermitian fom

V.

matrix C such that


VI. If
VII.

If

is positive definite if and only if there exists a


non-singular

= C'C.

is positive definite, every principal minor of

is positive semi- definite,

is positive,

every principal minor of

is

and conversely.
non-negative, and

conversely.

SOLVED PROBLEM
1
1.

Reduce

X'

\-2i

2 + 3i

-4 +

Prom Problem

7,

1-2/

+ 2i

to

canonical form (18.4).

13

2f
15,

2-Zi
-4 -

- 3i

-4-2i X

Chapter
2j

2j

o"

'l

2/\/io

i/^To

j/yTo

(-4-4j)/v^

j/a/TO

1/vTo

'^^

3i

-4 +

2i

13

-1

l_

Thus, the non-singular linear transformation


1

BY
,0

reduces the given Hermitian form

to

0'

HC

2/VlO

(-4 +

4i)/V^

1/VIO

-i/yflQ

-j/VTo

1/v^

Jiyr + Jsys - ysys

HERMITIAN FORMS

148

[CHAP.

18

SUPPLEMENTARY PROBLEMS
2.

Reduce each

of the following to canonical form.

+ 2il

l-2i

(c)

+
+

3j

3j

i"

(d)

For

(6), first

Ans.

{a)

(.b)

multiply the second row of

3i

3i

2i

2-J
2

2J

by

+J

and add to the

first row.

(-l-20/\/3l^
=

3t

1+j
3

Hint:

3i

1-i

jx
:]^

x'\

(b)

[1

tl

nyi - y-m

V2
(-1 + 30/3

(c)

-l"

-1

1/3

nn

- 7272

(-l+f)/A/2

'l

(d)

(-l +

Obtain the linear transformation

j_ri

Ans.

X= -^\

that X'

1-t

-1

l/v^

1+i

-1

-3+J

-3-J

5.

Prove Theorems V-VII.

6.

Obtain

7.

for

Prove:

BY

Show

Ti
(-i-2i)/v3i
:^'

v/2"Lo

4.

J<iyi

Jb

which followed by Y = IX carries

is positive definite

and X'

11

xi

X2

^n

X-L

All

hi2

hin

^2

/J21

h22

h^n

Use

(4.3).

K.

K.

^nn

(6).

\y

1+j

l+2i

1-i

l-2i

10

Hermitian forms theorems analogous to Theorems XIX- XXI, Chapter

^n

Problem 2 into

ij'

(a) of

is positive semi-definite.

17, for

quadratic forms.

n
_S Tj^-Xj^Xj

1=1 j=i

Hint:

- 7272 " 7373

2/VlO

.0

3.

30/\/^

(-3-2J)/\/lO

1/^2

where

77-

is the cofaotor of h^: in

= \h^j\

chapter 19
The Characteristic Equation of a Matrix
THE PROBLEM.

Let Y = AX,

where A = [o^],

(i,j

= 1,2

Z=

In general, the transformation carries a vector

X
vectors X

be a lineal transformation over F.

n).

x^Y

[%i,%2

into a vector

Y =

JriY

\.%,J-z

whose only connection with

is

possibility of certain

being carried by the transformation into KX, where A is either


which F is a subfield.

a scalar of

or of

some

We

through the transformation.

shall investigate

here the

field 3F of

Any vector X which by

the transformation is carried into KX, that is, any vector

AX

(19.1)

for

which

XX

is called an invariant vector under the transformation.

THE CHARACTERISTIC EQUATION.

\X-AX

(19.2)

From

(XI-A)X

(19.1),

we

obtain

A di^

O^ 2

^21

^ f^2

~
The system

of

homogeneous equations (19.2) has

\XI-A\

(19.3)

A Oil
a-zi

^271

X a-nn

On2

non-trivial solutions if and only if

Oi2
A 022

c-ni

0-tn

ci^n

A Onn

On2

The expansion of this determinant yields a polynomial 0(A) of degree re in A which is known as
the characteristic polynomial of the transformation or of the matrix A. The equation
<^(X) =
is called the characteristic equation of A and its roots Ai,A2
A are called the characterA = Ai is a characteristic root, then (19.2) has non-trivial solutions which are
the components of invariant or characteristic vectors associated with (corresponding to) that
root.

istic roots of i. If

Characteristic roots are also known as latent roots and eigenvalues; characteristic
vectors
are called latent vectors and eigenvectors.
r2

Example

1.

Determine the characteristic roots and associated invariant vectors, given A

The characteristic equation

characteristic roots are

Ai=

5,

A-2

-2

-1
-1

A-3

-1
-1

-2

A-2

is

A2=

1,

A3=

149

1.

A^'

- tA^ + llA

and the

THE CHARACTERISTIC EQUATION OF A MATRIX

150

When A

Ai =

(19.2)

5,

-2

-ll Vxi

-2

-1

since

becomes

2-1

-1

^2

or

-1

-1

-2

to

-1

X2

-1
-1

hence, associated with the characteristic root


x-i = x^ = xq = i
one-dimensional vector space spanned by the vector [l,l,l]'. Every vector

solution is given by

A=5

Xl

_X3

row equivalent

is

-1

xs

-2 -1

-1

[CHAP. 19

is the

[k.k.kY of this space is an invariant vector of A.

When A

Two

A2=1,

(19.2)

becomes

-2 -l'

Xl

-2

-1

X2

-2

-1

_X3_

Xl + 2x2

+^3

linearly independent solutions are (2,-1,0) and (1,0,-1).

Thus, associated with

two-dimensional vector space spanned by

Xi = [2,-l,0]'
Every vector hX^+kX^ = [2h+k,-h,-ky is an invariant vector of A.

the characteristic root A =

and X2 = [1,0,-1]'.

is the

See Problems

GENERAL THEOREMS.

In

Problem

3,

we prove a special case

(A:

3)

1-2.

of

Xj^
As
Xk ^f^ distinct characteristic roots of a matrix A and if Xi, X^
the
X's
are
linewith
these
roots,
respectively
vectors
associated
invariant
are non-zero
I.

If

Ai,

arly independent.

we prove a special case

In

Problem

k\

where A is re-square, with respect to A is


II. The Ath derivative of
cfi(X) = \XI A\
times the sum of the principal minors of order n-k of the characteristic matrix when k<n,

is

re!

4,

(ra

3) of

when k = n, and

As a consequence
III.

If

of

when k>n.

is

Theorem II, we have

Ai is an r-fold characteristic root of an re-square matrix A, the rank of Xil A


nr and the dimension of the associated invariant vector space is not

is not less than

greater than

r.

See Problem

5.

In particular
Iir.

is

re

Xi is a simple characteristic root of an re-square matrix A, the rank of Xil


and the dimension of the associated invariant vector space is 1.
If

Example 2. For the matrix A

of Ex.

1,

the characteristic equation is

<^(A) =

(A-5)(A-1) =

0.
The invariant vector [l,l,l]' associated with the characteristic root A=5 and the linearly
independent invariant vectors [2,-l,o]' and [l,0,-l]' associated with the multiple root A=l
are a linearly independent set (see Theorem I).

The

invariant vector space associated with the simple characteristic root

A=5

is of

CHAP.

THE CHARACTERISTIC EQUATION OF A MATRIX

19]

dimension

1.

multiplicity

2,

The invariant vector space associated with


Theorems III and III').

151

the characteristic root

= l, of

is of dimension 2 (see

See also Problem

6.

Since any principal minor of A" is equal to the corresponding principal minor ot A, we have

by (19.4) of Problem

1,

The characteristic

IV.

roots of

A and

A' are the same.

Since any principal minor of A is the conjugate of the corresponding principal minor of A,

we have
V.

The characteristic roots of A and

of

/I'

are the conjugates of the characteristic

roots of A.

By comparing
VI. If

Xi,

scalar, then
VII.

If

Problem
VIII.

A2

AAi,

kX^

we have

A are the characteristic roots of an ra-square matrix A and


AA are the characteristic roots of kA.

A are the characteristic roots of an n-square matrix A and


\-Lk,\2k
\ik are the characteristic roots of 4 -A/.

Ai, A2, ...,

scalar, then

In

characteristic equations,

7,

If

is a

if

A;

if

& is a

we prove

is

a characteristic root of a non-singular matrix A, then \A\/a is a character-

istic root of adj A.

SOLVED PROBLEMS
1.

If

is re-square,

(19.4)

where s^

We

show

0(A)

(m =

1,

that

|A/-^|

A" + 51 A"-^ + 52 A"-^ +

re-1) is (-1) times the

sum

...

+ s_,A + (-l)"|i|

of all the m-square principal minors of A.

rewrite \\! - A\ in the form

A-aii

0-ai2

0-021

A-a22

0-a2n

O-'^ni

O-Ons

X-

a-in

ar

and, each element being a binomial, suppose that the determinant


has been expressed as the sum of 2" determinants in accordance with Theorem VHI, Chapter 3. One of these
determinants has A as diagonal elements and zeroes elsewhere; its value is A". Another is free of
A; its value is (-1)"|^|
The remaining
determinants have m columns, {m = 1, 2, ....- 1), of -A and n~m
columns each of which contains just one
non-zero element A.

Consider one of these determinants and suppose that


of

-A

its

columns numbered h,

ia

are columns

After an even number of interchanges (count them) of adjacent


rows and of adjacent columns, the de-

terminant becomes

THE CHARACTERISTIC EQUATION OF A MATRIX

152

%.ii

[CHAP. 19

"iiA

"is.ii

"imA

"inA

"im.-im

(-1)'"
I

%.fe

"in-k

t^.to

im!

is an m-square principal minor of

i\

Aj

i
^'2

(-1)"

as

i^

(ii, is

%''''OT

..

where

^n

'i.'2

..

runs through the

n (n

1)

(n

...

-4

Now

S ^'^

H.^

-m + 1)

n taken

different combinations of 1, 2

at a

time.

-4 -1 -4

2.

Use

(19.4) of Problem

We have

to

1+0-2+6

Sl

-4

-3
1

3.

Let

\XI

-2

that Xi, X2,

Assume

--

(1)

8 + 2

A^'

+ 9A^ -

7A

the contrary, that is,

assume

-1

-4

-2

-1

-1

5-4
+

-2

4-1

2.

that there exist scalars 0%, 02, 03

+ 0^X2 + 03X3

A^^

(11)

aiAi^i + osAjA^s + a3A3A^s


(i), (11), (111)

may be

not all zero, such that

aiAi^i + 02^2X2 + osAgXg

by A and obtain

(III)

we have

a^AXi + oqAX^ + agAXg

Now

-2

A3, ^3 be distinct characteristic roots and associated invariant vectors of A.


X3 are linearly independent.

by A and recall that AX^ = A^

(II)

Multiply

-4 -4
-4

-1

a-]_Xi

Multiply

A'^- 5

(i)

(iv)

+ 16

-2
+

As, ^s)

Ai, A"!;

Show

- A\

-4 -1

-3

-1

-4

2-5+16-9

-1

SS

-4

-2

-1

Then

-1

-1
-1

S2

Ml

5-4
1-2 3
4-1 6

A =

expand |A/-^|, given

written as
1

aiXi

Ai
Ai

Ao
As

An
A3

02X2
ooXq

x\

\l

A\

"3^3

CHAP.

THE CHARACTERISTIC EQUATION OF A MATRIX

19]

By Problem

have

[aiX-j,

Chapter

5,

a^X^.asXaY

3,

Ai

As

As

\l

Al

x%

B~

hence,

0;

153

Multiplying (iv) by

exists.

B~

we

But this requires 01 = 02 = 03=0, contrary to the hypothesis.

= 0.

Thus. Xi. X^, Xq are linearly independent.

4.

From

(j!)(X)

\\1

A On

-012

-021

A 022

-ai3
- 123

-<31

-032

A- 033

A\

A-O22

-023

- Osi

- O32

A-O33

A-O22

-O23I

032

A Ossl

the

O12

A -Oil

-021

<^'(A)

we obtain

-031

A -Oil
Os 1

sum of the principal minors


1

<^"(A)

-032

A -022

-02

-Ol3

A
of

-012

-O2I

A 022

of order

-O23

two

OSI
c

A -01 1

A-o22

Ois
023
1

A -Oil

Og 3

XI - A

-ai2
A (322

A-a3g

-O32

A -033

-021

A -Oil

-Ol3

A 033

A -Oil

-oia
1

-0 12
1

2!(A-Oll) + (A-a22) + (A-Ogg)!


2!

times the sum of the principal minors of XI

-A

of order

one

0"'(A)

Also

5.

<j!.^"\A)

= cji^^hX) =

...

Prove: If X^ is an r-fold characteristic root of an re-square matrix A, the rank of X^J -A is not less
than n T and the dimension of the associated invariant vector space is not greater than r.
Since Ai is an r-fold root of <;6(A) = 0, ^SCA^) = ct>(X^) = 4' <.\) = = 4>''^~^\x^) =
and cfy'^AXi) + 0. Now
(^"^HX^ is r! times the sum of the principal minors of order n-r of A^/ -A; hence, not every principal minor
can vanish and X^l-A is of rank at least n-r. By (11.2), the associated invariant vector space otXj^l -A,
I.e., its

6.

null-space, is of dimension at most

For the matrix of Problem

2,

The characteristic roots

find the characteristic roots and the associated invariant vector spaces.
are

1, 1, 1, 2.

For A =

2:

XI

-2

-A

1
1

sion

1.

r.

2-5

-1

is of rank 3;

-3

-4

The associated invariant vector space

is that

spanned by

A=

1:

XI -A

-1

=
1

1-4
sion

1.

The associated

[2, 3,

-4
-2

-1

For

its null-space is of dimen-

-2, -3]'.

is of rank 3; its null-space is of

1-5

invariant vector space is that spanned by

[3, 6,

-4, -5]'

dimen-

THE CHARACTERISTIC EQUATION OF A MATRIX

154

7.

Prove:

a is a non-zero characteristic root of the non- singular

If

n-

[CHAP. 19

square matrix A, then \A\/a is

a characteristic root of adj A.

By Problem

1,

a"+ .!"-%

(i)

where

s^.

n - 1) is (-1)

= 1. 2

(i

l/xZ-adj/ll

where Sj

= 1, 2

(/

times the sum of


=

By
and the definitions of
llll-M

A\ = \AY' ^ then
(6.4)

s,-

and

S,-

all

Si/^""^

sum

l'^!

f-square principal minors of A, and

+ S^^^/s +

...

(-if

ladj^l

of the /-square principal minors of adj

Sj = (-1)" |-4|s._
Si= (-l)"s^_.,,
-^
To

Jl 1

and

m"

is (-1)-^ times the

1)

^n-i^ + (-1)

(-if Ml"-^si,

|adj

|;x/-adj A\

(-if !(-if ;x"

+ ._,A^"-' + ._2MlAi"-" +

s,|/irV

...

siMf-V

n-i

and

Mr"|/x/-adj

4|

(-if !i

s,(i^) +

...

\A\

(-if

._i(f^f-'
\A\

(^f Ml

/(/^)

\A\

Now
and by

/(R)

(-if ii

a"/(4^)

(-ifia"+ sia"-'+

+ ,,(1) +

._,(lf-

...

(-if (If Ml!

(i)

...

^_,a

(-i)"Ml

Hence, M|/Ot is a characteristic root of adj A.

8.

AAA

Prove: The characteristic equation of an orthoganal matrix

We have
=

</,(A)

|X/-P|

\\PIP'-P\

|-PA(^/-P')|

is a reciprocal equation.

A"|f/-P|

A"c^(i)

SUPPLEMENTARY PROBLEMS
9.

For the following matrices

determine the characteristic roots and a basis of each of the associated invariant

vector spaces.

-1

(a)

(6)

12

2
1

-2

-1

-12

1, [1,

-3

2,

[1,3,1]';

2,

[2,1,0]';

1,

[1,1,-1]';

(rf)

1,

[1,1,1]'

(g)

-3 -9

-12

(0

3,

6-10

(A)

9
6

-6
-4

-3 -3

-5

[1,-1,-2]'

7
n)

-1

-1

_i

1_

-1

-6

-2

-3

-1

-1

-1

3'

-5

'

1+t

(/)

'-1

-1

-1

'2

_0

-5 -4
-3 -2

1, [3, 2, 1]'

(O

1_

-1,-2]';

(b) -1, [1,0, 1]';


(c)

2-1
2, [2,

(/)

112-1

-1,0]';

2-4
2-1

-2 -8 -12
(e)

(rf)

(a)

(A:)

Ans.

(c)

2-i

CHAP.

THE CHARACTERISTIC EQUATION OF A MATRIX

19]

2,

[2,-1,0]';

0,

[4,-1,0]';

(/) 0,

[3,-1,0]';

1,

[12,-4, -1]'

(e)

(g) 1, [1,0,-1]', [0,1,-1]';


(A) 0,

[1,-1,0]';

-1, [0,1,-1]';

(j)

[l

i,

1+t,

(/) 2, [1,0,1]';

[4,0,-1]'

1,

[1,1,0]'

3,

[0,0,1]';

1,

[1,1,0]'

4,

+ j,l,l]';

[l-i.1,1]'

-J,

2-2j, [1,0,-1]'

1,0]';

[0,

(/c)

1,

[1,0,-1,0]', [1,-1,0,0]';

(/)

1,

[1,2,3,2]';

-1, [-3,0,1,4]'

(m)

0,

[2,1,0, 1]';

1,

[-2,4, 1.2]';

2,

AX

[0,3, 1,2]'

3,

-1, [3,0, 1,2]'

[3,0, 1,4]';

XX

X'AX

10.

Prove:

If A" is

H.

Prove:

The characteristic roots of a diagonal matrix are the elements

a unit vector and

if

155

then

of its diagonal and the associated in-

variant vectors are the elementary vectors E^


12.

13.

Prove Theorems

Prove Theorem
Hint.

16.

17.

=(A-Ai)(A-X2)...(A-A) then

The characteristic

14. Prove:

13.

VI.

VII.

|A/-.4|

If

and

Prove:

If

A and

A'

roots of the direct

! are n-square and

equation.

t:]

Prove:

If the n-stjuare

matrix A is of rank

Prove:

If

A and B

sum

are n-sauare and

r.

\(X

+ k)I

diag(.4i,

-A\

A2

r<n, show

then at least n

= (X + k~ X-O (X + k

(X + k -X^^.

A^) are the characteristic roots of A^ A^

that

-r

is non-singular, then

-X2)

NA

AN

and

have the same characteristic

of its characteristic roots are zero.

A~'^B and BA~'^ have the same characteristic

roots.

18.

For A and B of Problem

X9.

Let ^ be an n-square matrix.

17,

show

that

Write

B and A'^BA have the same characteristic

\XI -A''^\ = \-XA~'^(^I

-A)\

roots.

and conclude that

iAl IA2

lA

are

the characteristic roots of ^4"^

The characteristic

20. Prove:

Hint.

21.

If

are of absolute value

1.

X'

(PX.Y(PX-)

Prove: If A^ ^ \ is a characteristic root and A^ is the associated invariant vector of an orthogonal matrix
P, then XlXi = 0.

22. Prove:
23.

roots of an orthogonal matrix

A-, X. are a characteristic root and associated invariant vector of P, then

The characteristic

Obtain, using Theorem


=

(-i)"M|

<^ (0)

(-1)

(0)

(-1)

0^">(O)

n!

24. Substitute from

times the sum of the principal minors of order n -

r!

of

times the sum of the principal minors of order n -

of

.4

Problem 23 into
c/3(X)

to obtain (19.4).

1.

II,

0(0)

<P

roots of a unitary matrix are of absolute value

(?i(0)

+ (^'(0)-X +-^(f^'(0)-X^

...

+^0'^'^'(O)-A"

chapter 20
Similarity

TWO

ra-SQUARE MATRICES A and B over F are called similar over F


matrix R over F such that

if

there exists a non-singular

B = R'^-AR

(20.1)

Example

11

2J

R~'^AR

The matrices A

1.

of

Example

1,

-3 -3]

-110
-1
E7

ij

Chapter

19,

and

li
2

133"

12

13

[1

14

13]

iJ

are similar.

The characteristic equation (A 5)(A

An

invariant vector of

1)^ =

of

B associated with A=

is

is also the characteristic equationof ^.

Y^ = [l,0,0]' and

it

is readily

an invariant vector of A associated with the same


X-L
A = 5. The reader will show that Fg = [7, -2, O]' and Yg = [n, -3, -2]' are
arly independent invariant vectors of B associated with A = 1 while X^ = RY2
are a pair of linearly independent invariant vectors of A associated with the same

= RYi =

that

[1, 1, 1]' is

root

Example

characteristic

a pair of lineand Xg = RYg


root A =

1.

illustrates the following theorems:

Two

I.

shown

same characteristic

similar matrices have the

roots.

For a proof, see Problem


II.

If

A^ of B, then

is an invariant vector of

RY

1.

B = R~ AR corresponding to the characteristic root


A corresponding to the same characteristic

is an invariant vector of

root A^ of A.

For a proof, see Problem

DIAGONAL MATRICES. The

characteristic roots of a diagonal matrix

D =

a^)

diag(ai, 02

2.

are

simply the diagonal elements.

diagonal matrix always has n linearly independent invariant vectors.

vectors E^ are such a set since

DE^ = a^E^,

(i

As a consequence, we have (see Problems


III.

Any

The elementary

1, 2, ...,n).

and 4

for proofs)

re-square matrix A, similar to a diagonal matrix, has n linearly independent

invariant vectors.
IV.

If

an re-square matrix

A has

re

linearly independent invariant vectors,

it

is similar

to a diagonal matrix.

See Problem
In

Problem
V.

\IA

6,

5.

we prove

Over a

field

an re-square matrix A is similar to a diagonal matrix

factors completely in

F and

if

and only

if

the multiplicity of each A, is equal to the dimension of

the null-space of X.I A.


1/

156

CHAP.

SIMILARITY

20]

157

Not every ra-square matrix is similar to a diagonal matrix. The matrix of Problem 6, Chapexample. There, corresponding to the triple root X = 1, the null-space of A/-i
is of dimension 1.
ter 19, is an

We can

prove, however,

VI.

Every ra-square matrix A is similar

to

a triangular matrix

whose diagonal elements

are the characteristic roots of A.

See Problems

7-8.

As special cases, we have


VII.

If

is

orthogonal matrix

any real

n-

P such

that

square matrix with real characteristic roots, there exists an


P'^ AP = P'AP is triangular and has as diagonal elements

the characteristic roots of A.

See Problems 9-10.


VIII. If A is any re-square matrix with complex elements or a real re-square matrix with
complex characteristic roots, there exists a unitary matrix U such that W'^AU
UAU is
triangular and has as diagonal elements the characteristic roots of A.
=--

See Problem

The matrices A and P' AP

of

Theorem

The matrices A and U'^AU

of

Theorem VHI

11.

VII are called orthogonally similar.


are called unitarily similar.

DIAGONABLE MATRICES. A
Theorem IV

is

matrix A which is similar to a diagonal matrix is called diagonable.


basic to the study of certain types of diagonable matrices in the next chapter.

SOLVED PROBLEMS
1.

Prove:

Two

similar matrices have the

Let A and B

R~^AR

A/-B

(')

same characteristic

roots.

be the similar matrices; then

\I -

R'^AR

R-'^UR - R-''aR

R'^(XI-A)R

and

\\I-B\

\R-^.\XI-A\.\R\

\XI-A\

Thus, A and S have the same characteristic equation and the same characteristic roots.

2.

Prove:

RY

If

By hypothesis, BY

X^Y and RB

AX
and

3.

A"

Is an invariant vector of

Prove:

B = R ^AR corresponding to the characteristic root


A corresponding to the same characteristic root \,- of A.

is an invariant vector oi

is an invariant vector of

Any matrix A which

ARY

A,-,

then

AR; then
=

RBY

A corresponding

RX^Y

X-RY

X^X

to the characteristic root X^.

is similar to a diagonal matrix

has

re

linearly independent invariant

vectors.

Let R AR = diag(6i, 62. > b^) = B. Now the elementary vectors fii, Eg
^n ^'^ invariant vectors
Then, by Theorem n, the vectors Xj = RE. are invariant vectors of A. Since R is non-singular, its
column vectors are linearly independent.
of B.

SIMILARITY

158

4.

Prove:

[CHAP. 20

an n-square matrix A has n linearly independent invariant vectors,

If

is similar to a

it

diagonal matrix.

X^ be associated with the respective


L,et R = [X^,X2, ....X^]; then

Let the n linearly independent invariant vectors X^. Xq


teristic roots Ai, Xs. /^n ^ '^^"^

AR

,4A'^ = A^A'^,

AX^]

[AX^.AX^

0'

[Ai^i, ^2^2

Ai
1X1,

R ^AR

Hence,

5.

= diag(Ai,

^n^n^

...

A2

X2

...

'^n-'

A2

[Jf 1,

A)

Z2, ^3!

-1

then

10-1
R'UR

-2

1-2

-3

Example

of

1"

-1

2_

-3

,1

1,

Chapter

19, is

^3 = [1,0,-1]'

Z2 = [2,-1,0]',

Zi = [1,1,1]',

R =

diag(Ai, A2

A).

set of linearly independent invariant vectors of the matrix

Take

charac-

n).

1, 2

and

0"

-1

a diagonal matrix.

6.

Prove:

Over a

field

factors completely in
of

F an n-square matrix A is similar to a diagonal matrix if and only if Xl-A


F and the multiplicity of each A.^ is equal to the dimension of the null-space

X^I-A.

and that exactly k of these characteristic


B
R-'^AR
diag(Ai, A2,
^n'
Then X^I - B has exactly k zeroes in its diagonal and, hence, is of rankn-fe; its
thus, A.^/-^ has the same
null-space is then of dimension n-(n -k)=k. But \l-A = R (Xj^I - B) R~^
\^I
B
rank n -k and nullity k as has
Conversely, let Ai,A2
Ag be the distinct characteristic roots of A with respective multiplicities

suppose that

First,

roots are equal to A^.

r,

'i-''2

Take

where

Xi^.Xi^.

exist scalars a

.,Xir-

as a basis of the invariant vector space Vf..

not all zero,

s).

Suppose that there

XJ'

Now

= 1,2

(i

such that

(aiiA^ii + 012-^12 + + "iTi^iri)

(i)

the associated invariant vector spaces.

Denote by V^^. V^^,

ri+r2+...+rc

...

each vector

Yi

(osi^si
=

ai^Xu

"^

^s^Xs'z

ai^Xi^ +

"^

+
*

(o2i''''2i
"^

+ 022^^22 +

^'T^ ^STg
a' STg
XciT^))

+ air^Xir^) =

0,

''2'r2^2r2)

(i

1,

s).

for

otherwise,

Theorem I their totality is linearly independent. But this contradicts


X's constitute a basis of K and A is similar to a diagonal matrix by Theorem IV.
invariant vector and by

7.

Prove:

Every ra-square matrix A

is similar to a triangular matrix

(i);

whose diagonal elements

it

is

an

thus, the

are the

characteristic roots of A.

A and let X-l be an invariant vector of A corresponding to


column of a non-singular matrix Qi whose remaining columns
may be any whatever such that \Qi\ ^ 0. The first column of AQi_ is AX^ = X-^X^ and the first column of
O]'. Thus,
Ql^AQi is Ql^XiXi^. But this, being the first column of Qi'^XiQi. is [Ai,
Let the characteristic roots of A be Ai, Aj

the characteristic root Ai.

Take

X-i

as the

first

CHAP.

SIMILARITY

20]

Qi^AQ^

(i)

where

A-^

is of order n

Since

|A/

hi

Bil

[o

A,j

1.

AQ^\ = (X-Xi)\XI - Ai\,

-ft

159

and Qi^ AQi_ and A have the same characteristic roots, it


A. If n = 2, A-i = [A2] and the theorem is proved

follows that the characteristic roots of Ai are Xg. Ag


with Q = Qi.

Otherwise, let X^ be an invariant vector of Ai corresponding to the characteristic root As- Take X^ as
column of a non-singular matrix Qq whose remaining columns may be any whatever such that IftI /^ 0.

the first

Then
=

Q2^A^Q^

(il)

where A2

is of order n

2.

n =

If

3,

/I2 = [As],

[^2

52I

[0

A^j

and the theorem is proved with Q = Q^[0

Otherwise, we repeat the procedure and, after n -

I2

/i

n-2

(iii)
(?2

such that

8.

(J

/1()

is triangular

Q2J

steps at most, obtain

<?3

Qn-i

and has as diagonal elements the characteristic roots of A.

Find a non-singular matrix Q such that

Q AQ

is triangular, given

9-1

8-9
-5

-1

-4

-4

Here |A/-^| = (A^-l)(A^-4) and the characteristic roots are 1,-1,2,-2. Take [5,5,- 1, 3j', an
invariant vector corresponding to the characteristic root 1 as the first column of a non-singular matrix
Qx
whose remaining columns are elementary vectors, say

^5000'
5

10
10

-10
3

Then

-5

and

8-9

5-1

1
1

-3

Qi AQ^

-15
-12

20
16

Aq

[O

17

characteristic root of Ai is -1 and an associated invariant vector is

-l]'.

[4, 0,

Take ft

then
-20

o"!

ft A1Q2

and

II

J_
20

-15
-48

64

-11

48j

characteristic root of A2 is 2 and an associated invariant vector is

20]

[s, ll]'.

r_i

5I

"d

Take Qs

}
[.:

<?3'

H[-11
8

,!

2/5I
"I
sj

and

ft'-42ft

[2

-2J

then

[CHAP. 20

SIMILARITY

160

Now
32

pi

o"|

[o

Q^j

[h

o1

[o

Qsj

Q''

'

160

-1

11

20

-180 40 -220

1_

"l

-40 40
160_

-7 -9/5"

-1

and

2/5

-2

9.

If

any real re-square matrix with real characteristic roots then there exists an orthogonal
that P''^AP is triangular and has as diagonal elements the characteristic roots of ^.

is

matrix

P such

X be the characteristic roots of A. Since the roots are real the associated invariant
As in Problem 7, let Qi be formed having an invariant vector corresponding to Aj
as first column. Using the Gram-Schmidt process, obtain from Qt an orthogonal matrix Pi whose first column
is proportional to that of Q-^. Then
Let Ai.Xs

vectors will also be real.

hi

P'lAP^
where Ai

is of order n

fill

and has As, A3, ...Ayjas characteristic roots.

Next, form Qq having as first column an invariant vector of Ai corresponding to the root A2 and, using
the Gram-Schmidt process, obtain an orthogonal matrix P^- Then

TAs

Ss"!

[0

A^j

iji-g

After sufficient repetitions, build the orthogonal matrix

for

10.

which P~^AP

is triangular

pi

0]

|_o

pj

Pi.

pn-2
"I

P_J

with the characteristic roots of A as diagonal elements.

Find an orthogonal matrix P such that


"2

P-^AP

is triangular

2_

and has the characteristic roots of A as diagonal elements.

Prom Example
to

P""

1"

Chapter 19, the characteristic roots are

1,

5, 1, 1

and an invariant vector corresponding

is [1,0, -1]'.
"

We take

Qi

10

and, using the Gram-Schmidt process, obtain

-1

1
1

Pi

l/v^

/V2

-l/\/2
an orthogonal matrix whose

We

first

column is proportional to

1/1/2"
[l, 0, -l]'.

find

-I/V2

1/a/2

Pl'^Pi

l/>/2

I/V2"

3^/2"
\0

1/1/2"

l/^/T

-1/V2

1/

V2

21/2"

A^j

CHAP.

SIMILARITY

20]

Now Ax has A

a characteristic root and

[l,

161

-\/2]' as associated invariant vector. From Q2 =

r
we

"2=1 1/V3

obtain by the Gram-Schmidt process the orthogonal matrix

L-2/V6

1/\A2

-1/^^

1/^6

1/a/3

2/^6

-I/V2"

-I/a/S"

1/V6"

P-,

X]

\_

rn

Lv2

r-n

ij

2/v/6

Then

I/V3J

is orthogonal and

P~^ AP =

->/2

11.

Find a unitary matrix

f/

such that

All is triangular and has as diagonal elements the charac-

teristic roots of A, given

1 +t
2 - 2J

-6 -

-4 - 6i
+ 3i

-1 + i

-3 -

5 + 5i

The characteristic equation

of

is

A(A^

4f

6 + 4f

+(-4-J)A+

2i

i)

and the characteristic roots are


"
1

0,

1-i, 3 +

2i.

For A =

0,

take

[l,

-1, l]' as associated invariant vector and form

Qi

-1

The Gram -Schmidt process produces the

1_

unitary matrix

f/i

i/\fz

i/Ve'

-l/vA3

2/v^

1/V3"

l/\/6"

-i/\/2

1/^2"

Now

-2y/2a-i)

~(26 + 24:i)/y/6

1-i

(2 + 3i)/\A3"
3

so that, for this choice of Qi, the required matrix

12.

Find an orthogonal matrix

P such

that

P ^AP

+2J

= Ui.

is triangular

and has as diagonal elements the char-

acteristic roots of A, given


3

The
[1,1,1]',

[1,-2,1]' respectively.

-1

Now

may be taken as

[l,0, -l]'.

these three vectors are both linearly independent and mutually

Taking

l/V^
P

-1/-/2
find

characteristic roots are 2,3,6 and the associated invariant vectors

orthogonal.

we

-1

-1

P~^ AP

= diag(2,3, 6).

in the next chapter.

l/y/J

l/y/6

l/y/3

-2/^/6

l/y/3

l/yje

This suggests the more thorough study of the real symmetric matrix made

SIMILARITY

162

[CHAP. 20

SUPPLEMENTARY PROBLEMS
13.

Find an orthogonal matrix P such that f'^AP is triangular and has as diagonal elements the characteristic
A for each of the matrices A of Problem 9(a), (6), (c), (d). Chapter 19.

roots of

\/Zs[2

2/3

\/^^pi

2/3

-4/3a/2

1/3

\l\pZ
Ins.

(a)

-l/v/2

I/V2"

(rf)

1/^2"

14.

1/V^ -i/'v^

-i/Ve"'

1/V3"

2/V6"
I/a/2"

-1/\/6"

and (rf) are


Chapter 19 and determine those which are similar to a
diagonal matrix having the characteristic roots as diagonal elements.
of

Problem

9,

For each of the matrices A of Problem 9(j). (/), Chapter 19, find a unitary matrix V such that
angular and has as diagonal elements the characteristic roots of A.

(i)

l/v^

-(l+i)/2

l/>/2

a-i)/2^/2

\/\[2

(1

17.

Make the necessary modification

18.

Let B^ and C^ be similar matrices

is real

S
are similar.

Let

is tri-

(/)

\/^[2

\/y[2

and symmetric and P is orthogonal, then P''^ AP is real and symmetric.

Prove:

If

U'^AU

-1//!

\l\p2.

i
2

-0/2/2"

16.

of Problem 9 to prove

Suppose C^ =

Hint.

= diag(Bi, Sj)

for

(i

and

-1
i?^

B^ fl^ and form

B
B

to

that

and

VIII.

C)

= diag(Ci, C2

= diagCSj, B^).

Extend the result of Problem 19

Theorem

Show

m).

1, 2

= diag(5i, B2, ...,S)

of Bi and B^ respectively, and define

20.

1/^6"

of Problem 13 are similar to a diagonal matrix while (c)

(6)

Examine the matrices {a)-{m)

Ans.

19.

1/^

I/V3"

1/V2"

Explain why the matrices (a) and

not.

15.

2/>/6

-1/1/3

-1/^/6""

i/Va"

(c)

-l/V'2

(b)

1/-/2

I/V3"

Write

ij

= diag(Bi,

/ = diagC/i./g),

Show

Bg

where the orders of

BT^EK

that

B^).

to prove

/i

and

/g are

B and C are

those

similar.

oJ

= diag(Bi,

Bg

B)

and C any matrix obtained by rearranging the

S^ along the diagonal.


21. If

A and B are n-square, then

Hint.

Let P AQ =N; then

AE

and B/1 have the same characteristic roots.

PABP"^ =NQ''^BP''' and Q~^ BAQ

Q'^BP'^N.

See Problem

15,

Chapter 19

A^ are non-singular and of the same order, show thut A-^A^-.. A^, A2As-.. A^A^, A3... A^A-^^A^.
have the same characteristic equation.

22. If A-i.A^
...

23.

Q'^AQ

Let
of
(a)

B where B

is triangular

and has as diagonal elements the characteristic roots Ai, X2

Show that
S

A Q

is triangular

and has as diagonal elements the kth powers of the characteristic

roots of A.
(b)

.4.

Show

that

X,-

= trace

24.

Show

that similarity is an equivalence relation.

25.

Show

that

'2

_1

2_

-1

2-1

and
-3

-2

have the same characteristic roots but are not similar.

chapter 21
Similarity to

REAL SYMMETRIC MATRICES. The


we

be combined but

study of real symmetric matrices and Hermitian matrices may

shall treat them separately here.

The characteristic

I.

a Diagonal Matrix

For real symmetric matrices, we have:

roots of a real symmetric matrix are all real.

See Problem

The

II.

1.

invariant vectors associated with distinct characteristic roots of a real sym-

metric matrix are mutually orthogonal.

See Problem

When A

is real

and symmetric, each B^ of Problem

9,

Chapter 20, is

0;

hence,

is a real ra-square

symmetric matrix with characteristic roots Ai, \^,


then there exists a real orthogonal matrix P such that P'AP = P'^AP = diag(Ai, Aj
If

III.

Theorem
IV.

III

If

A^ is a characteristic root of multiplicity

In terms of a real quadratic form,

A,
A).

r^

of a real

Theorem

III

symmetric matrix, then

r^.

becomes

V. Every real quadratic form q = X' AX can be reduced by an orthogonal transformation


to a canonical form

BY

(21.1)

where

the

...,

implies

there is associated with A^ an invariant space of dimension

2.

Aiyi + X^yl +
r

is the rank of

A and

Ai,

Aj

...

A^ are

+ X^y"^

its

non-zero characteristic roots.

Thus, the rank of q is the number of non-zero characteristic roots of A while the index is
number of positive characteristic roots or, by Descartes Rule of signs, the number of varia-

tions of sign in |A/


VI.

^|

0.

real symmetric matrix is positive definite if and only if all of its characteristic

roots are positive.

ORTHOGONAL

SIMILARITY. If P is an orthogonal matrix and 8 = P"^ AP then B is said to be orthogonally similar to A. Since P~^ = P', B is also orthogonally congruent and orthogonally
equivalent to A. Theorem III may be restated as
.

VII. Every real symmetric matrix A is orthogonally similar to a diagonal matrix whose
diagonal elements are the characteristic roots of A.

See Problem

3.

Let the characteristic roots of the real symmetric matrix A be arranged so that Ai ^ A2 =
Then diag(Ai, A2, . A.) is a unique diagonal matrix similar to A.
... ^A.
The totality of
such diagonal matrices constitutes a canonical set for real symmetric matrices under orthogonal
similarity. We have
VIII.

the

Two

real symmetric matrices are orthogonally similar if and only if they


roots, that is, if and only if they are similar.

same characteristic

163

have

SIMILARITY TO A DIAGONAL MATRIX

164

PAIRS OF REAL QUADRATIC FORMS.

AX

IX. If X'

and

X'BX

In

Problem

4,

[CHAP. 21

we prove

are real quadratic forms in

(x-^^, x^
x^) and if
tive definite, there exists a real non- singular linear transformation X = CY

X'AX

X'BX

is posi-

which carries

into

AiXi + A2Y2 +
and X' BX into

22

yi

where A^ are the roots of

+ 72

+ Ay

+ Tn

\\BA\=0.
See also Problems

HERMITIAN MATRICES.
X.

Paralleling the theorems for real symmetric matrices,

The characteristic roots

4-5.

we have

of an Hermitian matrix are real.

See Problem
XI.

The

7.

invariant vectors associated with distinct characteristic roots of an Hermitian

matrix are mutually orthogonal.


XII.

If

is an re-square Hermitian matrix with characteristic roots Ai, A2.

exists a unitary matrix

U such

is called unitarily similar to


XIII. If

U'HU = U~^HU =

that

U'

HU

A,-

A.) is

r-

an invariant space of dimension

Let the characteristic roots of the Hermitian matrix

Then diag(Ai, A2

A).

...,

A, there

The matrix H

is a characteristic root of multiplicity

there is associated with

diag(Ai, A2,

of the Hermitian matrix H, then


r-

be arranged so that Ai i A2 ^

a unique diagonal matrix similar to H.

The

totality of

i Ansuch diago-

nal matrices constitutes a canonical set for Hermitian matrices under unitary similarity.

There

follows

Two

XIV.

Hermitian matrices are unitarily similar

characteristic roots, that

NORMAL MATRICES. An
diagonal,

and only

if

real

if

and only

they have the same

if

they are similar.

n-square matrix A is called normal

symmetric,

real

is, if

if

AA' = A'A. Normal matrices include

skew-symmetric, orthogonal, Hermitian,

skew-Hermitian, and

unitary matrices.

Let .4 be a normal matrix and f/ be a unitary matrix, and write B = U'AU.


and B'B = U'A'U -U'AU = U'A'AU = U'AJ'U = U'AU -U'A'U = BB'. Thus,

XV.
In

If

Problem
XVI.

mal

8,

is a

normal matrix and

is

a.

unitary matrix, then

U'AU

Then B' = U'A'U

is a normal matrix.

we prove

X^

If

rnatrix A,

is an invariant vector corresponding to the characteristic root A/ of a northen X^ is also an invariant vector of A' corresponding to the characteristic

root A^.
In

Problem
XVII.

9,

we prove
square matrix A is unitarily similar to a diagonal matrix

if

and only

if

is

normal.

As a consequence, we have
XVIII.

If

is normal,

the invariant vectors corresponding to distinct characteristic

roots are orthogonal.

See Problem

10.

CHAP.

SIMILARITY TO A DIAGONAL MATRIX

211

MX.

If

A- is a characteristic root of multiplicity

ated invariant vector space has dimension

r^

165

of a normal matrix A, the associ-

r^.

XX. Two normal matrices are unitarily similar

if

and only

they have the same char-

if

acteristic roots, that is, if and only if they are similar.

SOLVED PROBLEMS
1.

characteristic roots of an re-square real symmetric matrix

The

Prove:

Suppose that h+ik

which

BX

is real

is a

complex characteristic root of A.

{(h

and singular since

+ ik)I-A\[(h-ik)I-A\

{h

+ ik)I

-A

A are

all real.

Consider

(hl-Af+k'^I

There exists a non-zero real vector

is singular.

such that

and, hence,

X'BX
The vector

X\h!-AfX

(/(/- 4)

+ k^X'X

is real; hence,

A"

{(hi

X'(hl-A)'(hl

-A)X

- A)X\' {(hi -A)X\ ^

+ k^X'X
Also,

0.

X'X>0. Thus,

A:

and

there are no complex roots.

2.

Prove:

distinct characteristic roots of a real symmetric

The invariant vectors associated with

matrix A are mutually orthogonal.


Let

A'l

As of 4.

AX^

and X^ be invariant vectors associated respectively with the distinct characteristic roots Xi and

Then
=

Ai^i

AX^

and

= Aj'^s.

A^2'4^i =

also

X-iX^X^

and

Xj^AX2 = XqXiX^

Taking transposes
^1 A X^ = A 1 A*! ^2

Then

3.

X^X^X^

and

X[X2

and, since Ai ^ A2,

A2^iA^2

XqAX-j^ =

X2X2X1

Thus, X^ and X2 are orthogonal.

= 0.

Find an orthogonal matrix P such that P'^AP is diagonal and has as diagonal elements the characteristic roots of

A given
,

^7-2

The characteristic equation

-2

10

-2

-2

is

A-7

-1

A -10

-1

A-7

and the characteristic roots are

A - 24A^ + I8OA - 432

6, 6, 12.

- -

For A =

6,

we have

-1

Xl

-4

X2

-1

X3

variant vectors the mutually orthogonal pair X^ = [l,


[1, -2,1]' as associated invariant vector.

or

0,

Xi

- 2x2 + xs

-l]' and

X2

= [l,

1, l]'.

and choose as associated

When A

= 12,

in-

we take X3 =

SIMILARITY TO A DIAGONAL MATRIX

166

Using the normalized foim of these vectors as columns

1/VT
P

-i/x/T
is left as an exercise to

It

4.

Prove:

X'AX and X'BX

If

show

that

Xiyi + \2y2 +
\\B-A\ =0.

By Theorem

^'B^

^'^'^

^nTn

= diag(l/\/7H, X/sfJT^

for the real quadratic form


it

l/yjl

6, 12).

+ y^, where Xi, Ag, .,

...

GV

Then V =

BGH)W

G'AGH)W

(H

HW

which carries X'BX into

carries

(i)

into

+ wt

there exists an orthogonal transformation

X^y2 + X^y^ +

...

Y (K~ IK)Y
-'i

Since for

values of

all

KY

which

XK'h'g'BGHK - K'h'g'AGHK

=
=

follows that Xi, Xg

Prom Problem

3,

are the roots of |XB-/l|

there exists a real non-singular trans...

and X'BX into

+ Xy^

-'2

'n

diag(X,X

diag(X-Xi,

X)

- diag(Xi,X2

X-X2

X-X)

=0

the linear transformation

\/^pr

+ X^y^

X,

K'h'g'(\B-A)GHK

(GH)W
-l/\f2

l/vA3"

l/sfQ

l/VT

-2/'/6"

IATs

1/v^

\/2\fl

_-l/2>/3

carries

are the roots of

+ Ait;^

X are the characteristic roots ot H G AGH. Thus,


CY = GHKY which carries X'AX into X^y^ + X^y^ +

Y'(K'h'g'BGHK)Y

5.

into

where Xi.Xg

it

) and if X'BX is positive defi= CY which carries Z'/lZinto

transformation

jji^vf + fJ.2V2 +

(H G

Y'(K'h'g'AGHK)Y

formation

l/VT

into yi + y| +

l/i//!^)-

(ii)

carries

-2/sf6

/x are the characteristic roots (all positive) of B.

jjii.jj.^

X/\f&

I/VT

VII there exists an orthogonal transformation

V'(G'BG)V

Let

we have

are real quadratic forms in (xi,x2

(i)

where

1/x/y

= (iiag(6,

there exists a real non-singular linear

nite,

Now

P~^AP

of

[CHAP. 21

X'BX =

X'

The same transformation

-2

10

-2

-2

1//6"

1/2x^3"

1/3n/2

1/(

1/3-/2

-1/:

1/3\A2

1/f

into

ir'ZIF.

carries

1/3

X'AX

into

1/^r

tr'

0^

X)

CHAP.

SIMILARITY TO A DIAGONAL MATRIX

21]

Since this is a diagonal matrix, the transformation

Thus, the real linear transformation

and the quadratic form X' AX


into ri + y| +
7l
\kB-A\ = 36(3\-l)(2X-l)^.

6.

CY

KY

167

of Problem 4 is the identity transformation

into ^y^ + iy| + |y|.

A = CP

Every non-singular real matrix A can be written as


symmetric and P is orthogonal.

where C

Since A is non-singular, AA' is positive definite symmetric by Theorem X, Chapter


Q"''
with each
k^) = B
AA' Q = diag(A;i, ij
Q such that

diag(\/^,

VT^

C^

C'^AA'C'^^ C ^C'^C^

is orthogonal as required.

Prove:

pose

Then

C'^A.

The

QBQ-

QB^Q-^

and

= I

Then there

17.
A:^

>

Thus A

0.

Define

symmetric and

is positive definite

PP'

that

8.

QB^Q-^QB^Q-^

Now C

= QB-^Q''^.

positive definite symmetric and

Define

7.

and

\fk~^)

that

is positive definite

exists an orthogonal matrix


=

show

is left as an excersise to

It

= lY.

X'BX

carries the positive definite quadratic form

{GH)Y

Prove:

Bi

AA'
is orthogonal.

CP

with C

characteristic roots of an Hermitian matrix are real.

Let \j be a characteristic root of the Hermitian matrix H. Then there exists a non-zero vector X^ such
Now X'^HX^ = XjX'j^Xiis real and different from zero and so also is the conjugate transHXj^ = A jX^
.

\^X\X^.

X\HX;j^ =

Thus, X^=X.j andX^isreal.

If X: is an invariant vector corresponding to a characteristic root A,- of a normal matrix A,


then Z^ is an invariant vector of A corresponding to the characteristic root X^.

Prove:

Since A is normal,

(KI-A)(XI-A')

(XI-A)(XI-A)'

-XA+

XXI - XA'
so that XI

-A

is normal.

(SX^)'(SA'i)

By hypothesis, BX;
=

X-B'-BXi

X'iB.B'X^

-A)X^=

= (X^I

(W X ^)' {W X ^)

XA

XXI - \ a'

a' A
0;

(XI -A)' (XI

Prove:

An

re-square matrix

Suppose A

is normal.

By

AA
-A)

then

and

B'X^

Thus, X: is an invariant vector of A' corresponding to the characteristic root X

9.

is unitarily similar to a diagonal matrix if

(X^/-I')Zi

^.

and only

if

A is normal.

Theorem vni. Chapter 20, there exists a unitary matrix U such that
^1

6i2

bin

As

bin

U AU
....

B'b

By Theorem XV, B is normal so that B' B = BB


is XiXi while the corresponding element of BB

Now

Xfi-i

bn-i. n

the element in the first row and first column of

is

AjAi + 612^12 + bigbis +

...

bmbm

Since these elements are equal and since each b^j b-^j 2 0, we conclude that each b^j = 0. Continuing
we conclude that every b^; of B
with the corresponding elements in the second row and second column,
A). Conversely, let A be diagonal; then A is normal
is zero. Thus, B = diag(Ai,A2

SIMILARITY TO A DIAGONAL MATRIX

168

10. Prove:

If

[CHAP. 21

is normal, the invariant vectors corresponding to distinct characteristic roots are

orthogonal.

Let Ai,

A*!

and Xj, ^2 be distinct characteristic roots and associated invariant vectors of A. Then
= \^X^ and, by_Problem 8. jI'A'i =Xi^i,_I'A'2 = AaATa.
Now ^2/lA'i = Ai^a^i and,

AXi=\iXi, AX^

X[a' X^

taking the conjugate transpose,


and, since

11.

XiX2=

Xx iXo.,

x^ -

Consider the conic

X[a' X^

But

= X^X'-^X^.

- 4.xi = 40

12xix.2.

X'AX

(i)

referred to rectangular coordinate axes

The characteristic equation

of

OX^ and OX2.

A-1

(A-5)(A + 8)

For the characteristic roots Ai = 5 and A2 = -8, take

-2/\flf\

/v/T3

'VTs

The conic

form the orthogonal matrix

The transformation

vectors after normalization.

-6~]

40

::]

A is

|A/-^|

Now

Thus, Xi^^A's = Xa^i-^s

or

[-:

invariant vectors.

\^X[X2.

as required.

3/x/T3

PY

and

[2, 3]'

tsVTs
'__

2/V/T3I

-2/v/T3

3/vT3j

"

reduces

'^lll

respectively as associated

whose columns

fs

o"!

^-y|

y'\

13 J

are the two

to

(i)

13"!
Vis'

-\y
3/\/l3.

2/\/l3

3/VT3.

[3, -2]'

40

5yl

-Sj

Lo

is an hyperbola.

Aside from the procedure, this

is the familiar rotation of

axes

in plane analytic

geometry to effect the


Note that by Theorem VII the result is

elimination of the cross-product term in the equation of a conic.

known as soon as the characteristic roots are found.

12.

One problem

of solid analytic geometry is that of reducing, by translation and rotation of axes,

the equation of a quadric surface to simplest form.


to

determine the principal directions,

tempting to justify the steps,

The main tasks

are to locate the center and

the directions of the axes after rotation.

i.e.,

we show here

Without

at-

the role of two matrices in such a reduction of the

equation of a central quadric.


Consider the surface

Zx^ + 2xy + 2xz + iyz - 2x - 14y + 2z "

"

and

characteristic equation of

1A/-.11

The characteristic roots and associated


fi =

~.

-;=|

-1

-7

-1

-7

-9

all the terms.

is

A-3

1,

formed respectively from the terms of degree two and from

Ai =

and the symmetric matrices

The

A2

-1

-1

-1

-2

-1

-2

unit invariant vectors are


= 4,

V2

-2,

V3

^N-TfJ

CHAP.

SIMILARITY TO A DIAGONAL MATRIX

21]

169

Using only the elementary row transformations Hj(k) and H^:(k), where
3

-1

r-

Bi

-1

-7

-1

-7

4,

-4

1
--VJ

-9

ft]

-4
'3x +

Considering B^ as the augmented matrix of the system of equations

X + 2y

\^

from Di the solution x = -1, y =

The rank

of

is 3

0,

z =

C(-l,

or

and the rank of B is

4; the

XtX'^ + X^y'^ + XsZ^ + d

The

x = x'

of translation are

principal directions are

di,

y = y',

1,

we have

D^,

111
4F

z = z'

2Z

find

0, 4).

The required reduced

- 4

4.

Denote by E the inverse of

1-2. i^s-

we

d = -4.

quadrlc has as center C(-l,

equation is

The equations

Prom

0, 4).

0-1=0
+22-7=0
+1=0

y +

\_v\.v.2,v^.

The equations of the

rotation of axes to the principal directions are

\_X

Y Z\- E

[X Y Z]

i/\A3"

-i/i/s"

-1/vT

2/vT

i/vT

i/vT

-i/x/T

i/VT

SUPPLEMENTARY PROBLEMS
13.

For each of the following real symmetric matrices A find an orthogonal matrix
nal and has as diagonal elements the characteristic roots of A.
,

-1

2
(a)

(b)

-1

2_

_1

(c)
2_

1/VT

(a)

-4

-4

_ 2

-2

l/\f2
(b)

i/VT

-1/VT

-i/v^T

(d)

14.

i/\/^
Ans.

()

2/3

1/3

2/3

-2/3

2/3

1/3

-1/ 3

--2/3

2/3

21

-2

id)

-1_

\/sf&

l/v/

-2/vT

l/v/s"

1/\A6"

i/a/s"

(e)
4_

(c)

1/vT

P
that /""^.^P
is diago

2"

_2

i/x/T
(e)

"3

P such

"4

-1

-1

_1

-1

r
-1
4_

2/3

1/3

-2/3

2/3

1/3

1/3

2/3

-2/3

2/3

i/vT

2/^/6'

-l/v/T

yV 6

i/VT

-1/\A2

Find a linear transformation which reduces X'BX to


X' AX to Xtyl-^X^y^ + Asy^, where
yl-^y'^-^y'l and
of \XB - A\ =0, given

Xi are the roots

-2
{a)

[7
-2
1

10

-2
l/\/T

/4ns.

(a)

-\/\[2

r2

l\

-2

5=03

7J

\\

1/3VT

1/3

-2/3x^2"

1/3

1/31/2"

1/3

fj
(6)

-4

2J

(6)

-4

-4

-4

-8

_-4

-8

2/3

2/3

l/3x/l0

2/3

1/3

2/3^/10

1/3

-2/3

2/3\/l0

1_

2
4

SIMILARITY TO A DIAGONAL MATRIX

170

15.

21

Prove Theorem IV.


Hint.
'^i

/'"^^P = diag(Ai,

If

-^r + i'

'^1

Xi,

A.1

^1 ~^v)

~'^r+2

'^ ^

Modify the proof in Problem

17.

Prove Theorems XII, XIH, and XIX.

18. Identify

2 to

20x2-

(h)

3x^+ 2%2

24xi:>;2

27%^

+ 3%2

369,

"i'

= diag(0,

0,

r.

(a)

Each characteristic

(6)

+ ^ is non-singular,
(/

root of

\Q?,

(''^

*i + 2^:1x2 + Xg

is either zero or

900,

a pure imaginary.

non-singular.

/ -.4 is

(See Problem 35, Chapter 13.)

+ /iri(/-4) is orthogonal.

normal and non-singular so also is

20.

Prove:

If

Is

21.

Prove:

If

is normal, then

22. Prove:

square matrix A is normal

x\ - 312%ia:>, + 17 %|

(c)

Prove:

Let A be real and skew-symmetric.

B =

P"\Ai/ -/1)P

"^S"

^ri)'

'^r-i-2

each locus:

(o)

(c)

X^ + i-

''^"'^

prove Theorem XI.

16.

19.

[CHAP.

is similar to

and only

if

A~'^

if it

H + iK, where H

can be expressed as

and

commu-

are

tative Hermitian matrices.

23. If

is n-square with characteristic roots

roots of

ty =

for

Prove:

If

X2A2

^n^rv

where

are XiAi,
C/"^<4(7 =

Write

Hint.

24.

AA
J

T*

= [. ],

then

is normal if imd only if the characteristic

Xi, A,2

A.,

is unitary

and T is triangular.

_
C/

Now

tr(rT') =

tr(-4/l')

requires

is non-singular, then

AA'

is positive definite Hermitian.

Restate the theorem when A

is real

and non-singular.
25.

Prove:

A and B are n-square and normal and

If

if

A and S' commute, then

26. Let the characteristic function of the n-square matrix

and suppose there exists a non-singular matrix

P'^ AP

(/)

Define by

B.-

and Xj,

(j

= 1, 2

s) the

and

BA

are normal.

A be

(X-x^f^iX-x^f'^ ...a-'Kfs

<^(X)

^B

P such

that

diag(Ai/.ri' '^2-'r2

n-square matrix diag(0,

-^sV^)

0, ...,

0,/

0) obtained

by replacing A. by

'i

r a

i).

in the right

by

member
T?
E,

Show

DR. ,
PBiP~^.

= 1,2

s)

that

(a)

P'^AP

(6)

~-

AiBi

A282+

...

(c)

= Ail + A22 + +'^3^5


Every ^ is idempotent.

(d)

E^Ej

(e)

1 + 2 + + E3

=0

for

A5B,

^j.
=

rank of ^ is the multiplicity of the characteristic root A^.

(/)

The

(g)

(Xil-A)Ei

(h)

of (/) and define

IfpM
Hint.

= 0,

(J

= 1,2

s)

is any polynomial in x. then

Establish

^^

p(A)

Aii + A22 +

p(Ai)i + p(A2)2 +

xIe

+ P(^s^^s-

Aii + A22 +

+ X^E^.

CHAP.

SIMILARITY TO A DIAGONAL MATRIX

21]

Each E^

(i)

(/)

is a polynomial in A.

Define

Hint.

matrix

Hint.

/(A)

-Xi)(A-X2)

commutes with A

fi

B commutes

If

(A

with

if

it

(A -X^)

(k)

If

is normal, then each Ei is Hermitian.

is non-singular, then

A-^
If

and

/^(A)

/(A)/(A-A^),

(i

= l,2

Then

s).

and only if it commutes with every ^


commutes with every polynomial in A.

(I)

(m)

171

Al%

A-2%

+ A-^fig

...

is positive definite Hermitian, then


,1/2

A^'^

/f

vTii

VA22

...

VAs^s

is positive definite Hermitian.

Equation

(n)

27. (o)

(6) is called the spectral

24

-20

-20

24

10

-10

"

//^

Obtain

Use Problem

Prove:

If

-4/9
4/9

-2/9

2/9

-2/9

1/9

20
29

10

-10

10

44_

38/9

0/9

-10/ 9

26

and

Prove:

The square matrix A

is non-singular, then

such that

32.

Prove:

33.

Prove:

If .4 is

34.

Let A be normal, B =

4/9

-2/9'

5/9

2/9

-2/9

2/9

8/9

-10/9
23/9_

B commute.

(/).

by H^ = AA'

Hermitian matrix

36. If

5/9

4/9

10/9'

-20/9

-20/9

If

U and a positive

definite Hermitian matrix

f/

= H~'^A.

is

normal

if

and only

if

and U of Problem 29 commute.

is similar to a diagonal matrix if

H~^AH

and only

if

there exists a positive definite

is normal.

real symmetric (Hermitian) matrix is idempotent if and only if its characteristic roots are O's

(a)

Prove: If

+ 4

-lO"

29

Prove:

Prove:

2/9"

is non-singular then there exists a unitary matrix

Define

Hint.

35.

is unique.

A= HU

such that

31.

4/9

Prove: If A is normal and commutes with B, then A' and


Hint.

30.

49

20

--

29.

it

A-^

Obtain

9_

38/9

28.

that

-4/9

lO"

-10

196

(c)

Show

Obtain the spectral decomposition


"

(b)

decomposition of A.

tf

real

A and

symmetric (Hermitian) and idempotent, then


I

is n-square, the set of

(6)

The characteristic roots

(6)

Every diagonal element of


of values of A.

(c)

If

(d)

If

is real

is real

= tiA.

is unitary.

~iH)

numbers X' AX where

(a)

I's.

+ A be non-singular, and C = B^^B'.

(S')"^ commute,

is Hermitian, then {I + iHT'^{I

r.

and

of

is unitary.

is

2i

unit vector is called the field of values ot A.

A are in its field of values.


A and every diagonal element

of

U'^AU, where U

symmetric (Hermitian), every element in its field of values is

symmetric (Hermitian),

values
least and Ais the greatest characteristic root of A.
its field of

is the set of reals

Prove:

is unitary, is in the field

real.

Ai i A < A, where Ai

is the

chapter 22

Polynomials Over a Field


POLYNOMIAL DOMAIN OVER
commutative with

to be

Let X denote an abstract symbol (indeterminate) which is assumed


and with the elements of a field F. The expression

F.

itself

n-i

n
(22.1)

f(X)

where the

a^ are in

cirik

is called

On-^X

a^X +

p
a^X

a polynomial in \ over F.

(22.1) is called the zero polynomial and we write f(\) = 0. If a^ ^


of
degree n and a is called its leading coefficient. The polynomial /(A)
(22.1) is said to be
=
said
to
be of degree zero; the degree of the zero polynomial is not defined.
Oq^
oq ^
is
If

If

every

a =

oj;

polynomial is called raonic.

in (22.1), the

polynomials in A which contain, apart from terms with zero coefficients, the same
terms are said to be equal.

Two
The

totality of polynomials (22.1) is called a polynomial

SUM AND PRODUCT. Regarding

the individual polynomials of

tem, the polynomial domain has most but not


f(X) + g(\)

If

(i)

f(X) is of degree

^(X) + g(A)

is of

and g(X) is of degree

degree

TO

when m.>n,

F[X\ as elements

all of the properties of a field.

of a

number sys-

For example

f(X)-g(X) = g(X)-f(X)

and

= g(X) + f(X)

domain F[\] over F.

n,

of degree at most

m when m=n, and

of degree

re

when m<n.
(ii)

/(A)-g(A)
If

/(A)

If

g(A) ^

QUOTIENTS.

In
I.

?^

is of

while

m + ra.

f(X)-g(X) =

0,

1,

then

A(A) =

A:(A)

we prove

/(A) and g(X) ^

are polynomials in F[A], then there exist unique polynomials

and r(A) in F[A], where r(A)


that of g(A), such that
/((A)

(22.2)

g(X) =

then

and h(X)-g(X) = h(X)-g(X),

Problem
If

degree

/(A)

is either the

zero polynomial or is of degree less than

h{X)-g{X) +

r(A)

Here, r(A) is called the remainder in the division of /(A) by g(A).


to divide /(A) while g(A) and h{X) are called factors of /(A).

172

If

r(A) = 0, g{X) is said

CHAP.

POLYNOMIALS OVER A FIELD

22]

Let

= h(X)-g(X).

/(A)

When

Example

Over the rational field A, -3 is irreducible; over the real field


Over the real field (and hence over the rational field) A^+4
field it is factorable as (\+2i)(\-2i)

1,

THE REMAINDER THEOREM.

Let f(X) be any polynomial and g(X) =

(22.3)

By

is free of A.

n.

When

(22.3),

f(a) =

/(A) is divided by

h(X)-(X-a) +

/(A)
r

when g(A,) = c, a constant,


F is called irreducible over

of degree zero, that is,

non-constant polynomial over

only factorization is trivial.

if its

where

g(A.) is

the factorization is called trivial.

173

r,

is factorable

it

is irreducible;

X-

a.

Then

as (X+\/3)(X-^j3).
over the complex

(22.2)

becomes

and we have

A- a

until a remainder free of

A is obtained, that remainder

is f(a).

m. A

polynomial /(A) has

GREATEST COMMON

DIVISOR.

If

X-

a as a factor

h(X) divides both f(X)

and only

if

and

if

f(a) =

0.

g(A), it is called a

common

divisor of

f(X) and g(A).

polynomial d(X) is called the greatest common divisor of

(i)

d(X) is monic,

(ii)

d{X) is a

(Hi)

common

/"(A)

and g(X)

if

divisor of /(A) and g(X),

every common divisor of /(A) and g(X) is a divisor of d(X).


In

Problem

2,

we prove

IV. If /(A) and g(X) are polynomials in F[X],

a unique greatest

common

not both the zero polynomial, they have

divisor d(X) and there exist polynomials h(X) and ^(A) in F[X]

such that
(22-4)

d(X)

h(X)-f(X) +

k(X)

g(X)

See also Problem

When the only common divisors


is

d(X) =

of

/"(A)

3.

and g(X) are constants, their greatest common divisor

1.

Example?. The greatest common divisor

A +3A+5. and

of /(A)

(A^

+4)(A^+3A+

and g(A) =

5)

(A^

-1)(A^ + 3A+5)

is

(22.4) is
A.=

3A +

i/(A)

- lg(A)
5

We have also

(1-A)-/(A) + (X+4)-g(X)

0.

This illustrates

If the greatest common divisor of /(A) of degree re>0 and


g(X) of degree m>0 is
there exist non-zero polynomials a(X) of degree <m and 6(A) of degree <n such
that

V.

not

1,

a (A) -/(A) +

b(X)- g(X)

and conversely.

RELATIVELY PRIME POLYNOMIALS. Two


common

divisor is

i.

See Problem

polynomials are called relatively prime

if

4.

their greatest

POLYNOMIALS OVER A FIELD

174

VI. If g(A) is irreducible in

F[X] and /(A)

divides /(A) or g(X) is relatively prime to


VII.

If

VIII.

and

UNIQUE FACTORIZATION.

g:(A)

/(A)

h(X), it divides at least

are relatively prime and

if

each divides

one of

/(A)

and

h(X).

soalsodoes /(A)-g(A).

A(A),

Problems, we prove

In

Every non-zero polynomial f(X) of F[A] can be written as

IX.

(22.5)

/(A)

where

any polynomial of F[X], then either g(X)

is

/(A).

g(A) is irreducible but divides

If /"(A)

[CHAP. 22

is a

9i(A)

?2(A)

^^(A)

constant and the qi(X) are monic irreducible polynomials of F[X]-

SOLVED PROBLEMS
1.

Prove:

/(A) and g(X)^0 are polynomials in F[X], there exist unique polynomials h(X) and r(X)
where r(A) is either the zero polynomial or is of degree less than that of g(X), such that

If

in F[X],

(i)

h(X)-g(X) +

/(A)

r(X)

Let

n-i
+ %1-iA.
+

n
/(A)

o^jA

g(A)

h^X

+ a^A + Oq

and

Clearly, the theorem is true

/(A)

if

7^ A

/(A) =

i^-iA

or if

g(A)

n<m.

6iA +

n>m;

Suppose that

cpX

/i(A)

6o.

^m

?^

then

c^-iA

Co

is either the zero polynomial or is of degree less than that of /(A).


If /i(A)

r(X) = /i(A).

or is of degree less than that of g(A),

we have proved

the theorem with h(X) =

~X

and

Otherwise, we form

/(A)

^A"-\(A) - ;^A^-\(A)

/,(A)

f^X) =0 or is of degree less than that of g(A), we have proved the theorem. Otherwise, we repeat
Since in each step, the degree of the remainder (assumed ^ 0) is reduced, we eventually reach
a remainder r(A) = 4(A) which is either the zero polynomial or is of degree less than that of g(A).
Again,

if

the process.

To prove uniqueness, suppose


/(A)

where the degrees of

r(A)

h(X)-g(X)

and

+ r(X)

/(A)

k(X)-g(X)

'c(A)-g(A)

s(A)

Then

and s(\) are less than that of g(A).


h{X)-giX) + r(X)

+ s(X)

and
[k(X) -h(X)]g(X)

Now r(A)-s(A)

is of

to or greater than m.

degree less than

Thus, k{X) - h(X)

h(X) and r(A) are unique.

while, unless
=

0,

r(A)

A:(A)

s(A) =

r(X)

s(X)

h(X) = 0.

[k(X)

- A(A)]g(A)

is of degree equal

so that k{X) = h{X) and r(A) = s(A).

Then both

CHAP.

2.

POLYNOMIALS OVER A FIELD

22]

175

Prove:

If f(X) and g(A) are polynomials in F[X], not both zero, they have a unique greatest
divisor d(\) and there exist polynomials A(A) and k(\) in F such that

da)

(a)

/(A) =

say,

If,

A(A) =

then d(X) =

0,

and k(X) = 6

bin,

(*)

is not greater

r^^(X)

=0

r^(A)

If

7^

degree less than that of g(A).

or is of

and k(X)

6^

or is of

degree less than that of

it

or is of

rgCA) =

degree less than that of

'2(A)

it

b'^^giX)

and we have

(a)

with

+ r^iX)

/(A)

we have from

= 0,

If r^(X)

ri(A).

g(A)

-92(A) -/(A)

93(A)

'i(A)

and from

then d(X) =

( i

?i(A)-g(A)

we have

(i")

where

= 0,

obtain (a) by dividing by the leading coefficient of ri(A).

r^(X) 4 0,

If

we have

ri(A)

If r^(X)

g2(A)-ri(A)

HiX)

and from

I,

we have

0,

g(A)
/^(A)

(a) with

(ii)

where

By Theorem

than that of /(A).

?i(A)-g(A) +

/(A.)

where

we have

leading coefficient of g(k) and

is the

Suppose next that the degree of g(A)

h(X) =

ka)-g(\)

h{X)-f(X) +

where i^

g(X)

common

r^(X').

If rg(A) = 0,

?2(A)-'-i(A)

TsCA) + rsCA)

g(A)

we have

?2(A)[/(A)

from

i)

and

(ii)

9i(A)-g(A)]

[1 + 9i(A)-92(A)]g(A)

obtain (a) by dividing by the leading coefficient of r2(A),

Continuing the process under the assumption that each new remainder is different from

0,

we have,

in

general,
(v)

'i(A)

9i+2(A)-'-i+i(A)

ri+2(A)

moreover, the process must conclude with


(V)

's-2(A)

9s(A)-'-s-i(A)

&(A)?^0

r^cA),

and
(^^

's-i(A)

By

divides rs-i(A), and by

(vi), rjCA)

's-3(A)

so that rs(A) divides r^-gCA).

both /(A) and g(A).

Prom(l)

If

ri(A)

'2(A)

Prom

(iii),

/3(A)

'3(A)

Continuing,

d(X)

The proof

9s+i(A)-'-s(A)

also divides rs_2(A).

/(A)

91(A)

g(A)

qa(>^)

r^(X)

/!i(A)

[1 + 92(A) 93(A)]/(A)

h3(X)-f(X)

c-\^(X)

rf(A)

-/(A) + /ci(A)

g(A)

[-9i(A)

we have

we conclude

and substituting

93(A)

(vi),

that rs(A) divides

= e~^'s(A).

A2(A)-/(A)

Substituting for r^(X) and r2(A),

(iv).

r5_i(A)

then

c,

[l + 9i(A)-92(A)]g(A)

we obtain

Prom

Thus, by retracing the steps leading to

-52(A). /(A)

=
t^(K)

9s-i(A)-'s-2(A)

the leading coefficient of rs(A) is

in (ii)

+ i2(A)-g(A)

we have

9i(A)

92(A) 93(A)]g(A)

43(A) g(A)

finally,
rs(A)

Then

(v).

hs(X)'f(X)

c-i/5(A)-/(A) + c-iA:s(A)-g(A)

that d(X) is unique is left as an excercise.

ksiX)-g(.X)
=

h(X)-f(X) + k(X)-g(X)

as required.

POLYNOMIALS OVER A FIELD

176

3.

[CHAP. 22

Find the greatest common divisor d(X) of


=

/(A)

and express d(\)

We

and

3A^ + 1)C + llA + 6

Theorem

in the form of

g(\)

)C

2)i-)^-X+2

III.

find

+ 4A^+6A+4)

(i)

/(A)

(3A+l)g(A) +

(li)

g(A)

(A-2)(A^ +

(iil)

A^+4A2+6A + 4

(A-3)(A^ + 7A+10) + (17A + 34)

7A + 10

(^A + p^)(17A+34)

(A'

6A+4) +

4A^ +

+ 7A+10)

(A^

and
(Iv)

A^

The greatest common


Prom

+34

Substituting for A^ + 7A + 10

17A

for

A +

(iii),

17A

and

J-(17A+34)

divisor is

from

+34

+ 4A^ + 6A + 4 from

+ 4A^ + 6A + 4)

(A

(A^

+ 4>f + 6A + 4)

(A

- 3)[g(A) - (A- 2Xi^ +

(A^

- 5A +

(A^

- 5A + 7)/(A) + (-3A +

i(A^ - 5A +

3)(A^ + 7A + 10)

(ii)

7)(A^ + 4>f + 6A + 4) -

(A

4;? +

6A+4)]

- 3)g(A)

+34

17A

(A

14A^

- 17A - 4)g(A)

Then

A +

4.

Prove:

If

common

the greatest

divisor of /(A) of degree

there exist non-zero polynomials o(A) of degree

< m and

+ 14A^ - 17A

4)-g(A)

and g(X) of degree m >


such that

re>

is not 1,

b(X) of degree < n

b(X)-g(X)

a(X)-f(X) +

(a)

i(-3A

7) -/(A)

and conversely.
Let the greatest common divisor of /(A) and g(A) be d(X) ^
/(A)

where

/i(A) is of

degree

<n

and

rf(A)-/i(A)

and gi(A) is of degree


gi(A)-/(A)

then
d(X)-g^(X)

g(A)

<m. Now
=

gi(A)-rf(A)-/i(A)

g(A)-/i(A)

and
gi(A)-/(A)

Thus, taking a(A)

= gi(A)

and 6(A) = -/i(A),

we have

(a).

Conversely, suppose /(A) and g(A) are relatively prime and


polynomials h(X) and ^(A) such that
A(A)-/(A) +

Then, using

[-/i(A)-g(A)]

k(X)-g(X)

(a) holds.

Then by Theorem IV

there exist

(a),

a(X)

and g(A) divides

a(A).

a(X)-h(X)-f(X)

a(A)-A:(A)-g(A)

-b(X)-h(X)-g(X)

a(A)-i(A)-g(A)

But this is impossible; hence,

if (a)

holds, /(A) and g(A) cannot be relatively prime.

CHAP.

5.

POLYNOMIALS OVER A FIELD

22]

177

Prove: Every non-zero polynomial /(A) in F[\] can be written as

/(A)

where

c-q^(X).q^a)

...

qr(\)

constant and the q^(\) are monic irreducible polynomials in F[A].

is a

,^

Write
(i)

/(A)

where a

is the leading coefficient of /(A).

theorem.

Otherwise, there is a factorization

(")

and

are irreducible, then

/!(A)

a-/i(A)

If /^(A)

/(A)

If g(A)

is irreducible,

then (i) satisfies the conditions of the

n-g(A)-A(A)

satisfies the conditions of the theorem.

(ii)

Otherwise, further factor-

ization leads to a set of monic irreducible factors.

To

prove uniqueness, suppose that

"n
are two factorizations with

by a change

Pi(A) which,

Then

Pi(A).

Eventually,

ity is impossible,

?2(A)

r<s.

P2(A)-p3(A)

= s

.
.

a p^(X) PaCA)

and

9r(A)

p^CA)

... ps(A),
it must divide some one of the
Since Pi(A) is monic and irreducible, ?i(A) =

Pi(A)-p2(A)

may be taken as

p^(\).

and, after a repetition of the argument above, 92(A) = P2(A).


r and Pr-^i(A)
Pr+2(A) ... p^cA) = 1. Since the latter equaland uniqueness is established.

9i(A) = Pi(A) for

Since 91(A) divides

in numbering,

92(A) divides

we have

9i(A)

...

Ps(A)

= 1,2

SUPPLEMENTARY PROBLEMS
6.

Give an example

7.

Prove Theorem

8.

Prove:

9.

Find a necessary and sufficient condition that the two non-zero polynomials
/(A) and g(A)
each other.

10.

If /(A)

in

which the degree

divides g(A) and h{\).

it

()

/(A)

2A^-A=+2A2-6A-4,

(6)

/(A)

A^-

(c)

/(A)

2A^ + 5A^+ 4A=

(d)

/(A)

3A^ -

(a)

A=-2

(6)

A-

A"

4A='

3A=

+ A^

- 11A+

^+1

^"^^

g(A) + h(\).

divides

- ^ (A-

g(A)
g(A)

- A^ - A +
- 5A + 6,
1)/(A) +

A*-A=-A2+2A-2

A=

g(A)

1,

g(A)

in the form of

- 2A" - 2A -

A% 2A%

A= + 2A

-h

in

F[X] divide

Theorem IV

2A +

i (2A=+ l)g(A)

-l_(A+4)/(A) + l-(A^+5A+5)g(A)
to

('^)

degree of either /(A) or g(A).

III.

For each of the following, express the greatest common divisor

Ans.

11.

of /(A) + g(A) is less than the

lo

^(A+4)/(A)-^ ^(-2A=^-9A2_2A + 9)g(A)

jL(5A+2)/(A)

^^{-l5X^+4.A>?~55X+ib)sa)

Prove Theorem VI.


Hint.

Let d{\) be the greatest common divisor of /(A) and g(A)


h{X) is a constant.

then

g(A) = d{X)-h(X)

and either d(X)

or

POLYNOMIALS OVER A FIELD

178

12.

Prove Theorems VII and

13.

Prove:

14.

If

VIII.

15.

A=-

/(A)

A^-

(b)

/(A)

(A-l)(A+lf(A+2).

g.c.d.

A-l;

(b)

g.c.d.

(A+i)(A + 2);

l.c.m,

(A+l)(A+2f(A-3)

g(A)

(a)

Given 4

divides a( A).

Find the greatest common divisor and the least common multiple of

(a)

g(A)

it

g(A) is a monic polynomial which is a multiple of both /(A) and g(A),

The least common multiple of /(A.) and


and is of minimum degree.

Ans.

and divides g(X)-a(X),

/(A) is relatively prime to g(A)

1,

[CHAP. 22

(A^- l)(A^ + A+

l.c.m.

1)

(A- l)(A+ if (A + 2f (A-

3)

show

12 ;2 ij
(o)

(f)(k)

(b)

m(A)

= 0.

A^

SA^

when m(A)

16.

What property of a

17.

The scalar
and only

18.

if

c is

A-

Take
Show

- 4A -

field is not satisfied

(A-c)^g(A).

k>

(a)

of /(A)

if

that if D(A) is the greatest

Prove:

Let

/I.

sA!^

- 9A -

51

by a polynomial domain?

Show

/(c) = 0.

Prove: The scalar e is a root of /(A)

that c is a root of multiplicity

and only

common

if c

if

An n-square matrix A

is normal it

k-l

of /'(A),

(b)

common

divisor d(X). Let

/(A) = s(X)-D(X).

'

Show

that c is a

is a root of both /(A) and /'(A).

be any field containing F.

divisor of /(A) and g(A) considered in K[A], then

d(X) = h(\)- f(\) + k(X)- g(\).

a^A
in

/(A) and g(A), not both 0, in F[\] with greatest

Hint:

20.

A^

A -

(f)(A)

called a root of the polynomial /(A) if

root of multiplicity

19.

and

c is a factor of /(A).

/(A) =

Suppose

- 9A -

g(A) = t(X)-D(X).

can be expressed as a polynomial

+ a^^^A

...

+ aiA + oqI

D (A)

and D(X)

= d(X).

c{X)-d(X).

chapter 23
Lambda

Matrices

DEFINITIONS. Let F[X] be a polynomial domain consisting


in F.

mxn

non-zero

(23.1)

of all polynomials in A with coefficients

matrix over F[Aj

^(A)

[ij (A)]

oii(A)

ai2(A)

...

ai(A)

asiCA)

a22(A.)

...

CsnCA)

Omi(A)

a^^iX.)

...

aj^n(>^)

is called a A-matrix.

Let p be the maximum degree in A of the polynomials a^jiX) of


(23.1).
written as a matrix polynomial of degree p in A,
(23.2)

where the ^^ are


Example

Ap\^

^(A)

mxn

sp-l
A.^X^-^

...

A,\

+ Ao

matrices over F.

A^+A
1.

Then A(X) can be

A* + 2A= + 3A^ +
5]

'^(A)
.

A=

- 4

A^

- 3A=

J
1

A^ +

A +

-J

-4
L

is a A-matrix or matrix polynomial of degree four.

A(X) is ra-square, it is called singular or non-singular


according as \A(X)\ is oris not
Further. A(X) is called proper or improper according
as A^ is non-singular or singular
matrix polynomial of Example 1 is non-singular and
improper.
If

zero.

The

OPERATIONS WITH A-MATRICES.

Consider

two

the

n-square

A-matrices

over F(X)
(23.3)

A^X^

^(A)

.P-'

S^_,

or

matrix polynomials

+ Ai^X + Aq

p -1

and
(23.4)

(j

B(X)

a"^

'

...

SiA + So

The matrices (23.3) and (23.4) are said to be equal, ^(A) = 5(A), provided
= Bp=a andA-VI,'
^ ^
= 0,1,2
p).

The sum A(X)


two A-matrices.

+ S(A) is a A-matrix C(A) obtained by adding corresponding


elements of the

The product /1(A) B(A) is a A-matrix or matrix polynomial of degree at


most p+q. If either
^(A) or B(A) is non-singular, the degree of ^(A) 5(A) and also of
5(A) /1(A) is exactly p + 9.
The equality (23.3) is not disturbed when A is replaced throughout
by any scalar k of F
For example, putting A = A in (23.3) yields

A(k)

Apk^

+ Ap_.^k^~^ +

179

...

A^k ^ Ao

LAMBDA MATRICES

180

However, when \

[CHAP. 23

two results can be obtained due

is replaced by an re-square matrix C,

We

fact that, in general, two ra-square matrices do not commute.

Ag(C)

(23.5)

A^C^

. ^^
C^A^

.
C^
+ .

A^_,C^
p-i

to the

define

+ A,_C + Ao

...

and

4 (J)

(23.6)

A^
^p

+ ^^^i +

^o

called respectively the right and left functional values of A(\).

Example

Let

2.

(X)

A'
Jx^

X
XmI

\X-2
Then

X''

P
[O

+ 2j

Ajf(C)
[o

ij [3

o1.

To

i1.

To

ij

[l

oj

[-2

'

4J

[1

oJ

'

4)

[3

il

and
2j

[-2

_3

4_

[lo

15

14

26

12

[17

27

2J

j,nd

Al(C)
See Problem

DIVISION.

Problem

In

I.

If

1.

we prove

2,

A(\) and B{X) are matrix polynomials (23.3) and (23.4) and

polynomials

then there exist unique matrix

Q2(X), R2(X), where /?i(A) and

Qi_(X), RxiX);

degree less than that of 6(A), such that

/?2(A) are either zero or of

(23.7)

Bq is non-singular,

if

.4(A)

(?a(A)-S(A) + Ri(A)

A(X)

S(A)-(?2(A) + R2(X)

and
(23.8)

S(A) is called a right divisor of i(A);

/?i(A)=0,

If

visor of

/1(A).

Example

3.

jV
A

If

(A)

A"^

+ A -

2A^

A% A% A +

-A

Ta^-i
2A

|_

and
2A''

2A

JL

(A)

A'^+aJ

'1

'

a^aJ

ii

A-iirA^+i

6(A) is called a left di-

X%

21

'4(A)

/?2(A) = 0,

if

r 2A

2A+3

j_-5A

-2A_

Q-l(X)-B(X)

then

Ri(X)

and

A(X)
1_

A^ + aJLA-I

B{k)-Q2{X)

ij

Beie, B (A) is a left divisor of A (A)

See Problem

matrix polynomial of the form

is called scalar.

bqX"-!^ + 5,_,A'?"'-/ +

6(A)

(23.9)

trix

3.

scalar matrix polynomial

B(A) =

...

+ b,X

fc(A)-/

+ bol^

polynomial.
If in

(23.7) and (23.8),

A(X)

(23.10)

6(A) = b(X)

4.

Let

then

Q^(X)- B(X) + Rt(X)


rA^ +

Example

I.

2A

B(X)- Q^(X) +

A + ll
and

(A)

LA^^-I

2A+1J

commutes with every

B(A) = (A + 2)/2.

Then

R^X)

i(A)-/
ra-square ma-

CHAP.

LAMBDA MATRICES

23]

rA + 2

If

/?i(A)

If

-ll

To

ll

^(A) = 6(A)- /

in (23.10), then

181

and we have

(?i(A)

matrix polynomial i(A) = [oy (A)] of degree n is divisible by a scalar matrix polynomial S(A) = 6(A) / if and only if every oy (A) is divisible by 6(A).
II.

THE REMAINDER THEOREM.

Let A(X) be the A-matrix of (23.3) and let B =[6^,-] be an re-square


Since Xl - B is non-singular, we may write

matrix over F.
(23.11)

A(\)

(2i(A)-(A/-S) +

A(\)

(A/-fi)-(?2(A) + R2

/?!

and
(23.12)

where R^ and Kg are free of

m.

If

A.

It

can be shown

the matrix polynomial A(X) of (23.3) is divided by \I

re, until

remainders ^1 and R2, free of

-B, where B =[b^-]

is

ra-

''

A, are obtained,

then

Aj^(B)

A^B^

Ap_^B^~'

...

a,

Aj^(B)

B^Ap

B^~'a^_^

...

+ BA, +

A,B

+ Ao

and

Examples. Let

^(A)

."^M

LA-2

A='

[a +

\I - B

and

+ 2j
3

IfA-l

[4

A+

a'-JH'

4JL-3

r~^

"^

|_-3

A-4J

-2I
A-4J

"

^
The

flO

isl

[14

26j

[17

27]

'^'

and

'^'^

Prom Example

When ^(A)

2,

a!4J

R^ = Ag(B) and

j?2

(^'-B^Q^i^^-^^

= A^(B) in accordance with

Theorem

III.

is a scalar matrix polynomial

'4(A)

/(A)-/

apIX^ + ap.jx''^ +

...

+ ai/A + Oq/

the remainders in (23.11) and (23.12) are identical so that

^1

^2

apB^

ap_^B^~^

...

+ oifi + aol

and we have
IV. If a scalar matrix polynomial /(A)-/ is divided by
free of A, is obtained, then

As

a consequence,
V.

until a remainder/?.

we have

scalar matrix polynomial /(A)-/ is divisible by

CAYLEY-HAMILTON THEOREM.
trix

A/-B

= f(B).

KI^- B

if

and only

if

f(B) =

0.

Consider the re-square matrix A = [o,-,-] having characteristic maijj


Xl - A and characteristic equation 0(A) = \Xl - A\ = 0. By (6.2)

(A/-i)-adj(A/-i)

(f>(X)-I

LAMBDA MATRICES

182

Then ^(A)

6.

(^(A) =

by XI - A and, by Theorem V,

/ is divisible

Vl. Every square matrix A =

Example

[CHAP. 23

The characteristic equation

[oj,-]

of

Thus,

0.

satisfies its characteristic equation

is

V.i

A -

7>i

32

62

31

31

63

31

31

62

32

+ llX - 5 =

<fi(X)

0.

0.

Now

and
32

62

3l"

31

63

31

31

62

32_

- 7

"2

12

13

12

7.

11

_1

o"

1_

See Problem

4.

SOLVED PROBLEMS
1.

For the matrix

A(X)

[,
A+

^'<'^'

compute

ij

-4

and 4r(C)
^p(C)

when

C=

i o]C

"3

'2
'

o][o

'

2J

c 3

and

2.

there
^(A) and B(A) are the A-matrices (23.3) and (23.4) and if Bq is non-singular, then
are
either
and
R^iX)
exist unique polynomial matrices Q-,(X), i(A); QsO^). R2M. where Ri(A)
zero or of degree less than that of 6(A), such that

Prove:

If

(i)

^(A)

QAX)-B(X) +

^(A)

6(A) (?2(A) + R^iX)

Ri(X)

and
(ii)

If

p <

q,

then

(I)

and

holds with (3i(A) =

where C(A)
If

is either zero or of

degree at most p -

C(A) is zero or of degree less than


(3i(A)

q,

Suppose that p iq; then

fii(A) = A(X).

4(A) - ApB-^B(X)X^''^

C(A)

1.

we have

ApB'q'-^''^

(i)

with

and

R^iX) =

C(A)

CHAP.

LAMBDA MATRICES

23]

C^X +

C(X) =

If

...

where

A(\)
If

s>

q.

form

- ApB-^''B(\)\P-1 -

D(X) is either zero or of degree less than

Qi(\)

we continue

otherwise,

of decreasing degrees,

and we have

To

the process.

we

q,

C^S-1B(A)X^-'?

we have

and

Since this results in a sequence of matrix polynomials C(A), D(A),

(II),

begin with

- B(X)B-^''ApXP~1

This derivation together with the proof of uniqueness will be

rA^ +

and
A + A'
A"
A^ +

find matrices Q-l(X), Ri(X);

A(X)

as an exercise. See Problem

left

1,

Chapter 22.

A'-A-l]

2A=-l

^(A)
|_
L

(a)

A=
A"

(?2(A),

S(A)

L-A^ + 2

A^

- A

R2(X) such that

= Qi(X)-B(X) + Ri(X).

= S(A)-(?2(A) + R^iX)

A(X)

(b)

as in Problem

2.

We have

and

Here,

So

-ii

L-1
(a)

and

Tl

-,

S2

ii

[i

ij

2J

We compute
^(A)

C(A)

-^4S;^5(A)A^

- C3S2^S(A)A

1'

-2
A +

1]

-10
-6

and
D(A) - D^B^^BCX)

-6

5"]

(?i(A)

(A^X^ + Cs A

+O2 }B^-^

A +

-1'

f-l

11

C(A)

-1I

11
L

D{X)

-6A-13
-2A-9
A= +

q
+

fA^

P-i

^
A +

5A + 3
3A + 5

fli(A)

--

-l"!

fo

J
3

'1

Then

-9

[_

3J

3]

["-13

^ +

-2

10

A2 +

=
3

(b)

...

(i).

obtain

Given

Ri(X) = 0(A)

ultimately reach a matrix polynomial which is either zero or of degree less than
g

^(A)

3.

D(X)

with

(i)

Aj^B-^XP'^l .C^B'^^X'-^
ApB-'XP'^
+ C^B^^X^-'f

183

4A

2A+

+ 4
4

5]

A^ +

A +

sA +

3A +

^1

e]

We compute
^(A) - B(A)S2^'44A^

(A) - 5(A) B2

E(X)

^sA

and
F(X) - B(X)B2

F^

ftsCA)

LAMBDA MATRICES

184

Then

B2^(A^>? + 3 A +

QziX)

F2)

[A^+4A

4.

Given

and A

also, since

use the fact that A satisfies

2X +

+ 4

6A +

A^ +

[CHAP. 23

2I

3A + 5J

equation to compute A"

its characteristic

compute A

is non-singular, to

A-1

-1

-2

-3

A-1

-1

-2

-3

A-1

A/-^|

and A'

A^-3A^-7A-11

Then

"8

5"

_13

3A +1 A + 11/

^*

3/ +7/

From

11/

UA

-1 A -3-4

+A

"l

2"

8_

42

31

29"

45

39

31

_53

45

-J-7/

1_

5'

13

42_

0"

11

'8
+ 7

"1

_2

1_

42

31

29

45

39

31

53

45

42

193

160

224

177

160

272

224

193

144

we have
'1

"1

0'

34 +/II

'1

2"

1_

_13

8_

11

11
_0

1_

-1

-2

-1

-3

-1

-2

11

i{_7^-i -3I

+ A\

-1"

-3

-1

-2

121

-8

-24

40

-1

"1

- 33

0"

1
1

29'

-24

121

-27

5.

Let Ai, As,

,^n

of degree p in

x:.

40

-8

be the characteristic roots of an re-square matrix A and let h(x) be a polynomial

Prove that

\h(A)\

= h(\)- h(X2)

h(X^).

We have
(i)

\XI-A\

(A-Ai)(A-Ao)...(A-A^)

Let
(ii)

h(x)

c(,si- x)(s2-x) ...(Sp - x)

Then
h(A)

c(sil-A)(S2l-A)...(spI-A)

4
CHAP.

LAMBDA MATRICES

23]

185

and
|A(^)|

c'P\siI-A\-\s2l-A\ ...\sp!-A\

!c(si-Ai)(si-A2)

...

(si-A)i

Jc(S2-Ai)(S2-A.2)

using

(S2-X)!

...

{c(Sp -\t)(,Sp -As)

...

(Sp

jc(Sl-Ai)(S2-Xl) ... (S^- Xl)i


{c(si-A2)(S2-A2) ... (Sp-\2)\

...

ic(si-A)(s2-A)

...

(s^-A)!

...

-X)!

h(\i_)h(\2) ...h(\^)

(ii).

SUPPLEMENTARY PROBLEMS
6.

Given

^1

Pr^
[a^+i

A(\) =

_r2\2+2A
(a)

-4(A)

A(\) - B(A)

^(A)-S(A) =

2A -

r2A

-A^l

[a^-a

-ij

2A=+A=+A

L ^ +2A^ (rf)

r2A'' +

B(A)M(A)

1_

7.

Given

Ag(C)

^;?(C)

^^^^1
a

compute:

A^+2A1

+ A + 2

rA* +
(c)

[a +

+ B(A)
[_A^

(6)

B(A)=r^^

sna

A-iJ

2A='

ij

A* + 3A= + 3A=1
-,
A
o
A-'

3A'+A^+A

2A^

+ 3A^ + 3A

A'

2A^J

-a]

2\^

/1(A)

compute:

-2

B;p(C)

-1

-3

[-:
<?i?(C)

3"]

-3I

Bn(C)-4n(C)
i?^

4;f(C)-B^(C)
[l7

^i(C)-B^(C)

B^(C)-A^(C)

E
3-

^i(C)

[3

-7J

[::;]

:]

--M

where P(A) = 4(A)- B(A) and Q(X) = B(A)M(A).

8.

If /4(A)

and B(A) are proper n-square A-matrices of respective degrees p and q, and
show that the degree of the triple product in any order is at least p+q.

A-matrix,

if

C(A) is any non-zero

LAMBDA MATRICES

186

9.

For each pair of matrices A(X) and B(X), find matrices

|> + 2X
(a)

.4(X)

AiX)

-A.2

'

,,

La%i
f

(b)

"

,,

a^-aJ
+

U.2A-1

.4(A)

A'*

2A - A +

B(X)

(d)

A(X)

(a)

2A

A^

A-2

2A

A+

A"-l

A=-A^+l

A* + A^ +

A^ + A

A+

A^+3

-A+7

A^-1

3A + 5

-3A+2

A-6

A"

A+

A -

81A +

15A -

-7A

3A^+5A

+ 31

-3A -

46

-15A - 30

Verify in Problem 9(b) that

17A -

-21A + 4

-2A+3
loA +

- 2A -

2A^-4A

-A +

llA +

4A - 4

2A +

16A - 16

Ri(A) = .4^(0) and

4A -7
1

+ 3

2A - 2A -

-12A -

+ 8

-2A + 6A - 6

A"

-26A - 30

J,

23

12A -

-A^-A-4

A - 14

7lA +

-85A -

16

ll

figCA) = -4^(0)

A^ + A -

o]

[l

ij

f-

-fjA-3

-A^ +
2A'^

-12A -

A^-2A

fo

ll

-16A + 14

3A'^-7A

+ A

A +

|^_ 1

5A-7

-9A -

-A +

|^_^

A=^

46

4A -

fti(A)

- A -

A''

/?2(A) =

-2A -

fl2(A)

,,(A)=

-3A^-5A-16

''^'''-

f-A-l

-A+1

A^ +

a-i1
-A + 2j'

+2

_J,

3A^ + 6A + 31

<?2(A)

A^

A-2

^=^'^^

.,(A)^0;

10.

2A* + A -

-A-l]

,2

B(X)

ij'

A-l

A^-2A-1
A+1

r 2a

2A-3

Ri(X)

A -

0,(A)=

Qi(X)

A^-A

'^^''-'-

<?2(A)

8A -

A + A^ +

a1

6A+

4A^ +

A""

\^_^^^

(d)

+ 2

+ 4

3AnA"-l

(?i(A)

5A^+2A

A^ + 2A''

f-A

(c)

7A-2

+ A^ +

x4

[o

J'

3A^+ 2A

-ll

FA-I

''''-

3A-1

A^+1

(b)

-aJ

[\+i
Ans.

''''-'I

A'^

3A -

3A''

a]

A^-2A+1
(c)

Fa

-A^ + 2A + 2l

A.

Q^X), Rd^) satisfying (23.7) and (23.8).

(?i(A), /?i(A);

BiX)

[CHAP. 23

where B(A)=A/-C.

..(A).
-5A + 2

A-5
18A-7

CHAP.

11.

LAMBDA MATRICES

23]

3X+

A-2

^2
A^-3A

[\^

Given

compute

(b)

find (3(A) and fi(A) of degree at

most one such that

A^'+SA^+sA

L^- 5A=+llA Given

A 1
A
A+lJ

A +

B(A)- C(A)

Fa^'+SA^-VA-

12.

[A - 3
^A

=1fx

C(X)

''*

+ 2j

(a)

.4(A) =

187

4(A) = Q(X)- B(\) + R(\).

compute as

in

r-9 A +

si

A -

A^

^^1
i7j

|_i2

[I
Prove:

14.

A and B

If

Show

Prove:

If

and B
S^

= diag(Si,

are similar for any positive integer k.

B^) and g(A)

g(B)
15.

!;]

are similar matrices and g(A) is any scalar polynomial, then g(A) and g(fi) are similar.

first that

fi

10J

Problem 4

'1

Hint.

9A +

|_13A-6

ij

10

13.

-A - gl

S(A)

A-6

A='-A^-3A+2j

10

A +

[A + 4

l]

is

any scalar polynomial, then

g(B))

diag(g(Si), g(B2)

Prove Theorem ni.


Verify: A/ - B divides A(\)-Ajf(B).

Hint.
16.

The matrix C is called a root of the scalar matrix polynomial B(A) of (23.9) if B(C)
C is a root of B(A) if and only if the characteristic matrix of C divides B(A).

17.

Prove: If Ai, Aj
A are the characteristic roots of A and
characteristic roots of f(A) are /(Ai), /(A2)
/(A)Hint.

Write

\- f(x)

Nowuse

18.

c(xi- x)(x2-

|*j/-4|

- x)

x) ...(x^

so that

f(A) =

A'^

-2A+3,

given

19.

Obtain the theorem of Problem 5 as a corollary of Problem 17.

20.

Prove:

21.

Let A(t)

If

is an invariant vector of

= [a^,-()]

where the

A of Problem

a^j{t) are real

17, then

polynomials

Prove: The matrix

f(A) is any scalar polynomial in A, then the

\\I

and

(xj-Ai)(::i-A2)...(*^-A)

Find the characteristic roots of

if

= 0.

- f(A)\ = c'^\xj - A\ \x2l - A\ ...\xsl -A\.


c(xi -Xj) (x2-\j) ... (x^ -\j) = \- f(\j).

-1

is an invariant vector of f(A)

in the real variable

Take

t.

A(t)

and differentiate the last member as

if it

nition

22.

Derive formulas
(a)

-r-\A(t)

were a polynomial
Sfnomial with
wit! constant coefficients to suggest the defi,

for:

+ B{t)\;

(b)

-T-\cA(t)\.

dt

where

c is

a constant or c =[ciA;

(c)

^{A(t)-

B(t)\;

dt

(d)

^A

dt

'it).

n
Hint.

use

For (c). write


A{t)-A-\t) = /.

A(t)- B(t) = C(t) = [ci;.(t)]


'

and differentiate

cv,-()

a,-i,(0 6i,,-().

k=i ^^

"^^

For

(d),

chapter 24
Smith Normal Form

BY AN ELEMENTARY TRANSFORMATION
(7)

The interchange

of the ith and /th tow, denoted by

column, denoted by K^j


(2)

on a A.-matrix A(X) over F[\] is meant:

The multiplication

ff|--

the interchange of the ith and /th

of the ith row by a non-zero constant k, denoted by H^(k);

the multiplication of the ith column by a non-zero constant k, denoted by K^ik).


(3)

The addition

row of the product of

to the ith

/(X),

any polynomial of F[\], and the /th row,

denoted by ^^(/(X));
the addition to the ith column of the product of /(X) and the ;th column, denoted by K^j (/(X)).

These

are the elementary transformations of Chapter 5 except that in (3) the word scalar has

been replaced by polynomial. An elementary transformation and the elementary matrix obtained
by performing the elementary transformation on / will again be denoted by the same symbol.
Also, a row transformation on ^(X) is effected by multiplying it on the left by the appropriate
H and a column transformation is effected by multiplying ^(X) on the right by the appropriate K.
Paralleling Chapter
I.

5,

we have

Every elementary matrix in F[\] has an inverse which in turn

is an elementary

ma-

trix in F[\].
II.

III.

Two

If

\A(\)\ = k

The rank

0,

with k in F, A(\) is a product of elementary matrices.

of a X-matrix is invariant under elementary transformations.

re-square X-matrices i(X) and B(X) with elements in F[X] are called equivalent pro-

vided there exist P(\) = H^

ff 2

'

^1 and Q(\) = K^- K^

B(X)

(24.1)

...

K^ such

that

P(\)-A(X)-Qa)

Thus,
IV.

Equivalent

THE CANONICAL SET.

In

mxn

X-matrices have the same rank.

Problems

and

2,

we prove

Let A(\) and B(X) be equivalent matrices of rank r; then the greatest common divisor of all s-square minors of A(\), s r, is also the greatest common divisor of all ssquare minors of B(\).
V.

In

Problem
VI.

3,

we prove

Every X-matrix A(X) of rank

can be reduced by elementary transformations

Smith normal form

188

to the

CHAP.

SMITH NORMAL FORM

24]

189

A (A)
/2(A)

N(\)

(24.2)

...

where each

monic and

/^(A) is

When a A-matrix A(\)

of rank

f^(\) divides f^^^ (A),


r

has been reduced

(i

0_

1, 2, ...

1).

to (24.2), the greatest

common

divisor of

s-square minors of A(\), s ^ r, is the greatest common divisor of all s-square minors of A^(A)
by Theorem V. Since in N(\) each f^(X) divides /^^j(A), the greatest common divisor of all ssquare minors of N(\) and thus of A(\) is
all

g,(A)

(24.3)

Suppose A(\) has been reduced

= l,2,

(s

/i(\)-f2(A)-...-4(A),

.r)

to

/V(A)

diag(/-i(A), /2(A)

/Vi(A)

diag(Ai(A). AsCA)

/^(A), 0,

and to

By

O)

h^iX),

(24.3),

gs(A)

Now

we

go(A) =

define

(24.4)

Ai(A)-A2(A)-...-As(A)

g2(A) =/i(A)-/2(A) =Ai(A)-A2(A)

gi(A) =/i(A) =Ai(A),

eral, if

/i(A)-/2(A)-...-/s(A)

1,

so that

U\)

h^W,

in gen-

then
h^(\).

4(^)

s:s(^)/s-i(^)

(s

1, 2

r)

and we have
VII.

The matrix N{X)

of (24.2) is uniquely determined by the given matrix /1(A).

Thus, the Smith normal matrices are a canonical set


A +

Example

1.

Consider

A'+2A^+A

.4(A)

A^ +
It

A +

3A +

2A +

A^ +

gi(A) =

and g3(A)
/i(A)

1,

= i\A(X)\

gi(A)

the greatest
= A^

+X^

1,

equivalence over F[A].


A.

+ 3

2A%3A'^+A
3A^ +

6A

common divisor

is readily found that the greatest

A(X) is

A^+A'^ + A

for

common

+ 3

of the one-row minors (elements) of

divisor of the two-row minors of A(X) is g2(A)

=X,

Then, by (24.4),
/2(A)

g2(A)/gi(A)

/3(A)

A,

g3(A)/g2(A)

A^+A

and the Smith normal form of A(X) is

10
A

A'(A)

A^+A
For another reduction, see Problem

INVARIANT FACTORS. The

polynomials

4.

f^(X),f-2{X)

/V(A)

in the diagonal of the Smith normal

form of A(X) are called invariant factors of ^(A). If 4(A) =


fk(X) = 1 and each is called a trivial invariant factor.

As

a consequence of Theorem VII,

we have

1,

k r,

then

/i(A) = /2(A) =

...

SMITH NORMAL FORM

190

VIII.

Two

[CHAP. 24

re-square X-matrices over F[X] are equivalent over F[X\ if and only if they

have the same invariant factors.

ELEMENTARY

DIVISORS. Let A(X) be an re-square X-matrix over F[X] and

let its invariant factors

be expressed as
(24.5)

{p,(X)\'^i^\p^(\)fi^...\PsMi^^^.(i = i.2

f^(\)

r)

where pi(A), p2(X), ., PgCX) are distinct monic, irreducible polynomials of F[X\.
q^j may be zero and the corresponding factor may be suppressed; however, since
r-l;/ = l,2, .s).
/i+i(A), qi+t,j^gij. (f = 1.2

The

factors

!p-(X)S'^*i

which appear

Some
fj

(A)

of the

divides

elementary divisors over F[X]

in (24.5) are called

of A(X).

Example

2.

Suppose a 10-square A-matrlx A(X) over the rational

has the Smith normal form

field

10

10

(A.-1)(A^+1)

'

(A-1)(A^ + 1)^A

(A-l)^(X" + l)^A^(A"-3)

The rank

is 5.

The invariant
=

/i(A)

The elementary divisors

(X-l)^

/2(A)

1,

(A-l)''(A.''+lf A^(A''-3)

/5(A)

(A-1)(A^+1),

/3(A)

1,

(\-l)(X^+lfX.

U(k)

factors are

are

A-1,

A-1.

(A^'+l)'',

(A^+l)^,

Note that the elementary divisors are not necessarily


mentary divisor appears as often as

Example

3.

(a)

Over the

it

A^

(A'^+l),

distinct;

A,

A""

- 3

in the listing

each ele-

appears in the invariant factors.

Example

real field the invariant factors of A(X) of

2 are

unchanged but the

ele-

mentary divisors are


(A-l)"^,

since
(6)

A^-3

A-1,

A-1,

(A^lf,

(A^'

l)''

(A^'

l),

X',

A,

X+x/l

X-\fz,

can be factored.

Over the complex

field the invariant factors remain

unchanged but the elementary divisors

are

A-1,

A-1,

(X-if,

X-i,

(A-1)^,

The

{X + if,

(X + if,

A^,

X-\/l,

A,

X + i.

(X-if.

X+\/l

Invariant factors of a A-matrix determine its rank and its elementary divisors;

conversely, the rank and elementary divisors determine the invariant factors.

Example

4.

Let the elementary divisors of the 6-square A-matrix A(X) of rank


A=,

A"",

A,

(A-l)^

(A-^)^

A-l,

be

(A+l)^

A+

Find the invariant factors and write the Smith canonical form.

To form

/5(A), form the

lowest common multiple of the elementary divisors,


/5(A)

a" (A-1)'' (A

if

i.e.,

CHAP.

SMITH NORMAL FORM

24]

To form

f^i\),

form the lowest

191

remove the elementary divisors used


of those remaining, i.e.,

in /sCX) from the original list

and

common multiple

UiX.)

Repeating, /sCX) =

Now

A(X-l).

The Smith canonical form

A^(X-lf(A+l)

the elementary divisors are exhausted; then /sCX) = /i(A) =

1.

is

\(XN(\)

1)

A^(A-lf (A + 1)
A=(A--lf(X+lf

Since the invariant factors of a A-matrix are invariant under elementary transformations, so
Thus,

also are the elementary divisors.

IX. Two re-square X-matrices over F[\] are equivalent over F[X]
have the same rank and the same elementary divisors.

if

and only

if they

SOLVED PROBLEMS
1.

If P(\) is a product of elementary matrices, then the greatest common divisor of all ssquare minors of P(\)- A(X) is also the greatest common divisor of all s-square minors of A(\).

Prove:

It

is

necessary only

to

consider P(A)-.4(A) where P(A) is each of the three types of elementary matrices H.

Let R(\) be an s-square minor of .4(A) and let S(A) be the s-square minor of P(X)- A(\) having the same
position as R(X). Consider P(A) = H^j
its effect on ^(A) is either (i) to leave R(X) unchanged, (u) to interchange two rows of R(X). or (iii) to interchange a row of R(X) with a row not in R(X). In the case of (i),
S(X) = R(X); in the case of (ii), S(A) = -R(X); in the case of (Iii), S(X) is except possibly for sign another
s-square minor of ^(A).
;

Consider P(X)

= H^ (k);

Finally, consider

then either S(A) = R(X)

P(A) = H^j (/(A)).

Its effect

or S(X) = kR(X).

on ^(A) is either

(i) to leave R{X) unchanged,


(U) to increase one of the rows of R(X) by /(A) times another row of R(X). or (iii) to increase one of the rows of R(X)
by /(A) times a row not of R(X). In the case of (i) and (li), S(X) = R(Xy, in the case of (iii),

S(X)

where 7(A) is an s-square minor of

R(X) f(X)-T(X)

.4(A).

Thus, any s-square minor of P(X)' A(X) is a linear combination of s-square minors of /4(A). If g(A) is
the greatest common divisor of all s-square minors of .4(A) and gi(A) is the greatest common divisor of all
s-square minors of P(A)--4(A), then g(A) divides gi(A). Let B(X) = P(XyA(X).

Now

.4(A) =

P"^(A)-

fi(A)

and P~'^(X) is a product of elementary matrices.

Thus, gi(A) divides g(A) and

gi(A) = g(A).

2.

Prove:

If P(X) and Q(X) are products of elementary matrices, then the greatest common divisor of all
s-square minors of P(A) -^(A) -QiX) is also the greatest common divisor of all s-square minors otA(X).

Let B(A) = P(A)M(A) and C(A) = B(X) Q(X). Since C'(A) = Q'(X)- b'(X) and Q'(X) is a product of elementary matrices, the greatest common divisor of all s-square minors of C'(A) is the greatest common divisor
of all s-square minors of s'(A). But the greatest common divisor of all s-square minors of C'(X) is the great

common divisor of all s-square minors of C(A) and the same is true
common divisor of all s-square minors of C(A) = P(X)- A(X)- Q(.X) is

est

square minors of ^(A).

for B'(A)

and B(A). Thus, the greatest

the greatest

common

divisor of all s-

SMITH NORMAL FORM

192

3.

Every A-matrix A(\) =

Prove:

of rank

[a^j(\)]

[CHAP. 24

can be reduced by elementary transformations to

the Smith normal form


fi(X)

/2(A)

N(X)

where each

fi(\) is

L(A)

monic and

f^(X) divides /^^j^(A),

(i

1, 2,

...,r-

1).

The theorem is true for A(\) = 0. Suppose A(X) ^ 0; then there is an element a::(K)
of minimum
J
By means of a transformation of type 2, this element may be made monic and, by the proper interchanges of rows and of columns, can be brought into the (l,l)-position in thematrix to become the new aii(A).
7^

degree.

(a)

Suppose a 11 (A) divides every other element of ^(A).


duced to

Then by transformations

of type

3,

4(A) can be re-

/i(A)
(i)

B(A)

where /i(A) = a 11(A).


(6)

Suppose that an(A) does not divide every element of A(X).


which is not divisible by

an (A).

By Theorem

ay (A)
where

ri^-(A) is

I,

Let

ai--(A)

of degree less than that of aii(A).

Prom the

/th

(l,l)-position as the

new oii(A).

If

row

column subtract the product of q{X) and

now

ri,-(A).

By a transformation

replace this element by one which is monic and, by an interchange of columns bring

2,

first

g(A)an(A) + ry(A)

the first column so that the element in the first row and /th column is

type

be an element in the

Chapter 23, we can write

now an(A) divides every element

of A{X),

we proceed

it

of

into the

to obtain

(i).

Otherwise, after a finite number of repetitions of the above procedure, we obtain a matrix in which every

element
If

in the first

row and the

column is divisible by the element occupying the (l,l)-position.

first

this element divides every element of A(X),

not divisible by

On (A).

Now

row

we proceed

to obtain

(i).

Otherwise, suppose

a,-,-(A)

is

"J

Let

aii(X) - qix(X)- a^^iX) and aij(A) = gijXA)- aii(A). From the ith row subtract the product of gii(A) and the first row. This replaces aii(A) by
and aij (A) by aij{X) - qii(X) a^jiX).

add the

jth

to the first.

aij (A)

This leaves oii(A) unchanged but replaces aij(A) by

- qii(X)- aij(X) + aj(\)

Since this is not divisible by aii(A), we divide

^ij(X) +
it

gij(A)U- ?ii(A)!aii(A)

by aii(A) and as before obtain a new replacement (the

remainder) for aii(A). This procedure is continued so long as the monic polynomial last selected as aii(A)
does not divide every element of the matrix. After a finite number of steps we must obtain an a 11(A) which

does divide every element and then reach


Next,

we

treat B(A) in the

(i).

same manner and reach


i(A)

/2(A)

C(A)
Ultimately,

we have the Smith normal

form.

Since /i(A) is a divisor of every element of B(A) and /2(A) is the greatest common divisor of the elements
of S(A), /i(A) divides /2(A).

Similarly,

it

is found that

each /^(A) divides fi^^iX).

CHAP.

4.

SMITH NORMAL FORM

24]

193

Reduce
X +
^(A)

A='

A^

A+

+ 2A=

+A

3A +

A +

X" + 2

A + 3

+ A

a""

A +

+A

+ 3A^

2A''

+ 6a + 3

a""

Smith normal form.

to its

necessary to follow the procedure of Problem 3 here. The element /i(A) of the Smith normal
common divisor of the elements of A(\); clearly this is 1. We proceed at once to obtain
such an element in the (l,l)-position and then obtain (1) of Problem 3. After subtracting the second column
is not

It

form is the greatest

from the

we obtain

first,

A+

A+

a"

A^

'4(A)

A^ +

A+

"

A% A

2A +

3A%A

2A +

3A^ +

6A

A +

A +

A%A

+ 3

2A^ + 2A.

A + A

Now

the greatest

common

S(A)J

Lo

2A + 2A_

Then

divisor of the elements of S(A) is A.

A+A
2A^ + 2A

2A^ + 2A

A" + A

and this is the required form.

5.

Reduce
A =

-4(A)

A""

+ A

A + 2

A^

A''

- 3A +

A=- 2A

2A

A= + A -

Smith normal form.

to its

We

find

A-

^(A)

A=

A-

A^-3A

+ 2

"

"l

-A -

^\^

A +

A-

A=+2A
A^+A-3_

2"

A +

A+1
1

-V-

-1

-A -1

A+1

A+

using the elementary transformations ^i2(-l);


l),

A +

"l

A+1
^23(1); Hs2(A +

A +
-A'

^2i(-A), ^3i(-A +

2);

-A

/f2i(-A +

A(A+i)
l),

.Ksi(-A -2);

HqsC-I);

H^i-iy. K32(-A-l), Xs(-l)-

SUPPLEMENTARY PROBLEMS
6.

Show

7.

Prove:

that

H.jK^j

H^{k)K^a/k)

An n-square A-matrix A(\)

constant.

H^j{f{\))

Kj^{-f(,X))

/.

is a product of elementary matrices if

and only

if |.4(X)| is

a non-zero

SMITH NORMAL FORM

194

8.

An n-square A-matrix ^(A) may be reduced

Prove:

to / by

[CHAP. 24

elementary transformations

if

and only if\A(X)\

is a non-zero constant.

9.

Prove:

A-matrix .4(A) over F[AJ has an inverse with elements in F[\]

if

and only

if /1(A) is

a product of

elementary matrices.

10.

Obtain matrices P(\) and Q(k) such that

P(\)- A(\)- Q(X) =

A(Xr^

and then obtain

Q(X)-P(X)

given

A+
=

-4(A)

A+

A+2

A+1

-A-1

A+2
Hint.

See Problem

Chapter

6,

A-1

Ans

5.

-A^ - 3 A - 2

-A

11.

Reduce each of the following

A%A
_2A''

>+

\^-2A

A
A= +

A^

A-1

(b)

+ 2A

>?

_ A^

A^- 2A+
2A=

A^

A+

2A''

- 2A +

3\^

-7A

l_

A=

A-

(c)

a''-

A^

2A

A"

A^A""

+ 4

2A''

2A''-2A''

A +

A^ + 2

A" + A

A +

A^

+A

A"

A%A^

A=

A^ +

A""

A + 4

A""

- 2A^

a''

\^ + l

A^ +

+ A -

A"

(d)

00

-2A=

a""

- A

A^-

A=
A^ +

A"-

1
1
'-\j

'a=^

A= +

3A

A%

A -

+ 3

A^ +

A= +

4A -

A-2

A-1

A+

3\ +

4A+3

2\ + 2

A-1

A^

A-^

A^

A^ +

2A

A^ +

6A

A%

+ 4

"a^

A-2

-2\

'-VJ

A+

6A -

i_

A ^+ 2A + 3J

Obtain the elementary divisors over the rational


matrices of Problem 11.

3A + 2

\^

a""

A= + 3

(e)

12.

'-Xj

- A -

a""

if)

'Xy

2A-2

A+1

A% A

- A=

A+

A^ + 2

A^

-A^A

2A

1
'-\>

2A^ - 3A + 2

-A^'-A +

normal form:

A-1
A^-1

X
(a)

to its Smith

2A-1

A^ +

A=( A-

ifOV+l I

field, the real field,

and the complex field

for

each of the

CHAP.

13.

SMITH NORMAL FORM

24]

The following polynomials

195

Find

are non-trivial invariant factors of a matrix.

its

elementary divisors in the

real field.
(a)

X^-X. X=-A^ A^-2A^

(b)

A+1, X^-

14.

(X^-lf. (X^-lf

1,

(c)

X,

X" + X,

A'

(d)

X.

A^ + A,

A''

+ 2A +

Ans.

+ X^

(A

(a)

X^,

(b)

A+1, A+1,

(c)

A,

A,

A^

(d)

A,

A,

A.

A,

A^,

+ 2X'' -

X''

2A^+

A''

X"^

A''+A''+2A'' + 2A +

A,

A^ +

A,

The following polynomials

(A^+lf,

l,

-if,

(A

(A

-if

A-1

(A^'+lf,

l,

+A

A-1

-if, A-1,

(A+lf, (A+lf, A-1,

A^'

A''

(A^

A+1

+ lf.

are the elementary divisors of a matrix

whose rank

What are

is six.

its invariant

factors?
(a)

A,

(6)

A^

.4ns.

15.

A+

A,

A=,

A+2, A +

1,

A,

(A-

1)2,

A+

3,

A-1

-if,

(c)

(A

(d)

A^,

A^

(A

-if,

(A

(A + 2f,

A,

-if, A-1, (A+lf


(A + 2f,

(A+2f

A(A + l)(A + 2)(A+3)(A + 4)

(a)

1,

1,

1,

1,

A,

(b)

1,

1,

1,

A,

A^cA-l),

(c)

1,

1,

A-1, (A-

(d)

1,

1,

1,

A^A-lf

if, (A- if, (A- if (A+lf

A(A + 2f,

A''(A

+ 2f,

A''(A

2f

Solve the system of ordinary linear differential equations

!Dxx

+ (D + i)x2

(0 + 2)^1

(D + l)x2

where

xj, x^,

unknown

xs are

the polynomials in

computing form similar to


.ffi(-l),

Hs(2). Ks(l/5)

D
D+2

and

4.

D+1

'o'

Xl

-D + 1
D+2

X2

e\

3_

K2i(D +

l),

/I

H2i(-D-2). ff3i(D+l), K^siD). H^si-i). K^ih. K^gi^D

+1).

Hs^i-kD),

obtain

k(D +

5D + 6

e*

combine as do the polynomials in A of a A-matrix; hence, beginning with a


that of Problem 6, Chapter 5, and using in order the elementary transformations:
of

-1

PAQ

at

D+1

:i2(-l),

(D+ 2)X2

system is

AX
Now

real functions of a real variable

In matrix notation, the

Hint.

- (D-l)xs

-4

.4

^(5D2+12D + 7)

l)

-jD

-1

-5D2-8D-2 -D 4D + 2

^-^(SD^ + TD)

i(5D2+7D + 2)

\D

A'l

d=+|d + | 5

the Smith normal form of A.

Use the

linear transformation

yi = 0,

y2=-4e*

QY

to carry

AX

into

AQY

and from

PAQY

N-J

PH

get

Finally, use
i

QY

(D2+|D

+ i)y3

6e*-l

and

ys

K^e"^^^ + K^e-^ + \e^ ~ ^


3
4

to obtain the required solution

3Cie-**/5 + it _ |,

^2

12Cie-'^*/5 + Cje"* - i,

Xs

-2Cie-**/s+ ie*+ i

chapter 25
The Minimum Polynomial of a Matrix
THE CHARACTERISTIC MATRIX \I-A

of an -square matrix A over F is a non-singular A-matrix


having invariant factors and elementary divisors. Using (24.4) it is easy to show
If

I.

Problem

In

II.

is a

we prove

1,

Two

diagonal matrix, the elementary divisors of \I- D are its diagonal elements.

A and B over F are similar over F if and only if their charsame invariant factors or the same rank and the same elemen-

re-square matrices

acteristic matrices have the


tary divisors in F[X.].

Prom Theorems
HI.

An

and

II,

we have

re-square matrix

A over F

is similar to a diagonal matrix if and only if

\I- A

has linear elementary divisors in F[\].

SIMILARITY INVARIANTS. The

invariant factors of \I -

are called similarity Invariants of A.

Let P(X) and Q(\) be non-singular matrices such that P(X) (\1-A)-Q(\)
mal form
diag(fi(X), 4(X),

Now

\P(\).(XI-A).Q(\)\
Since <^(X) and
IV.

/^(A) are

...,

monic,

P(A)

(?(A)

Smith nor-

j^a))

\P(X)\.\Qa)\4,a)

is the

A(A)-/2(A)-.../na).

and we have

The characteristic polynomial of an re-square matrix A


XI- A or of the similarity invariants of A.

is the

product of the invar-

iant factors of

THE MINIMUM POLYNOMIAL. By

the Cay ley-Hamilton Theorem (Chapter 23), every re-square matrix


A satisfies its characteristic equation <^(A) =
of degree re. That monic polynomial m(X) of
minimum degree such that m(A) =
is caUed the minimum polynomial of A and m(X) =
is
called the minimum equation of A. (m(X) is also called the minimum function oi A.)

The most elementary procedure for finding the minimum polynomial of i


following routine:
(I)

If

(II)

If

(III)

If

4 =
A ^

oq/,

then m{X) =

al for all a but

A^ ^ aA + hi

X-ao:

i^ = a^A +

for all a

aj

and b but
're(A)

A''

A^

then

m(A) =

a^A"^

a^)?

122
1.

Find the minimum polynomial of ^

196

- a^X - uq

+ a^A + a^I

- a^X - Oq

and so on.

Example

A^

then

5.^

involves the

CHAP.

THE MINIMUM POLYNOMIAL OF A MATRIX

25]

Clearly A

Using the

first

a^= i and oq
A^ = iA +

Problem

In

V.
if

If

is

and only

Problem

In

VI.
fn(X) of

is

impossible.

'9

two elements of the

'100"

'122"
=

oo

1
1

row of each matrix, we have

first

+ ao_

9 =

Oj^

8 =

2ai

then

After (and not before) checking for every element of A^, we conclude that
and the required minimum polynomial is X^ - 4X - 5.

= 5.

51

if

any re-square matrix over F and /(A) is any polynomial over F, then f(A) =
the minimum polynomial m{\) of A divides /(A).

we prove

3,

The minimum polynomial m(\) of an


A which has the highest degree.

Since the similarity invariants

f^(X), f^(k)

Vn. The characteristic polynomial


A and certain monic factors of m(X).

of

Set

we prove

2,

oqI

197

(fi(\)

ra-square matrix

is that similarity invariant

fn-iM all divide


A is the product

we have

^(A),

of

of the

minimum polynomial

and
Vin. The characteristic matrix of an ra-square matrix A has distinct linear elementary
if m(A), the minimum polynomial of A, has only distinct linear factors.

divisors if and only

NON-DEROGATORY MATRICES. An

re-square matrix

A whose

polynomial are identical is called non-derogatoiy;


IX.

An

re-square matrix

is

characteristic polynomial and

otherwise, derogatory.

non-derogatory

if

and only

if

minimum

We have

A has

just one non-trivial

similarity invariant.
It

is also

X.

If

easy

to

show

Si and Sg have minimum polynomials

polynomial
and m2(A).

ro(A) of the direct

is the least

This result may be extended to the direct sum of


XI.
let

Let gi(X),g2(X),

Aj be a non-derogatory matrix such that

B = diag(^i, ^2
acteristic and

COMP ANION MATRIX.


(25.1)

We

^n)

has

matrices.

^(A)

\XI - Aj\ = \g-(X)\ "i


fii

fgi(A)!

ig2(A)P-ig^(A)!

>

= 1,2,
'^

Let A be non-derogatory with non-trivial similarity invariant

^(A)

C(g)

re

(;

,02

a" +

a-iA"'^

define as the companion matrix of g(X),

and for

multiple of ^^(A)

minimum polynomial.

g(X)

(25-2)

common

be distinct, monic, irreducible polynomials

..gnCA)

and m^iX) respectively, the minimum

/rei(A)

sum D = diag (61,55)

= [-a],

if

g(X)

X+a

a^X +

Oq

in

F[A] and

...,m).

Then

as both char-

THE MINIMUM POLYNOMIAL OF A MATRIX

198

[CHAP. 25

1
1

(25.3)

C(g)
1
1

Oq

Problem

In

4,

-%

-e

-an-2

-'^n-3

-Gj

we prove

The companion matrix C(g) of a polynomial g(\) has g(\) as both


and minimum polynomial.

XII.

istic

(Some authors prefer to define C(g) as the transpose


Both forms will be used here.)

its character-

of the matrix given in (25.3).

See Problem

XIII.

If /4 is

non-derogatory with non-trivial similarity invariant f^(X) = {k-af,


"a

/ =

(25.4)

[o],

if

ra

and

1,

...

if

ra

>

...

f^{\)

then

O'

...

...

has

5.

easy to show

is

It

as its characteristic and minimum polynomial.

SOLVED PROBLEMS
1.

Prove: Two ra-square matrices A and S over F are similar over F if and only if their characteristic
matrices have the same invariant factors or the same elementary divisors in F[X\-

Suppose A and B are similar. From (i) of Problem 1, Chapter 20. it follows that Xl - A and XI -B are
Then by Theorems VHI and IX of Chapter 24, they have the same invariant factors and the same

equivalent.

elementary divisors.
let Xl - A and Xl - B have the same invariant factors or elementary divisors.
Chapter 24 there exist non-singular A-matrices P(X) and Q{X) such that

Conversely,

Theorem

VIII,

Xl - B

P{X)-{XI-A)-Q{X)
or

P{X)-{XI-A)

(i)

(Xl-B)-Q

(X)

Let
(ii)

P{X)

(X/-S).Si(A) + Ri

(iii)

Q(X)

S2(X)-(A/-S) +

/?2

(iv)

Q'\X)

S3(A)-(A/-.4) +

/?3

where R^, Rq, and R3 are free of

A.

(Xl-B)-S^(X)-(Xl-A)

Substituting in

+ Rj^(Xl-A)

i ),

we have
(A/-B)-S3(A)-(A/-4)

(XI-B)Rs

or
(V)

{Xl-B){S^{X)-Sa{X)\{Xl-A)

(A/-B)Rg

R^{XI-A)

Then by

CHAP.

THE MINIMUM POLYNOMIAL OF A MATRIX

25]

Then

S^(X)

and

Sg(A) =

(Xl-B)Rg

(vi)

since otherwise the

199

left

member

R^(\I-A)

of (v) Is of degree at least two while the right

member

is of degree at

most

one.

Using

(ill), (Iv),

and

(vi)

Q(^)-Q~\\)

Q(\)\Ss(\)-(Xl-A) + Rs\

Q(X.)-S3(X).(X!-A) +

Q(X)

Q(X)-Ss(X)-(Xl-A) + S2(X)-R^-(XI-A) + R^Rg

Ss(X)

(XI

\S2(X)-(XI-B) + R^lRg

- A) + S2(X)-(Xl-B)Rs + R^Rs

or
(vii)

- R^Rs

Now

Q{X)-Sq(X) + S^(X)R-^ =

X while the

right

member

and

is of

/ =

Since A.B.R^, and R^ are free of A.


to be proved.

Prove:
only

If

is

R2R3

Thus, Rg = R^

/?i(A/-^)/?2

F and

any ra-square matrix over

if

the minimum polynomial m(X) of

By

the division algorithm. Chapter 22,

(vii) is of

degree zero

XI - R'^AR^ and A and B are similar, as was

f(X) is any polynomial in F[X]. then f(A) =

A divides

in

and, from (vi)

ARi2 - R^AR^

R^: then XI - B

/Ji =

RxUXl - A)

since otherwise the left member of

degree at least one.

XI - B

2.

\Q{X) Sg(X) ^ S^iX)

if

and

/(A).

f(X)

q(X)-n(X) +

f(A)

q(A)-n(A) + r(A)

r(A)

and then
=

r(A)

Suppose f(A) = 0; then r(A) = 0. Now if r(A) ji 0, its degree is less than that of m(X), contrary
to the
hypothesis that m(A) is the minimum polynomial of ^. Thus, r(A) =
and m(A) divides /(A).
Conversely, suppose /(A) = ?(A)-m(A).

3.

Then f(A)

= q(A)-m(,A) = 0.

Prove: The minimum polynomial m(X) of an ra-square matrix A


which has the highest degree.
Let

gri-i(X)

denote the greatest common divisor of the

\XI-A\

cf,(X)

(n

is that similarity invariant fn(X) of

- l)-square minors

g-i(A)-/(A)

and

adj(A/-/l)

g_i(A)-S(A)

where the greatest common divisor of the elements of B(X)

Now

(A/-^).adj(A/-^)

(f)(X)-I

is 1.

sothat

(X/-A)-gn_^(X).B(X)

gn-i(A)-/(A)-/

(A/-^).S(A)

fn(X)-I

or

(i)

Then XI

-A

is a divisor of f^(X)-I

and by Theorem V, Chapter 23, f^(A) =

0.

of XI

-A. Then

THE MINIMUM POLYNOMIAL OF A MATRIX

200

By Theorem

V, m(\) divides

Suppose

fn(X-)-

(ii)

fn(\)

Since m(A) =

0,

XI

-A

Then, using (i) and

[CHAP. 25

is a divisor of

m(X)

g(X.)-m(\)

say

/,

m(\)-l

(\I-A).C(\)

fn(X.)-l

9(A)-m(A)-/

5(A)

?(A)

(ii).

(\I-A)-B(\)

9(A)

(A/ - ^). C(A)

and

Now

9(A) divides every element of iS(A); hence ?(A)

=1

C(A)
and, by

(ii),

m(A)

/n(A)

as was to be proved.

4.

Prove: The companion matrix C(g) of a polynomial g(\) has g(A) as both
minimum polynomial.

The characteristic matrix

its

characteristic and

of (25.3) is

-1

-1

^1

"2

an-2

To

the first column add A times the second column,


column to obtain

A + a^_j

A times the

third column,

,n-l

times the last

-1

A
G(A)

Since

|G(A)| = g(A),

a^

Oi

g(A)

-1

a_2

A+a^-l

the characteristic polynomial of C(g) is g(A). Since the minor of the element
g(A)
common divisor of all (n-l)-square minors of G(\) is 1. Thus, C(g) is non -derog-

in G(X) is 1, the greatest

atory and its

5.

minimum polynomial

The companion matrix

of

g(\)

is g(A).

A^ + 2A

A=

+ 6A -

is

10

10
10
1

5-61-20

or, if

preferred.

10
0-6
10
1

10-2
1

CHAP.

THE MINIMUM POLYNOMIAL OF A MATRIX

25]

201

SUPPLEMENTARY PROBLEMS
6.

Write the companion matrix of each of the following polynomials:

- 2\ -

+ \^

- X^

+ 2X

X''

(d)

X""

(b)

(X^-i)(X+2)

(e)

]?()? + I)

(c)

(X-lf

(/)

(A+2)(A^-2A^ + 4A-8)

'o

Ans.

(a)

-1

--3

(d)

(/)

0-10

7.

Prove: Every 2-square matrix

8.

Reduce G(X) of Problem

A=

[a^]

4 to diag (1

16

which (011-022)^ +

for

o'

(e)

-2

-2

o"

(c)

2X!'

o"

(b)

1
1

9.

(a)

g(X)

4012021/0

is non-derogatory.

).

For each of the following matrices ^. (i) find the characteristic and minimum polynomial and

(ii) list

non-trivial invariant factors and the elementary divisors in the rational field.
1

(a)

(b)

'211
if)

Ans.

2
(c)

1"

2-3
(g)

m(X)

(A-l)(A-2)(A-3);

(b) <^( A)

m(,\.)

X^;

if)

if. = e.d

(^(A)

(A-if(A-2);

i.f.

m(X)

(A-1) (A-2)

e.d.

i.f.

(A+ihA-5)

i.f.

A+1, (A+i)(A-5)

(A+l)(A-5)

e.d.

A+i, A+l, A-5

,^(A)

A(A-H)^(A-1)

m(A)

A(A^-l)

(/)(A)

A2(A+1)2

m(A) =

A(A+1)2

(A-2) (A""-

m(X)

A^-A-2

Prove Theorems VII and

A,

i.f.

A+l,

ih)

_1

4-6 3 s"
3-2
2
4-3 4-1 -6
4-2 4 0-4
-1
0-2 1 2
1

e.d. (A-1), (A-2).

(A-3)

2)'';

A^-A

A+l, A+l, A-1

A(A+l)2

e.d. A. A,

A-

A-

e.d. A,

oS(A)
(h)

i.f.

A''-4A

e.d. A, A,

c^(A)

is)

A,

(e)

A-1, A-1, A-2

i.f.

A-1, (A-i)(A-2)

- 4A=

-5
-2

(A-l)(A-2)(A-3);

m(A)

A=

= A=

(/)(A)

m(A) = A^ - 4A

1-3

-1 -6 -3 -6
-3 -3 -4 -3

ie)

id)

1
1

<^(A)

(a)

id)

11.

-6 -2 -3 -2
-3 -1 -1 -2

(c)

10.

-2 -1 -3

(A+D^

i.f.

e.d.

A-2, A'^-A-2, A^-A-2


A-2. A-2, A-2, A+l, A+l

VIII.

Prove Theorem X.
Hint.

m(D)

= diag(m(Bi), m(B^)

requires

m(Si) = miB^) =

thus, mi(A) and

m^X)

divide m(X).

the

THE MINIMUM POLYNOMIAL OF A MATRIX

202

12.

Prove Theorem XI.

A
Show

n-square and if k is the least positive integer such that A = 0, .4 is culled nilpotent of index
that A is nilpotent of index k if and only if its characteristic roots are all zero.

13. If

14.

is

Prove: (a) The characteristic roots of an n-square idempotent matrix A are either

The rank

(b)

15.

Prove: Let

of

A.B.C.D

same invariant factors


Follow the proof

Hint.

Prove:

number of characteristic roots which are

is the

be ra-square matrices over

P and Q such

matrices

16.

PCQ

that

or the

Problem

in

18.

Use the minimum polynomial

F
it

with C and

and only

if

There exist non-singular

non-singular.

R(X) = \C - A

and

S(A.)

XD-B

have the

1,

noting that similarity is replaced by equivalence.

s,

then A~^ is expressible

slmA.

to find the inverse of the matrix

of

Problem

9(h).

Prove: Every linear factor X-A.J of 0(A) is a factor of m(A).

20.

PDQ

1.

1.

the minimum polynomial m(X) of a non-singular matrix A is of degree

If

The theorem follows from Theorem VII or assume the contrary and
Then {A -\^I)q(A) + rl =
and A -X^l has an inverse.

Hint.

19.

= A.

or

A:.

same elementary divisors.

as a scalar polynomial of degree

17.

[CHAP. 25

^ 0.

Use

-4

Prove:

If

^
to

[o

show

is not the product of the distinct factors of 0(A).

g(\) is any scalar polynomial in A, then g(^) is singular if and only if the greatest

Suppose d{\)
Suppose d(A)

(i)

(ii)

minimum polynomial
i-

common

divi-

oi A, is d(k) ^ 1.

and use Theorem V, Chapter 22.


and use Theorem IV, Chapter 22.

Problem 20 that when

21. Infer from

r,

;]

sor of g(A) and m(A), the


Hint,

minimum polynomial

that the

m(A) = {\-X^)q{X) +

w:rite

g(.4) is

non-singular, then [g(/l)]

is

expressible as a polynomial in A of

degree less than that of m(\).

22.

Prove:

If

the

minimum polynomial m(A) of A over F is irreducible in F\X\ and is of degree


A with coefficients in F of degree < s constitutes a field.

s in A,

then the

set of all scalar polynomials in

23.

Let A and S be square matrices and denote by m(A) and (A) respectively the minimum polynomials oi
and BA. Prove:
(a) m(A) = n(A)
(&) m(A)

Hint.

24.

when not both

and n(A) differ

B.m(^S)./l

Let A be of dimension

at

.A

and B are singular.

most by a factor A when both A and B are singular.

(B,4).m(B.4)

mxn

Let

X.I

and

A-n{BA)-B

m >

n,

and S be of dimension nxm,

the characteristic polynomials of

25.

AB

and BA.

Show 0(A)

(AB)-n(AB)

Xj^ is

26. If the matrices

0.

and denote by 0(A) and 0(A) respectively


(/f(A).

be an invariant vector associated with a simple characteristic root of A.

mute, then

AB

Prove:

If

A and B com-

an invariant vector of B.

A and B commute, state a theorem concerning the invariant vectors

simple characteristic roots.

of

B when A has only

chapter 26
Canonical Forms Under Similarity

THE PROBLEM.

it was shown that the characteristic matrices of two similar n-square


R'^AR over F have the same invariant factors and the same elementary divisors.
we establish representatives of the set of all matrices R'^AR which are (i) sim-

Chapter 25

In

matrices A and
In this chapter,

ple in structure and

These matrices,
ical matrix

(ii)

put into view either the invariant factors or the elementary divisors.

four in number, are called canonical forms of A.

'[^

introduced earlier for


'

o]

all

mxn

They correspond

matrices of rank

to the

canon-

under equivalence.

THE RATIONAL CANONICAL FORM.

Let A be an ra-square matrix over F and suppose first that its


characteristic matrix has just one non-trivial invariant factor /(\). The companion matrix
C(f )
of /(A) was shown in Chapter 25 to be similar to A. We define it to be the rational
canonical
form S of all matrices similar to A

Suppose next that the Smith normal form of \I - A


diag

<26-l)

(1, 1

with the non-trivial invariant factor

1,

f^(\.)

f.(X), f.^^iX),

...,

of degree s^,

(i

tional canonical form of all matrices similar to

(26-2)

To show
D^ = diag

is
f^(X))

= j.j +

n).

We define as the

di^s {C(fj), C(fJ ^^)

C(/))

A and S have the same similarity invariants we note that C(f-)

that

l,/i(A))

(1, 1

and, thus, S is similar to diag(D^-

l.fja), fj^^(\)

is similar to

By a sequence

D).

.^^

interchanges of two rows and the same two columns, we have S similar
diag (1,1

ra-

of

to

f^iX))

We have proved
I.

Every square matrix A is similar to the direct sum (26.2) of the companion matrices

of the non-trivial invariant factors of XI - A.

Example

1.

Let the non-trivial similarity invariants of A over the rational field be


/8(A)

X+1.

fg(X)

A%L

/io(A)

A^ +

2A +

Then
1

1
1

C(fa)

= [-1],

C(f9)

C(fio)

10

-10

-10
203

-2

CANONICAL FORMS UNDER SIMILARITY

204

[CHAP. 26

and
-1

1
1

-1
1

diag(C(/g),C(/9),C(/io))
1
1

1
1

-1
Is the required form of

Note.
immaterial.

The

Theorem

-2

I.

order in which the companion matrices are arranged along the diagonal is

Also

-1
-1
1
1

-1
1
1

-2

1
1

using the transpose of each of the companion matrices above is an alternate form.

A SECOND CANONICAL FORM.

Let the characteristic matrix of A have as non-trivial invariant


tors the polynomials f^(X) of (26.1). Suppose that the elementary divisors are powers of t
tinct irreducible polynomials in F[X]: pi(A,), paCX), ..-, PjCA). Let
(26.3)

fi(\)

ipi(X)l^"!p2(A)!^2^..{p,(X)!'^is(pi(A)i'"''iP2(A)i'"''...iPi(

(i

= i,i+'^

dis-

n)

The companion matrix

where not every factor need appear since some of the ^'s may be zero.
C{p?^^) of any factor present has

fac-

\])^(X)\ ^^ as the only non-trivial similarity invariant;

hence,

C{{^) is similar to

diag (C(p^?"

).

Cipl-'i

Cip^^ti))

We have
Every square matrix A over F is similar to the direct sum of the companion matriII.
ces of the elementary divisors over F of XI A.
Example

2.

For the matrix A of Example


(A.+ l)^,

A^-A,+

l,

(X

-A+1)

1,
.

the elementary divisors over the rational field are

Their respective companion matrices are


1
1

[-1],

[-1],

[-:

.;]

[.:

\
1

and the canonical form of Theorem

II is

-3

A+1, X+1,

CHAP.

CANONICAL FORMS UNDER SIMILARITY

26]

205

-1

-1
1

-1 -2
1

-1

1
1
1
1

-1

THE JACOBSON CANONICAL FORM.

-3

Let A be the matrix of the section above with the elementary

divisors of its characteristic matrix expressed as powers of irreducible polynomials in F\_\].

Consider an elementary divisor {p(X)!^.

If

q =

use C(p), the companion matrix;

1,

if

q>l,

build

C(p)

C(p)
(26.4)

where

Cq(P)

C(p)

C(p)

same order as C(p) having the element 1 in the lower left hand corner
The matrix C^^Cp) of (26.4), with the understanding that Ci(p) = C(p) is

is a matrix of the

and zeroes elsewhere.

called the hypercompanion matrix of \p(X)]'^.


I's just

..

Note that

in (26.4), there is a

continuous line of

above the diagonal.

When the alternate companion matrix

C'(p) is used, the hypercompanion matrix of jp(A)!''is

C'ip)

C'ip)

C'ip)

Ca(P)
C'ip)

C'ip)

where A' is a matrix of the same order as C'ip) having the element 1 in the upper right hand corner and zeroes elsewhere. In this form there is a continuous line of I's just below the diagonal.
Examples. Let

IpiX.)]'^ = (\^

2\-

Then

1)'^.

C(p) =

M,

M=y

1
1

-2

1
1

-2

CqiP)
1
1

-2

1
1
1

-2

and

CANONICAL FORMS UNDER SIMILARITY

206

In

Problem

1, it

is

Thus, Cq(p) is similar

shown
to

[CHAP. 26

that Cq(p) has {pCA)}*^ as its only non-trivial similarity invariant.

C(p^) and

may be substituted

for it in the

canonical form of Theorem

II.

We have
Every square matrix A over F

III.

is similar to the direct

matrices of the elementary divisors over

Example

4.

For the matrix A of Example

A+1 and A -A+1

2,

of

sum of the hypercompanion

\I - A.

the hypercompanion matrices of the elementary divisors A +

are their companion matrices, the hypercompanion matrix of

1,

(A+l)^is

r-1

10

-11
and that of (A^-A + 1)^ is

Thus, the canonical form of Theorem

0-11
nils
-1

-1
-1

-1
1

-1

1
1

-1

1
1

-1

The use

of the term "rational" in connection with the canonical form of

what misleading.

It

was used

Theorem

is

some-

originally to indicate that in obtaining the canonical form only

A are necessary. But this is, of course, true


Theorems n and III. To further add to the consometimes called the rational canonical form.

rational operations in the field of the elements of

also of the canonical forms (introduced later) of


fusion, the canonical form of

Theorem

III

is

THE CLASSICAL CANONICAL FORM.

Let the elementary divisors of the characteristic matrix of A


be powers of linear polynomials. The canonical form of Theorem III is then the direct sum of
hypercompanion matrices of the form
i

(26.5)

..

1
1

C^(P)
..
.

corresponding to the elementary divisor

!p(A)!

1
<^.,

(A-a^)^

This special case of the canonical form of Theorem


canonical form.

III is

For an example, see Problem

known as the Jordan


We have
]

2.

or classical

[Note that C^(p) of (26.5) is of the type / of (25.4).

Let

be the field in which the characteristic polynomial of a matrix A factors


Then A is similar over ^to the direct sum of hypercompanion matrices of the form (26.5), each matrix corresponding to an elementary divisor (X-a^)'^.
IV.

into linear polynomials.

Example

5.

Let the elementary divisors over the complex field of \I

-A

be:

A-i, X + i, (X-i)

(X+i)

CHAP.

CANONICAL FORMS UNDER SIMILARITY

26]

The classical canonical form

of

207

is

-1
1

-i

-i

From Theorem IV follows


V.

An

re-square matrix

is similar to a diagonal matrix if

and only

divisors oi XI -

are linear polynomials, that is, if and only if the


is the product of distinct linear polynomials.

if

the elementary

minimum polynomial of
See Problems 2-

4.

A REDUCTION TO RATIONAL CANONICAL FORM.

In concluding this discussion of canonical forms,


that a reduction of any re-square matrix to its rational canonical
form can be
made, at least theoretically, without having prior knowledge of the invariant
factors of \I - A.
A somewhat different treatment of this can be found in Dickson, L. E., Modern Algebraic
it

will be

shown

Benj.H. Sanborn, 1926. Some improvement on purely computational


aspects
Browne, E. T., American Mathematical Monthly, vol. 48 (1940).
ries,

We
in
to

Theo-

is

made

in

shall need the following definitions:

If A is an re-square matrix and X is an re-vector


over F and if g(X) is the monic polynomial
F[\] of minimum degree such that g(A)-X = 0. then with respect
to A the vector X is said
belong to g(X).
If,

vectors

Example

with respect to A

X,AX.A''X

6.

Let A

while

[1,0, -ij',

/4^

-6

-3

-2

= X.

AY

the vector

belongs

to g(X) of

are called a chain having

The vectors
Then

(A

Y;

thus,

and

A"

degree

p,

the linearly independent

as its leader.

AX

= [l, 0, oj' and

-I)X=0

=[-1,0,1]'

= [2,

1, l]'

are linearly independent

belongs to the polynomial A,^-l


For y =
and F belongs to the polynomial A+ 1.

(A+I)Y=0

m(\) is the minimum polynomial of an re-square matrix A,


then m(A)- X =
for every reThus, there can be no chain of length greater than the degree of
m(A). For the matrix
of Example 6, the minimum polynomial is A^ - 1.
If

vector X.

Let S be the rational canonical form of the re-square matrix A


over F.
R over F such that

Then, there exists a

non-singular matrix
(26-6)

R'^AR

diag(C.,C.^^

C)

where, for convenience, Q/^) in (26.2) has been replaced


by C^.
companion matrix of the invariant factor
/iCA)

A^i

c^ ^_X'i-

c,.

We
A

has the form


-^tl
1

-^i2
1

-^i3

Ci
.

shall

assume

that

C,-

the

CANONICAL FORMS UNDER SIMILARITY

208

Prom

(26.6),

we have

AR

(26.7)

RS

Rdiag(Cj,Cj^^

R-,R-^

Let R be separated into column blocks


have the same number of columns.

AR

A[R..R.^^

[CHAP. 26

From

RJ

R^

C)
so that R^

and C^, (i=/,/ +

n)

(26.7),

IRj.Rj,,

iRjCj.Rj^^C.^^

C)

/?Jdiag(C,,..C^.^^

R,,CJ

and

AR^ = R^Ci.
Denote the

si

(i=j,j +

column vectors of Ri by Ri^, R^s.

n)

Ris- and form the product


t

RiCi

[Rii, Ri2,

, Risi\Ci

st

[Ri2,Ri3, ,Risj,> -

Rik^ik]

^__

Since

ARi

A[Ri^,Ri^, -.Risi]

[ARi^.ARi2,. ..ARisil

RiCi

we have

ARn,

Ri2 =

(26.8)

Ris - ARi2 = A Rii,

Risi = A^'-'Ri.

and

-|>i.%

(26.9)

^Ris,

we obtain

Substituting into (26.9) from (26.8),

-i=

A'iRu

c^kA^-'Ri^
l

or

(A^^ + cis.A^'i'^

(26.10)

Prom the

definition of C^ above, (26.10)

(26.11)

...

may be

a^A

+ cirDRir

written as

fi(A)-Ri^

Let Rit be denoted by X, so that (26.11) becomes fj(A)-X: =0; then, since X-,AXj,
A ^ X^ are linearly independent, the vector X^ belongs to the invariant factor f^iX).
Thus, the column vectors of fi^ consist of the vectors of the chain having X^, belonging to /^(X),

Xj^

as leader.

To summarize:

the n linearly independent columns of R, satisfying (26.2), consist of re-/+l

chains

Xi,AXi
whose leaders belong

A^^"'Xi

(i=j,i +

to the respective invariant factors

<

satisfy the condition

s,-

s- ^^
+1

...

/ (A),

n)

fj.AX.)

/^(X)

and whose lengths

^ s.
n

We have
VI.

Por a given re-SQuare matrix A over F:


(i)

let

X^ be

(it)

let

X^_^ be the leader

the leader of a chain

(iii)

let

X^^^he

is linearly

F which

and so on.

Then,

for

^_^

maximum length

for all

ra-

vectors over F\

of

are linearly independent of the vectors of ;

the leader of a chain -n-2 ^ maximium length (any member of which


independent of the preceding members and those of and fi'^.i)

for all n- vectors over

and_i;

of

maximum length (any member of which


preceding members and those of for all n-

of a chain

is linearly independent of the

vectors over

F which

are linearly independent of the vectors of

CHAP.

CANONICAL FORMS UNDER SIMILARITY

26]

AR

Example

7.

209

is the rational canonical form of A.

Let

Take ;f=[l,0,o]'; then ^,

4X

= [l,

^;f = [14,25,30]'= 54^X- a:.


m(X) = A - 5A^ + 1 = <jS(A). Taking

Independent while

arly

belongs to

/gCX) =

= Q

and

A'

15
6

we

[3, 5. 6].' are line-

4A',4=A']

[a;.

^^A: =

{A-bA^+I)X

Thus,

1, l]',

find

-3

6-5
-1

AR

[AX.A'^X^a'^X]

14

25

30

0-1
and

R ^AR

Here A is non-derogatory with minimum polynomial m(A) irreducible over the rational
Every 3-vector over this field belongs to m(X). (see Problem 11), and leads a chain

field.

of length

three.

The matrix R having

Example

8.

Let

the vectors of any chain as column vectors is such that

Tak

A"

then ^ A' =

= [l,-l,o]';

cannot be the minimum polynomial m(A) of A.


11). and could be a similarity invariant of A.
Next, take

F=[l,0,0]',

independent while

A - 5A - 3A +

A^Y

The vectors

= [54,43,46]'

7 = <^(A).

It is,

A-

and

AT

belongs to

A-

1.

S.

NowA-1

however, a divisor of m(A), (see Problem

Y, -4? = [2,

= 5A'^Y +

The polynomial

AT

R'^AR =

1, 2]',

SAY -77.

^^^ =[ 11,

8, s]'

are linearly

Thus, Y belongs

is not a similarity invariant;

m(A) =

to

in fact,

the first choice of vector belongs to a polynomial which could reasonably


be the
function, it should be considered a false start. The reader may verify
that

unless

minimum

R'^AR
11

when

[Y,AY,A'^Y]

See Problems

5-6.

SOLVED PROBLEMS
Prove:

The matrix C^ip)

Let C^(p) be of order


so that the greatest
tors of

of (26.4) has !p(A)!'? as its only non-trivial similarity invariant.

s.

common

The minor of the element in the last row and first column of A/ - C
(p) is 1
divisor of all (s-l)-square minors of A/ - C (p) is 1.
Then the invanant fac-

A/ - C^(p) are 1,1


0(A)

l.f^(\).
=

But /^(A) =

|A/-C^(p)|

ip(A)!'?

since

\XI-C(pf

jp(A)!9

CANONICAL FORMS UNDER SIMILARITY

210

2.

The canonical form

(a) is that of

X^+4X +6A^ +4X +

divisor being

Theorems I and II, the non-trivial invariant factor and elementary


The canonical form of Theorem III is (b).

1.

-1

(b)

(a)

-1

-4

-1

The canonical form

[CHAP. 26

(a) is that of

-6

-1

-4

Theorem

and the elementary divisors being X +


Theorems II and III is (b).

2,

I,

2,\ -4, A'+3A -4A-12


The canonical form of both

the invariant factors being \ +

X+

2,

X+

2,

X-

2,

X-

X+

2,

3.

-2

-2

4
(b)

(a)

12

4.

The canonical form


X+

2,

X+

2,

(X'^

-3

-3

Theorem III. Over the rational field the elementary divisors


+2X- if and the invariant factors are

(a) is that of

+2X-lf

(X^

(X + 2)(X^ + 2X -

The canonical form

of

Theorem

is (b)

2)(X^+2X-1)^

(X +

1)^

and that of Theorem

II

is (c).

-2

-2
1

-2

1
--2

1
(a)

-2

1
1

-2

1
1

-2

1
1
1

-10

--6
1

(b)
1

ID

1
1
1

11

12

17

-14

--21

-8

are

CHAP.

CANONICAL FORMS UNDER SIMILARITY

26]

211

-200000000000
-2
1
1

-1

-2

-4

(c)
1
1
1
1

-6

-2

-1

-6

-2

-1

-1

-1

0_

-1

A =

Let

5.

-2
1

-2

AX

Then

[-2

-9

-6

Take X

-1

pendent while
m(\) = A* - 2X^ +

1, 1, 1. 1, 1]',

A^X

= [1,0,-1,0,0

= [l, 0, -2, 0, 0, -2]' = 2^=A:

-4

A"

and write Xg for X.

-Z;

-ir.

A''

belongs to

=[-3.1.1,1.

A*-2A^ + 1

1,2]'

We

are linearly inde-

tentatively

assume

Y = [o, 0, 0, 1, 0, O]' is linearly independent of the members


of the chain led by
-1, 1,0]' is linearly independent of Y and the members
2,,
= Y
of the chain. Now A^Y

Xg and

The vector

AY

- [-1,0,

Y belongs

Xs

1,

to

A -1.

Since the two polynomials complete the set of non-trivial


invariant factors

When

for Y.

-1

[Xs.AXs.Xg.AXe.A^Xe.A^'Xfi
1

-2

-3

"o

-1

-1

-1

write

-1

R~'^AR 1

so that

we

2
1

the rational canonical form 'of ^.

Note.

and

AZ

The vector Z =

- [3, 0, -2,

1,

-2,

[o, 1, 0, 0, 0. Oj' is linearly

o]' is

linearly independent of

independent of the members of the chain led by Xg


Z and the members of the chain However A'^Z =

1-1,1,0,0,0,1]'= -AXg + A Xg + Z; then (A'' ~ 1)(Z - AXg) =


and W=Z-AXe = [2.0 -1 -1 ll -i]'
belongs to A - 1. Using this as X^. we may form another R with which
to obtain the rational canonical' form.

-2

6.

A =

Let

Then
r

-1

-1

-1

-2

-1

-1

-4
-4

-1

-2

-2

-2

-2

-2

^.^5

2,

Take X = [1,0,0,0,0]'.

[-2,1,-1,-1,-2]',

X-3X

-2, -2,0J - 2A
polynomial m(A) and label

L-1,

-1

and

as X^.

^^^=[1,1,-1,-1,0]' are linearly independent while a'^X^


3.
We tentatively assume this to be the minimum

belongs to A - 2A^ +

CANONICAL FORMS UNDER SIMILARITY

212

[CHAP. 26

When, in A, the fourth column is subtracted from the first, we have [-1,0,0,1,0]'; hence, if y =
Again, when the fourth column of A is subtracted from
-1, o]', AY = -Y and Y belongs to X+ 1.
the third, we have [o, 0, -1, 1, O]'; hence, if Z = [O, 0, 1, -1, O]', AZ = -Z find Z belongs to A+ 1. Since
y, Z, and the members of the chain led by X^ are linearly independent, we label Y as X^ and Z as X^. When
[l, 0, 0,

11-2

-1

11
R

[Xs.X^.X^.AXs,.A%]

0-1-1

-1

0-100

-1

R~ AR

0-3

10

-1

-1

12

0-2
the rational canonical form of A.

SUPPLEMENTARY PROBLEMS
7.

For each of the matrices (a)-{h) of Problem

Can any

over the rational field,

9,

Chapter 25, write the canonical matrix of Theorems

of these matrices be

I,

n, in

changed by enlarging the number field?

Partial Ans.

(a)

I,

-11

"o

(b)

II,

m.

I, II,

3)

o"

0_

"o

o'

_0

n, in.

I,

diag(l,2,

_0

(e)

ni,

-1

-1

1
(/)

n, ni.

I.

-1

1
o'

m.

n.

(g)

-1

-1

-2.

-1

-1

'2

(h)

8.

I,

10
10

n. in,

diag(2, 2,2, -1, -1)

Under what conditions will (a) the canonical forms of Theorems I and n be identical? (b) the canonical
forms of Theorems n and in be identical? (c) the canonical form of Theorem H be diagonal?

CHAP.

CANONICAL FORMS UNDER SIMILARITY

26]

"o
9.

o"

Identify the canonical form

Check with the answer

to

Let the non-singular matrix A have non-trivial invariant factors


Write the canonical forms of
that of

Theorem

Ans.

(a)

Problem

8(6).

0_

_0

10.

213

(a) X + 1
Theorems I, n,

A,

III

(X + i)^

IV.

-1
1
1

-1

10
1
1
1
1

-10

_0

-2

0_

-1

-1
1

-1

-2
1

n.

-1

1
1

1
1
-

-1

-3

2
6"

-1

-1
-1

-1
1

in,

-1

1
1
--1

-1

-1

1_

o"

-1
-1

-1

a
IV,

where a,^ = ^(li\/3).

a
/S
/3

(b)

X^+

1,

over the rational field and

CANONICAL FORMS UNDER SIMILARITY

214

11.

Hint.

12.

Prove: If with respect to an n-square matrix A, the vector


polynomial m(A) of A.

In

Suppose the contrary and consider

Example

show

6,

X,AX. and Y

that

m(A)

A (A)

belongs to g(X) then g(X) divides the minimum

g (A) +

"

[CHAP. 26

r (X).

are linearly independent and then reduce

to its rational canonical

form.

13.

In

Problem

6:

Take 7

(a)

= [o,

linearly independent of the chain led by ^5, and obtain

1, 0, 0, o]',

X4

y_

(3/1

_2/)

A^g

belonging to A+1.

Take

(i>)

= [O, 0,

belonging to A +

Compute R~

(c)

14.

led by X5, and obtain

X^ = Z

-X^

1.

using the vectors X3 and X^ of (b) and

For each of the matrices A of Problem 9(a)-(h), Chapter


ical form of

15.

AR

X^ and the chain

linearly independent of

1, 0, O]',

(a) to build R.

25, find

R such

R' AR

that

is the rational canon-

Solve the system of linear differential equations


dxi

2Xx

X2

Xg

4^1

2x2

3At3

6*1

2a:2

- 3*3 -

2xi^

3ai

X2

2x4.

X4 +

dt

dx2
dt

dxs

dt

dX4
dt

where the x^ are unknown functions of the


TT.

i.

Hint.

Leti

^V

U, . .3. -JT

J
define

(i)

dt

real variable

"^^

dxQ

dxi

|^^,

111

dX_

-6

-2

-3

-2

-3

-1

-1

-2

Since the non-singular linear transformation

dY

RY

Xq

t.

dxA'

dxn

-jf. -jf,

^J

AX

carries

R ^ARY +

and rewrite the system as

(i)

into

R^H

dt

choose R so that R~

AR

is leader of the chain

The elementary 4-vector 1 belonging to A - A


Now
4 yields X2 = E^- Xi + 2AXi belonging to A + 1.

is the rational canonical form of A.

X-i

= Ei, AX^.A'^X-i

while

12-1

with

4-2
8
0-6 4 -12
0-3 2-5

[X.L,AXi,A^Xi,X2]

0'
t

dY^
dt

Y +

yi + 73

y2

-1

-74

Then
2Ci +

C^+it'^
Cge* + Cge-* -

-Ci +

-t_l,2^
J. Cse"Cse^Sr
46"*

and

RY

C2e*+

3(Cs + C4)e"*+

^-2t+l'

2C1 + 2C2e*+ 2(3C3 + 4C4)e-* + ^ -4C1 - 2C2e*- 2(5C3+6C4)e-* - 2^+

-2C1 -

Cae*-

5(C3+C4)e"* -

4 + 2
6t

- 4

2+3{-2_

INDEX
Absolute value of a complex number, 110
Addition
of matrices,

2,

Characteristic roots (cont.)


of Hermitian matrices, 164
of inverse A, 155

of vectors, 67

of real orthogonal matrices, 155


of real skew-symmetric matrices, 170

Adjoint of a square matrix


definition of, 49

of real symmetric matrices, 163

determinant of, 49
inverse from, 55
rank of, 50
Algebraic complement, 24
Anti-commutative matrices, 11
Associative laws for

of unitary matrices, 155


Characteristic vectors
{see Invariant vectors)
Classical canonical form, 206

addition of matrices, 2
fields, 64
multiplication of matrices, 2
Augmented matrix, 75

Cofactor, 23
Cogredient transformation, 127

Closed, 85
Coefficient matrix, 75

Column
space of a matrix, 93
transformation, 39
Commutative law for
addition of matrices, 2
fields, 64
multiplication of matrices, 3
Commutative matrices, 11
Companion matrix, 197
Complementary minors, 24
Complex numbers, 12, 110
Conformable matrices

Basis

change

of,

95

of a vector space, 86

orthonormal, 102, 111


Bilinear form(s)
canonical form of, 126
definition of, 125
equivalent, 126
factorization of, 128

rank of, 125


reduction of, 126

for addition, 2

for multiplication, 3
Congruent matrices, 115

Canonical form

Conjugate
of a complex number, 12

classical (Jordan), 206

Jacobson, 205
of bilinear form, 126
of Hermitian form, 146
of matrix, 41, 42
of quadratic form, 133
rational, 203
row equivalent, 40
Canonical set
under congruence, 116, 117
under equivalence, 43, 189
under similarity, 203
Cayley-Hamilton Theorem, 181
Chain of vectors, 207
Characteristic
equation, 149
polynomial, 149
Characteristic roots
definition of, 149
of adj A, 151
of a diagonal matrix, 155
of a direct sum, 155

of a matrix,

of a product,

12
13

of a sum, 13

Conjugate transpose, 13
Conjunctive matrices, 117
Contragredient transformation, 127
Coordinates of a vector, 88
Cramer's rule, 77
Decomposition of a matrix into
Hermitian and skew-Hermitian parts, 13
symmetric and skew-symmetric parts, 12
Degree
of a matrix polynomial, 179
of a (scalar) polynomial, 172

Dependent
forms, 69
matrices, 73
polynomials, 73
vectors, 68

Derogatory matrix, 197

215

INDEX

216

Hermitian form
canonical form

Determinant
definition of, 20

derivative of, 33

of,

146

147
index of, 147
rank of, 146

definite,

expansion of
along first row and column, 33
along a row (column), 23
by Laplace method, 33
multiplication by scalar, 22
of conjugate of a matrix, 30
of conjugate transpose of a matrix, 30
of elementary transformation matrix, 42
of non-singular matrix, 39
of product of matrices, 33
of singular matrix, 39
of transpose of a matrix, 21

Diagonal
elements of a square matrix,

semi-definite, 147

signature

of,

147

Hermitian forms
equivalence

of,

146

Hermitian matrix, 13, 117, 164


Hypercompanion matrix, 205
Idempotent matrix, 11
Identity matrix, 10

Image
1

matrix, 10, 156

Diagonable matrices, 157


Diagonalization
by orthogonal transformation, 163
by unitary transformation, 164
Dimension of a vector space, 86
Direct sum, 13
Distributive law for
fields, 64
matrices, 3
Divisors of zero, 19
Dot product, 100

Eigenvalue, 149
Eigenvector, 149

Elementary
matrices, 41
n-vectors, 88
transformations, 39

of a vector, 94
of a vector space, 95

Index
of an Hermitian form, 147
of a real quadratic form, 133
Inner product, 100, 110
Intersection space, 87

Invariant vector(s)
definition of, 149
of a diagonal matrix, 156
of an Hermitian matrix, 164
of a normal matrix, 164
of a real symmetric matrix, 163
of similar matrices, 156
Inverse of a (an)
diagonal matrix, 55
direct sum, 55
elementary transformation, 39
matrix, 11, 55
product of matrices, 11
symmetric matrix, 58
Involutory matrix, 11

Equality of
matrices, 2
matrix polynomials, 179
(scalar) polynomials, 172

Equations, linear
equivalent systems
solution of, 75

Kronecker's reduction, 136


of,

75

Lagrange's reduction, 132

system of homogeneous, 78
system of non-homogeneous, 77
Equivalence relation, 9
Equivalent
bilinear forms, 126

Hermitian forms, 146


matrices, 40, 188
quadratic forms, 131, 133, 134
systems of linear equations, 76

Factorization into elementary matrices, 43, 188


Field, 64
Field of values, 171
First minor, 22

common

Lambda

matrix, 179
Laplace's expansion, 33
Latent roots (vectors), 149
Leader of a chain, 207
Leading principal minors, 135
Left divisor, 180
Left inverse, 63

Linear combination of vectors, 68


Linear dependence (independence)
of forms, 70
of matrices, 73
of vectors, 68

Lower triangular matrix, 10


Matrices
congruent, 115

Gramian, 103, 111


Gram-Schmidt process,
Greatest

Jacobson canonical form, 205


Jordan (classical) canonical form, 206

equal, 2
102, 111

divisor, 173

equivalent, 40

over a

field,

65

INDEX
Matrices (cont.)
product of, 3

Null space, 87
Nullity, 87

scalar multiple of, 2


similar, 95, 156
square, 1

sum

of, 2

Matrix
derogatory, 197
diagonable, 157
diagonal, 10

elementary row (column), 41


elementary transformation of, 39
Hermitian, 13, 117, 164
idempotent, 11
inverse of, 11, 55
lambda, 179
nilpotent, 11

nonderogatory, 197
non-singular, 39
normal, 164
of,

w-vector, 85

Order of a matrix,
Orthogonal

congruence, 163
equivalence, 163
matrix, 108

definition of, 1

normal form

217

41

nullity of, 87

similarity, 157, 163


transformation, 103
vectors, 100, 110

Orthonormal

basis,

102,

111

Partitioning of matrices, 4
Periodic matrix, 11

Permutation matrix, 99
Polynomial
domain, 172
matrix, 179
monic, 172
scalar, 172
scalar matrix, 180

of a bilinear form, 125

Positive definite (semi-definite)

of an Hermitian form, 146


of a quadratic form, 131

Hermitian forms, 147


matrices, 134, 147
quadratic forms, 134
Principal minor

order of, 1
orthogonal, 103, 163
periodic, 11

permutation, 99
polynomial, 179
positive definite (semi-definite), 134, 147

rank

of,

39

scalar,

10
singular, 39

skew-Hermitian, 13, 118


skew-symmetric, 12, 117
symmetric, 12, 115, 163
triangular, 10, 157
unitary, 112, 164

Matrix Polynoniial(s)
definition of, 179
degree of, 179
product of, 179
proper (improper), 179
scalar, 180
singular (non-singular), 179
sum of, 179
Minimum polynomial, 196

Multiplication
in partitioned form, 4
of matrices, 3

definition of, 134

leading, 135

Product of matrices
adjoint of, 50
conjugate of, 13
determinant of, 33
inverse of, 11
of, 43

rank

transpose

of, 12

Quadratic form
canonical form of, 133, 134
definition of, 131
factorization of, 138
rank of, 131
reduction of

Kronecker, 136
Lagi-ange, 132
regular, 135
Quadratic form, real
definite, 134
index of, 133
semi-definite, 134
signature of, 133

Quadratic forms
equivalence

Negative
definite form (matrix), 134, 147
of a matrix, 2
semi-definite form (matrix), 134, 147
Nilpotent matrix, 11

of, 131, 133,

Rank
of adjoint, 50
of bilinear form, 125
of Hermitian form, 146

Non-derogatory matrix, 197


Non-singular matrix, 39

of matrix, 39
of product, 43

Normal form of a matrix, 41


Normal matrix, 164

of quadratic form, 131


of sum, 48

134

INDEX

218
Right divisor, 180
Right inverse, 63
Root
of polynomial, 178
of scalar matrix polynomial, 187

Row

(cont.)

deiinition of, 12

invariant vectors of, 163


System(s) of Equations, 75
Trace, 1

equivalent matrices, 40
space of a matrix, 93
transformation, 39

Scalar
matrix, 10
matrix polynomial, 180
multiple of a matrix, 2
polynomial, 172
product of two vectors (see inner product)
Schwarz Inequality, 101, 110
Secular equation (see
characteristic equation)
Signature
of Hermitian form, 147
of Hermitian matrix, 118
of real quadratic form, 133
of real symmetric matrix, 116
Similar matrices, 95, 196
Similarity invariants, 196
Singular matrix, 39
Skew-Hermitian matrix, 13, 118
Skew-symmetric matrix, 12, 117
Smith normal form, 188

Span, 85
Spectral decomposition, 170
Spur (see Trace)
Sub-matrix, 24
Sum of
matrices, 2
vector spaces, 87
Sylvester's

Symmetric matrix

law

of inertia, 133
of nullity, 88

Symmetric matrix
characteristic roots of, 163

Transformation
elementary, 39
linear, 94
orthogonal, 103
singular, 95
unitary, 112

Transpose
of a matrix, 11
of a product, 12
of a sum, 11

Triangular inequality, 101, 110


Triangular matrix, 10, 157

Unit vector, 101


Unitary
matrix, 112
similarity, 157
transformation, 112
Upper triangular matrix, 10
Vector(s)

belonging to a polynomial, 207


coordinates of, 88
definition of, 67
inner product of, 100
invariant, 149
length of, 100, 110
normalized, 102
orthogonal, 100
vector product of, 109

Vector space
basis of, 86
definition of, 85

dimension of, 86
over the complex field, 110
over the real field, 100

Index of Symbols

Symbol

Page

Page
88

Hj

E^, (vector)

[^i;]

X-Y; X Y

100, 110

\\n

100, 110

103, 111

10

zxy

109

11

115

116

116

131

146

^k
A'

A-'-

A'-

Symbol

A':

11

I;

12

A*- A^

13

\A\;

det

20

22

\^ij\

A. A,-

149

23

<^(A)

149

a,.

23

E^, (matrix)

170

39

/(A)

172

h-^ij

39

F[A]

172

Hi(k), K^ik)

39

^(A)

H^.{k). K.j(k)

39

-V.

40

^ii'i2
ii, is, ....

im
i^

AjfiC),

A^ (C)

179
180

NiX)

189

fi(^)

189

49

m(A)

196

64

C(g)

198

X. X^

67

198

yn(f)

85

203

V^tiF)

86

Cq(p)

205

N
adj

^A

43

87

219

Vous aimerez peut-être aussi